Go Math Grade 6 Answer Key Chapter 1 Divide Multi-Digit Numbers

go-math-grade-6-chapter-1-divide-multi-digit-numbers-answer-key

Students who are in search of the Answer Key of Go Math Grade 6 Chapter 1 Divide Multi Digit Numbers can get them on this page. Refer to our Go Math Grade 6 Answer Key  and solve the problems. We provide the step by step explanations for all the question by using the pictures here. By using Go Math 6th Grade Chapter, 1 Answer Key the scholars will not find any difficulty in solving the questions.

Go Math Grade 6 Answer Key Chapter 1 Divide Multi-Digit Numbers 

In order to excel in exam we suggest the students of 6th standard to refer the Go Math Answer Key Chapter 1 Divide Multi-Digit Numbers. The solutions in Go Math Grade 6 Answer Key Chapter 1 Divide Multi-Digit Numbers are prepared by the math experts. You can understand the concepts in depth with the help of Go Math Grade 6 Solution Key.

Lesson 1: Divide Multi-Digit Numbers

Lesson 2: Prime Factorization

Lesson 3: Least Common Multiple

Lesson 4: Greatest Common Factor

Lesson 5: Problem Solving • Apply the Greatest Common Factor

Mid-Chapter Checkpoint

Lesson 6: Add and Subtract Decimals

Lesson 7: Multiply Decimals

Lesson 8: Divide Decimals by Whole Numbers

Lesson 9: Divide with Decimals

Chapter 1 Review/Test 

Page No. 7

Estimate. Then find the quotient. Write the remainder, if any, as a fraction.

Question 3.
6,114 ÷ 63

Answer:  Quotient is 97 3/63= 97 1/21 and the remainder is 3

Explanation:

Go Math Grade 6 Answer Key Chapter 1 Divide Multi-Digit Numbers

Question 4.
11050 ÷ 26

Answer: Quotient is 425 and the remainder is 0.

Explanation:
Go Math Grade 6 Answer Key Chapter 1 Divide Multi-Digit Numbers

On Your Own

Estimate. Then find the quotient. Write the remainder, if any, as a fraction.

Question 5.
3150 ÷ 9

Answer: Quotient is 350 and the remainder is 0.

Explanation:
Go Math Grade 6 Answer Key Chapter 1 Divide Multi-Digit Numbers

Question 6.
2115 ÷ 72

Answer: Quotient is 29 27/72= 29 3/8 and the remainder is 27.

Explanation:
Go Math Grade 6 Answer Key Chapter 1 Divide Multi-Digit Numbers

Question 7.
20835 ÷ 180

Answer: Quotient is 115 135/180= 115 3/4 and the remainder is 135

Explanation:
Go Math Grade 6 Answer Key Chapter 1 Divide Multi-Digit Numbers

Question 8.
Find the least whole number that can replace ? to make the statement true.
110 < ? ÷ 47

Answer: Least whole number is 5,171.

Explanation: 110×47= 5,170

Question 9.
Use Reasoning Name two whole numbers that can replace ? to make both statements true.
2 × ? < 1800 ÷ 12                         ? > 3744 ÷ 52

Answer:

Explanation:

Question 10.
The 128 employees of a company volunteer 12,480 hours in 26 weeks. On average, how many hours do they all volunteer per week? On average, how many hours does each employee volunteer per week?

Answer: 3.75 hours.

Explanation: In 26  weeks 128 employees volunteer 12,480 hours, so in 1 week they volunteer 12,480÷26= 480 hours.
And each employee volunteer per week is 480÷128= 3.75 hours.

Question 11.
A factory produces 30,480 bolts in 12 hours. If the same number of bolts are produced each hour, how many bolts does the factory produce in 5 hours?

Answer: 12,700.

Explanation: As the factory produces 30,480 bolts in 12 hours, so in a 1-hour company produces 30,480÷12= 2,450 boults and in 5 hours it produces 2,450×5= 12,700.

Problem Solving + Applications – Page No. 8

Use the table for 12-15.
Go Math Grade 6 Answer Key Chapter 1 Divide Multi-Digit Numbers img 1

Question 12.
A Smooth Flight jet carried 6,045 passengers last week, and all of its flights were full. How many flights did the jet make last week?

Answer: 15 flights.

Explanation: As there are 403 seats in Smooth flight and 6,045 passengers are carried in last week, so no.of flights did the jet make in last week are 6045÷403= 15

Question 13.
Last month an airline made 6,322 reservations for flights from Newark, New Jersey, to Frankfurt, Germany. If there were 21 full flights and 64 reservations cancelled, which airplane made the flights?

Answer: Jet Set.

Explanation: Total reservations made by the airline are 6,322 and cancelled are 64, so completed reservations are 6,322-64= 6,258 and 21 flights are full so 6258÷21= 298 passengers in each flight and that airplane is Jet set

Question 14.
An airline carries about 750 passengers from Houston to Chicago each day. How many Blue Sky jets would be needed to carry this many passengers, and how many empty seats would there be?

Answer: 5 Sky jets would be needed and 50 empty seats would be there.

Explanation: Let’s round off 750 to 800, As there are 800 passengers each day so no. of Sky jets needed are 800÷160= 5. and 50 empty seats would be there.

Question 15.
Pose a Problem Refer back to Problem 12. Use the information in the table to write a similar problem involving airplane passenger seats.

Answer:

Explanation:

Question 16.
For numbers 16a – 16d, choose Yes or No to indicate whether the equation is correct.
16a. 1,350 ÷ 5 = 270 O Yes O No
16b. 3,732 ÷ 4 = 933 O Yes O No
16c. 4,200 ÷ 35 = 12 O Yes O No
16d. 1,586 ÷ 13 = 122 O Yes O No

16a. Answer: Yes

Explanation: 1,350÷5= 270.

16b. Answer: Yes

Explanation: 3732÷4= 933

16c. Answer: No

Explanation: 4200÷35= 120

16d. Answer: Yes

Explanation: 1586÷13= 122.

Divide Multi-Digit Numbers – Page No. 9

Estimate. Then find the quotient. Write the remainder, if any, with an r.

Question 1.
180)\(\overline { 20835 } \)

Answer: Quotient is 115 and remainder r135

Explanation:
Go Math Grade 6 Answer Key Chapter 1 Divide Multi-Digit Numbers

Question 2.
19)\(\overline { 800 } \)

Answer: Quotient is 42 and remainder r2

Explanation:
Go Math Grade 6 Answer Key Chapter 1 Divide Multi-Digit Numbers

Question 3.
68)\(\overline { 1025 } \)

Answer: Quotient is 15 and remainder r5

Explanation:
Go Math Grade 6 Answer Key Chapter 1 Divide Multi-Digit Numbers

Estimate. Then find the quotient. Write the remainder, if any, as a fraction.

Question 4.
20)\(\overline { 1683 } \)

Answer: Quotient is 84 and remainder r3.

Explanation:
Go Math Grade 6 Answer Key Chapter 1 Divide Multi-Digit Numbers

Question 5.
14124 ÷ 44

Answer: Quotient is 321 and remainder r0

Explanation:
Go Math Grade 6 Answer Key Chapter 1 Divide Multi-Digit Numbers

Question 6.
11629 ÷ 29

Answer: Quotient is 401 and remainder r0

Explanation:
Go Math Grade 6 Answer Key Chapter 1 Divide Multi-Digit Numbers

Find the least whole number that can replace ? to make the statement true.

Question 7.
? ÷ 7 > 800

Answer: The least whole number to makes the statement true is 5600

Explanation: 5600÷7> 800

Question 8.
? ÷ 21 > 13

Answer: The least whole number to makes the statement true is 273

Explanation:  273÷21>13

Question 9.
15 < ? ÷ 400

Answer: The least whole number to makes the statement true is 6000

Explanation: 15< 6000÷400

Problem Solving

Question 10.
A plane flew a total of 2,220 miles. Its average speed was 555 miles per hour. How many hours did the plane fly?

Answer: 4 hours

Explanation: Total miles does a plane flew is 2,220 miles and average speed is 555 miles per hour. So total hours did plane fly are 2,220÷555= 4 hours

Question 11.
A van is carrying 486 pounds. There are 27 boxes in the van. What is the average weight of each box in the van?

Answer: 18 lbs

Explanation: No.of pounds did the van carry are 486 pounds and no.of boxes in a van are 27. So the average weight of each box is 486÷27= 18

Question 12.
Find 56,794 ÷ 338. Write the quotient twice, once with the remainder as a fraction and once with an r.

Answer: 56,794÷338= 168 10/338= 168 5/169, r=10.

Explanation: 56,794÷338= 168 10/338= 168 5/169 and reminder is 10

Lesson Check – Page No. 10

Question 1.
A caterer’s fee is based on the number of meals she provides. How much is the price per meal if the total fee is $1,088 for 64 meals?

Answer: $17.

Explanation: No.of meals are 64 and the total fee is $1,088. Therefore the price per meal is $1,088÷64= $17.

Question 2.
Amelia needs 24 grains of beads to make a bracelet. She has 320 grams of beads. How many bracelets can she make?

Answer: 13 bracelets.

Explanation: No.of grains of beads Amelia need are 24 and she has 320 grams of beads. So no.of bracelets can Amelia make are 320÷24= 13.33 will round off to 13.

Spiral Review

Question 3.
Hank bought 2.4 pounds of apples. Each pound cost $1.95. How much did Hank spend on the apples?

Answer: $4.68

Explanation: No.of pounds of apples Hank bought is 2.4 pounds and each pound cost is $ 1.95, so total Hank spend on apples is 2.4×$1.95= $4.68

Question 4.
Gavin bought 4 packages of cheese. Each package weighed 1.08 kilograms. How many kilograms of cheese did Gavin buy?

Answer: 4.32kg

Explanation: No.of cheese packages Gavin bought are 4 and each package weight is 1.08 kg. So total weight of cheese is 4×1.08= 4.32 kg

Question 5.
Mr. Thompson received a water bill for $85.98. The bill covered three months of service. He used the same amount of water each month. How much does Mr. Thompson pay for water each month?

Answer: $28.66

Explanation: Water bill received to Mr. Thompson is $85.98 as he covered for 3 months the amount Mr.Thompson paid for each month is $85.98÷3= $28.66

Question 6.
Layla used 0.482 gram of salt in her experiment. Maurice use 0.51 gram of salt. Who used the greater amount of salt?

Answer: Maurice as 0.51 is greater than 0.482.

Explanation: Salt used by Layla is 0.482 grams and salt used by Maurice is 0.51 grams, so the greatest amount of salt used is Maurice as 0.51 is greater than 0.482.

Find the prime factorization – Page No. 13

Question 3.
75

Answer: 5×5×3.

Explanation:     75
15         5
5×3       5
5×5×3

Question 4.
12

Answer: 3×2×2.

Explanation:     12
6×2
3×2×2

Question 3.
65

Answer: 13×5

Explanation:   65
13×5

On Your Own

Write the number whose prime factorization is given.

Question 6.
2 × 2 × 2 × 7

Answer: 56

Question 7.
2 × 2 × 5 × 5

Answer: 100

Question 8.
2 × 2 × 2 × 2 × 3 × 3

Answer: 144

Practice: Copy and Solve Find the prime factorization.

Question 9.
45

Answer: 5×3×3

Explanation:
45
5×9
5×3×3

Question 10.
50

Answer: 5×5×2

Explanation:
50
5×10
5×5×2

Question 11.
32

Answer: 2×2×2×2×2

Explanation:
32
2×16
2×2×8
2×2×2×4
2×2×2×2×2

Question 12.
76

Answer:  2×2×19

Explanation:
76
2×38
2×2×19

Question 13.
108

Answer: 2×2×3×3×3

Explanation:
108
2×54
2×2×27
2×2×3×9
2×2×3×3×3

Question 14.
126

Answer:  2×7×3×3

Explanation:
126
2×63
2×7×9
2×7×3×3

Question 15.
The area of a rectangle is the product of its length and width. A rectangular poster has an area of 260 square inches. The width of the poster is greater than 10 inches and is a prime number. What is the width of the poster?

Answer: Width is 13.

Explanation: Area of a rectangular poster is 260 square inches i.e width×length= 260 sq inches. And the width of the poster is greater than 10 inches and it is a prime number, so the width will be 13 as 13 is a prime number and also 260 is divisible by 13, and length is 13×length= 260 in which length is 260÷13= 20.

Question 16.
Look for Structure Dani says she is thinking of a secret number. As a clue, she says the number is the least whole number that has three different prime factors. What is Dani’s secret number? What is its prime factorization?

Answer: Dani’s secret number is 30 and the prime factorization is 2,3,5.

Explanation: The least three prime numbers are 2,3,5, so the product of three prime numbers is 2×3×5= 30.

Problem Solving + Applications – Page No. 14

Use the table for 17–19. Agent Sanchez must enter a code on a keypad to unlock the door to her office.
Go Math Grade 6 Answer Key Chapter 1 Divide Multi-Digit Numbers img 2

Question 17.
In August, the digits of the code number are the prime factors of 150. What is the code number for the office door in August?

Answer: 2355.

Explanation: Prime Factors of 150 are 2×3×5×5, so code number for office door in August is 2355

Question 18.
In September, the fourth digit of the code number is 2 more than the fourth digit of the code number based on the prime factors of 225. The prime factors of what number were used for the code in September?

Answer: 315.

Explanation: Prime factors of 225 are 3×3×5×5 which is 3355 as the fourth digit of the code number is 2 more than the fourth digit, s0 5+2=7 and by replacing 7 in 3×3×5×5, then 3×3×5×7= 315.

Question 19.
One day in October, Agent Sanchez enters the code 3477. How do you know that this code is incorrect and will not open the door?

Answer: 4 is not a prime number.

Explanation: The code 3477 is incorrect as the code contains only a prime number and 4 is not a prime number.

Question 20.
Use the numbers to complete the factor tree. You may use a number more than once.
2 3 6 9 18

Answer: 36= 2×2×3×3

Explanation:
Go Math Grade 6 Answer Key Chapter 1 Divide Multi-Digit Numbers

Prime Factorization – Page No. 15

Find the prime factorization.

Question 1.
44

Answer: 2×2×11

Explanation:
44
2×22
2×2×11

Question 2.
90

Answer: 2×3×3×5

Explanation:
90
2×45
2×3×15
2×3×3×5

Question 3.
48

Answer:

Explanation:
48
2×24
2×2×12
2×2×2×6
2×2×2×2×3

Question 4.
204

Answer: 2×2×3×17

Explanation:
204
2×102
2×2×51
2×2×3×17

Question 5.
400

Answer: 2×2×2×2×5×5

Explanation:
400
2×200
2×2×100
2×2×2×50
2×2×2×2×25
2×2×2×2×5×5

Question 6.
112

Answer: 2×2×2×2×7

Explanation:
112
2×56
2×2×28
2×2×2×14
2×2×2×2×7

Problem Solving

Question 7.
A computer code is based on the prime factorization of 160. Find the prime factorization of 160.

Answer: 2×2×2×2×2×5

Explanation: Prime factors of 160 is 2×2×2×2×2×5

Question 8.
The combination for a lock is a 3-digit number. The digits are the prime factors of 42 listed from least to greatest. What is the combination for the lock?

Answer: 237.

Explanation: Prime factors of 42 is 2×3×7.

Question 9.
Describe two methods for finding the prime factorization of a number.

Answer:
1. Divison Method.
2. Factor Tree Method.

Explanation:
1. Division Method: In Division method first we will divide the number by smallest prime number, and repeat the process until the quotient became 1.
2. Factor Tree Method: In Factor Tree Method we will write a pair of factors as the branches of the tree and then we will factorize.

Lesson Check – Page No. 16

Question 1.
Maritza remembers her PIN because it is between 1,000 and 1,500 and it is the product of two consecutive prime numbers. What is her PIN?

Answer: Two consecutive prime numbers are 31 and 37 and PIN is 1147.

Explanation: As 31 and 37 are two consecutive prime numbers and their product is 1147 which is between 1,000 to 1,500.

Question 2.
Brent knows that the 6 -digit number he uses to open his computer is the prime factorization of 5005. If each digit of the code increases from left to right, what is his code?

Answer: 111357.

Explanation: Factors of 5005 are 5×7×11×13, as the increases from left to right so the code is 111357

Spiral Review

Question 3.
Piano lessons cost $15. What expressions could be used to find the cost in dollars of 5 lessons?

Answer: $15×5= $75

Explanation: We will use multiplication to find the cost in dollars of 5 lessons.

Question 4.
A jet plane costs an airline $69,500,000. What is the place value of the digit 5 in this number?

Answer: Hundred thousand.

Explanation: The place value of 5 in $69,500,000 is 500,000.

Question 5.
A museum has 13,486 butterflies, 1,856 ants, and 13,859 beetles. What is the order of the insects from least number to greatest number?

Answer: Ants, Butterflies, Beetles.

Explanation: The order of insects from least to greatest are Ants, Butterflies, Beetles.

Question 6.
Juan is reading a 312-page book for school. He reads 12 pages each day. How long will it take him to finish the book?

Answer: 26 days.

Explanation: As Juan reads 12 pages each day and the book has 312 pages, so he will finish in 312÷12= 26 days

Find the LCM – Page No. 19

Question 2.
3, 5

Answer: 15

Explanation:
Multiples of 3: 3,6,9,12,15
Multiples of 5: 5,10,15.
LCM is 15

Question 3.
3, 9

Answer: 9

Explanation:
Multiples of 3: 3,6,9
Multiples of 9: 9
LCM is 9

Question 4.
9, 15

Answer: 135

Explanation:
Multiples of 9: 9,18,27,36,45,54,63,72,81,90,99,108,117,126,135.
Multiples of 15: 15,30,45,60,75,90,105,120,135.
LCM is 135

On Your Own

Find the LCM.

Question 5.
5, 10

Answer: 10

Explanation:
Multiples of 5: 5,10
Multiples of 10: 10
LCM is 10

Question 6.
3, 8

Answer: 24

Explanation:
Multiples of 3: 3,6,92,15,18,21,24
Multiples of 8: 8,16,24
LCM is 24

Question 7.
9, 12

Answer: 108

Explanation:
Multiples of 9: 9,18,27,36,45,54,63,72,81,90,99,108
Multiples of 12: 12,24,36,48,60,72,84,96,108
LCM is 108

Use Reasoning Algebra Write the unknown number for ?.

Question 8.
5, 8      LCM : ?
? =

Answer: 40

Explanation:
Multiples of 5: 5,10,15,20,25,30,35,40
Multiples of 8: 8,16,24,32,40
LCM is 40

Question 9.
?, 6      LCM : 42
? =

Answer: 7

Explanation: 6×7= 42

Question 10.
How can you tell when the LCM of two numbers will equal one of the numbers or equal the product of the numbers?

Answer: If the other number is 1 then the LCM of two numbers will equal one.

Question 11.
Verify the Reasoning of Others Mr. Haigwood is shopping for a school picnic. Veggie burgers come in packages of 15, and buns come in packages of 6. He wants to serve veggie burgers on buns and wants to have no items left over. Mr. Haigwood says that he will have to buy at least 90 of each item, since 6 × 15 = 90. Do you agree with his reasoning? Explain.

Answer: No. We must find the least number of burgers and buns, so we must find LCM of 15 and 6.

Explanation:
Multiples of 15: 15,30
Multiples of 6: 6,12,18,24,30
LCM is 30.

Question 12.
A deli has a special one -day event to celebrate its anniversary. On the day of the event, every eighth customer receives a free drink. Every twelfth customer receives a free sandwich. If 200 customers show up for the event, how many of the customers will receive both a free drink and a free sandwich?

Answer: 24,48,72,96,120,144,168,192 are the customers who get both free drink and free sandwich.

Explanation: To find how many customers have received both a free drink and a sandwich, first we have to find who got a free sandwich and a free drink separately, so
Multiples of 8 are 8,16,24,32,40,48,56,64,72,80,88,96,104,112,120,128, 136,144,152,160,168,176,184,192 and 200 and
Multiples of 12 are 12,24,36,48,60,72,84,96,108,120,132,144,156,168,180 and 192. So common customers are 24,48,72,96,120,144,168,192 are the customers who get both free drink and free sandwich.

Unlock The Problem – Page No. 20

Question 13.
Katie is making hair clips to sell at the craft fair. To make each hair clip, she uses 1 barrette and 1 precut ribbon. The barrettes are sold in packs of 12, and the precut ribbons are sold in packs of 9. How many packs of each item does she need to buy to make the least number of hair clips with no supplies left over?
a. What information are you given?

Answer: 3 packs of barrettes and 4 packs of precut ribbons.

Explanation: As barrettes are sold in packs of 12 and precut ribbons are sold in packs of 9, so we need to find the number of packs of each item does she need to make the least number of hair clips with no supplies left over. So the LCM of 12 and 9.
Multiples of 12 are: 12,24,36
Multiples of 9 are: 9,18,27,36
LCM is 36
So Katie needs 36 barrettes and ribbons to make the least number of hair clips with no supplies left over, and she needs 3 packs of barrettes and 4 packs of precut ribbons.

Question 13.
b. What problem are you being asked to solve?

Answer: To find the number of packs of each item does she need to make the least number of hair clips with no supplies left over

Question 13.
c. Show the steps you use to solve the problem.

Answer:
Multiples of 12 are: 12,24,36
Multiples of 9 are: 9,18,27,36
LCM is 36

Question 13.
d. Complete the sentences.
The least common multiple of 12 and 9 is _____ .
Katie can make _____ hair clips with no supplies left over.
To get 36 barrettes and 36 ribbons, she needs to buy _____ packs of barrettes and _____ packs of precut ribbons.

Answer: 36, 3, 4.

Explanation:
The least common multiple of 12 and 9 is 36.
Katie can make 36 hair clips with no supplies left over.
To get 36 barrettes and 36 ribbons, she needs to buy 3 packs of barrettes and 4 packs of precut ribbons.

Question 14.
Reptile stickers come in sheets of 6 and fish stickers come in sheets of 9. Antonio buys the same number of both types of stickers and he buys at least 100 of each type. What is the least number of sheets of each type he might buy?

Answer: 108

Explanation: As Reptile stickers come in sheets of 6 and fish stickers come in sheets of 9, so we will find the LCM of 6 and 9 to get the least number of sheets,
Multiples of 6 are 6,12,18
Multiples of 9 are 9,18
LCM is 18
As Antonio buys at least 100 of each type, so multiples of 18 are 18,36,54,72,90,108 as 108 is the least number and more than 100 and nearest to 100, so the least number of sheets he might buy= 108

Question 15.
For numbers 15a -15d, choose Yes or No to indicate whether the LCM of the two numbers is 16.
15a. 2,8 O Yes O No
15b. 2,16 O Yes O No
15c. 4,8 O Yes O No
15d. 8,16 O Yes O No

15a. 2,8 O Yes O No

Answer: No

Explanation:
Multiples of 2 are 2,4,6,8
Multiples of 8 are 8
LCM is 8

15b. 2,16 O Yes O No

Answer: Yes

Explanation:
Multiples of 2 are 2,4,6,8,10,12,14,16
Multiples of 16 are 16
LCM is 16

15c. 4,8 O Yes O No

Answer: No

Explanation:
Multiples of 4 are 4,8
Multiples of 8 are 8
LCM is 8

15d. 8,16 O Yes O No

Answer: 16

Explanation:
Multiples of 8 are 8,16
Multiples of 16 are 16
LCM is 16

Least Common Multiple – Page No. 21

Find the LCM.

Question 1.
2, 7

Answer: 14

Explanation:
Multiples of 2 are 2,4,6,8,10,12,14.
Multiples of 7 are 7,14.
LCM is 14.

Question 2.
4, 12

Answer: 12

Explanation:
Multiples of 4 are 4,8,12
Multiples of 12 are 12
LCM is 12

Question 3.
6, 9

Answer: 54

Explanation:
Multiples of 6 are 6,12,18,24,30,36,42,48,54
Multiples of 9 are 9,18,27,36,45,54
LCM is 54

Question 4.
5, 4

Answer: 8

Explanation:
Multiples of 5 are 5,10,15
Multiples of 4 are 8
LCM is 8

Question 5.
5, 8, 4

Answer: 40

Explanation:
Multiples of 5 are 5,10,15,20,25,30,35,40
Multiples of 8 are 8,16,24,32,40
Multiples of 4 are 4,8,12,16,20,24,28,32,36,40
LCM is 40

Question 6.
12, 8, 24

Answer: 24

Explanation:
Multiples of 12 are 12,24
Multiples of 8 are 8,16,24
Multiples of 24 are 24
LCM is 24

Write the unknown number for the?

Question 7.
3, ?        LCM : 21
? =

Answer: 7

Explanation: 3×7= 21

Question 8.
?, 7        LCM : 63
? =

Answer: 9

Explanation: 9×7=63

Question 9.
10, 5     LCM : ?
? =

Answer: 10

Explanation:
Multiples of 10 are 10
Multiples of 5 are 5,10
LCM is 10

Problem Solving

Question 10.
Juanita is making necklaces to give as presents. She plans to put 15 beads on each necklace. Beads are sold in packages of 20. What is the least number of packages she can buy to make necklaces and have no beads left over?

Answer: 3 packages.

Explanation:
Multiples of 15: 15,30,45,60
Multiples of 20: 20,40,60
LCM is 60
As beads are sold in packages of 20 Juanita needs 3 least number of packages to make necklaces with no beads leftover.

Question 11.
Pencils are sold in packages of 10, and erasers are sold in packages of 6. What is the least number of pencils and erasers you can buy so that there is one pencil for each eraser with none left over?

Answer: 30 pencils and 30 erasers are the least numbers we can buy without any leftover.

Explanation:
Multiples of 10: 10,20,30.
Multiples of 6: 6,12,18,24,30.
LCM is 30.
So 30 pencils and 30 erasers are the least numbers we can buy without any leftover.

Question 12.
Explain when you would use each method (finding multiples or prime factorization) for finding the LCM and why.

Answer: When the numbers are smaller we can use finding multiples and when the numbers are larger then we can use prime factorization.

Lesson Check – Page No. 22

Question 1.
Martha is buying hot dogs and buns for the class barbecue. The hot dogs come in packages of 10. The buns come in packages of 12. What is the least number she can buy of each so that she has exactly the same number of hot dogs and buns? How many packages of each should she buy?
_________ packages of hot dogs
_________ packages of buns

Answer: 6 packages of hot dogs and 5 packages of buns she can buy.

Explanation:
Multiples of 10: 10,20,30,40,50,60.
Multiples of 12: 12,24,36,48,60.
LCM is 60.
So 60 is the least number she can buy and 6 packages of hot dogs and 5 packages of buns she can buy.

Question 2.
Kevin makes snack bags that each contain a box of raisins and a granola bar. Each package of raisins contains 9 boxes. The granola bars come 12 to a package. What is the least number he can buy of each so that he has exactly the same number of granola bars and boxes of raisins? How many packages of each should he buy?
_________ packages of raisins
_________ packages of granola bars

Answer: 4 packages of raisins and 3 packages of granola bars he should buy.

Explanation: Kevin’s every package contains 9 raisins boxes and 12 granola bars in each package, so LCM of 9 and 12 are
Multiples of 9: 9,18,21,36
Multiples of 12: 12,24,36
LCM is 36.
So 4 packages of raisins and 3 packages of granola bars he should buy.

Spiral Review

Question 3.
John has 2,456 pennies in his coin collection. He has the same number of pennies in each of 3 boxes. Estimate to the nearest hundred the number of pennies in each box.

Answer: 800 pennies.

Explanation: Let’s round off 2,456 to 2400, as he has the same no. of pennies in each of 3 boxes, so in each box no.of pennies are 2400÷3= 800 pennies.

Question 4.
What is the distance around a triangle that has sides measuring 2 \(\frac{1}{8}\) feet, 3 \(\frac{1}{2}\) feet, and 2 \(\frac{1}{2}\) feet?

Answer: 8 1/8 feet

Explanation: Distance around the triangle is 2 1/8+3 1/2+ 2 1/2= 8 1/8 feet

Question 5.
The 6th grade class collects $1,575. The class wants to give the same amount of money to each of 35 charities. How much will each charity receive?

Answer: $45

Explanation: The 6th-grade class collects $1575 and wants to give the same amount to 35 charities each, so each charity receives $1575÷35= $45.

Question 6.
Jean needs \(\frac{1}{3}\) cup of walnuts for each serving of salad she makes. She has 2 cups of walnuts. How many servings can she make?

Answer: 6.

Explanation: No.of servings made by 1/3 cup of walnuts is 1, so for 1 cup Jean serves 1/(1/3+1/3+1/3)= 3. So for 2 cups, no.of servings can she make are 3×2= 6.

Share and Show – Page No. 25

Question 1.
List the factors of 12 and 20. Circle the GCF.
Factors of 12 : __________
Factors of 20 : __________

Answer: 4

Explanation:
Factors of 12: 1,2,3,4,6,12
Factors of 20: 1,2,4,5,10,20
Common factors are 1,2,4
GCF is 4

Find the GCF.

Question 2.
16, 18

Answer: 2

Explanation:
Factors of 16: 1,2,4,8,16
Factors of 18: 1,2,3,6,9,18
Common factors are 1,2
GCF is 2

Question 3.
25, 40

Answer: 5

Explanation:
Factors of 25: 1,2,5,25
Factors of 40: 1,2,4,5,8,10,20,40
Common factors are 1,2,5
GCF is 5

Question 4.
24, 40

Answer: 8

Explanation:
Factors of 24: 1,2,3,4,6,8,12,24
Factors of 40: 1,2,4,5,8,10,20,40
Common factors are 1,2,4,8
GCF is 8

Question 5.
14, 35

Answer: 7

Explanation:
Factors of 14: 1,2,7,14
Factors of 35: 1,2,5,7,35
Common factors are 1,2,7
GCF is 7

Use the GCF and the Distributive Property to express the sum as a product.

Question 6.
21 + 28

Answer: 7×(3+4)

Explanation:
21+28= (7×3)+(7×4)
=7×(3+4)

Question 7.
15 + 27

Answer: 3×(5+9)

Explanation:
15+27= (3×5)+(3×9)
=3×(5+9)

Question 8.
40 + 15

Answer: 5×(8+3)

Explanation:
40+15= (5×8)+(5×3)
= 5×(8+3)

Question 9.
32 + 20

Answer: 4×(8+5)

Explanation:
32+20= (4×8)+(4×5)
= 4×(8+5)

On Your Own

Find the GCF.

Question 10.
8, 25

Answer: 1

Explanation:
Factors of 8: 1,2,4,8
Factors of 25: 1,5,25
Common factors are 1
GCF is 1

Question 11.
31, 32

Answer: 1

Explanation:
Factors of 31: 1,31
Factors of 32: 1,2,4,8,16,32
Common Factors are 1
GCF is 1

Question 12.
56, 64

Answer: 8

Explanation:
Factors of 56: 1,2,4,7,8,14,28,56
Factors of 64:  1,2,4,8,16,32,64
Common Factors are 1,2,4,8
GCF is 8

Question 13.
150, 275

Answer: 25

Explanation:
Factors of 150: 1,2,3,5,6,10,15,25,30,50,75,150
Factors of 275: 1,5,11,25,55,275
Common Factors are 1,5,25.
GCF is 25.

Use the GCF and the Distributive Property to express the sum as a product.

Question 14.
24 + 30

Answer: 6×(4+5)

Explanation:
24+30= (6×4)+(6×5)
=6×(4+5)

Question 15.
49 + 14

Answer: 7×(7+2)

Explanation:
49+14= (7×7)+(7×2)
=7×(7+2)

Question 16.
63 + 81

Answer: 9×(7+9)

Explanation:
63+81= (9×7)+(9×9)
=9×(7+9)

Question 17.
60 + 12

Answer: 12×(5+1)

Explanation:
60+12= (12×5)+(12×1)
=12×(5+1)

Question 18.
Describe the difference between the LCM and the GCF of two numbers.

Answer: In LCM we will get the Least Common Multiples of two numbers, and in GCF we will get Greatest Common Factor.

Problem Solving + Applications – Page No. 26

Use the table for 19-22. Teachers at the Scott School of Music teach only one instrument in each class. No students take classes for more than one instrument.
Go Math Grade 6 Answer Key Chapter 1 Divide Multi-Digit Numbers img 3

Question 19.
Francisco teaches group lessons to all of the violin and viola students at the Scott School of Music. All of his classes have the same number of students. What is the greatest number of students he can have in each class?

Answer: 6

Explanation: No. of students for Viola instrument is 30 and 36 for Violin,
Factors of 30: 1,2,3,5,6,10,15,30
Factors of 36: 1,2,3,4,6,9,12,18,36
GCF is 6
So the greatest number of students he can have in each class is 6

Question 20.
Amanda teaches all of the bass and viola students. All her classes have the same number of students. Each class has the greatest possible number of students. How many of these classes does she teach?
__________ bass classes
__________ viola classes

Answer: 2 bass classes and 3 viola classes.

Explanation:
Factors of 20: 1,2,4,5,10,20
Factors of 30: 1,2,3,5,6,10,15,30
GCF is 10
As the greatest number of possible students in each class is 10, So Amanda teaches 2 bass classes and 3 viola classes.

Question 21.
Mia teaches jazz classes. She has 9 students in each class, and she teaches all the classes for two of the instruments. Which two instruments does she teach, and how many students are in her classes?

Answer: 63 students.

Explanation:
Factors of 27: 1,3,9,27
Factors of 36: 1,2,3,4,6,9,12,18,36
GCF is 9
As 9 is the GCF of 27 and 36, So Mia teaches Cello and Violin classes for a total of 63 students.

Question 22.
Explain how you could use the GCF and the Distributive Property to express the sum of the number of bass students and the number of violin students as a product.

Answer: GCF is 4
Distributive property is 4×(5+9)

Explanation: The no.of bass students are 20 and no.of violin students are 36,
Factors of 20: 1,2,4,5,10,20
Factors of 36: 1,2,3,4,6,12,18,36
GCF is 4
And the Distributive property is 20+36
= (4×5)+(4×9)
= 4×(5+9)

Question 23.
Go Math Grade 6 Answer Key Chapter 1 Divide Multi-Digit Numbers img 4

Answer: 6

Explanation:
Factors of 6: 1,2,3,6
Factors of 12: 1,2,3,4,6
GCF is 6

Go Math Grade 6 Answer Key Chapter 1 Divide Multi-Digit Numbers

Greatest Common Factor – Page No. 27

List the common factors. Circle the greatest common factor. 

Question 1.
25 and 10

Answer: 5

Explanation:
Factors of 25: 1,5,25.
Factors of 10:  1,2,5,10
Common factors are 1,5
GCF is 5

Question 2.
36 and 90

Answer: 18

Explanation:
Factors of 36: 1,2,3,4,6,9,12,18,36
Factors of 90: 1,2,3,5,6,9,10,15,18,30,45,90
Common Factors are 1,2,3,6,9,18
GCF is 18

Question 3.
45 and 60

Answer: 15

Explanation:
Factors of 45: 1,3,5,9,15,45
Factors of 60: 1,2,3,4,5,6,10,12,15,20,30,60
Common Factors are 1,3,5,15
GCF is 15

Find the GCF.

Question 4.
14, 18

Answer: 2

Explanation:
Factors of 14: 1,2,7,14
Factors of 18: 1,2,3,6,9,18
Common Factors are 1,2
GCF is 2

Question 5.
6, 48

Answer: 6

Explanation:
Factors of 6: 1,2,3,6
Factors of 48: 1,2,3,4,6,8,12,24,48
Common Factors are 1,2,3,6
GCF is 6

Question 6.
16, 100

Answer: 4

Explanation:
Factors of 16: 1,2,4,8,16
Factors of 100: 1,2,4,5,10,20,25,50,100
Common Factors are 1,2,4
GCF is 4

Use the GCF and the Distributive Property to express the sum as a product.

Question 7.
20 + 35

Answer: 5×(4+7)

Explanation:
20+35= (5×4)+(5×7)
=5×(4+7)

Question 8.
18 + 27

Answer: 9×(2+3)

Explanation:
18+27= (9×2)+(9×3)
=9×(2+3)

Question 9.
64 + 40

Answer: 8×(8+5)

Explanation:
64+40= (8×8)+(8×5)
= 8×(8+5)

Problem Solving

Question 10.
Jerome is making prizes for a game at the school fair. He has two bags of different pins, one with 15 square pins and one with 20 round pins. Every prize will have one kind of pin. Each prize will have the same number of pins. What is the greatest number of pins Jerome can put in each prize?

Answer: 5

Explanation:
Factors of 15: 1,3,5,15
Factors of 20: 1,2,4,5,10,20
Common factors are 1,5
So the greatest number of pins Jerome can put in each prize is 5

Question 11.
There are 24 sixth graders and 40 seventh graders. Mr. Chan wants to divide both grades into groups of equal size, with the greatest possible number of students in each group. How many students should be in each group?

Answer: 8.

Explanation:
Factors of 24: 1,2,3,4,6,8,12,24
Factors of 40: 1,2,4,5,8,10,20,40
Common Factors are 1,2,4,8
So the greatest possible number of students are 8
Question 12.
Write a short paragraph to explain how to use prime factorization and the Distributive Property to express the sum of two whole numbers as a product.

Answer:
Prime Factorization is the product of prime numbers

Lesson Check – Page No. 28

Question 1.
There are 15 boys and 10 girls in Miss Li’s class. She wants to group all the students so that each group has the same number of boys and the same number of girls. What is the greatest number of groups she can have?

Answer: 5

Explanation:
Factors of 15: 1,3,5,15
Factors of 10: 1,2,5,10
Common Factors are 1,5
The greatest number of groups she can have is 5.

Question 2.
A pet shop manager wants the same number of birds in each cage. He wants to use as few cages as possible, but can only have one type of bird in each cage. If he has 42 parakeets and 18 canaries, how many birds will he put in each cage?

Answer: 6

Explanation:
Factors of 42: 1,2,3,6,7,14,21,42
Factors of 18: 1,2,3,6,9,18
Common Factors are 1,2,3,6
GCF is 6
So he will put 6 birds in each cage.

Spiral Review

Question 3.
There are 147 people attending a dinner party. If each table can seat 7 people, how many tables are needed for the dinner party?

Answer: 21 tables.

Explanation: Total no.of people attending a dinner party are 147 and 7 people can seat in each table, so 147÷7= 21 tables are needed for a dinner party.

Question 4.
Sammy has 3 pancakes. He cuts each one in half. How many pancake halves are there?

Answer: 6

Explanation: Sammy has 3 pancakes, as he cut each one into half so there are 3×2= 6 pancake halves.

Question 5.
The Cramer Company had a profit of $8,046,890 and the Coyle Company had a profit of $8,700,340 last year. Which company had the greater profit?

Answer: Coyle company

Explanation: Coyle company had a profit of $8,700,340 and Cramer Company had $8,046,890, So $8,700,340-$8,046,890= $653,450 Coyle company have greater profits.

Question 6.
There are 111 guests attending a party. There are 15 servers. Each server has the same number of guests to serve. Jess will serve any extra guests. How many guests will Jess be serving?

Answer: 6.

Explanation:
Total guests attending a party are 111 and no.of servers are 15, as each server has the same number of guests to serve so we will divide total guests by the number of servers 111÷15= 7.4 round off to 6. Therefore, no.of guests, will Jess be serving is 6.

Share and Show – Page No. 31

Question 1.
Toby is packaging 21 baseball cards and 12 football cards to sell at a swap meet. Each packet will have the same number of cards. Each packet will have cards for only one sport. What is the greatest number of cards he can place in each packet? How many packets will there be for each sport?

Answer: 7 packets of baseball cards and 4 packets of football cards and each packet contains 3 cards.

Explanation: The GCF of 21 and 12 are
Factors of 21: 1,3,7,21
Factors of 12: 1,2,3,4,6,12
GCF is 3
By Distributive property 21+12
= (3×7)+(3×4)
= 3×(7+4)
So there will be 7 packets of baseball cards and 4 packets of football cards and each packet contains 3 cards.

Question 2.
What if Toby had decided to keep one baseball card for himself and sell the rest? How would your answers to the previous problem have changed?

Answer: 5 packets of baseball cards and 3 football and each packet contains 4 cards.

Explanation: If Toby had decided to keep one baseball card for himself, so he will have 20 baseball cards and 12 football cards
Factors of 20: 1,2,4,5,10,20
Factors of 12: 1,2,3,4,6,12
GCF is 4
By Distributive property 20+12
= (4×5)+(4×3)
=4×(5+3)
So there will be 5 packets of baseball cards and 3 football and each packet contains 4 cards.

Question 3.
Melissa bought 42 pine seedlings and 30 juniper seedlings to plant in rows on her tree farm. She wants each row to have the same number of seedlings. She wants only one type of seedling in each row. What is the greatest number of seedlings she can plant in each row? How many rows of each type of tree will there be?

Answer: 7 rows of pine seedlings and 5 rows of juniper seedling with 6 seedlings in each row.

Explanation:
Factors of 42: 1,2,3,6,7,14,21,42
Factors of 30: 1,2,3,6,10,15,30
GCF is 6
By Distributive 42+30
=(6×7)+(6×5)
=6×(7+5)
So there will be 7 rows of pine seedlings and 5 rows of juniper seedling with 6 seedlings in each row.

On Your Own – Page No. 32

Question 4.
Make Sense of Problems A drum and bugle marching band has 45 members who play bugles and 27 members who play drums. When they march, each row has the same number of players. Each row has only bugle players or only drummers. What is the greatest number of players there can be in each row? How many rows of each type of player can there be?

Answer: 9 people in each row, And there will be 5 rows of bugle players and 3 rows of drummers.

Explanation:
Factors of 45: 1,3,5,9,15,45
Factors of 27: 1,3,9,27
GCF is 9
So there will be 9 people in each row and by the distributive law 45+27
= (9×5)+(9×3)
= 9×(5+3)
And there will be 5 rows of bugle players and 3 rows of drummers.

Question 5.
The “color guard” of a drum and bugle band consists of members who march with flags, hoops, and other props. How would your answers to Exercise 4 change if there were 21 color guard members marching along with the bugle players and drummers?

Answer: 15 rows of bugle players, 9 rows of drummers, and 7 rows color guard members with 3 marchers in each row.

Explanation:
Factors of 21: 1,3,7,21
Factors of 45: 1,3,5,9,15,45
Factors of 27: 1,3,9,27
GCF is 3
So there would be 15 rows of bugle players, 9 rows of drummers, and 7 rows color guard members with 3 marchers in each row.

Question 6.
If you continue the pattern below so that you write all of the numbers in the pattern less than 500, how many even numbers will you write?
4, 9, 14, 19, 24, 29…

Answer: 50

Explanation: You can write 50 numbers.

Question 7.
Mr. Yaw’s bookcase holds 20 nonfiction books and 15 fiction books. Each shelf holds the same number of books and contains only one type of book. How many books will be on each shelf if each shelf has the greatest possible number of books? Show your work.

Answer: 5

Explanation:
Factors of 15: 1,3,5,15
Factors of 20: 1,2,4,5,10,20.
GCF is 5
5 books will be on each self.

Problem Solving Apply the Greatest Common Factor – Page No. 33

Read the problem and solve.

Question 1.
Ashley is bagging 32 pumpkin muffins and 28 banana muffins for some friends. Each bag will hold only one type of muffin. Each bag will hold the same number of muffins. What is the greatest number of muffins she can put in each bag? How many bags of each type of muffin will there be?

Answer: 8 pumpkin muffins and 7 banana muffins with 4 greatest number of muffins in each bag.

Explanation:
Factors of 32: 1,2,4,8,16,32
Factors of 28: 1,2,4,7,14,28
GCF is 4
By distributive property 32+28
= (4×8)+(4×7)
=4×(8+7)
So there will be 8 pumpkin muffins and 7 banana muffins with 4 greatest number of muffins in each bag.

Question 2.
Patricia is separating 16 soccer cards and 22 baseball cards into groups. Each group will have the same number of cards, and each group will have only one kind of sports card. What is the greatest number of cards she can put in each group? How many groups of each type will there be?

Answer: Patricia has 8 soccer cards and 11 baseball cards and 2 groups each.

Explanation:
Factors of 16: 1,2,4,8,16
Factors of 22: 1,2,11,22
GCF is 2
By distributive property 16+22
= (2×8)+(2×11)
=2×(8+11)
Patricia has 8 soccer cards and 11 baseball cards and 2 groups each.

Question 3.
Bryan is setting chairs in rows for a graduation ceremony. He has 50 black chairs and 60 white chairs. Each row will have the same number of chairs, and each row will have the same color chair. What is the greatest number of chairs that he can fit in each row? How many rows of each color chair will there be?

Answer: 10 chairs per row and 5 black chairs and 6 white chairs.

Explanation:
By distributive law 50+60
= (10×5)+(10×60)
= 10×(5+6)
So there will 10 chairs per row and 5 black chairs and 6 white chairs.

Question 4.
A store clerk is bagging spices. He has 18 teaspoons of cinnamon and 30 teaspoons of nutmeg. Each bag needs to contain the same number of teaspoons, and each bag can contain only one spice. What is the maximum number of teaspoons of spice the clerk can put in each bag? How many bags of each spice will there be?

Answer: 6 no. of teaspoons of spices and 3 teaspoons of cinnamon 5 teaspoons of nutmeg.

Explanation:
By distributive property (18+30)
= (6×3)+(6×5)
= 6×(3+5)
So there will be 6 no. of teaspoons of spices and 3 teaspoons of cinnamon 5 teaspoons of nutmeg.

Question 5.
Write a problem in which you need to put as many of two different types of objects as possible into equal groups. Then use the GCF, Distributive Property, and a diagram to solve your problem

Answer: Jack has a bag full of 20 red apples and 32 green apples. Each bag needs to contain same number of apples and each bag can contain only one type of apple. What is the maximum number of apples can Jack put in each bag? How many bags of each apple will be there?

Explanation: By distributive property (20+32)
= (4×5)+(4×8)
= 4×(5+8)
So there will be 4 bags and in that 5 red apples and 8 green apples.

Lesson Check – Page No. 34

Question 1.
Fred has 36 strawberries and 42 blueberries. He wants to use them to garnish desserts so that each dessert has the same number of berries, but only one type of berry. He wants as much fruit as possible on each dessert. How many berries will he put on each dessert? How many desserts with each type of fruit will he have?

Answer: 6 berries on each dessert and 6 strawberries and 7 blueberries in each type of fruit.

Explanation:
By distributive property 36+42
= (6×6)+(6×7)
= 6×(6+7)
So he put 6 berries on each dessert and 6 strawberries and 7 blueberries in each type of fruit.

Question 2.
Dolores is arranging coffee mugs on shelves in her shop. She wants each shelf to have the same number of mugs. She only wants one color of mug on each shelf. If she has 49 blue mugs and 56 red mugs, what is the greatest number she can put on each shelf? How many shelves does she need for each color?
__________ shelves for blue mugs
__________ shelves for red mugs

Answer: 7 blue mugs and 8 red mugs.

Explanation:
By distributive property 49+56
= (7×7)+(7×8)
= 7×(7+8)
So the greatest number she can put on each shelf is 7, 7 blue mugs and 8 red mugs.

Spiral Review

Question 3.
A rectangle is 3 \(\frac{1}{3}\) feet long and 2 \(\frac{1}{3}\) feet wide. What is the distance around the rectangle?
_____ \(\frac{□}{□}\)

Answer: 11 1/3 feet

Explanation: Distance of a rectangle= 2(L+W)
= 2(3 1/3+ 2 1/3)
= 2(10/3+7/3)
= 2(17/3)
= 34/3
= 11 1/3 feet.

Question 4.
Lowell bought 4 \(\frac{1}{4}\) pounds of apples and 3 \(\frac{3}{5}\) pounds of oranges. How many pounds of fruit did Lowell buy?
_____ \(\frac{□}{□}\)

Answer: 7 17/20 pounds

Explanation: Lowell bought 4 1/4 pounds of apples and 3 3/5 pounds of oranges, so total pounds of fruits Lowell bought is 4 1/4+ 3 3/5=
= 17/4+ 18/5
= 157/20
= 7 17/20 pounds

Question 5.
How much heavier is a 9 \(\frac{1}{8}\) pound box than a 2 \(\frac{5}{6}\) pound box?
_____ \(\frac{□}{□}\)

Answer: 6 7/4 much heavier.

Explanation: 9 1/8 – 2 5/6
= 73/8 – 17/6
= 151/24
= 6 7/4

Question 6.
The combination of Clay’s locker is the prime factors of 102 in order from least to greatest. What is the combination of Clay’s locker?

Answer: 2317.

Explanation:
Prime Factors of 102 are 2,3,17, so the combination of Clay’s locker is 2317

Vocabulary – Page No. 35

Choose the best term from the box to complete the sentence.
Go Math Grade 6 Answer Key Chapter 1 Divide Multi-Digit Numbers img 5

Question 1.
The _____ of two numbers is greater than or equal to the numbers.

Answer: LCM

Question 2.
The _____ of two numbers is less than or equal to the numbers.

Answer: Greatest Common

Concepts and Skills

Estimate. Then find the quotient. Write the remainder, if any, with an r.

Question 3.
2,800 ÷ 25

Answer: Quotient is 112 and remainder is 0

Explanation:
Go Math Grade 6 Answer Key Chapter 1 Divide Multi-Digit Numbers

Question 4.
19,129 ÷ 37

Answer: Quotient is 517 and remainder is 0

Explanation:
Go Math Grade 6 Answer Key Chapter 1 Divide Multi-Digit Numbers

Question 5.
32,111 ÷ 181

Answer: Quotient is 177 and the remainder is 74

Explanation:
Go Math Grade 6 Answer Key Chapter 1 Divide Multi-Digit Numbers

Find the prime factorization.

Question 6.
44

Answer: 2×2×11

Explanation:
44= 4×11
2×2×11

Question 7.
36

Answer: 2×2×3×3

Explanation:
36= 2×18
=2×2×9
=2×2×3×3

Question 8.
90

Answer: 3×3×5×2

Explanation:
90= 9×10
=3×3×10
=3×3×5×2

Find the LCM.

Question 9.
8, 10

Answer: 40

Explanation:
Multiples of 8: 8,16,24,32,40
Multiples of 10: 10,20,30,40
LCM is 40

Question 10.
4, 14

Answer: 28

Explanation:
Multiples of 4:  4,8,12,16,20,24,28
Multiples of 14: 14,28
LCM is 28

Question 11.
6, 9

Answer: 18

Explanation:
Multiples of 6: 6,12,18
Multiples of 9: 9,18
LCM is 18

Find the GCF.

Question 12.
16, 20

Answer: 4

Explanation:
Factors of 16: 1,2,4,8,16
Factors of 20: 1,2,4,5,10,20
Common Factors are 1,2,4
GCF is 4

Question 13.
8, 52

Answer: 4

Explanation:
Factors of 8: 1,2,4,8
Factors of 52: 1,2,4,13,26,52
Common Factors are 1,2,4
GCF is 4

Question 14.
36, 54

Answer: 18

Explanation:
Factors of 36: 1,2,3,4,6,9,12,18,36
Factors of 54:  1,2,3,6,9,18,27,54
Common Factors are 1,2,3,6,9,18
GCF is 18

Page No. 36

Question 15.
A zookeeper divided 2,440 pounds of food equally among 8 elephants. How many pounds of food did each elephant receive?

Answer: 305 Pounds.

Explanation: Zookeeper divides 2,440 pounds of food equally among 8 elephants, so no. of pounds is
2,440÷8= 305 pounds.

Question 16.
DVD cases are sold in packages of 20. Padded mailing envelopes are sold in packets of 12. What is the least number of cases and envelopes you could buy so that there is one case for each envelope with none left over?

Answer: 60

Explanation:
Multiples of 20: 20,40,60
Multiples of 12: 12,24,36,48,60
LCM is 60
So the Least number of cases and envelopes without any leftover is 60.

Question 17.
Max bought two deli sandwich rolls measuring 18 inches and 30 inches. He wants them to be cut into equal sections that are as long as possible. Into what lengths should the rolls be cut? How many sections will there be in all?

Answer: 6 inches and 8 sections.

Explanation:
By distributive property 18+30
= (6×3)+(6×5)
= 6(3+5)
So Length of the rolls should cut at 6 inches and sections are (3+5)= 8 sections.

Question 18.
Susan is buying supplies for a party. If spoons only come in bags of 8 and forks only come in bags of 6, what is the least number of spoons and the least number of forks she can buy so that she has the same number of each?

Answer: So least no. of forks and spoons are 24.

Explanation:
Multiples of 8: 8,16,24
Multiples of 6: 6,2,18,24
LCM is 24
So least no. of forks and spoons are 24.

Question 19.
Tina is placing 30 roses and 42 tulips in vases for table decorations in her restaurant. Each vase will hold the same number of flowers. Each vase will have only one type of flower. What is the greatest number of flowers she can place in each vase? If Tina has 24 tables in her restaurant, how many flowers can she place in each vase?

Answer: Maximum flowers in a vase is 3.

Explanation: Tina is placing 30 roses and 42 tulips, so total flowers are 30+42= 72 flowers. The total number of tables are 24, as each vase hold same no. of flowers, Let the no. of flowers in each vase be X, so total no.of flowers to be decorate 24X,
24X = 72
X= 3.
So maximum flowers in a vase is 3.

Share and Show – Page No. 39

Question 1.
Find 3.42 − 1.9.

Answer: 1.52

Explanation: 3.42 − 1.9= 1.52.

Estimate. Then find the sum or difference.

Question 2.
2.3 + 5.68 + 21.047

Answer: 29.027

Explanation: 2.3 + 5.68 + 21.047= 29.027

Question 3.
33.25 − 21.463

Answer: 11.787

Explanation: 33.25 − 21.463= 11.787

Question 4.
Evaluate (8.54 + 3.46) − 6.749.

Answer: 5.251

Explanation:
(8.54 + 3.46) − 6.749= (12)-6.749
= 5.251

On Your Own

Estimate. Then find the sum or difference.

Question 5.
57.08 + 34.71

Answer: 91.79

Explanation:
57.08 + 34.71= 91.79

Question 6.
20.11 − 13.27

Answer: 33.38

Explanation:
20.11−13.27= 33.38

Question 7.
62 − 9.817

Answer: 52.183

Explanation:
62 − 9.817= 52.183

Question 8.
35.1 + 4.89

Answer: 39.99

Explanation:
35.1 + 4.89= 39.99

Practice: Copy and Solve Evaluate using the order of operations.

Question 9.
8.01 − (2.2 + 4.67)

Answer: 1.14

Explanation:
8.01 − (2.2 + 4.67)
= 8.01-(6.87)
= 1.14

Question 10.
54 + (9.2 − 1.413)

Answer: 61.787

Explanation: 54 + (9.2 − 1.413)
= 54+(7.787)
=61.787

Question 11.
21.3 − (19.1 − 3.22)

Answer: 5.42

Explanation: 21.3 − (19.1 − 3.22)
= 21.3-(15.88)
=5.42

Question 12.
Make Arguments A student evaluated 19.1 + (4.32 + 6.9) and got 69.2. How can you use estimation to convince the student that this answer is not reasonable?

Answer: The answer is not reasonable, because 19.1+4.32+6.9= 30.32

Explanation: 19.1 + (4.32 + 6.9)
= 19.1+(11.22)
= 30.32

Question 13.
Lynn paid $4.75 for cereal, $8.96 for chicken, and $3.25 for soup. Show how she can use properties and compatible numbers to evaluate (4.75 + 8.96) + 3.25 to find the total cost.

Answer: 16.96

Explanation: Total cost is (4.75 + 8.96) + 3.25
= (13.71)+3.25
= 16.96

Page No. 40

Question 14.
For numbers 14a–14d, select True or False for each equation.
14a. 3.76 + 2.7 = 6.46 True False
14b. 4.14 + 1.8 = 4.32 True False
14c. 2.01 – 1.33 = 0.68 True False
14d. 51 – 49.2 = 1.8 True False

14a. 3.76 + 2.7 = 6.46

Answer: True

Explanation: 3.76 + 2.7 = 6.46

14b. 4.14 + 1.8 = 4.32

Answer: False

Explanation: 4.14 + 1.8 = 5.94

14c. 2.01 – 1.33 = 0.68

Answer: True

Explanation: 2.01 – 1.33 = 0.68

14d. 51 – 49.2 = 1.8

Answer: True

Explanation: 51 – 49.2 = 1.8

Comparing Eggs

Different types of birds lay eggs of different sizes. Small birds lay eggs that are smaller than those that are laid by larger birds. The table shows the average lengths and widths of five different birds’ eggs.
Go Math Grade 6 Answer Key Chapter 1 Divide Multi-Digit Numbers img 6

Use the table for 15–17.

Question 15.
What is the difference in average length between the longest egg and the shortest egg?

Answer: 0.073

Explanation: The length of the longest egg is 0.086 and the shortest egg is 0.013, so the difference is
0.086-0.013= 0.073

Question 16.
Which egg has a width that is eight thousandths of a meter shorter than its length?

Answer: Turtledove

Explanation: The length of the turtledove egg is 0.031 and the width is 0.023, so 0.031-0.023= 0.08m shorter than length.

Question 17.
How many robin eggs, laid end to end, would be about equal in length to two raven eggs? Justify your answer

Answer: 5 robin eggs should be laid.

Explanation: The length of Two raven eggs is 0.049+0.049=0.098, so 5 robin eggs should be laid.

Add and Subtract Decimals – Page No. 41

Estimate. Then find the sum or difference.

Question 1.
43.53 + 27.67

Answer: 71.2

Explanation: 43.53 + 27.67=71.2

Question 2.
17 + 3.6 + 4.049

Answer: 24.649

Explanation: 17 + 3.6 + 4.049
=17+7.649
=24.649

Question 3.
3.49 − 2.75

Answer: 0.74

Explanation:
3.49-2.75= 0.74

Question 4.
5.07 − 2.148

Answer: 2.922

Explanation:
5.07-2.148= 2.922

Question 5.
3.92 + 16 + 0.085

Answer: 20.005

Explanation: 3.92 + 16 + 0.085
= 3.92+16.085
= 20.005

Question 6.
41.98 + 13.5 + 27.338

Answer: 82.818

Explanation: 41.98 + 13.5 + 27.338
= 41.98+ 40.838

Evaluate using the order of operations.

Question 7.
8.4 + (13.1 − 0.6)

Answer: 20.9

Explanation: 8.4 + (13.1 − 0.6)
= 8.4+(12.5)
= 20.9

Question 8.
34.7 − (12.07 + 4.9)

Answer: 17.73

Explanation: 34.7 − (12.07 + 4.9)
= 34.7-(16.97)
= 17.73

Question 9.
(32.45 − 4.8) − 2.06

Answer: 25.59

Explanation: (32.45 − 4.8) − 2.06
= 27.65- 2.06
= 25.59

Problem Solving

Question 10.
The average annual rainfall in Clearview is 38 inches. This year, 29.777 inches fell. How much less rain fell this year than falls in an average year?

Answer: 8.23

Explanation: Average annual rainfall in last year is 38 inches and this year is 29.777 inches, so 38-29.77= 8.23 inches less rainfall

Question 11.
At the theater, the Worth family spent $18.00 on adult tickets, $16.50 on children’s tickets, and $11.75 on refreshments. How much did they spend in all?

Answer: $46.25

Explanation: As the family spent $18.00 on adult tickets, $16.50 on children’s tickets, and $11.75 on refreshments,
So the total spent by the family is $18.00+$16.50+$11.75= $46.25

Question 12.
Write a word problem that involves adding or subtracting decimals. Include the solution.

Answer: Mark and Jack went to the park and cost of ticket is $6.50. Mark has $20. How much remaining did mark has left?

Explanation: As MArk and jack went to the park where ticket price is $6.50, so for both it will be
$6.50+$6.50= $13.00. As Mark has $20, remaining amount left with Mark is $20-$13= $7

Lesson Check – Page No. 42

Question 1.
Alden fills his backpack with 0.45 kg of apples, 0.18 kg of cheese, and a water bottle that weighs 1.4 kg. How heavy are the contents of his backpack?

Answer: 2.03kg

Explanation: The total weight of a backpack is 0.45+0.18+1.4= 2.03kg

Question 2.
Gabby plans to hike 6.3 kilometers to see a waterfall. She stops to rest after hiking 4.75 kilometers. How far does she have left to hike?

Answer: 1.55kms

Explanation: Gobby hikes 6.3kms and stops at 4.75kms, so she left at 6.3-4.75= 1.55kms
Spiral Review

Question 3.
A 6-car monorail train can carry 78 people. If one train makes 99 trips during the day, what is the greatest number of people the train can carry in one day?

Answer: 7,722.

Explanation: The greatest number of people the train can carry in one day is 78×99= 7,722.

Question 4.
An airport parking lot has 2,800 spaces. If each row has 25 spaces, how many rows are there?

Answer: 112 rows

Explanation: As parking lot has 2,800 spaces and each row has 25 spaces, no. of rows is 2800÷25= 112 rows

Question 5.
Evan brought 6 batteries that cost $10 each and 6 batteries that cost $4 each. The total cost was the same as he would have spent buying 6 batteries that cost $14 each. So, 6 × $14 = (6 × 10) + (6 × 4). What property does the equation illustrate?

Answer: Distributive property

Explanation: By distributive property (a×b)+(a×c)= a×(b+c), here a= 6, b=10, c=4.

Question 6.
Cups come in packages of 12 and lids come in packages of 15. What is the least number of cups and lids that Corrine can buy if she wants to have the same number of cups and lids?

Answer: 60 cups and 60 lids.

Explanation:
Multiples of 12: 12,24,36,48,60
Multiples of 15: 15,30,45,60
LCM is 60
So the least number of cups and lids she can buy is 60 cups and 60 lids.

Share and Show – Page No. 45

Estimate. Then find the product.

Question 1.
12.42 × 28.6

Answer: 355.212

Explanation: 12.42 × 28.6
= 355.212

Question 2.
32.5 × 7.4

Answer: 240.5

Explanation: 32.5 × 7.4
=240.5

Attend to Precision Algebra Evaluate using the order of operations.

Question 3.
0.24 × (7.3 + 2.1)

Answer: 2.256

Explanation: 0.24 × (7.3 + 2.1)
= 0.24×9.4
= 2.256

Question 4.
0.075 × (9.2 − 0.8)

Answer: 0.63

Explanation: 0.075 × (9.2 − 0.8)
= 0.075×(8.4)
= 0.63

Question 5.
2.83 + (0.3 × 2.16)

Answer: 3.478

Explanation: 2.83 + (0.3 × 2.16)
= 2.83+0.648
= 3.478

On Your Own

Estimate. Then find the product.

Question 6.
29.14 × 5.2

Answer: 151.528

Explanation: 29.14 × 5.2

= 151.528

Question 7.
6.95 × 12

Answer: 83.4

Explanation: 6.95 × 12
= 83.4

Question 8.
0.055 × 1.82

Answer: 0.1001

Explanation: 0.055 × 1.82
= 0.1001

Attend to Precision Algebra Evaluate using the order of operations.

Question 9.
(3.62 × 2.1) − 0.749

Answer: 6.853

Explanation: (3.62 × 2.1) − 0.749
= 7.602- 0.749
= 6.853

Question 10.
5.8 − (0.25 × 1.5)

Answer: 5.425

Explanation: 5.8 − (0.25 × 1.5)
= 5.8- (0.375)
= 5.425

Question 11.
(0.83 + 1.27) × 6.4

Answer: 13.44

Explanation: (0.83 + 1.27) × 6.4
= (2.1)×6.4
= 13.44

Question 12.
Jamal is buying ingredients to make a large batch of granola to sell at a school fair. He buys 3.2 pounds of walnuts for $4.40 per pound and 2.4 pounds of cashews for $6.25 per pound. How much change will he receive if he pays with two $20 bills?

Answer: $40-$29.08= $10.92.

Explanation: Jamal bought 3.2 pounds of walnuts for $4.40 per pound, so for 3.2 pounds it will be 3.2×4.40= 14.08,
and 2.4 pounds of cashew for $6.25 per pound, so for 2.4 pounds it will be 2.4×6.25= 15. Total Jamal spend is 14.08+15= 29.08. As he have two $20 so he will receive $40-$29.08= $10.92.

Unlock the Problem – Page No. 46

The table shows some currency exchange rates for 2009.
Go Math Grade 6 Answer Key Chapter 1 Divide Multi-Digit Numbers img 7

Question 13.
When Cameron went to Canada in 2007, he exchanged 40 U.S. dollars for 46.52 Canadian dollars. If Cameron exchanged 40 U.S. dollars in 2009, did he receive more or less than he received in 2007? How much more or less?
a. What do you need to find?

Answer: We need how much or less 40 US dollars are worth in Canadian dollars in 2009 compared to 2007.

Question 13.
b. How will you use the table to solve the problem?

Answer: The table provides exchange rates for 2009, will multiply to find the value of 40 US dollars in Canadian dollars in 2009.

Question 13.
c. Complete the sentences.
40 U.S. dollars were worth _____ Canadian dollars in 2009.
So, Cameron would receive _____ Canadian dollars in 2009.

Answer: 42.08 Canadian dollars in 2009
4.44 Canadian dollars in 2009

Explanation: In 2009 1 US dollar is 1.052, so 40 US dollars is 40×1.052= 42.08 and in 2007 Cameron received 46.52, so in 2009 Cameron would receive 46.52-42.08= 4.44 Canadian dollars in 2009.

Question 14.
At a convenience store, the Jensen family puts 12.4 gallons of gasoline in their van at a cost of $3.80 per gallon. They also buy 4 water bottles for $1.99 each, and 2 snacks for $1.55 each. Complete the table to find the cost for each item.
Go Math Grade 6 Answer Key Chapter 1 Divide Multi-Digit Numbers img 8
Mrs. Jensen says the total cost for everything before tax is $56.66. Do you agree with her? Explain why or why not.

Answer: No, the answer is not reasonable.

Explanation: As the total cost is 58.18
12.4×3.80= 47.12
4×1.99= 7.96
2×1.55= 3.1
So 47.12+7.96+3.1= $58.18

Multiply Decimals – Page No. 47

Estimate. Then find the product.

Question 1.
5.69 × 7.8

Answer: 44.382

Explanation: 5.69 × 7.8
= 44.382

Question 2.
3.92 × 0.051

Answer: 0.19992

Explanation: 3.92 × 0.051
= 0.19992

Question 3.
2.365 × 12.4

Answer: 29.326

Explanation: 2.365 × 12.4
= 29.326

Question 4.
305.08 × 1.5

Answer: 457.62

Explanation: 305.08 × 1.5
= 457.62

Evaluate the expression using the order of operations.

Question 5.
(61.8 × 1.7) + 9.5

Answer: 114.56

Explanation: (61.8 × 1.7) + 9.5
= 105.06+ 9.5
= 114.56

Question 6.
205 − (35.80 × 5.6)

Answer: 4.52

Explanation: 205 − (35.80 × 5.6)
= 205- 200.48
= 4.52

Question 7.
1.9 × (10.6 − 2.17)

Answer: 16.017

Explanation: 1.9 × (10.6 − 2.17)
= 1.9×( 8.43)
= 16.017

Problem Solving

Question 8.
Blaine exchanges $100 for yen before going to Japan. If each U.S. dollar is worth 88.353 yen, how many yen should Blaine receive?

Answer: 8835.3 yen

Explanation: As 1 US dollar is 88.353 yen, so when Blaine exchanges $100 to yen it will be $100×88.353=8835.3 yen

Question 9.
A camera costs 115 Canadian dollars. If each Canadian dollar is worth 0.952 U.S. dollars, how much will the camera cost in U.S. dollars?

Answer: 109.48.

Explanation: As 1 Canadian dollar is 0.952 US dollars, so camers cost is 115×0.952= 109.48.

Question 10.
Explain how to mentally multiply a decimal number by 100.

Answer: Move the decimal point two places right.

Lesson Check – Page No. 48

Question 1.
A gallon of water at room temperature weighs about 8.35 pounds. Lena puts 4.5 gallons in a bucket. How much does the water weigh?

Answer: 37.575

Explanation: As 1 gallon= 8.35 pounds, Lena put 4.5 gallons in a bucket. So water weight is 4.5×8.35= 37.575

Question 2.
Shawn’s rectangular mobile home is 7.2 meters wide and 19.5 meters long. What is its area?

Answer: 140.4

Explanation: Area= Length×wide, so 7.2×19.5= 140.4

Spiral Review

Question 3.
Last week, a store sold laptops worth a total of $3,885. Each laptop cost $555. How many laptops did the store sell last week?

Answer: 7 Laptops.

Explanation: Total Laptops sold is $3885 and each laptop cost is $555, so 3885÷555= 7 laptops were sold by the store.

Question 4.
Kyle drives his truck 429 miles on 33 gallons of gas. How many miles can Kyle drive on 1 gallon of gas?

Answer: 13 miles.

Explanation: As Kyle drives 429 miles on 33 gallons gas, so 429÷33= 13 miles he can drive on 1 gallon of gas.

Question 5.
Seven busloads each carrying 35 students arrived at the game, joining 23 students who were already there. Evaluate the expression 23 + (7 × 35) to find the total number of students at the game.

Answer: 268 students.

Explanation: 23+(7×35)
=23+(245)
=268.
Total students are 268.

Question 6.
A store is giving away a $10 coupon to every 7th person to enter the store and a $25 coupon to every 18th person to enter the store. Which person will be the first to get both coupons?

Answer: 126th person will get both coupons.

Explanation: LCM of 7 and 18 is 18×7= 126. So 126th person will get both coupons.

Estimate. Then find the quotient – Page No. 51

Question 2.
7)\(\overline { $17.15 } \)

Answer: 2.45

Explanation: $17.15÷7= 2.45

Question 3.
4)\(\overline { 1.068 } \)

Answer: 0.267

Explanation: 1.068÷4= 0.267

Question 4.
12)\(\overline { 60.84 } \)

Answer: 5.07

Explanation: 60.84÷12= 5.07

Question 5.
18.042 ÷ 6

Answer: 3.007

Explanation: 18.042÷6= 3.007

On Your Own

Estimate. Then find the quotient.

Question 6.
$21.24 ÷ 6

Answer: 3.54

Explanation: $21.24 ÷ 6= 3.54

Question 7.
28.63 ÷ 7

Answer: 4.09

Explanation: 28.63 ÷ 7= 4.09

Question 8.
1.505 ÷ 35

Answer: 0.043

Explanation: 1.505 ÷ 35= 0.043

Question 9.
0.108 ÷ 18

Answer: 0.006

Explanation: 0.108 ÷ 18= 0.006

Attend to Precision Algebra Evaluate using the order of operations.

Question 10.
(3.11 + 4.0) ÷ 9

Answer: 0.79

Explanation: (3.11 + 4.0) ÷ 9
= (7.11)+9
= 0.79

Question 11.
(6.18 − 1.32) ÷ 3

Answer: 1.62

Explanation: (6.18 − 1.32) ÷ 3
= (4.86)÷3
= 1.62

Question 12.
(18 − 5.76) ÷ 6

Answer: 2.04

Explanation: (18 − 5.76) ÷ 6
= (12.24)÷6
= 2.04

Question 13.
Use Appropriate Tools Find the length of a dollar bill to the nearest tenth of a centimeter. Then show how to use division to find the length of the bill when it is folded in half along the portrait of George Washington

Answer: 3.07 inches or 7.8 centimeter.

Explanation: As the length of a dollar bill to the nearest tenth of a centimeter is 15.6 cm, and length of the bill when it is folded in half along the portrait of George Washington is 3.07 inches or 7.8 centimeter.

Question 14.
Emilio bought 5.65 pounds of green grapes and 3.07 pounds of red grapes. He divided the grapes equally into 16 bags. If each bag of grapes has the same weight, how much does each bag weigh?

Answer: 0.545 pounds.

Explanation: Total weight of grapes is 5.65+3.07= 8.72 pounds, so each bag weight is 8.72÷16= 0.545 pounds.

Problem Solving + Applications – Page No. 52

Pose a Problem

Question 15.
This table shows the average height in inches for girls and boys at ages 8, 10, 12, and 14 years. To find the average growth per year for girls from age 8 to age 12, Emma knew she had to find the amount of growth between age 8 and age 12, then divide that number by the number of years between age 8 and age 12.
Go Math Grade 6 Answer Key Chapter 1 Divide Multi-Digit Numbers img 9
Emma used this expression: (60.50−50.75)÷4
She evaluated the expression using the order of operations.
Write the expression. (60.50−50.75)÷4
Perform operations in parentheses. 9.75÷4
Divide. 2.4375
So, the average annual growth for girls ages 8 to 12 is 2.4375 inches. Write a new problem using the information in the table for the average height for boys. Use division in your problem.

Answer: Find the average growth per year for girls 8 to 14.

Explanation: As (62.50-50.75)÷6
= (11.75)÷6
= 1.96
So the average annual growth for girls age 8 to age 14 is 1.96 inches.

Question 16.
The table shows the number of books each of three friends bought and the cost. On average, which friend spent the most per book? Use numbers and words to explain your answer
Go Math Grade 6 Answer Key Chapter 1 Divide Multi-Digit Numbers img 10

Answer: Nabil spent the most per book.

Explanation:
Joyce purchased 1 book which costs $10.95
Nabil purchased 2 books which costs $40.50, so 1 book cost is 40.50÷2= $20.26
Kenneth purchased 3 books for $51.15 , so 1 book cost is 51.15÷3= $17.05
So, Nabil spent the most per book.

Divide Decimals by Whole Numbers – Page No. 53

Estimate. Then find the quotient.

Question 1.
1.284 ÷ 12

Answer: 0.107

Explanation: 1.284÷12= 0.107

Question 2.
9)\(\overline { 2.43 } \)

Answer: 0.27

Explanation: 2.43÷9 = 0.27

Question 3.
25.65 ÷ 15

Answer: 1.71

Explanation: 25.65÷15= 1.71

Question 4.
12)\(\overline { 2.436 } \)

Answer: 0.203

Explanation: 2.436÷12 = 0.203

Evaluate using the order of operations.

Question 5.
(8 − 2.96) ÷ 3

Answer: 1.68

Explanation: (8 − 2.96) ÷ 3
= (5.04)÷3
= 1.68

Question 6.
(7.772 − 2.38) ÷ 8

Answer: 0.674

Explanation: (7.772 − 2.38) ÷ 8
= (5.392)÷8
= 0.674

Question 7.
(53.2 + 35.7) ÷ 7

Answer: 12.7

Explanation: (53.2 + 35.7) ÷ 7
= (88.9)÷7
= 12.7

Problem Solving

Question 8.
Jake earned $10.44 interest on his savings account for an 18-month period. What was the average amount of interest Jake earned on his savings account per month?

Answer: $0.58.

Explanation: Jake earned $10.44 interest on his savings account for an 18 month period, so average amount interest is 10.44÷18= $0.58.

Question 9.
Gloria worked for 6 hours a day for 2 days at the bank and earned $114.24. How much did she earn per hour?

Answer: $9.52.

Explanation: As gloria worked for 6 hours for 2 days, so total hours is 6×2= 12 hours earned $114.24. So per hour she earns $114.24÷12= $9.52.

Question 10.
Explain the importance of correctly placing the decimal point in the quotient of a division problem.

Answer: If you don’t have the decimals in right spot your answer could be differ.

Lesson Check – Page No. 54

Estimate each quotient. Then find the exact quotient for each question.

Question 1.
Ron divided 67.6 fluid ounces of orange juice evenly among 16 glasses. How much did he pour into each glass?

Answer: 4.225 ounces.

Explanation: As there are 16 glasses, he pours into each glass 67.6÷16= 4.225 ounces.

Question 2.
The cost of a $12.95 pizza was shared evenly by 5 friends. How much did each person pay?

Answer: $2.59.

Explanation: The cost of pizza is $12.95 which was shared by 5 friends, so each person pays $12.95÷5= $2.59

Spiral Review

Question 3.
What is the value of the digit 6 in 968,743,220?

Answer: 60 Lakhs.

Explanation: The place value of 6 is 60,00,000.

Question 4.
The Tama, Japan, monorail carries 92,700 riders each day. If the monorail runs 18 hours each day, what is the average number of passengers riding each hour?

Answer: 5150 passengers.

Explanation: No. of riders each day is 92,700 and he runs for 18 hours in each day, so average no. of passengers riding each hour is 92,700÷18= 5150 passengers.

Question 5.
Ray paid $812 to rent music equipment that costs $28 per hour. How many hours did he have the equipment?

Answer: 29 hours.

Explanation: As Ray paid $812 which costs $28 per hour, so no.of hours did he have the equipment is
$812÷$28= 29 hours.

Question 6.
Jan has 35 teaspoons of chocolate cocoa mix and 45 teaspoons of french vanilla cocoa mix. She wants to put the same amount of mix into each jar, and she only wants one flavor of mix in each jar. She wants to fill as many jars as possible. How many jars of french vanilla cocoa mix will Jan fill?

Answer: 9 jars.

Explanation: By distributive property (35+45)
= (5×7)+(5×9)
= 5(7+9)
So she will fill 9 jars.

Share and Show – Page No. 57

Question 1.
Find the quotient.
14.8)\(\overline { 99.456 } \)

Answer: 6.72

Explanation: 99.456÷14.8= 6.72

Estimate. Then find the quotient.

Question 2.
$10.80 ÷ $1.35

Answer: 8

Explanation:
$10.80 ÷ $1.35
= 8

Question 3.
26.4 ÷ 1.76

Answer: 15.113

Explanation:
26.4 ÷ 1.76
= 15.113

Question 4.
8.7)\(\overline { 53.07 } \)

Answer: 6.1

Explanation: 53.07÷8.7= 6.1

On Your Own

Estimate. Then find the quotient.

Question 5.
75 ÷ 12.5

Answer: 6

Explanation:

Question 6.
544.6 ÷ 1.75

Answer: 311.2

Explanation:

Question 7.
0.78)\(\overline { 0.234 } \)

Answer: 0.3.

Explanation: 0.234÷0.78= 0.3.

Attend to Precision Algebra Evaluate using the order of operations.

Question 8.
36.4 + (9.2 − 4.9 ÷ 7)

Answer: 44.9

Explanation: By BODMAS rule
36.4+(9.2−4.9÷ 7)
= 36.4+(9.2−(4.9÷7))
= 36.4+(9.2-(0.7))
= 36.4+(8.5)
= 44.9

Question 9.
16 ÷ 2.5 − 3.2 × 0.043

Answer: 6.2624

Explanation: 16 ÷ 2.5 − 3.2 × 0.043
= (16÷2.5) − (3.2 × 0.043)
= (6.4)-(3.2 × 0.043)
= 6.4 – 0.1376
= 6.2624

Question 10.
142 ÷ (42 − 6.5) × 3.9

Answer: 15.6

Explanation: 142 ÷ (42 − 6.5) × 3.9
= (142÷ 35.5) × 3.9
= 4×3.9
= 15.6

Question 11.
Marcus can buy 0.3 pound of sliced meat from a deli for $3.15. How much will 0.7 pound of sliced meat cost?

Answer: $7.35

Explanation: As 0.3 pound of sliced meat is $3.15, so cost of 1 pound of sliced meat is 3.15÷0.3= $10.5. And for 0.7 pound of sliced meat cost is 10.5×0.7= $7.35

Page No. 58

Question 12.
The table shows the earnings and the number of hours worked for three employees. Complete the table by finding the missing values. Which employee earned the least per hour? Explain.
Go Math Grade 6 Answer Key Chapter 1 Divide Multi-Digit Numbers img 11

Answer: Employee 2 has earned least per hour.

Explanation:
1. No. of hours worked is 34.02÷ 9.72= 3.5 hours.
2. Earnings per hour is 42.75÷4.5= $9.5
3. No.of hours worked is 52.65÷9.75= 5.4 hours
Employee 2 has earned least per hour.

Amoebas

Amoebas are tiny one-celled organisms. Amoebas can range in size from 0.01 mm to 5 mm in length. You can study amoebas by using a microscope or by studying photographic enlargements of them.

Jacob has a photograph of an amoeba that has been enlarged 1,000 times. The length of the amoeba in the photo is 60 mm. What is the actual length of the amoeba?
Divide 60 ÷ 1,000 by looking for a pattern.
60 ÷ 1 = 60
60 ÷ 10 = 6.0 The decimal point moves _____ place to the left.
60 ÷ 100 = ____ The decimal point moves _____ place to the left.
60 ÷ 1000 =____ The decimal point moves _____ place to the left.
So, the actual length of the amoeba is _____ mm.

Answer: 0.06mm

Explanation:
60 ÷ 10 = 6.0 The decimal point moves one place to the left.
60 ÷ 100 =0.6  The decimal point moves two place to the left.
60 ÷ 1000 =0.06 The decimal point moves three place to the left.
Actual length of amoeba is 0.06mm

Question 13.
Explain the pattern.

Answer: 0.06mm

Explanation: 60÷1000= 0.06mm.

Question 14.
Pelomyxa palustris is an amoeba with a length of 4.9 mm. Amoeba proteus has a length of 0.7 mm. How many Amoeba proteus would you have to line up to equal the length of three Pelomyxa palustris? Explain.

Answer: 21

Explanation:
Let N be the number, then
N×(proteus length)= 3× (palustris length)
N× 0.7= 3× 4.9
N×0.7= 14.7
N= 14.7÷0.7
N= 21

Divide with Decimals – Page No. 59

Estimate. Then find the quotient.

Question 1.
43.18 ÷ 3.4

Answer: 12.7

Explanation: 43.18 ÷ 3.4= 12.7

Question 2.
4.185 ÷ 0.93

Answer: 4.5

Explanation: 4.185 ÷ 0.93= 4.5

Question 3.
6.3)\(\overline { 25.83 } \)

Answer: 0.244

Explanation: 6.3÷25.83= 0.244

Question 4.
0.143 ÷ 0.55

Answer: 0.26

Explanation: 0.143 ÷ 0.55= 0.26

Evaluate using the order of operations.

Question 5.
4.92 ÷ (0.8 – 0.12 ÷ 0.3)

Answer: 12.3

Explanation: 4.92 ÷ (0.8 – 0.12 ÷ 0.3)
= 4.92÷(0.8-(0.12÷0.3))
= 4.92÷(0.8-(0.4))
= 4.92÷(0.4)
= 12.3

Question 6.
0.86 ÷ 5 – 0.3 × 0.5

Answer: 0.022

Explanation: 0.86 ÷ 5 – 0.3 × 0.5
= (0.86÷5) – (0.3 × 0.5)
= (0.172)-(0.3 × 0.5)
= 0.172 – (0.15)
= 0.022

Question 7.
17.28 ÷ (1.32 – 0.24) × 0.6

Answer: 9.6

Explanation: 17.28 ÷ (1.32 – 0.24) × 0.6
= (17.28 ÷ (1.32 – 0.24))× 0.6
= (17.28 ÷( 1.08))×0.6
= (16)×0.6
= 9.6

Problem Solving

Question 8.
If Amanda walks at an average speed of 2.72 miles per hour, how long will it take her to walk 6.8 miles?

Answer: 2.5 hours.

Explanation: Amanda walks at an average speed of 2.72 miles per hour, so for 6.8 miles it will be
6.8÷2.72= 2.5 hours.

Question 9.
Chad cycled 62.3 miles in 3.5 hours. If he cycled at a constant speed, how far did he cycle in 1 hour?

Answer: 17.8 miles.

Explanation: Chad cycled 62.3 miles in 3.5 hours, so in 1 hour 62.3÷3.5= 17.8 miles.

Question 10.
Explain how dividing by a decimal is different from dividing by a whole number and how it is similar.

Answer: By moving the decimals first the dividing will be different, and after that it will be same.

Lesson Check – Page No. 60

Question 1.
Elliot drove 202.8 miles and used 6.5 gallons of gasoline. How many miles did he travel per gallon of gasoline?

Answer: 31.2 miles.

Explanation: Elliot drove 202.8 miles and used 6.5 gallons of gasoline, so per gallon of gasoline he will travel 202.8÷6.5= 31.2 miles.

Question 2.
A package of crackers weighing 8.2 ounces costs $2.87. What is the cost per ounce of crackers?

Answer: 0.35 per ounce.

Explanation: As 8.2 ounces costs $2.87, so per ounce of crackers it will be 2.87÷8.2= 0.35 per ounce.

Spiral Review

Question 3.
Four bags of pretzels were divided equally among 5 people. How much of a bag did each person get?
\(\frac{□}{□}\)

Answer: 0.8

Explanation: As 4 bags were divided equally among 5 people, so each person gets 4÷5= 0.8 of a bag

Question 4.
A zebra ran at a speed of 20 feet per second. What operation should you use to find the distance the zebra ran in 10 seconds?

Answer: Multiplication.

Explanation: Per second zebra ran at a speed of 20 feet, so for 10 seconds 20×10= 200 feet.

Question 5.
Nira has $13.50. She receives a paycheck for $55. She spends $29.40. How much money does she have now?

Answer: $39.10.

Explanation: As Nira has $13.50 and she receives paycheck of $55, so total she had is $13.50+$55= $68.5. As she spent $29.40, so she has now $68.5-$29.40= $39.10.

Question 6.
A piece of cardboard is 24 centimeters long and 15 centimeters wide. What is its area?
_____ cm2

Answer: 360 cm2

Explanation:
Area= Length×wide
= 24×15
= 360 cm2

Chapter 1 Review/Test – Page No. 61

Question 1.
Use the numbers to complete the factor tree. You may use a number more than once.
Go Math Grade 6 Answer Key Chapter 1 Divide Multi-Digit Numbers img 12
Write the prime factorization of 54

Answer: 54=2×3×3×3

Explanation:
Go Math Grade 6 Answer Key Chapter 1 Divide Multi-Digit Numbers

Question 2.
For numbers 2a–2d, choose Yes or No to indicate whether the LCM of the two numbers is 15.
2a. 5, 3 Yes No
2b. 5, 10 Yes No
2c. 5, 15 Yes No
2d. 5, 20 Yes No

2a. 5, 3

Answer: Yes

Explanation:
Multiples of 5: 5,10,15
Multiples of 3: 53,6,9,12,15
LCM of 5,3 is 15

2b. 5, 10

Answer: No

Explanation:
Multiples of 5: 5,10
Multiples of 10: 10
LCM of 5,10 is 10

2c. 5, 15

Answer: Yes

Explanation:
Multiples of 5: 5,10,15
Multiples of 15: 15
LCM is 15

2d. 5, 20

Answer:  No

Explanation:
Multiples of 5: 5,10,15,20
Multiples of 20: 20
LCM is 20

Question 3.
Select two numbers that have 9 as their greatest common factor. Mark all that apply.
Options:
a. 3, 9
b. 3, 18
c. 9, 18
d. 9, 36
e. 18, 27

Answer: c,d,e

Explanation:
a. 3,9
Factors of 3: 1,3.
Factors of 9: 1,3,9.
GCF is 3

b. 3,18
Factors of 3: 1,3
Factors of 18: 1,2,3,6,9,18
GCF is 3

c. 9,18
Factors of 9: 1,3,9
Factors of 18: 1,2,3,6,9,18.
GCF is 9

d. 9,36
Factors of 9: 1,3,9
Factors of 36: 1,2,3,4,6,9,18,36
GCF is 9

e. 18,27
Factors of 18: 1,2,3,6,9,18
Factors of 27: 1,3,9,27
GCF is 9

Page No. 62

Question 4.
The prime factorization of each number is shown.
15 = 3 × 5
18 = 2 × 3 × 3
Part A
Using the prime factorization, complete the Venn diagram
Go Math Grade 6 Answer Key Chapter 1 Divide Multi-Digit Numbers img 13

Answer:
Prime factors of 15: 3×5
Prime factors of 18: 2×3×3
Common factors are: 3

Explanation:
Go Math Grade 6 Answer Key Chapter 1 Divide Multi-Digit Numbers

Question 4.
Part B
Find the GCF of 15 and 18.

Answer: 3

Explanation:
Factors of 15: 1,3,5
Factors of 18: 1,2,3,6,9,18
GCF is 3

Question 5.
For numbers 5a–5d, choose Yes or No to indicate whether each equation is correct.
5a. 222.2 ÷ 11 = 22.2 Yes No
5b. 400 ÷ 50 = 8 Yes No
5c. 1,440 ÷ 36 = 40 Yes No
5d. 7,236 ÷ 9 = 804 Yes No

5a. 222.2 ÷ 11 = 22.2

Answer: No

Explanation:
222.2 ÷ 11 = 20.2

5b. 400 ÷ 50 = 8

Answer: Yes

Explanation:
400 ÷ 50 = 8

5c. 1,440 ÷ 36 = 40

Answer: Yes

Explanation:
1,440 ÷ 36 = 40

5d. 7,236 ÷ 9 = 804

Answer: Yes

Explanation:
7,236 ÷ 9 = 804

Page No. 63

Question 6.
For numbers 6a–6d, select True or False for each equation.
6a. 1.7 + 4.03 = 6 True False
6b. 2.58 + 3.5 = 6.08 True False
6c. 3.21 − 0.98 = 2.23 True False
6d. 14 − 1.3 = 0.01 True False

6a. 1.7 + 4.03 = 6

Answer: False

Explanation:
1.7 + 4.03 = 5.73

6b. 2.58 + 3.5 = 6.08

Answer: True

Explanation:
2.58 + 3.5 = 6.08

6c. 3.21 − 0.98 = 2.23

Answer: True

Explanation:
3.21 − 0.98 = 2.23

6d. 14 − 1.3 = 0.01

Answer: False

Explanation:
6d. 14 − 1.3 = 12.7

Question 7.
Four friends went shopping at a music store. The table shows the number of CDs each friend bought and the total cost. Complete the table to show the average cost of the CDs each friend bought.
Go Math Grade 6 Answer Key Chapter 1 Divide Multi-Digit Numbers img 14
What is the average cost of all the CDs that the four friends bought? Show your work.

Answer: $8.94.

Explanation:
Lana purchased 4 CDs and total cost is $36.68, so cost of 1 CD is $36.68÷4= $9.17
Troy purchased 5 CDs and total cost is $40.55, so cost of 1 CD is $40.55÷5= $8.11
Juanita purchased 5 CDs and total cost is $47.15, so cost of 1 CD is $47.15÷5= $9.43
Alex purchased 6 CDs and total cost is $54.42, so cost of 1 CD is $54.42÷6= $9.07
Average cost of all CD’s i= (cost of all CD’s)÷(No.of CD’S)
=($36.68+$40.55+$47.15+$54.42)÷20
= (178.8) ÷20
= $8.94

Question 8.
The table shows the earnings and the number of hours worked for five employees. Complete the table by finding the missing values.
Go Math Grade 6 Answer Key Chapter 1 Divide Multi-Digit Numbers img 15

Answer:
1. No. of hours worked is 2.5 hours.
2. Earnings per hour is $93.654.
3. No. of hours worked is 4.4 hours.
4. Earnings per hour is $302.5.
5. Earnings per hour is $150.

Explanation:
1. No. of hours worked is $23.75÷$9.50= 2.5 hours.
2. Earnings per hour is $28.38×3.3= $93.654.
3. No. of hours worked is $38.50÷8.75= 4.4 hours.
4. Earnings per hour is $55×5.5= $302.5.
5. Earnings per hour is $60×2.5= $150.

Page No. 64

Question 9.
The distance around the outside of Cedar Park is 0.8 mile. Joanie ran 0.25 of the distance during her lunch break. How far did she run? Show your work.

Answer: 0.2 miles.

Explanation: Joanie ran 0.25 miles and the distance around the outside of cedar park is 0.8 mile, so she ran
0.25×0.8= 0.2 miles.

Question 10.
A one-celled organism measures 32 millimeters in length in a photograph. If the photo has been enlarged by a factor of 100, what is the actual length of the organism? Show your work.

Answer: 3200 millimeters.

Explanation: Length of one celled organism is 32 millimeters, as the photo was enlarged by a factor of 100, it’s actual length is 32×100= 3200 millimeters.

Question 11.
You can buy 5 T-shirts at Baxter’s for the same price that you can buy 4 T-shirts at Bixby’s. If one T-shirt costs $11.80 at Bixby’s, how much does one T-shirt cost at Baxter’s? Use numbers and words to explain your answer.

Answer: $9.44.

Explanation: As one T-shirt costs $11.80, so 4 T-shirts cost is 4×$11.80= 47.2. So 5 T-shirts at Baxter’s is 47.2 and one T-shirt cost is 47.2÷5= $9.44.

Page No. 65

Question 12.
Crackers come in packages of 24. Cheese slices come in packages of 18. Andy wants one cheese slice for each cracker. Patrick made the statement shown.
If Andy doesn’t want any crackers or cheese slices left over, he needs to buy at least 432 of each.
Is Patrick’s statement correct? Use numbers and words to explain why or why not. If Patrick’s statement is incorrect, what should he do to correct it?

Answer: Patrick’s statement is wrong.

Explanation:
Multiples of 18: 18,36,54,72
Multiples of 24: 24,48,72
LCM is 72
So the least packages he need to buy is 72.

Question 13.
There are 16 sixth graders and 20 seventh graders in the Robotics Club. For the first project, the club sponsor wants to organize the club members into equal-size groups. Each group will have only sixth graders or only seventh graders.
Part A
How many students will be in each group if each group has the greatest possible number of club members? Show your work.

Answer: Each group will have 4 members, and 4 groups of sixth grade and 5 groups of seventh grade.

Explanation: By distributive property 16+20
=(4×4)+(4×5)
=4×(4+5)
So each group will have 4 members, and 4 groups of sixth grade and 5 groups of seventh grade.

Question 13.
Part B
If each group has the greatest possible number of club members, how many groups of sixth graders and how many groups of seventh graders will there be? Use numbers and words to explain your answer
__________ groups of sixth graders
__________ groups of seventh graders

Answer: Each group will have 4 members, and 4 groups of sixth grade and 5 groups of seventh grade.

Explanation: By distributive property 16+20
=(4×4)+(4×5)
=4×(4+5)
So each group will have 4 members, and 4 groups of sixth grade and 5 groups of seventh grade.

Page No. 66

Question 14.
The Hernandez family is going to the beach. They buy sun block for $9.99, 5 snacks for $1.89 each, and 3 beach toys for $1.49 each. Before they leave, they fill up the car with 13.1 gallons of gasoline at a cost of $3.70 per gallon.
Part A
Complete the table by calculating the total cost for each item.
Go Math Grade 6 Answer Key Chapter 1 Divide Multi-Digit Numbers img 16

Answer: Total cost is $48.47+$9.45++$4.47+$9.99= $72.38

Explanation:
Gasoline 13.1×$3.70= $48.47
Snacks 5×$1.89= $9.45
Beach toys 3×$1.49= $4.47
Sun block 1×$9.99= $9.99
Total cost is $48.47+$9.45++$4.47+$9.99= $72.38

Question 14.
Part B
What is the total cost for everything before tax? Show your work.

Answer: $72.38.

Explanation: Total cost is $48.47+$9.45++$4.47+$9.99= $72.38.

Question 14.
Part C
Mr. Hernandez calculates the total cost for everything before tax using this equation.
Total cost = 13.1 + 3.70 × 5 + 1.89 × 3 + 1.49 × 9.99
Do you agree with his equation? Use numbers and words to explain why or why not. If the equation is not correct, write a correct equation.

Answer: No

Explanation: Mr. Hernandez calculated in a wrong way.
Total cost is (13.1×$3.70)+(5×$1.89)+(3×$1.49)+(1×$9.99)= $72.38.

Conclusion:

I hope the information provided in the Go Math Grade 6 Answer Key Chapter 1 Chapter 1 Divide Multi-Digit Numbers. For any queries you can post your comments in the below comment section.

Go Math Grade 3 Answer Key Chapter 6 Understand Division Extra Practice

go-math-grade-3-chapter-6-understand-division-extra-practice-answer-key

If you looking to practice Go Math 3rd Grade Textbook Questions then take the help of the Go Math Grade 3 Answer Key Chapter 6 Understand Division Extra Practice. You need to have strong fundamentals in Maths in order to become a pro in the Subject. You can easily understand the basics of the division with the Go Math Grade 3 Answer Key Chapter 6 Understand Division Extra Practice. Solve as Many Questions as possible from the Extra Practice and Clear the Final Exams with better grades.

3rd Grade Go Math Answer Key Ch 6 Understand Division Extra Practice

Repeated subtraction, Equal groups, Number line, related multiplication, and division facts are all the topics covered in the 3rd Grade Go Math Answer Key Ch 6. Before you begin your preparation firstly know the syllabus i.e. concepts in Chapter 6 Understand Division and prepare accordingly. Check out the Step by Step Solutions provided for 3rd Grade Go Math Answer Key Chapter 6 Understand Division Extra Practice and learn the concepts efficiently.

Common Core – Page No. 123000

Lessons 6.1–6.3 Make equal groups.

Complete the table.

Counters Number of Equal Groups Number in Each Group
1. 18 9 ________
2. 24 ________ 8
3. 12 6 ________
4. 35 7 ________
5. 32 ________ 4
6. 25 ________ 5

Answer:

Counters Number of Equal Groups Number in Each Group
1. 18 9 2
2. 24 3 8
3. 12 6 2
4. 35 7 5
5. 32 8 4
6. 25 5 5

Explanation:

1. Number of counters = 18
Number of equal groups = 9
Number in each group = x
x × 9 = 18
x= 18/9 = 2
Therefore number in each group = 2

2. Number of counters = 24
Number in each group = 8
Number of equal groups = x
x × 8 = 24
x = 24/8 = 3
Thus the number of equal groups = 3

3. Number of counters = 12
Number of equal groups = 6
Number in each group = x
x × 6 = 12
x = 12/6 = 2
So, the number in each group = 2

4. Number of counters = 35
Number of equal groups = 7
Number in each group = x
x × 7 = 35
x = 35/7 = 5
x = 5
Therefore number in each group = 5

5. Number of counters = 32
Number of equal groups = x
Number in each group = 8
x × 8 = 32
x = 32/8 = 4
Thus the number of equal groups = 4

6. Number of counters = 25
Number of equal groups = x
Number in each group = 5
x × 5 = 25
x = 25/5 = 5
So, the number of equal groups = 5

Lesson 6.4

Write a division equation for the picture.

Question 1.
Go Math Grade 3 Answer Key Chapter 6 Understand Division Extra Practice Common Core img 1
Type below:
__________

Answer: 27 ÷ 3 = 9 or 27 ÷ 9 = 3

Explanation:

Total number of counters = 27
Number of equal groups = 3
Number in each group = 9
The division equation is
Number of counters by number of groups = 27 ÷ 3 = 9
or
Number of counters by number in each group = 27 ÷ 9 = 3

Question 2.
Go Math Grade 3 Answer Key Chapter 6 Understand Division Extra Practice Common Core img 2
Type below:
__________

Answer: 24 ÷ 4 = 6 or 24 ÷ 6 = 4

Explanation:

Total number of counters = 24
Number of equal groups = 4
Number in each group = 6
The division equation is
Number of counters by number of groups = 24 ÷ 4 = 6
or
Number of counters by number in each group = 24 ÷ 6 = 4

Lesson 6.5

Write a division equation.

Question 3.
Go Math Grade 3 Answer Key Chapter 6 Understand Division Extra Practice Common Core img 3
Type below:
__________

Answer: 3 groups, 15 ÷ 5 = 3

Explanation:

Step 1:

Starts at 15

Step 2:

Count back by 5s as many times as you can.

Step 3:

Count the number of times you jumped back 5.
You jumped back by 15 three times
There are 3 jumps of 5 in 15.

Question 4.
Go Math Grade 3 Answer Key Chapter 6 Understand Division Extra Practice Common Core img 4
Type below:
__________

Answer: 24 ÷ 6 = 4

Explanation:

Step 1:

Begins at 24

Step 2:

Subtract with 6 until you get 0.

Step 3:

Count the number of times you subtract with 6.

You subtract 4 times
There are 4 groups of 6 with 24
So, 24 ÷ 6 = 4

Common Core – Page No. 124000

Lesson 6.6

Make an array. Then write a division equation.

Question 1.
12 tiles in 4 rows
______ ÷ ______ = ______

Answer: 12 ÷ 4 = 3

Explanation:

■ ■ ■
■ ■ ■
■ ■ ■
■ ■ ■
Total number of tiles = 12
Number of rows = 4
Number of tiles in each row = x
Divide the number of tiles by number of rows = 12 ÷ 4 = 3

Question 2.
18 tiles in 3 rows
______ ÷ ______ = ______

Answer: 18 ÷ 3 = 6

Explanation:

■ ■ ■ ■ ■ ■
■ ■ ■ ■ ■ ■
■ ■ ■ ■ ■ ■
Total number of tiles = 18
Number of rows = 3
Number of tiles in each row = y
Divide the number of tiles by no. of rows = 18 ÷ 3 = 6

Question 3.
35 tiles in 5 rows
______ ÷ ______ = ______

Answer: 35 ÷ 5 = 7

Explanation:

■ ■ ■ ■ ■ ■ ■
■ ■ ■ ■ ■ ■ ■
■ ■ ■ ■ ■ ■ ■
■ ■ ■ ■ ■ ■ ■
■ ■ ■ ■ ■ ■ ■

Total number of tiles = 35
Number of rows = 5
Number of tiles in each row = p
Divide the number of tiles by number of rows = 35 ÷ 5 = 7

Question 4.
28 tiles in 7 rows
______ ÷ ______ = ______

Answer: 28 ÷ 7 = 4

Explanation:

■ ■ ■ ■
■ ■ ■ ■
■ ■ ■ ■
■ ■ ■ ■
■ ■ ■ ■
■ ■ ■ ■
■ ■ ■ ■

Total number of tiles = 28
Number of rows = 7
Number of tiles in each row = x
Divide the number of tiles by number of rows = 28 ÷ 7 = 4

Lesson 6.7

Complete the equations.

Question 5.
8 × ______ = 40 40 ÷ 8 = ______

Answer: 5, 5

Explanation:

Let x be the unknown factor
8 × x = 40
x = 40/8
x = 5
Check whether the related multiplication and division facts are the same or not.
40 ÷ 8 = 5
The related facts of 40 and 8 are 5.

Question 6.
6 × ______ = 36 36 ÷ 6 = ______

Answer: 6, 6

Explanation:

Let y be the unknown factor
6 × y = 36
y = 36/6 = 6
Check if the related multiplication and division facts are the same or not.
36 ÷ 6 = 6
The related facts of 36 and 6 are 6.

Question 7.
3 × ______ = 21 21 ÷ 3 = ______

Answer: 7, 7

Explanation:

Let x be the unknown factor
3 × x = 21
x = 21
Check whether the related facts are the same or not.
21 ÷ 3 = 7
The quotient is 7.

Question 8.
2 × ______ = 18 18 ÷ 2 = ______

Answer: 9, 9

Explanation:

Let b be the unknown factor
2 × b = 18
b = 18/2 = 9
Check the related multiplication and division facts
18 ÷ 2 = 9
The related facts of 18 and 2 are 9.

Lesson 6.8 (pp. 239–243)

Write the related facts for the array.

Question 9.
■ ■ ■ ■ ■
■ ■ ■ ■ ■
■ ■ ■ ■ ■
______ × ______ = ______
______ × ______ = ______
______ ÷ ______ = ______
______ ÷ ______ = ______

Answer:

3 × 5 = 15
5 × 3 = 15
15 ÷ 3 = 5
15 ÷ 5 = 3

Explanation:

Total number of tiles = 15
Number of equal rows = 3
Number of rows in each group = 5
So, the related 5, 3 and 15 is 5× 3 = 15, 3×5 = 15, 15 ÷ 3 = 5 and 15÷ 5 = 3

Question 10.
■ ■ ■ ■ ■ ■
■ ■ ■ ■ ■ ■
■ ■ ■ ■ ■ ■
______ × ______ = ______
______ × ______ = ______
______ ÷ ______ = ______
______ ÷ ______ = ______

Answer:

3 × 6 = 18
6 × 3 = 18
18 ÷ 3 = 6
18 ÷ 6 = 3

Explanation:

Total number of tiles = 18
Number of equal rows = 3
Number of rows in each group = 6
So, the related 18, 3 and 6 is 3 × 6 = 18, 6 × 3 = 18, 18 ÷ 3 = 6 and 18 ÷ 6 = 3

Question 11.
■ ■ ■ ■ ■
■ ■ ■ ■ ■
______ × ______ = ______
______ × ______ = ______
______ ÷ ______ = ______
______ ÷ ______ = ______

Answer:

2 × 5 = 10
5 × 2 = 10
10 ÷ 2 = 5
10 ÷ 5 = 2

Explanation:

Total number of tiles = 10
Number of equal rows = 2
Number of rows in each group = 5
So, the related 2, 5 and 10 is 2 × 5 = 10, 5 × 2 = 10, 10 ÷ 2 = 5 and 10 ÷ 5 = 2

Lesson 6.9

Find the quotient.

Question 12.
7 ÷ 1 = ______

Answer: 7

Explanation:

Any number divided by 1 will be the same number. Thus the quotient is 7.

Question 13.
4 ÷ 4 = ______

Answer: 1

Explanation:

The number divided by the same number will be always 1. Thus the quotient is 1.

Question 14.
9 ÷ 1 = ______

Answer: 9

Explanation:

Any number divided by 1 will be always the same number. So, the quotient is 9.

Question 15.
0 ÷ 1 = ______

Answer: 0

Explanation:

0 divided by any number is always 0. So, the quotient is 0.

Question 16.
Anton has 8 flower pots. He plants 1 seed in each pot. How many seeds does Anton use?
______ seeds

Answer: 8 seeds

Explanation:

Anton has 8 flower pots.
He plants 1 seed in each pot.
Number of seeds Anton used = x
x × 1 = 8
x = 8/1
x = 8
Therefore there are 8 seeds in 8 flower pots.

All the Questions in Go Math Grade 3 Answer Key Chapter 6 Understand Division Extra Practice helps the students to be prepared for their exams. For any assistance needed you can always look upto  Go Math Grade 3 Answer Key Chapter 6 Understand Division.  You can get All Lessons Solutions in Chapter 6 Understand Division here.

Go Math Grade 4 Answer Key Chapter 13 Algebra: Perimeter and Area

go-math-grade-4-chapter-13-algebra-perimeter-and-area-answer-key

Students who are looking for a great study resource or prep resource can refer to this page. Here, we have curated a Grade 4 Answer Key of Go Math Chapter 13 Algebra: Perimeter and Area. Download HMH Go Math Grade 4 Answer Key Chapter 13 Algebra: Perimeter and Area pdf by accessing the links available over here. Save them and use the Go Math Grade 4 Answer Key Chapter 13 Algebra: Perimeter and Area as a reference purpose during your practice sessions & score good marks in the exam.

Go Math Grade 4 Answer Key Chapter 13 Algebra: Perimeter and Area

Students can find various concepts questions and solutions covered in the chapter 13 Algebra: Perimeter and Area from this Go math Gerade 4 Answer Keys. All these solutions are prepared by the subject experts in a well-organized and understanding manner. So, practice all exercise and homework problems through Go Math 4th Grade Key of Chapter 13 Perimeter and Area. Also, test your knowledge by answering the given sums and learn your mistakes using HMH Go Math Grade 4 Solution Key Chapter Perimeter and Area.

Common Core – New – Page No. 721

Perimeter

Find the perimeter of the rectangle or square.

Question 1.
Go Math Grade 4 Answer Key Chapter 13 Algebra Perimeter and Area Common Core - New img 1
9+3+9+3=24
24 inches

Explanation:

Length = 9 inches
Width = 3 inches
Perimeter of the rectangle = l + w + l + w
9+3+9+3=24
Therefore the Perimeter of the rectangle = 24 inches.

Question 2.
Go Math Grade 4 Answer Key Chapter 13 Algebra Perimeter and Area Common Core - New img 2
_____ meters

Answer: 32 meters

Explanation:

Side of a square = 8 meters
The perimeter of a square = 4a
= 4 × 8 meters = 32 meters
Thus the perimeter of a square = 32 meters.

Question 3.
Go Math Grade 4 Answer Key Chapter 13 Algebra Perimeter and Area Common Core - New img 3
_____ feet

Answer: 44 feet

Explanation:

Length = 10 ft
Width = 12 ft
Perimeter of the rectangle = l + w + l + w
P = 10 + 12 + 10 + 12 = 20 + 24 = 44 feets
Thus the perimeter of the rectangle = 44 feet.

Remember: perimeter is the total distance around the outside, which can be found by adding together the length of each side. In the case of a rectangle, opposite sides are equal in length, so the perimeter is twice its width plus twice its height.

Question 4.
Go Math Grade 4 Answer Key Chapter 13 Algebra Perimeter and Area Common Core - New img 4
_____ centimeters

Answer: 108 centimeters

Explanation:

Length = 30 cm
Width = 24 cm
Perimeter of the rectangle = l + w + l + w
= 30 + 24 + 30 + 24 = 60 + 48
= 108 centimeters
Therefore the perimeter of the rectangle = 108 centimeters

Question 5.
Go Math Grade 4 Answer Key Chapter 13 Algebra Perimeter and Area Common Core - New img 5
_____ inches

Answer: 216 inches

Explanation:

Length = 25 in.
Width = 83 in.
Perimeter of the rectangle = l + w + l + w
= 25 + 83 + 25 + 83
= 216 inches
Thus the perimeter of the rectangle = 216 inches

Question 6.
Go Math Grade 4 Answer Key Chapter 13 Algebra Perimeter and Area Common Core - New img 6
_____ meters

Answer: 240 meters

Explanation:

The side of a square = 60 meters
The perimeter of the square = 4a
= 4 × 60 meters = 240 meters
Thus the perimeter of the square = 240 meters.

Problem Solving

Question 7.
Troy is making a flag shaped like a square. Each side measures 12 inches. He wants to add ribbon along the edges. He has 36 inches of ribbon. Does he have enough ribbon? Explain.
_____

Answer: No. He needs 48 inches of ribbon.

Explanation:

Troy is making a flag shaped like a square. Each side measures 12 inches.
He wants to add a ribbon along the edges.
He has 36 inches of ribbon.
36 inches + 12 inches = 48 inches

Question 8.
The width of the Ochoa Community Pool is 20 feet. The length is twice as long as its width. What is the perimeter of the pool?
_____ feet

Answer: 120 feet

Explanation:

The width of the Ochoa Community Pool is 20 feet.
The length is twice as long as its width.
Length = 2 × 20 feet = 40 feet
Perimeter of the rectangle = l + w + l + w
= 40 + 20 + 40 + 20 = 120 feet
Thus the perimeter of the pool is 120 feet.

Common Core – New – Page No. 722

Lesson Check

Question 1.
What is the perimeter of a square window with sides 36 inches long?
Options:
a. 40 inches
b. 72 inches
c. 144 inches
d. 1,296 inches

Answer: 144 inches

Explanation:

Given, Side of a square = 36 inches
The perimeter of the square = 4 × side = 4a
= 4 × 36 inches = 144 inches
Thus the perimeter of the square = 144 inches
The correct answer is option C.

Question 2.
What is the perimeter of the rectangle below?
Go Math Grade 4 Answer Key Chapter 13 Algebra Perimeter and Area Common Core - New img 7
Options:
a. 11 meters
b. 14 meters
c. 18 meters
d. 400 meters

Answer: 18 meters

Explanation:

Length of the rectangle = 5 meter
Width of the rectangle = 4 meters
The perimeter of the rectangle = l + w + l + w
= 5 + 4 + 5 + 4 = 18 meters
Thus the correct answer is option C.

Spiral Review

Question 3.
Which is the most reasonable estimate for the measure of the angle Natalie drew?
Go Math Grade 4 Answer Key Chapter 13 Algebra Perimeter and Area Common Core - New img 8
Options:
a. 30°
b. 90°
c. 180°
d. 210°

Answer: 90°

Explanation:

By seeing the above figure we can say that it is the right angle.
The correct answer is option B.

Question 4.
Ethan has 3 pounds of mixed nuts. How many ounces of mixed nuts does Ethan have?
Options:
a. 30 ounces
b. 36 ounces
c. 48 ounces
d. 54 ounces

Answer: 48 ounces

Explanation:

Given that, Ethan has 3 pounds of mixed nuts.
1 pound = 16 ounces
3 pounds = 3 × 16 ounces = 48 ounces
Therefore the correct answer is option C.

Question 5.
How many lines of symmetry does the shape below appear to have?
Go Math Grade 4 Answer Key Chapter 13 Algebra Perimeter and Area Common Core - New img 9
Options:
a. 0
b. 1
c. 2
d. more than 2

Answer: 1

Explanation:

The above shape has 1 line of symmetry.
The correct answer is option B.

Question 6.
Which of the following comparisons is correct?
Options:
a. 0.70 > 7.0
b. 0.7 = 0.70
c. 0.7 < 0.70
d. 0.70 = 0.07

Answer: 0.7 = 0.70

Explanation:

a. 0.70 > 7.0
7.0 = 7
0.7 is less than 7

b. 0.7 = 0.70
0.7 is nothing but 0.70
So, the comparision is correct.
The answer is option B.

Page No. 725

Question 1.
Find the area of the rectangle.
Go Math Grade 4 Answer Key Chapter 13 Algebra Perimeter and Area img 10
A = _____ square cm

Answer: 143 square cm

Explanation:

Length = 11 cm
Width = 13 cm
Area of the rectangle = l × w
= 11 cm × 13 cm = 143 square cm
Therefore the area of the rectangle = 143 square cm

Find the area of the rectangle or square.

Question 2.
Go Math Grade 4 Answer Key Chapter 13 Algebra Perimeter and Area img 11
A = _____ square inches

Answer: 14 square inches

Explanation:

Length = 7 inches
Width = 2 inches
Area of the rectangle = l × w
= 7 inches × 2 inches = 14 inches
Therefore the area of the rectangle = 14 square inches

Question 3.
Go Math Grade 4 Answer Key Chapter 13 Algebra Perimeter and Area img 12
A = _____ square meters

Answer: 81 square meters

Explanation:

Side of the square = 9 m
Area of a square = s × s
= 9 m × 9 m = 81 square meters
Thus the area of a square = 81 square meters

Question 4.
Go Math Grade 4 Answer Key Chapter 13 Algebra Perimeter and Area img 13
A = _____ square feet

Answer: 112 square feet

Explanation:

Length = 8 feet
Width = 14 feet
Area of the rectangle = l × w
= 8 feet × 14 feet = 112 square feet
Therefore, area of the rectangle = 112 square feet

Find the area of the rectangle or square.

Question 5.
Go Math Grade 4 Answer Key Chapter 13 Algebra Perimeter and Area img 14
A = _____ square feet

Answer: 65 square feet

Explanation:

Length of the rectangle = 13 ft
Width of the rectangle = 5 feet
Area of a rectangle = l × w
= 13 feet × 5 feet = 65 square feet
Thus, the area of the rectangle = 65 square feet

Question 6.
Go Math Grade 4 Answer Key Chapter 13 Algebra Perimeter and Area img 15
A = _____ square yards

Answer: 169 square yards

Explanation:

Side of the square = 13 yards
Area of a square = s × s
= 13 yards × 13 yards = 169 square yards
Therefore, the area of a square = 169 square yards

Question 7.
Go Math Grade 4 Answer Key Chapter 13 Algebra Perimeter and Area img 16
A = _____ square centimeters

Answer: 40 square centimeters

Explanation:

Length of the rectangle = 20 cm
Width of the rectangle = 2 cm
Area of a rectangle = l × w
= 20 cm × 2 cm = 40 square centimeters
Therefore the area of the rectangle = 40 square centimeters.

Practice: Copy and Solve Find the area of the rectangle.

Question 8.
base: 16 feet
height: 6 feet
A = _____ square feet

Answer: 96 square feet

Explanation:

base: 16 feet
height: 6 feet
Area of a rectangle = b ×h
= 16 feet × 6 feet = 96 square feet
Thus the area of the rectangle = 96 square feet

Question 9.
base: 9 yards
height: 17 yards
A = _____ square yards

Answer: 153 square yards

Explanation:

base: 9 yards
height: 17 yards
Area of a rectangle = b × h
9 yards × 17 yards = 153 square yards
The area of the rectangle = 153 square yards

Question 10.
base: 14 centimeters
height: 11 centimeters
A = _____ square centimeters

Answer: 154 square centimeters

Explanation:

base: 14 centimeters
height: 11 centimeters
Area of a rectangle = b × h
14 centimeters × 11 centimeters = 154 square centimeters
The area of the rectangle = 154 square centimeters

Question 11.
Terry’s rectangular yard is 15 meters by 18 meters. Todd’s rectangular yard is 20 meters by 9 meters. How much greater is the area of Terry’s yard than Todd’s yard?
_____ square meters

Answer: 90 square meters

Explanation:

Given,
Terry’s rectangular yard is 15 meters by 18 meters.
Todd’s rectangular yard is 20 meters by 9 meters.
Terry’s rectangular yard:
Area of a rectangle = b × h
= 15 meters × 18 meters = 270 square meters
Todd’s rectangular yard:
Area of a rectangle = b × h
20 meters × 9 meters = 180 square meters
270 square meters – 180 square meters = 90 square meters
Terry’s yard is 90 square meters greater than Todd’s yard.

Question 12.
Reason Quantitatively Carmen sewed a square baby quilt that measures 36 inches on each side. What is the area of the quilt?
A = _____ square inches

Answer: 1296 square inches

Explanation:

Carmen sewed a square baby quilt that measures 36 inches on each side.
Area of a square = s × s
= 36 inches × 36 inches = 1296 square inches
Therefore the area of the quilt is 1296 square inches.

Page No. 726

Go Math Grade 4 Answer Key Chapter 13 Algebra Perimeter and Area img 17

Question 13.
Nancy and Luke are drawing plans for rectangular flower gardens. In Nancy’s plan, the garden is 18 feet by 12 feet. In Luke’s plan, the garden is 15 feet by 15 feet. Who drew the garden plan with the greater area? What is the area?
a. What do you need to find?
Type below:
__________

Answer: I need to find who drew the garden plan with the greater area.

Question 13.
b. What formula will you use?
Type below:
__________

Answer: I will Area of rectangle and Area of a square formula

Question 13.
c. What units will you use to write the answer?
Type below:
__________

Answer: Square feet units

Question 13.
d. Show the steps to solve the problem.
Type below:
__________

Answer:
First, we need to find the area of Nancy’s plan
Length = 18 feet
Width = 12 feet
Area of a rectangle = l × w
A = 18 feet × 12 feet = 216 square feet
And now we need to find the area of Luke’s plan
A = s × s
A = 15 feet × 15 feet = 225 square feet

Question 13.
e. Complete the sentences.
The area of Nancy’s garden is _______.
The area of Luke’s garden is _______.
_______ garden has the greater area.
Type below:
__________

Answer:
The area of Nancy’s garden is 216 square feet.
The area of Luke’s garden is 225 square feet.
Luke’s garden has a greater area.

Question 14.
Victor wants to buy fertilizer for his yard. The yard is 35 feet by 55 feet. The directions on the bag of fertilizer say that one bag will cover 1,250 square feet. How many bags of fertilizer should Victor buy to be sure that he covers the entire yard?
______ bags

Answer: 2 bags

Explanation:
Given that,
Victor wants to buy fertilizer for his yard. The yard is 35 feet by 55 feet.
The directions on the bag of fertilizer say that one bag will cover 1,250 square feet.
A = b × h
A = 35 feet × 55 feet
A = 1925 square feet
1925 square feet is greater than 1,250 square feet.
So, Victor has to buy 2 bags to be sure that he covers the entire yard.

Question 15.
Tuan is an artist. He is painting on a large canvas that is 45 inches wide. The height of the canvas is 9 inches less than the width. What is the area of Tuan’s canvas?
A = ______ square inches

Answer: 1620 square inches

Explanation:
Tuan is an artist. He is painting on a large canvas that is 45 inches wide.
The height of the canvas is 9 inches less than the width.
So, h = 45 – 9 = 36 inches
A = b × h
A = 45 inches × 36 inches
A = 1,620 square inches
Therefore the area of Tuan’s canvas is 1620 square inches.

Common Core – New – Page No. 727

Area

Find the area of the rectangle or square.

Question 1.
Go Math Grade 4 Answer Key Chapter 13 Algebra Perimeter and Area Common Core - New img 18

Question 2.
Go Math Grade 4 Answer Key Chapter 13 Algebra Perimeter and Area Common Core - New img 19
______ square yards

Answer: 64 square yards

Explanation:

Side of the square = 8 yards
Area of the square = s × s
8 yards × 8 yards = 64 square yards
Therefore, The area of the square is 64 square yards.

Question 3.
Go Math Grade 4 Answer Key Chapter 13 Algebra Perimeter and Area Common Core - New img 20
_____ square meters

Answer: 45 square meters

Explanation:

Length of the rectangle = 15 m
Width of the rectangle = 3 m
Area of the rectangle = b × h
= 15 m × 3 m = 45 square meters
Thus the area of the rectangle is 45 square meters.

Question 4.
Go Math Grade 4 Answer Key Chapter 13 Algebra Perimeter and Area Common Core - New img 21
______ square inches

Answer: 78 square inches

Explanation:

The base of the rectangle = 13 in.
Height of the rectangle = 6 in.
Area of the rectangle = b × h
13 in. × 6 in. = 78 square inches
Thus the area of the rectangle is 78 square inches.

Question 5.
Go Math Grade 4 Answer Key Chapter 13 Algebra Perimeter and Area Common Core - New img 22
______ square centimeters

Answer: 150 square centimeters

Explanation:

The base of the rectangle = 30 cm
Height of the rectangle = 5 cm
Area of the rectangle = b × h
30 cm × 5 cm = 150 square centimeters
Therefore, the area of the rectangle = 150 square centimeters

Question 6.
Go Math Grade 4 Answer Key Chapter 13 Algebra Perimeter and Area Common Core - New img 23
______ square feet

Answer: 56 square feet

Explanation:

The base of the rectangle = 14 feet
Height of the rectangle = 4 feet
Area of the rectangle = b × h
14 feet × 4 feet = 56 square feet
Therefore, the area of the rectangle = 56 square feet.

Problem Solving

Question 7.
Meghan is putting wallpaper on a wall that measures 8 feet by 12 feet. How much wallpaper does Meghan need to cover the wall?
______ square feet wallpaper

Answer: 96 square feet

Explanation:

Meghan is putting wallpaper on a wall that measures 8 feet by 12 feet.
The base of the rectangle = 8 feet
Height of the rectangle = 12 feet
Area of the rectangle = b × h
8 feet × 12 feet = 96 square feet
Thus the Area of the rectangle = 96 square feet

Question 8.
Bryson is laying down sod in his yard to grow a new lawn. Each piece of sod is a 1-foot by 1-foot square. How many pieces of sod will Bryson need to cover his yard if his yard measures 30 feet by 14 feet?
______ pieces

Answer: 420 pieces

Explanation:

Bryson is laying down sod in his yard to grow a new lawn.
Each piece of sod is a 1-foot by 1-foot square.
The base of the rectangle = 30 feet
Height of the rectangle = 14 feet
Area of the rectangle = b × h
= 30 feet × 14 feet = 420 sq. ft.
Therefore Bryson needs 420 pieces of sod to cover his yard.

Common Core – New – Page No. 728

Lesson Check

Question 1.
Ellie and Heather drew floor models of their living rooms. Ellie’s model represented 20 feet by 15 feet. Heather’s model represented 18 feet by 18 feet. Whose floor model represents the greater area? How much greater?
Options:
a. Ellie; 138 square feet
b. Heather; 24 square feet
c. Ellie; 300 square feet
d. Heather; 324 square feet

Answer: Heather; 24 square feet

Explanation:

Given,
Ellie and Heather drew floor models of their living rooms.
Ellie’s model represented 20 feet by 15 feet.
Heather’s model represented 18 feet by 18 feet.
Area of Ellie’s model = 20 feet × 15 feet = 300 square feet
Area of Heather’s model = 18 feet × 18 feet = 324 square feet
Now subtract the area of Ellie’s model from Heather’s model = 324 square feet – 300 square feet = 24 square feet
Thus the area of Heather’s model is greater than Ellie’s model
The correct answer is option B.

Question 2.
Tyra is laying down square carpet pieces in her photography studio. Each square carpet piece is 1 yard by 1 yard. If Tyra’s photography studio is 7 yards long and 4 yards wide, how many pieces of square carpet will Tyra need?
Options:
a. 10
b. 11
c. 22
d. 28

Answer: 28

Explanation:

Tyra is laying down square carpet pieces in her photography studio.
Each square carpet piece is 1 yard by 1 yard. Tyra’s photography studio is 7 yards long and 4 yards wide
Area of the rectangle = b × h
= 7 yards × 4 yards
= 28 square yards
Thus the correct answer is option D.

Spiral Review

Question 3.
Typically, blood fully circulates through the human body 8 times each minute. How many times does blood circulate through the body in 1 hour?
Options:
a. 48
b. 240
c. 480
d. 4,800

Answer: 480

Explanation:

Blood fully circulates through the human body 8 times each minute.
1 minute = 60 seconds
8 × 60 seconds = 480 seconds
The correct answer is option C.

Question 4.
Each of the 28 students in Romi’s class raised at least $25 during the jump-a-thon. What is the least amount of money the class raised?
Options:
a. $5,200
b. $700
c. $660
d. $196

Answer: $700

Explanation:

Each of the 28 students in Romi’s class raised at least $25 during the jump-a-thon.
Multiply number od students with $25
28 × $25 = $700
The correct answer is option B.

Question 5.
What is the perimeter of the shape below if 1 square is equal to 1 square foot?
Go Math Grade 4 Answer Key Chapter 13 Algebra Perimeter and Area Common Core - New img 24
Options:
a. 12 feet
b. 14 feet
c. 24 feet
d. 28 feet

Answer: 28 feet

Explanation:

Given that 1 square is equal to 1 square foot
There are 14 squares
Length = 14 squares
Width = 2 squares
Area of the rectangle = l × w = 14 × 2 = 28 sq. feets
The correct answer is option D.

Question 6.
Ryan is making small meat loaves. Each small meat loaf uses \(\frac{3}{4}\) pound of meat. How much meat does Ryan need to make 8 small meat loaves?
Options:
a. 4 pounds
b. 6 pounds
c. 8 pounds
d. 10 \(\frac{2}{3}\) pounds

Answer: 6 pounds

Explanation:

Ryan is making small meatloaves.
Each small meatloaf uses \(\frac{3}{4}\) pound of meat.
Ryan need to make 8 small meatloaves.
\(\frac{3}{4}\) × 8 = 6 pounds
The correct answer is option B.

Page No. 731

Question 1.
Explain how to find the total area of the figure.
Go Math Grade 4 Answer Key Chapter 13 Algebra Perimeter and Area img 25
A = ______ square units

Answer: 23 square units

Explanation:
Rectangle:
Each square box = 1 unit
There are 7 units
Base = 7 units
Height = 2 units
The area of the figure = b × h
A = 7 units × 2 units = 14 square units
Square:
The side is 3 units
Area of the square = 3 units × 3 units = 9 square units
Add both the areas = 14 square units + 9 square units = 23 square units
Therefore the area of the above figure is 23 square units.

Find the area of the combined rectangles.

Question 2.
Go Math Grade 4 Answer Key Chapter 13 Algebra Perimeter and Area img 26
A = ______ square mm

Answer: 72 square mm

Explanation:
Area of top rectangle = b × h
Base = 12 mm
Height = 3 mm
A = 12 mm × 3 mm = 36 square mm
Area of square = s × s
s = 6 mm
A = 6 mm × 6 mm = 36 square mm
Area of the figure = 36 square mm + 36 square mm = 72 square mm
Thus the area of the above figure is 72 square mm.

Question 3.
Go Math Grade 4 Answer Key Chapter 13 Algebra Perimeter and Area img 27
A = ______ square miles

Answer: 146 square miles

Explanation:
Area of rectangle = b × h
Area of the first rectangle = 10 mi × 9 mi
A = 90 square miles
Area of the second rectangle = 8 mi × 7 mi
A = 56 square miles
Area of the figure = Area of first rectangle + Area of the second rectangle
Area of the figure = 90 square mi + 56 square miles
Thus the Area of the figure = 146 square miles

Question 4.
Go Math Grade 4 Answer Key Chapter 13 Algebra Perimeter and Area img 28
A = ______ square feet

Answer: 96 square feet

Explanation:
There are 2 squares and one rectangle in this figure
Area of the square = s × s
A = 4 ft × 4 ft = 16 square ft
Area of the square = s × s
A = 4 ft × 4 ft = 16 square ft
Area of the rectangle = b × h
A = 16 ft × 4 ft = 64 square ft
Area of the figure = 16 square ft + 16 square ft + 64 square ft
Thus the Area of the figure = 96 square feet.

Find the area of the combined rectangles.

Question 5.
Attend to Precision Jamie’s mom wants to enlarge her rectangular garden by adding a new rectangular section. The garden is now 96 square yards. What will the total area of the garden be after she adds the new section?
Go Math Grade 4 Answer Key Chapter 13 Algebra Perimeter and Area img 29
A = ______ square yards

Answer: 180 square yards

Explanation:

There are 2 rectangles in the above figure
Area of rectangle = b × h
A = 12 yard × 8 yards  = 96 square yards
Area of rectangle = b × h
A = 6 yards × 14 yards = 84 square yards
Area of the figure = 96 square yards + 84 square yards
Therefore the area of the figure = 180 square yards.

Question 6.
Explain how to find the perimeter and area of the combined rectangles at the right.
Go Math Grade 4 Answer Key Chapter 13 Algebra Perimeter and Area img 30
P = ______ feet; A = ______ square feet

Answer: A = 92 square feet; P = 52 feet

Explanation:
There are 2 rectangle in the figure
Area of rectangle = b × h
A = 5 ft × 4 ft = 20 square ft
Area of rectangle = b × h
A = 8 ft × 9 ft = 72 square ft
Area of the figure = 20 square ft + 72 square ft = 92 square ft
Perimeter of the rectangle = 2l + 2w
P = 2 × 5 + 2 × 4 = 10 + 8 = 18 feet
Perimeter of the rectangle = 2l + 2w
P = 2 × 8 + 2 × 9 = 16 + 18 = 34 feet
Perimeter of the figure = 52 feet

Page No. 732

Go Math Grade 4 Answer Key Chapter 13 Algebra Perimeter and Area img 31

Question 7.
The diagram shows the layout of Mandy’s garden. The garden is the shape of combined rectangles. What is the area of the garden?
a. What do you need to find?
Type below:
__________

Answer: I need to find the area of the garden.

Question 7.
b. How can you divide the figure to help you find the total area?
Type below:
__________

Answer: I will divide the figure into 3 parts to find the total area

Question 7.
c. What operations will you use to find the answer?
Type below:
__________

Answer: I will use the addition operation to find the area.

Question 7.
d. Draw a diagram to show how you divided the figure. Then show the steps to solve the problem.
Type below:
__________

Answer:
Go-Math-Grade-4-Answer-Key-Chapter-13-Algebra-Perimeter-and-Area-img-31
There are 2 rectangles and 1 square in this figure.
Area of rectangle = b × h
Base = 1 ft
H = 7 ft
A = 1 ft × 7 ft = 7 square ft
Area of rectangle = b × h
Base = 5 ft
H = 2 ft
A = 5 ft × 2 ft = 10 square ft
Area of the square = s × s
A = 3 ft × 3 ft = 9 square ft
Total area = 7 square ft + 10 square ft + 9 square ft
= 26 square ft

Question 8.
Workers are painting a large letter L for an outdoor sign. The diagram shows the dimensions of the L. For numbers 8a–8c, select Yes or No to tell whether you can add the products to find the area that the workers will paint.
Go Math Grade 4 Answer Key Chapter 13 Algebra Perimeter and Area img 32
8a. 2 × 8 and 2 × 4
i. yes
ii. no

Answer: Yes
Explanation:
There are 2 rectangles in the above figure
B = 2 ft
H = 8 ft
A = 2 × 8
B = 4 ft
H = 2 ft
A = 4 × 2
Thus the above statement is correct.

Question 8.
8b. 2 × 6 and 2 × 8
i. yes
ii. no

Answer: No
There are 2 rectangles in the above figure
B = 6 ft
H = 2 ft
A = 2 × 6
Then 2 will be subtracted from 8 = 6
So, the above statement 2 × 6 and 2 × 8 is false.

Question 8.
8c. 2 × 6 and 6 × 2
i. yes
ii. no

Answer: Yes
Explanation:
There are 2 rectangles in the above figure
B = 6 ft
H = 2 ft
A = 6 × 2
B = 2 ft
H = 6 ft
A = 2 × 6
Thus the above statement is true.

Common Core – New – Page No. 733

Area of Combined Rectangles

Find the area of the combined rectangles.

Question 1.
Go Math Grade 4 Answer Key Chapter 13 Algebra Perimeter and Area Common Core - New img 33

Question 2.
Go Math Grade 4 Answer Key Chapter 13 Algebra Perimeter and Area Common Core - New img 34
______ square feet

Answer: 143 square feet

Explanation:

Go-Math-Grade-4-Answer-Key-Chapter-13-Algebra-Perimeter-and-Area-img-34

Area of A = 9 ft × 5 ft = 45 sq. ft.
Area of B = 14 ft. × 7 ft. = 98 sq. ft.
Total Area = Area of A + Area of B
= 45 sq. ft. + 98 sq. ft. = 143 square feet
Therefore the total Area = 143 square feet

Question 3.
Go Math Grade 4 Answer Key Chapter 13 Algebra Perimeter and Area Common Core - New img 35
______ square inches

Answer: 63 square inches

Explanation:

Go-Math-Grade-4-Answer-Key-Chapter-13-Algebra-Perimeter-and-Area-img-35

Area of A = 9 in. × 5 in. = 45 square inches
Area of B = 6 inches × 3 inches = 18 square inches
Total Area = Area of A + Area of B
Total Area = 45 square inches + 18 square inches
Total Area = 63 square inches

Question 4.
Go Math Grade 4 Answer Key Chapter 13 Algebra Perimeter and Area Common Core - New img 36
______ square feet

Answer: 50 square feet

Explanation:

Go-Math-Grade-4-Answer-Key-Chapter-13-Algebra-Perimeter-and-Area-img-36

Area of A = 4 feet × 2 feet = 8 square feet
Area of B = 7 feet × 6 feet = 42 square feet
Total Area = Area of A + Area of B
Total Area = 8 square feet + 42 square feet
Total Area = 50 square feet

Question 5.
Go Math Grade 4 Answer Key Chapter 13 Algebra Perimeter and Area Common Core - New img 37
______ square centimeters

Answer: 180 square centimeters

Explanation:

Go-Math-Grade-4-Answer-Key-Chapter-13-Algebra-Perimeter-and-Area-img-37

Area of A = 12 cm × 7 cm = 84 square cm
Area of B = 16 cm × 6 cm = 96 square cm
Total Area = Area of A + Area of B
Total Area = 84 square cm + 96 square cm
Total Area = 180 square centimeters

Question 6.
Go Math Grade 4 Answer Key Chapter 13 Algebra Perimeter and Area Common Core - New img 38
______ square yards

Answer: 68 square yards

Explanation:

Go-Math-Grade-4-Answer-Key-Chapter-13-Algebra-Perimeter-and-Area-img-38

Area of A = 14 yd × 1 yd = 14 square yards
Area of B = 9 yd × 6 yd = 54 square yards
Total Area = Area of A + Area of B
Total Area = 14 square yards + 54 square yards
Total Area = 68 square yards

Problem Solving

Use the diagram for 7–8.

Nadia makes the diagram below to represent the counter space she wants to build in her craft room.
Go Math Grade 4 Answer Key Chapter 13 Algebra Perimeter and Area Common Core - New img 39

Question 7.
What is the area of the space that Nadia has shown for scrapbooking?
______ square feet

Answer: 52 square feet

Explanation:

Length = 13 feet
Width = 9 feet – 5 feet = 4 feet
Area of scrapbooking = l × w
= 13 feet × 4 feet
= 52 square feet
Therefore the area of the space that Nadia has shown for scrapbooking is 52 square feet.

Question 8.
What is the area of the space she has shown for painting?
______ square feet

Answer: 25 square feet

Explanation:
The space for painting is a square.
Side of the square is 5 feet
Area of the square = 5 feet × 5 feet
= 25 square feet
Thus the area of the space she has shown for painting is 25 square feet.

Common Core – New – Page No. 734

Lesson Check

Question 1.
What is the area of the combined rectangles below?
Go Math Grade 4 Answer Key Chapter 13 Algebra Perimeter and Area Common Core - New img 40
Options:
a. 136 square yards
b. 100 square yards
c. 76 square yards
d. 64 square yards

Answer: 76 square yards

Explanation:
Area of 1st rectangle = 5 yards × 8 yards = 40 square yards
Area of 2nd rectangle = 12 yards × 3 yards = 36 square yards
Area of the figure = Area of 1st rectangle + Area of 2nd rectangle
Area of the figure = 40 square yards + 36 square yards
Therefore, the Area of the figure is 76 square yards.
So, the correct answer is option C.

Question 2.
Marquis is redecorating his bedroom. What could Marquis use the area formula to find?
Options:
a. how much space should be in a storage box
b. what length of wood is needed for a shelf
c. the amount of paint needed to cover a wall
d. how much water will fill up his new aquarium

Answer: the amount of paint needed to cover a wall

Spiral Review

Question 3.
Giraffes are the tallest land animals. A male giraffe can grow as tall as 6 yards. How tall would the giraffe be in feet?
Options:
a. 2 feet
b. 6 feet
c. 12 feet
d. 18 feet

Answer: 18 feet

Explanation:
Giraffes are the tallest land animals. A male giraffe can grow as tall as 6 yards.
6 yards + 6 yards + 6 yards = 18 yards
The correct answer is option D.

Question 4.
Drew purchased 3 books for $24. The cost of each book was a multiple of 4. Which of the following could be the prices of the 3 books?
Options:
a. $4, $10, $10
b. $4, $8, $12
c. $5, $8, $11
d. $3, $7, $14

Answer: $4, $8, $12

Explanation:
Given that,
Drew purchased 3 books for $24.
The cost of each book was a multiple of 4.
So, the prices of books will be multiple of 4.
That means $4 × 1, $4 × 2, $4 × 3
=  $4, $8, $12
The correct answer is option B.

Question 5.
Esmeralda has a magnet in the shape of a square. Each side of the magnet is 3 inches long. What is the perimeter of her magnet?
Options:
a. 3 inches
b. 7 inches
c. 9 inches
d. 12 inches

Answer: 12 inches

Explanation:
Esmeralda has a magnet in the shape of a square. Each side of the magnet is 3 inches long.
Side = 3 inches
The perimeter of the square = 4s
P = 4 × 3 = 12 inches
The correct answer is option D.

Question 6.
What is the area of the rectangle below?
Go Math Grade 4 Answer Key Chapter 13 Algebra Perimeter and Area Common Core - New img 41
Options:
a. 63 square feet
b. 32 square feet
c. 18 square feet
d. 16 square feet

Answer: 63 square feet

Explanation:
Area of the rectangle = base × height
Base = 9 feet
Height = 7 feet
A = 9 feet × 7 feet
A = 63 square feet
Thus the correct answer is option A.

Page No. 735

Choose the best term from the box.
Go Math Grade 4 Answer Key Chapter 13 Algebra Perimeter and Area img 42

Question 1.
A square that is 1 unit wide and 1 unit long is a ________.
__________

Answer: Square unit

Question 2.
The _______ of a two-dimensional figure can be any side.
__________

Answer: Base

Question 3.
A set of symbols that expresses a mathematical rule is called a ______.
__________

Answer: Formula

Question 4.
The ______ is the distance around a shape.
__________

Answer: Perimeter

Find the perimeter and area of the rectangle or square.

Question 5.
Go Math Grade 4 Answer Key Chapter 13 Algebra Perimeter and Area img 43
Perimeter = ______ cm
Area = ______ square cm

Answer:
Perimeter = 52 cm
Area = 169 square cm

Explanation:
P = 4s
P = 4 × 13 = 52 cm
A = s × s
A = 13 × 13 = 169 square cm

Question 6.
Go Math Grade 4 Answer Key Chapter 13 Algebra Perimeter and Area img 44
Perimeter = ______ ft
Area = ______ square ft

Answer:
Perimeter: 48 ft
Area = 63 square ft

Explanation:
Base = 21 ft
Height = 3 ft
P = 2l +2w
P = 2 (21 ft + 3 ft)
P = 2 × 24 = 48 feet
A = b × h
A = 21 × 3
A = 63 square ft

Question 7.
Go Math Grade 4 Answer Key Chapter 13 Algebra Perimeter and Area img 45
Perimeter = ______ in.
Area = ______ square in.

Answer:
Perimeter = 46 in.
Area = 120 square in.

Explanation:
P = 2l +2w
P = 2 × 15 + 2 × 8
P = 30 + 16 = 46 inches
A = l × w
A = 15 × 8 = 120 square inches

Question 8.
Go Math Grade 4 Answer Key Chapter 13 Algebra Perimeter and Area img 46
Area = ____ square yd

Answer:
Area of the rectangle = 20 yards × 5 yards = 100 square yards
Area of the rectangle = 18 yards × 5 yards = 90 square yards
Area of the figure = 100 square yards + 90 square yards = 190 square yards

Question 9.
Go Math Grade 4 Answer Key Chapter 13 Algebra Perimeter and Area img 47
Area = ____ square meters

Answer:
A = b × h
A = 5 m × 2 m = 10 square meters
A = b × h
A = 5 m × 2 m = 10 square meters
A = b × h
A = 4 m × 2 m = 8 square meters
Now add all the areas
10 square meters + 10 square meters + 8 square meters
= 28 square meters
Therefore the area of the figures is 28 square meters

Question 10.
Go Math Grade 4 Answer Key Chapter 13 Algebra Perimeter and Area img 48
Area = ____ square feet
Answer:
Area of the rectangle = b × h
A = 14 ft × 2 ft = 28 square feet
A = s × s
A = 8 ft × 8 ft = 64 square feet
Area of the figures = 64 square feet + 28 square feet
Therefore Area of the figure = 92 square feet

Page No. 736

Question 11.
Which figure has the greatest perimeter?
Go Math Grade 4 Answer Key Chapter 13 Algebra Perimeter and Area img 49
Go Math Grade 4 Answer Key Chapter 13 Algebra Perimeter and Area img 50
Go Math Grade 4 Answer Key Chapter 13 Algebra Perimeter and Area img 51
Go Math Grade 4 Answer Key Chapter 13 Algebra Perimeter and Area img 52
________

Answer: Figure B has the highest perimeter.

Explanation:
Go Math Grade 4 Answer Key Chapter 13 Algebra Perimeter and Area img 49
P = 2l +2w
P = 2 × 3 + 2 ×5 = 6 + 10 = 16
Go Math Grade 4 Answer Key Chapter 13 Algebra Perimeter and Area img 51

P = 2 × 6 + 2 × 3 = 12 + 6 = 18
Go Math Grade 4 Answer Key Chapter 13 Algebra Perimeter and Area img 50
P = 4a = 4 × 4 = 16
Go Math Grade 4 Answer Key Chapter 13 Algebra Perimeter and Area img 52
P = 2 × 4 + 2 × 3 = 8+ 6 = 14
Thus the greatest perimeter is figure B.

Question 12.
Which figure has an area of 108 square centimeters?
Go Math Grade 4 Answer Key Chapter 13 Algebra Perimeter and Area img 53
Go Math Grade 4 Answer Key Chapter 13 Algebra Perimeter and Area img 54
Go Math Grade 4 Answer Key Chapter 13 Algebra Perimeter and Area img 55
Go Math Grade 4 Answer Key Chapter 13 Algebra Perimeter and Area img 56
________

Answer: Figure C

Explanation:
Go Math Grade 4 Answer Key Chapter 13 Algebra Perimeter and Area img 53
A = 13 cm × 6 cm = 78 square cm.
Go Math Grade 4 Answer Key Chapter 13 Algebra Perimeter and Area img 55
A = 11 cm × 11 cm = 121 square cm.
Go Math Grade 4 Answer Key Chapter 13 Algebra Perimeter and Area img 54
A = 12 cm × 9 cm = 108 square cm.
Go Math Grade 4 Answer Key Chapter 13 Algebra Perimeter and Area img 56
A = 16 cm × 38 cm = 608 square cm.
Thus the area of 108 square centimeters is Figure C.

Question 13.
Which of the combined rectangles has an area of 40 square feet?
Go Math Grade 4 Answer Key Chapter 13 Algebra Perimeter and Area img 57
Go Math Grade 4 Answer Key Chapter 13 Algebra Perimeter and Area img 58
Go Math Grade 4 Answer Key Chapter 13 Algebra Perimeter and Area img 59
Go Math Grade 4 Answer Key Chapter 13 Algebra Perimeter and Area img 60
________

Answer: Figure A

Explanation:
Go Math Grade 4 Answer Key Chapter 13 Algebra Perimeter and Area img 57
Area of top rectangle = 6 ft × 2 ft = 12 square feet
Area of bottom rectangle = 6 ft × 2 ft = 12 square feet
Area of square = 4 ft × 4 ft = 16 square feet
Add Area of top rectangle, Area of bottom rectangle and Area of square
= 12 square feet +  12 square feet + 16 square feet = 40 square feet.
Thus the correct answer is option A.

Page No. 739

Question 1.
Find the unknown measure. The area of the rectangle is 36 square feet.
Go Math Grade 4 Answer Key Chapter 13 Algebra Perimeter and Area img 61
A = b × h
The base of the rectangle is ________ .
base = _____ ft

Answer: 12 feet

Explanation:
Given,
The area of the rectangle = 36 square feet
Height = 3 feet
Base =?
A = b × h
36 square feet = b × 3 feet
b × 3 feet = 36 square feet
b = 36/3 = 12 feet
The base of the rectangle is 12 feets.

Find the unknown measure of the rectangle.

Question 2.
Go Math Grade 4 Answer Key Chapter 13 Algebra Perimeter and Area img 62
Perimeter = 44 centimeters
width = _____ cm

Answer: 10 cm

Explanation:
Given,
Perimeter = 44 centimeters
Length = 12 cm
width =?
The perimeter of the rectangle = 2 (l + w)
P = 2l + 2w
44 cm = 24 cm + 2w
2w = 44 cm – 24 cm
2w = 20 cm
w = 20/2 = 10
Therefore width = 10 cm

Question 3.
Go Math Grade 4 Answer Key Chapter 13 Algebra Perimeter and Area img 63
Area = 108 square inches
height = _____ in.

Answer: 12 inches

Explanation:
Given,
Area = 108 square inches
Base = 9 inches
height = _____ in.
A = b × h
108 square inches = 9 inches × h
h = 108/9
Height = 12 inches
Therefore the height of the rectangle = 12 inches

Question 4.
Go Math Grade 4 Answer Key Chapter 13 Algebra Perimeter and Area img 64
Area = 90 square meters
base = _____ cm

Answer: 18 meters

Explanation:
Given,
Area = 90 square meters
Height = 5 meters
base = _____ cm
A = b × h
90 square meters = b × 5 meters
b × 5 meters = 90 square meters
b = 90/5 = 18 meters
Therefore the base of the rectangle = 18 meters

Question 5.
Go Math Grade 4 Answer Key Chapter 13 Algebra Perimeter and Area img 65
Perimeter = 34 yards
length = _____ yd

Answer: 12 yards

Explanation:
Given,
Perimeter = 34 yards
Width = 5 yards
Length =?
The perimeter of the rectangle = 2 (l + w)
P = 2l + 2w
34 yards = 2 × l + 2 × 5 yards
34 yards = 2 × l + 10 yards
2 × l + 10 yards = 34 yards
2l = 34 yards – 10 yards
2l = 24 yards
l = 24/2 = 12 yards
Therefore the length of the rectangle = 12 yards.

Question 6.
Go Math Grade 4 Answer Key Chapter 13 Algebra Perimeter and Area img 66
Area = 96 square feet
base = ______ ft

Answer: 12 feet

Explanation:
Given,
Area = 96 square feet
Height = 8 feet
Base =?
A = b × h
96 square feet = b × 8 feet
b × 8 feet = 96 square feet
b = 96/8 = 12 feet
Thus base of the rectangle = 12 feet.

Question 7.
Go Math Grade 4 Answer Key Chapter 13 Algebra Perimeter and Area img 67
Area = 126 square centimeters
height = _____ centimeters

Answer: 14 centimeters

Explanation:
Given,
Area = 126 square centimeters
Base = 9 cm
height = _____ centimeters
A = b × h
126 square centimeters = 9 cm × h
9 cm × h = 126 square centimeters
h = 126/9 = 14 centimeters
Therefore the Height of the rectangle = 14 centimeters

Question 8.
A square has an area of 49 square inches. Explain how to find the perimeter of the square.
Type below:
________

Answer:

Explanation:
Given that,
A square has an area of 49 square inches.
A = 49 square inches
s^2 = 49 square inches
The square root of 49 is 7
So, each side of the square is 7 inches
The perimeter of the square = 4 × s
4 × 7 inches = 28 inches.
Therefore the perimeter of the square is 28 inches.

Page No. 740

Question 9.
Identify Relationships The area of a swimming pool is 120 square meters. The width of the pool is 8 meters. What is the length of the pool in centimeters?
length = _____ centimeters

Answer:
Given that the area of a swimming pool is 120 square meters.
The width of the pool is 8 meters.
We have to find the length of the pool in centimeters.
We know that Area of the rectangle = l × w
A = l × w
120 square meters = l × 8 meters
l × 8 meters = 120 square meters
l = 120/8 = 15 meters
Therefore, the length of the pool = 15 meters
Convert meters to centimeters
1 meter = 100 centimeters
15 meters = 1500 centimeters.
The length of the pool in centimeters = 1500 centimeters

Question 10.
An outdoor deck is 7 feet wide. The perimeter of the deck is 64 feet. What is the length of the deck? Use the numbers to write an equation and solve. A number may be used more than once.
Go Math Grade 4 Answer Key Chapter 13 Algebra Perimeter and Area img 68
P=(2 × l) + (2 × w)
So, the length of the deck is _______ feet.
length = _____ ft

Answer:
An outdoor deck is 7 feet wide.
The perimeter of the deck is 64 feet.
We know that,
P=(2 × l) + (2 × w)
64 feet = (2 × l) + (2 × 7)
64 feet = 2l + 14 feet
2 × l = 64 feet – 14 feet
2 × l = 50 feet
l = 50/2 = 25 feet
Therefore the length of the deck = 25 feet.

Question 11.
A male mountain lion has a rectangular territory with an area of 96 square miles. If his territory is 8 miles wide, what is the length of his territory?
Go Math Grade 4 Answer Key Chapter 13 Algebra Perimeter and Area img 69
length = _____ miles

Answer:
A male mountain lion has a rectangular territory with an area of 96 square miles.
Width = 8 miles
Length =?
A = l × w
96 square miles = l × 8 miles
l × 8 miles = 96 square miles
l = 96/8
l = 12 miles
Therefore, length of his territory = 12 miles

Common Core – New – Page No. 741

Find Unknown Measures

Find the unknown measure of the rectangle.

Question 1.
Go Math Grade 4 Answer Key Chapter 13 Algebra Perimeter and Area Common Core - New img 70
Perimeter = 54 feet
width = 7 feet
Think: P = (2 × l) + (2 × w)
54 = (2 × 20) + (2 × w)
54 = 40 + (2 × w)
Since 54 = 40 + 14, 2 × w = 14, and w = 7.

Question 2.
Go Math Grade 4 Answer Key Chapter 13 Algebra Perimeter and Area Common Core - New img 71
Perimeter = 42 meters
length = _____ meters

Answer: length = 12 meters

Explanation:

Given, Perimeter = 42 meters
Width = 9 meters
P = (2 × l) + (2 × w)
P = (2 × l) + (2 × 9 m)
42 m = 2l + 18 m
42 m – 18 m = 2l
2l = 24 meters
l = 24 meters/2 = 12 meters
Therefore length = 12 meters

Question 3.
Go Math Grade 4 Answer Key Chapter 13 Algebra Perimeter and Area Common Core - New img 72
Area = 28 square centimeters
height = _____ centimeters

Answer: height = 7 centimeters

Explanation:

Given,
Area = 28 square centimeters
Base = 4 cm
A = b × h
28 square centimeters = 4 cm × h
4 × h = 28
h = 28/4 = 7 cm
The height of the rectangle = 7 centimeters

Question 4.
Go Math Grade 4 Answer Key Chapter 13 Algebra Perimeter and Area Common Core - New img 73
Area = 200 square inches
base = _____ inches

Answer: base = 8 inches

Explanation:

Given,
Area = 200 square inches
Height = 25 inches
Base = ?
Area of the rectangle = b × h
200 square inches = b × 25 inches
b × 25 inches = 200 square inches
b = 200/25 = 8 inches
The base of the rectangle = 8 inches.

Problem Solving

Question 5.
Susie is an organic vegetable grower. The perimeter of her rectangular vegetable garden is 72 yards. The width of the vegetable garden is 9 yards. How long is the vegetable garden?
length = _____ yards

Answer: 27 yards

Explanation:

Susie is an organic vegetable grower.
The perimeter of her rectangular vegetable garden is 72 yards.
The width of the vegetable garden is 9 yards.
P = 72 yards
W = 9 yards
L =?
We know that,
P = (2 × l) + (2 × w)
72 yards = (2 × l) + (2 × 9)
72 yards – 18 yards = (2 × l)
(2 × l) = 72 yards – 18 yards
2l = 54 yards
l = 54/2 = 27 yards
Thus the vegetable garden is 27 yards long.

Question 6.
An artist is creating a rectangular mural for the Northfield Community Center. The mural is 7 feet tall and has an area of 84 square feet. What is the length of
the mural?
length = _____ feet

Answer: 12 feet

Explanation:

An artist is creating a rectangular mural for the Northfield Community Center.
The mural is 7 feet tall and has an area of 84 square feet.
A = 84 square feet
W = 7 feet
L =?
A = l × w
84 square feet = l × 7 feet
l × 7 feet = 84 square feet
l = 84/7 = 12 feet
Thus the length of Murali is 12 feet.

Common Core – New – Page No. 742

Lesson Check

Question 1.
The area of a rectangular photograph is 35 square inches. If the width of the photo is 5 inches, how tall is the photo?
Options:
a. 5 inches
b. 7 inches
c. 25 inches
d. 30 inches

Answer: 7 inches

Explanation:

The area of a rectangular photograph is 35 square inches.
Width = 5 inches
A = l × w
35 square inches = l × 5 inches
Length = 35/5 = inches
Thus the photo is 7 inches tall.
The correct answer is option B.

Question 2.
Natalie used 112 inches of blue yarn as a border around her rectangular bulletin board. If the bulletin board is 36 inches wide, how long is it?
Options:
a. 20 inches
b. 38 inches
c. 40 inches
d. 76 inches

Answer: 20 inches

Explanation:

Natalie used 112 inches of blue yarn as a border around her rectangular bulletin board.
Width = 36 inches
A = 112 inches
A = l × w
112 inches = l × 36 inches
l × 36 inches = 112 inches
l = 112/36 = 20 inches
Length = 20 inches
The correct answer is option A.

Spiral Review

Question 3.
A professional basketball court is in the shape of a rectangle. It is 50 feet wide and 94 feet long. A player ran one time around the edge of the court. How far did the player run?
Options:
a. 144 feet
b. 194 feet
c. 238 feet
d. 288 feet

Answer: 288 feet

Explanation:

A professional basketball court is in the shape of a rectangle.
It is 50 feet wide and 94 feet long.
A player ran one time around the edge of the court.
P = (2 × l) + (2 × w)
P = (2 × 94 feet) + (2 × 50 feet)
P = 188 feet + 100 feet = 288 feet
Therefore the perimeter of the rectangle is 288 feet.

Question 4.
On a compass, due east is a \(\frac{1}{4}\) turn clockwise from due north. How many degrees are in a \(\frac{1}{4}\) turn?
Options:
a. 45°
b. 60°
c. 90°
d. 180°

Answer: 90°

Explanation:

On a compass, due east is a \(\frac{1}{4}\) turn clockwise from due north.
\(\frac{1}{4}\) × 360° = 360°/4 = 90°
The correct answer is option C.

Question 5.
Hakeem’s frog made three quick jumps. The first was 1 meter. The second jump was 85 centimeters. The third jump was 400 millimeters. What was the total length of the frog’s three jumps?
Options:
a. 189 centimeters
b. 225 centimeters
c. 486 centimeters
d. 585 millimeters

Answer: 225 centimeters

Explanation:

Hakeem’s frog made three quick jumps.
The first was 1 meter. The second jump was 85 centimeters. The third jump was 400 millimeters.
Convert other units to centimeters
1 meter = 100 centimeters
400 millimeters = 40 centimeters
100 + 85 + 40 = 225 centimeters
Thus the correct answer is option B.

Question 6.
Karen colors in squares on a grid. She colored \(\frac{1}{8}\) of the squares blue and \(\frac{5}{8}\) of the squares red. What fraction of the squares are not colored in?
Options:
a. \(\frac{1}{8}\)
b. \(\frac{1}{4}\)
c. \(\frac{1}{2}\)
d. \(\frac{3}{4}\)

Answer: \(\frac{1}{4}\)

Explanation:

Karen colors in squares on a grid.
She colored \(\frac{1}{8}\) of the squares blue and \(\frac{5}{8}\) of the squares red.
\(\frac{1}{8}\) + \(\frac{5}{8}\) = \(\frac{6}{8}\)
Total number of fractions = \(\frac{8}{8}\)
\(\frac{8}{8}\) – \(\frac{6}{8}\) = \(\frac{2}{8}\)
\(\frac{1}{4}\) fraction of the squares are not colored.

Page No. 745

Question 1.
Lila is wallpapering one wall of her bedroom, as shown in the diagram. She will cover the whole wall except for the doorway. How many square feet of wall does Lila need to cover?
Go Math Grade 4 Answer Key Chapter 13 Algebra Perimeter and Area img 74
First, find the area of the wall.
A = b × h
Awall = _____ square feet

Answer:
Base = 12 feet
Height = 8 feet
A = b × h
Awall = 12 feet × 8 feet
Awall = 96 square feet

Question 1.
Next, find the area of the door.
A = b × h
Adoor = _____ square feet

Answer:
Base = 3 feet
Height = 7 feet
A = b × h
Adoor = 3 feet × 7 feet
Adoor = 21 square feet

Question 1.
Last, subtract the area of the door from the area of the wall.
_____ – _____ = _____ square feet
So, Lila needs to cover _____ of wall.
Type below:
________

Answer:
Adoor = 21 square feet
Awall = 96 square feet
Last, subtract the area of the door from the area of the wall.
A = Awall – Adoor
A = 96 square feet – 21 square feet
A = 75 square feet
So, Lila needs to cover 75 square feet

Question 2.
What if there was a square window on the wall with a side length of 2 feet? How much wall would Lila need to cover then? Explain.
______ square feet

Answer:
If there is a square window of length 2 feet
Area of square = s × s
Awindow = 2 × 2 = 4 square feet
Now Subtract the area of the door, area of the window from the area of the wall.
A = 96 square feet – 21 square feet – 4 square feet
A = 71 square feet
Therefore Lila need to cover 71 square feet.

Question 3.
Ed is building a model of a house with a flat roof, as shown in the diagram. There is a chimney through the roof. Ed will cover the roof with square tiles. If the area of each tile is 1 square inch, how many tiles will he need? Explain.
Go Math Grade 4 Answer Key Chapter 13 Algebra Perimeter and Area img 75
_____ tiles

Answer:
Roof:
Base = 20 inches
Height = 30 inches
Area of the roof = b × h
Aroof = 20 inches × 30 inches
Aroof = 600 inches
Chimney:
Base = 3 inches
Height = 4 inches
Area of the chimney = b × h
Achimney = 3 × 4 = 12 inches
Now subtract Area of Chimney from Area of the roof
A = 600 inches – 12 inches
A = 588 inches
Therefore Ed needs 588 tiles.

Page No. 746

Question 4.
Make Sense of Problems Lia has a dog and a cat. Together, the pets weigh 28 pounds. The dog weighs 3 times as much as the cat. How much does each pet weigh?
cat weight = _____  pounds dog weight = _____ pounds

Answer:
Given that, the pets weigh 28 pounds.
28 = 7 + 7 + 7 + 7
The dog weighs 3 times as much as the cat.
= 3 × 7 = 21 pounds
The dog weighs 21 pounds
28 – 21 = 7
The cat weighs = 7 pounds.

Question 5.
Mr. Foster is covering two rectangular pictures with glass. One is 6 inches by 4 inches and the other one is 5 inches by 5 inches. Does he need the same number of square inches of glass for each picture? Explain.
_____

Answer: No

Explanation:
Mr. Foster is covering two rectangular pictures with glass.
One is 6 inches by 4 inches and the other one is 5 inches by 5 inches.
Area of first rectangular picture = 6 × 4 = 24 square inches
Area of second rectangular picture = 5 × 5 = 25 square inches
Area of two rectangular pictures = 25 square inches – 24 square inches
1 square inch.
Therefore, he doesn’t need the same number of square inches of glass for each picture.

Question 6.
Claire says the area of a square with a side length of 100 centimeters is greater than the area of a square with a side length of 1 meter. Is she correct? Explain.
_____

Answer: No

Explanation:
Claire says the area of a square with a side length of 100 centimeters is greater than the area of a square with a side length of 1 meter.
Her statement is not correct because 1 meter = 100 centimeters.
So, the area of a square with a side length of 100 centimeters is equal to the area of a square with a side length of 1 meter.

Question 7.
A rectangular floor is 12 feet long and 11 feet wide. Janine places a rug that is 9 feet long and 7 feet wide and covers part of the floor in the room. Select the word(s) to complete the sentence.
To find the number of square feet of the floor that is NOT covered by the rug,
Go Math Grade 4 Answer Key Chapter 13 Algebra Perimeter and Area img 76 the Go Math Grade 4 Answer Key Chapter 13 Algebra Perimeter and Area img 77 Go Math Grade 4 Answer Key Chapter 13 Algebra Perimeter and Area img 78 the area of the floor.
_____ square feet

Answer:
Length = 12 feet
Width = 11 feet
Area of the rectangular floor = l × w
= 12 feet × 11 feet = 132 square feet
Room:
Length = 9 feet
Width = 7 feet
Area of the floor in the room = l × w
= 9 feet × 7 feet
= 63 square feet
Subtract the area of the rug from the area of the floor
= 132 square feet – 63 square feet = 69 square feet
The number of square feet of the floor that is NOT covered by the rug is 69 square feet.

Common Core – New – Page No. 747

Problem Solving Find the Area

Solve each problem.

Question 1.
A room has a wooden floor. There is a rug in the center of the floor. The diagram shows the room and the rug. How many square feet of the wood floor still shows?
Go Math Grade 4 Answer Key Chapter 13 Algebra Perimeter and Area Common Core - New img 79
82 square feet
Area of the floor: 13 × 10 = 130 square feet
Area of the rug: 8 × 6 = 48 square feet
Subtract to find the area of the floor still showing: 130 – 48 = 82 square feet

Question 2.
A rectangular wall has a square window, as shown in the diagram.
Go Math Grade 4 Answer Key Chapter 13 Algebra Perimeter and Area Common Core - New img 80
What is the area of the wall NOT including the window?
The area of the wall NOT including the window = _____ square feet

Answer: 96 square feet

Explanation:
Wall:
Base = 14 feet
Height = 8 feet
Area of the wall = b × h
A = 14 feet × 8 feet
A = 112 square feet
Window:
Length = 4 feet
Area of the square = s × s
Area of the window = 4 feet × 4 feet = 16 square feet
Now subtract Area of the window from the area of the rectangular wall
= 112 square feet – 16 square feet
= 96 square feet
Therefore the area of the wall NOT including the window = 96 square feet.

Question 3.
Bob wants to put down new sod in his backyard, except for the part set aside for his flower garden. The diagram shows Bob’s backyard and the flower garden.
Go Math Grade 4 Answer Key Chapter 13 Algebra Perimeter and Area Common Core - New img 81
How much sod will Bob need?
The area covered with new sod = _____ square yards

Answer: 235 square yards

Flower Garden:
Base = 20 yards
Height = 14 yards
Area of the rectangular flower garden = b × h
A = 20 yards × 14 yards
A = 280 square yards
Sod:
Base = 5 yards
Height = 9 yards
Area of sod = b × h
= 5 yards × 9 yards = 45 square yards
Now subtract area of sod from area of flower garden
= 280 square yards – 45 square yards
= 235 square yards
Thus the area covered with new sod = 235 square yards

Question 4.
A rectangular painting is 24 inches wide and 20 inches tall without the frame. With the frame, it is 28 inches wide and 24 inches tall. What is the area of the frame not covered by the painting?
The area of the frame = _____ square inches

Answer: 192 square inches

Explanation:
A rectangular painting is 24 inches wide and 20 inches tall without the frame.
A = b × h
A = 24 inches × 20 inches
A = 480 square inches
With the frame, it is 28 inches wide and 24 inches tall.
A = b × h
A = 28 inches × 24 inches
A = 672 square inches
The area of the frame not covered by the painting
= 672 square inches – 480 square inches
= 192 square inches
Therefore, The area of the frame = 192 square inches

Question 5.
One wall in Jeanne’s bedroom is 13 feet long and 8 feet tall. There is a door 3 feet wide and 6 feet tall. She has a poster on the wall that is 2 feet wide and 3 feet tall. How much of the wall is visible?
The area of the wall visible = _____ square feet

Answer: 80 square feet

Explanation:
One wall in Jeanne’s bedroom is 13 feet long and 8 feet tall.
Area of Jeanne’s bedroom = 13 feet × 8 feet = 104 square feet
Area of door = 3 feet × 6 feet = 18 square feet
Area of the wall = 2 feet × 3 feet = 6 square feet
To find the area of the wall visible we have to subtract Area of the wall, Area of the door from Area of Jeanne’s bedroom.
104 square feet – 18 square feet – 6 square feet
= 80 square feet
The area of the wall visible = 80 square feet

Common Core – New – Page No. 748

Lesson Check

Question 1.
One wall in Zoe’s bedroom is 5 feet wide and 8 feet tall. Zoe puts up a poster of her favorite athlete. The poster is 2 feet wide and 3 feet tall. How much of the wall is not covered by the poster?
Options:
a. 16 square feet
b. 34 square feet
c. 35 square feet
d. 46 square feet

Answer: 34 square feet

Explanation:
One wall in Zoe’s bedroom is 5 feet wide and 8 feet tall.
Area of the wall in Zoe’s bedroom = b × h
A = 5 feet × 8 feet
A = 40 square feet
Zoe puts up a poster of her favorite athlete. The poster is 2 feet wide and 3 feet tall.
Area of the poster = b × h
A = 2 feet × 3 feet = 6 square feet
Now subtract Area of the poster from the Area of the wall in Zoe’s bedroom
= 40 square feet – 6 square feet
= 34 square feet
Thus the area of the wall is not covered by the poster = 34 square feet.
The correct answer is option B.

Question 2.
A garage door is 15 feet wide and 6 feet high. It is painted white, except for a rectangular panel 1 foot high and 9 feet wide that is brown. How much of the garage door is white?
Options:
a. 22 square feet
b. 70 square feet
c. 80 square feet
d. 81 square feet

Answer: 81 square feet

Explanation:
A garage door is 15 feet wide and 6 feet high.
Area of the garage door = b × h
A = 15 feet × 6 feet
A = 90 square feet
It is painted white, except for a rectangular panel 1 foot high and 9 feet wide that is brown.
b = 9 feet
h = 1 foot
A = b × h
A = 9 feet × 1 feet
A = 9 square feet
Area of the garage door is white = 90 square feet – 9 square feet
Area of the garage door is white = 81 square feet
The correct answer is option D.

Spiral Review

Question 3.
Kate baked a rectangular cake for a party. She used 42 inches of frosting around the edges of the cake. If the cake was 9 inches wide, how long was the cake?
Options:
a. 5 inches
b. 12 inches
c. 24 inches
d. 33 inches

Answer: 12 inches

Explanation:
Kate baked a rectangular cake for a party. She used 42 inches of frosting around the edges of the cake.
Width = 9 inches
P = (2 × l) + (2 × w)
42 inches = (2 × l) + (2 × 9)
(2 × l) + (2 × 9) = 42 inches
(2 × l) = 42 inches – 18 inches
2l = 24 inches
l = 24/2 = 12 inches
Therefore the cake is 12 inches long.
Thus the correct answer is option B.

Question 4.
Larry, Mary, and Terry each had a full glass of juice. Larry drank \(\frac{3}{4}\) of his. Mary drank \(\frac{3}{8}\) of hers. Terry drank \(\frac{7}{10}\) of his. Who drank less than \(\frac{1}{2}\) of their juice?
Options:
a. Larry
b. Mary
c. Mary and Terry
d. Larry and Terry

Answer: Mary

Explanation:
Larry, Mary, and Terry each had a full glass of juice.
Larry drank \(\frac{3}{4}\), Mary drank \(\frac{3}{8}\) and Terry drank \(\frac{7}{10}\) of \(\frac{1}{2}\)
\(\frac{3}{8}\) is less than \(\frac{1}{2}\) of their juice.
The correct answer is Option B.

Question 5.
Which of the following statements is NOT true about the numbers 7 and 9?
Options:
a. 7 is a prime number.
b. 9 is a composite number.
c. 7 and 9 have no common factors other than 1.
d. 27 is a common multiple of 7 and 9.

Answer: 27 is a common multiple of 7 and 9

Explanation:
a. 7 is a prime number is true.
b. 9 is a composite number is true
c. 7 and 9 have no common factors other than 1 is true.
d. 27 is a common multiple of 7 and 9 is not true because 7 is not the multiple of 27.
Thus the correct answer is option D.

Question 6.
Tom and some friends went to a movie. The show started at 2:30 P.M. and ended at 4:15 P.M. How long did the movie last?
Options:
a. 1 hour 35 minutes
b. 1 hour 45 minutes
c. 1 hour 55 minutes
d. 2 hours 15 minutes

Answer: 1 hour 45 minutes

Explanation:
Tom and some friends went to a movie. The show started at 2:30 P.M. and ended at 4:15 P.M.
Subtract 2:30 P.M. from 4:15 P.M.
4 hour 15 minutes
-2 hour 30 minutes
1 hour 45 minutes
The movie last for 1 hour 45 minutes
Thus the correct answer is option B.

Page No. 749

Question 1.
For numbers 1a–1e, select Yes or No to indicate if a rectangle with the given dimensions would have a perimeter of 50 inches.
a. length: 25 inches; width: 2 inches
i. yes
ii. no

Answer: No

Explanation:
P = (2 × l) + (2 × w)
50 inches = (2 × 25 in.) + (2 × w)
(2 × w) = 50 inches – 50 inches
w = 0
Thus the above statement is false

Question 1.
b. length: 20 inches; width: 5 inches
i. yes
ii. no

Answer: Yes

Explanation:
P = (2 × l) + (2 × w)
50 inches = (2 × 20 in.) + (2 × 5)
50 inches = 40 in. + 10 in.
Thus the above statement is true.

Question 1.
c. length: 17 inches; width: 8 inches
i. yes
ii. no

Answer: Yes

Explanation:
P = (2 × l) + (2 × w)
50 inches = (2 × 17 in.) + (2 × 8 in.)
50 inches = 34 in. + 16 in.
Thus the above statement is true.

Question 1.
d. length: 15 inches; width: 5 inches
i. yes
ii. no

Answer: No

Explanation:
P = (2 × l) + (2 × w)
50 inches = (2 × 15 in.) + (2 × 5 in.)
50 inches = 30 in. + 10 in.
50 inches = 40 inches
Thus the above statement is false.

Question 1.
e. length: 15 inches; width: 10 inches
i. yes
ii. no

Answer: Yes

Explanation:
P = (2 × l) + (2 × w)
50 inches = (2 × 15 in.) + (2 × 10 in.)
50 inches = 30 in. + 20 in.
50 inches = 50 inches
Thus the above statement is true.

Question 2.
The swimming club’s indoor pool is in a rectangular building.
Marco is laying tile around the rectangular pool.
Go Math Grade 4 Answer Key Chapter 13 Algebra Perimeter and Area img 82
Part A
What is the area of the pool and the area of the pool and the walkway? Show your work.
A(pool) = ____ m2    A(building) = ____ m2

Answer:
Pool:
Base = 20 m
Height = 16 m
A = b × h
Area of the pool = 20 m × 16 m = 320 square meters
Pool and the walkway:
Area of the pool and the walkway = 26 m × 22 m = 572 square meters

Question 2.
Part B
How many square meters of tile will Marco need for the walkway?
Explain how you found your answer.
A(walkway) = ____ m2

Answer: 252 square meters

Explanation:
Area of walkway = Area of the pool and the walkway – Area of pool
Area of the walkway = 572 square meters – 320 square meters
= 252 square meters
Therefore the Area of walkway = 252 square meters

Page No. 750

Question 3.
Match the dimensions of the rectangles in the top row with the correct area or perimeter in the bottom row
Go Math Grade 4 Answer Key Chapter 13 Algebra Perimeter and Area img 83

Answer:

Go-Math-Grade-4-Answer-Key-Chapter-13-Algebra-Perimeter-and-Area-img-83

Question 4.
Kyleigh put a large rectangular sticker on her notebook. The height of the sticker measures 18 centimeters. The base is half as long as the height. What area of the notebook does the sticker cover?
________ square centimeters

Answer: 162 square centimeters

Explanation:
Kyleigh put a large rectangular sticker on her notebook.
The height of the sticker measures 18 centimeters.
The base is half as long as the height.
Base = h/2 = 18/2 = 9 centimeters
Area of the rectangle = b × h
A = 9 cm × 18 cm
A = 162 square centimeters
Thus the area of the notebook the sticker cover is 162 square centimeters.

Question 5.
A rectangular flower garden in Samantha’s backyard has 100 feet around its edge. The width of the garden is 20 feet. What is the length of the garden? Use the numbers to write an equation and solve. A number may be used more than once.
Go Math Grade 4 Answer Key Chapter 13 Algebra Perimeter and Area img 84
□ = (2 × l) + (2 × □)
□ = 2 × l + □
□ = 2 × l
□ = l
So, the length of the garden _____ feet.

Answer:
P = (2 × l) + (2 × w)
100 = (2 × l) + (2 × 20)
100 – 40 = 2 × l
2 × l = 60
l = 60/2 = 30 feet
Length = 30 feet
So, the length of the garden 30 feet.

Question 6.
Gary drew a rectangle with a perimeter of 20 inches. Then he tried to draw a square with a perimeter of 20 inches.
Draw 3 different rectangles that Gary could have drawn. Then draw the square, if possible.
Type below:
__________

Answer:
The possible rectangles with a perimeter of 20 inches are:
Go Math Grade 4 Chapter 13 Answer Key review solution image-1HMH Grade 4 Go Math Answer Key review solution image-2Go Math 4th Grade Solution Key Review solution image-3
The possible square with a perimeter of 20 inches is:
Go Math Grade 4 Chapter 13 solution key review solution image-4

Page No. 751

Question 7.
Ami and Bert are drawing plans for rectangular vegetable gardens. In Ami’s plan, the garden is 13 feet by 10 feet. In Bert’s plan, the garden is 12 feet by 12 feet. For numbers 7a−7d, select True or False for each statement.
a. The area of Ami’s garden is 130 square feet.
i. True
ii. False

Answer: True

Explanation:
A = b × h
Area of Ami’s garden = 13 feet × 10 feet =
Area of Ami’s garden = 130 square feet
The above statement is true.

Question 7.
b. The area of Bert’s garden is 48 square feet.
i. True
ii. False

Answer: False

Explanation:
Area of Bert’s garden = 12 feet × 12 feet = 144 square feet
The above statement is false.

Question 7.
c. Ami’s garden has a greater area than Bert’s garden.
i. True
ii. False

Answer: False

Explanation:
Area of Ami’s garden = 13 feet × 10 feet = 130 square feet
Area of Bert’s garden = 12 feet × 12 feet = 144 square feet
130 square feet is less than 144 square feet
The area of Ami’s garden is less than Area of Bert’s garden.
The above statement is false.

Question 7.
d. The area of Bert’s garden is 14 square feet greater than Ami’s.
i. True
ii. False

Answer: True

Explanation:
Area of Ami’s garden = 13 feet × 10 feet = 130 square feet
Area of Bert’s garden = 12 feet × 12 feet = 144 square feet
144 square feet – 130 square feet = 14 square feet
The above statement is true.

Question 8.
A farmer planted corn in a square field. One side of the field measures 32 yards. What is the area of the cornfield? Show your work.
_______ square yards

Answer: 1024 square yards

Explanation:
A farmer planted corn in a square field. One side of the field measures 32 yards.
Area of the square = 32 yards × 32 yards
A = 1,024 square yards
Therefore the area of the cornfield is 1,024 square yards.

Question 9.
Harvey bought a frame in which he put his family’s picture.
Go Math Grade 4 Answer Key Chapter 13 Algebra Perimeter and Area img 85
What is the area of the frame not covered by the picture?
_______ square inches

Answer: 136 square inches

Explanation:
Area of the picture = 12 in. × 18 in.
A = 216 square inches
Area of the frame = 16 in. × 22 in.
A = 352 square inches
The area of the frame not covered by the picture = 352 square inches – 216 square inches
= 136 square inches
Therefore the area of the frame not covered by the picture is 136 square inches.

Question 10.
Kelly has 236 feet of fence to use to enclose a rectangular space for her dog. She wants the width to be 23 feet. Draw a rectangle that could be the space for Kelly’s dog. Label the length and width.
Type below:
________

Answer:

Kelly has 236 feet of fence to use to enclose a rectangular space for her dog.
She wants the width to be 23 feet.
Perimeter = (2 × l) + (2 × w)
236 = (2 × l) + (2 × w)
236 = (2 × l) + (2 × 23)
236 – 46 = (2 × l)
(2 × l) = 190HMH Go Math Answer Key Grade 4 Chapter 13 review solution img- 5
l = 190/2
l = 95 feet
Therefore length = 95 feet.

Page No. 752

Question 11.
The diagram shows the dimensions of a new parking lot at Helen’s Health Food store.
Go Math Grade 4 Answer Key Chapter 13 Algebra Perimeter and Area img 86
Use either addition or subtraction to find the area of the parking lot. Show your work.
_______ square yards

Answer: 1100 square yards

Explanation:
Addition:
Top:
Base = 40 yards
Height = 20 yards
Area of the top rectangle = b × h
A = 40 yards × 20 yards = 800 square yards
Bottom:
Base = 30 yards
Height = 10 yards
Area of the rectangle = b × h
A = 30 yards × 10 yards = 300 square yards
Area of the parking = Area of top + Area of bottom
A = 800 square yards + 300 square yards
Area of parking = 1100 square yards.

Question 12.
Chad’s bedroom floor is 12 feet long and 10 feet wide. He has an area rug on his floor that is 7 feet long and 5 feet wide. Which statement tells how to find the amount of the floor that is not covered by the rug? Mark all that apply.
Options:
a. Add 12 × 10 and 7 × 5.
b. Subtract 35 from 12 × 10
c. Subtract 10 × 5 from 12 × 7.
d. Add 12 + 10 + 7 + 5.
e. Subtract 7 × 5 from 12 × 10.
f. Subtract 12 × 10 from 7 × 5.

Answer: B, F

Chad’s bedroom floor is 12 feet long and 10 feet wide.
A = 12 feet × 10 feet = 120 square feet
Area rug on his floor = 7 feet × 5 feet = 35 square feet
To find the amount of the floor that is not covered by the rug we have to subtract 120 square feet from 35 square feet or 35 square feet from 12 × 10.
So, the correct answers are B and F.

Question 13.
A row of plaques covers 120 square feet of space along a wall. If the plaques are 3 feet tall, what length of the wall do they cover?
____ feet

Answer: 40 feet

Explanation:
Given that,
A row of plaques covers 120 square feet of space along a wall.
Height = 3 feet
A = b × h
120 square feet = b × 3 feet
b = 120/3 = 40
Therefore the base is 40 feet.

Page No. 753

Question 14.
Ms. Bennett wants to buy carpeting for her living room and dining room.
Go Math Grade 4 Answer Key Chapter 13 Algebra Perimeter and Area img 87
Explain how she can find the amount of carpet she needs to cover the floor in both rooms. Then find the amount of carpet she will need.
____ square feet

Answer:
She can find the area of each rectangle and then find the sum. The area of the living room is 20 × 20 = 400 square feet.
The area of the dining room is 15 × 10 = 150 square feet.
The sum of the two rooms = 400 + 150 = 550 square feet.
She needs 550 square feet of carpeting.

Question 15.
Lorenzo built a rectangular brick patio. He is putting a stone border around the edge of the patio. The width of the patio is 12 feet. The length of the patio is two feet longer than the width.
How many feet of stone will Lorenzo need? Explain how you found your answer.
____ feet

Answer: 52 feet

Explanation:
Width = 12 feet
Length = 2 × width
Length = 2 + 12 feet = 14 feet
Perimeter = (2 × l) + (2 × w)
P = (2 × 14) + (2 × 12)
P = 28 + 24
P = 52 feet

Page No. 754

Question 16.
Which rectangle has a perimeter of 10 feet? Mark all that apply.
Go Math Grade 4 Answer Key Chapter 13 Algebra Perimeter and Area img 88
Rectangle: ____
Rectangle: ____

Answer: A, C

Explanation:
i. Perimeter of A = (2 × l) + (2 × w)
P = (2 × 1) + (2 × 4) = 2 + 8 = 10 feet
ii. Perimeter of B = (2 × l) + (2 × w)
P = (2 × 2) + (2 × 5) = 4 + 10 = 14 feet
iii. Perimeter of C = (2 × l) + (2 × w)
P = (2 × 2) + (2 × 3) = 4 + 6 = 14 feet
iv. Perimeter of D = (2 × l) + (2 × w)
P = (2 × 4) + (2 × 6) = 8 + 12 = 20 feet
The correct answer is option A and C.

Question 17.
A folder is 11 inches long and 8 inches wide. Alyssa places a sticker that is 2 inches long and 1 inch wide on the notebook. Choose the words that correctly complete the sentence.
To find the number of square inches of the folder that is NOT covered by the sticker,
Go Math Grade 4 Answer Key Chapter 13 Algebra Perimeter and Area img 89
Type below:
________

Answer: Subtract the area of the sticker from the area of the notebook.

Question 18.
Tricia is cutting her initial from a piece of felt. For numbers 18a–18c, select Yes or No to tell whether you can add the products to find the number of square centimeters Tricia needs.
Go Math Grade 4 Answer Key Chapter 13 Algebra Perimeter and Area img 90
a. 1 × 8 and 5 × 2 _______
b. 3 × 5 and 1 × 8 _______
c. 2 × 5 and 1 × 3 and 1 × 3 _______

Answer:
a. 1 × 8 and 5 × 2 _______
Yes
b. 3 × 5 and 1 × 8 _______
No
c. 2 × 5 and 1 × 3 and 1 × 3 _______
No

Question 19.
Mr. Butler posts his students’ artwork on a bulletin board.
Go Math Grade 4 Answer Key Chapter 13 Algebra Perimeter and Area img 91
The width and length of the bulletin board are whole numbers. What could be the dimensions of the bulletin board Mr. Butler uses?
Type below:
________

Answer: 5 feet long by 3 feet wide
Area of the rectangle = l × w
A = 15 square feet
The factor of 15 is 5 and 3.
So, the length = 5 feet long
Width = 3 feet long.

Quick learning is not only important but also understanding is important to learn the concepts. You can’t love maths if you don’t understand the subject. So, to help you guys we have provided the images for your better understanding. Learn the simple techniques to solve the problem in less time in our Go Math Answer Key.

Conclusion:

We wish you all have satisfied with the solutions exists in the Go Math Grade 4 Answer Key Chapter 13 Algebra: Perimeter and Area. For better practice sessions refer to the questions given at the end of the chapter and solve them properly with the help of topic-wise chapter 13 Go Math 4th Grade Answer Key. Practice all problems easily and score well in any standard tests or exams.

Go Math Grade 5 Answer Key Chapter 1 Place Value, Multiplication, and Expressions

go-math-grade-5-chapter-1-place-value-multiplication-and-expressions-answer-key

Help your kid be on the track and ensure him Math Proficiency with our Go Math Grade 5 Answer Key Chapter 1 Place Value, Multiplication, and Expressions. Go Math Grade 5 Chapter 1 Place Value, Multiplication, and Expressions Answer Key enhances your subject knowledge. Use the Best resource Go Math Grade 5 Chapter 1 Answer Key and ace up your preparation. Try to solve as many problems as possible from the Go Math Grade 5 Answer Key and get to know the concepts behind them.

Go Math Grade 5 Answer Key Chapter 1 Place Value, Multiplication, and Expressions

It’s really difficult to find Solutions for all the Problems in Go Math Grade 5 Chapter 1 all in one place. Now, you will no longer have such difficulties. We are providing Go Math Grade 5 Answer Key Chapter 1 Place Value, Multiplication, and Expressions. Check out Step by Step Solutions provided for various lessons and the topics in it. Practice using the 3rd Grade Go Math Answer Key Ch 1 Place Value, Multiplication, and Expressions and score better grades in the exam.

Lesson 1: Investigate • Place Value and Patterns

Lesson 2: Place Value of Whole Numbers

Lesson 3: Algebra • Properties

Lesson 4: Algebra • Powers of 10 and Exponents

Lesson 5: Algebra • Multiplication Patterns

Mid-Chapter Checkpoint

Lesson 6: Multiply by 1-Digit Numbers

Lesson 7: Multiply by Multi-Digit Numbers

Lesson 8: Relate Multiplication to Division

Lesson 9: Problem Solving • Multiplication and Division

Lesson 10: Algebra • Numerical Expressions

Lesson 11: Algebra • Evaluate Numerical Expressions

Lesson 12: Algebra • Grouping Symbols

Review/Test

Place Value and Patterns – Share and Show – Page No. 7

Complete the sentence.

Question 1.
500 is 10 times as much as ______

Answer:
50

Explanation:
Let the unknown number is S.
500 = 10S
S = 500/10 = 50.
500 is 10 times as much as 50.

Question 2.
20,000 is \(\frac{1}{10}\) of ______

Answer:
2,00,000

Explanation:
Let the unknown number is S.
20,000 = \(\frac{1}{10}\) S
S = 20,000 X 10 = 2,00,000

Question 3.
900 is \(\frac{1}{10}\) of ______

Answer:
9,000

Explanation:
Let the unknown number is S.
900 = \(\frac{1}{10}\) S
S = 900 X 10 = 9,000

Question 4.
600 is 10 times as much as ______

Answer:
60

Explanation:
Let the unknown number is S.
600 = 10S
S = 600/10 = 60.

Use place-value patterns to complete the table

Question 5.

Numbers 10 times as much as \(\frac{1}{10}\) of
10 ______ ______
3,000 ______ ______
800 ______ ______
50 ______ ______

Answer:

Numbers 10 times as much as \(\frac{1}{10}\) of
10 ___1___ ___100___
3,000 ___300___ ___30,000___
800 ___80___ ___8,000___
50 ___5___ ___500___

Explanation:
1. 10 is 10 times as much as ______
Let the unknown number is S.
10 = 10S
S = 10/10 = 1.
10 is 10 times as much as 1.
10 is \(\frac{1}{10}\) of ______
Let the unknown number is S.
10 = \(\frac{1}{10}\) S
S = 10 X 10 = 100.
2. 3,000 is 10 times as much as ______
Let the unknown number is S.
3,000 = 10S
S = 3,000/10 = 300.
3,000 is 10 times as much as 300.
3,000 is \(\frac{1}{10}\) of ______
Let the unknown number is S.
3,000 = \(\frac{1}{10}\) S
S = 3,000 X 10 = 30,000.
3. 800 is 10 times as much as ______
Let the unknown number is S.
800 = 10S
S = 800/10 = 80.
800 is 10 times as much as 80.
800 is \(\frac{1}{10}\) of ______
Let the unknown number is S.
800 = \(\frac{1}{10}\) S
S = 800 X 10 = 8,000.
4. 50 is 10 times as much as ______
Let the unknown number is S.
50 = 10S
S = 50/10 = 5.
50 is 10 times as much as 5.
50 is \(\frac{1}{10}\) of ______
Let the unknown number is S.
50 = \(\frac{1}{10}\) S
S = 50 X 10 = 500.

Question 6.

Numbers 10 times as much as \(\frac{1}{10}\) of
400 ______ ______
90 ______ ______
6,000 ______ ______
200 ______ ______

Answer:

Numbers 10 times as much as \(\frac{1}{10}\) of
400 __40____ ___4,000___
90 ___9___ ___900___
6,000 __600____ __60,000____
200 ___20___ ___2,000___

Explanation:
1. 400 is 10 times as much as ______
Let the unknown number is S.
400 = 10S
S = 400/10 = 40.
400 is 10 times as much as 40.
400 is \(\frac{1}{10}\) of ______
Let the unknown number is S.
400 = \(\frac{1}{10}\) S
S = 400 X 10 = 4,000.
2. 90 is 10 times as much as ______
Let the unknown number is S.
90 = 10S
S = 90/10 = 9.
90 is 10 times as much as 9.
90 is \(\frac{1}{10}\) of ______
Let the unknown number is S.
90 = \(\frac{1}{10}\) S
S = 90 X 10 = 900.
3. 6,000 is 10 times as much as ______
Let the unknown number is S.
6,000 = 10S
S = 6,000/10 = 600.
6,000 is 10 times as much as 600.
6,000 is \(\frac{1}{10}\) of ______
Let the unknown number is S.
6,000 = \(\frac{1}{10}\) S
S = 6,000 X 10 = 60,000.
4. 200 is 10 times as much as ______
Let the unknown number is S.
200 = 10S
S = 200/10 = 20.
200 is 10 times as much as 20.
200 is \(\frac{1}{10}\) of ______
Let the unknown number is S.
200 = \(\frac{1}{10}\) S
S = 200 X 10 = 2,000.

Complete the sentence with 100 or 1,000.

Question 13.
200 is ______ times as much as 2

Answer:
200 is 100 times as much as 2

Explanation:
Let the unknown number is S.
200 = 2S
S = 200/2 = 100

Question 14.
4,000 is ______ times as much as 4

Answer:
4,000 is 1000 times as much as 4

Explanation:
Let the unknown number is S.
4,000 = 2S
S = 4,000/2 = 1,000

Question 15.
700,000 is ______ times as much as 700

Answer:
700,000 is 1,000 times as much as 700

Explanation:
Let the unknown number is S.
700,000 = 700S
S = 700,000/700 = 1,000

Question 16.
600 is ______ times as much as 6

Answer:
600 is 100 times as much as 6

Explanation:
Let the unknown number is S.
600 = 6S
S = 600/6= 100

Question 17.
50,000 is ______ times as much as 500

Answer:
50,000 is 100_ times as much as 500

Explanation:
Let the unknown number is S.
50,000 = 500S
S = 50,000/500= 100

Question 18.
30,000 is ______ times as much as 30

Answer:
30,000 is 1,000 times as much as 30

Explanation:
Let the unknown number is S.
30,000 = 30S
S = 30,000/30 = 1,000

Question 19.
Explain how you can use place-value patterns to describe how 50 and 5,000 compare.
Type below:
__________

Answer:
5,000 is 100 times as much as 50

Explanation:
5,000/50 = 100

Place Value and Patterns – Problem Solving – Page No. 8

Sense or Nonsense?

Question 20.
Mark and Robyn used base-ten blocks to show that 300 is 100 times as much as 3. Whose model makes sense? Whose model is nonsense? Explain your reasoning.
Go Math Grade 5 Answer Key Chapter 1 Place Value, Multiplication, and Expressions Place Value, Multiplication, and Expressions; Place Value and Patterns img 1
Type below:
__________

Answer:
Robyn’s model makes sense. Because the given data 300 is 100 times as much as 3. It clearly states that there are 100 model-blocks and one model blocks should take to solve the problem.

Question 20.
Explain how you would help Mark understand why he should have used small cubes instead of longs.
Type below:
__________

Answer:
Mark’s drew 100 model-blocks and 10 model-blocks which. To get 300 is 100 times as much as 3, he needs to do 300/3 = 100 model blocks.

Place Value of Whole Numbers – Share and Show – Page No. 11

Complete the place-value chart to find the value of each digit.

Question 1.
Go Math Grade 5 Answer Key Chapter 1 Place Value, Multiplication, and Expressions Place Value, Multiplication, and Expressions; Place Value of Whole Numbers img 2
Type below:
__________

Answer:
Grade 5 Chapter 1 Image 1
7,333,820

Explanation:
7 x 1,000,000 = 7,000,000
3 x 100,000 = 300,000
3 x 1,000 = 3000
8 x 100 = 800
2 x 10 = 20

Write the value of the underlined digit.

Question 2.
1,574,833
__________

Answer:
4,000

Explanation:
(1 x 1,000,000) + (5 x 1,00,000) + (7 x 10,000) + (4 x 1,000) + (8 x 100) + (3 x 10) + (3 x 1)
4 x 1,000 = 4 thousands = 4,000

Question 3.
598,102
__________

Answer:
100

Explanation:
(5 x 1,00,000) + (9 x 10,000) + (8 x 1,000) + (1 x 100) + (0 x 10) + (2 x 1)
1 x 100 = 4 hundreds = 100

Question 4.
7,093,455
__________

Answer:
90,000

Explanation:
(7 x 1,000,000) + (0 x 1,00,000) + (9 x 10,000) + (3 x 1,000) + (4 x 100) + (5 x 10) + (5 x 1)
9 x 10,000 = 9 ten-thousands = 90,000

Question 5.
301,256,878
__________

Answer:
3,00,000,000

Explanation:
(3 x 1,00,000,000) + (0 x 10,000,000) + (1 x 1,000,000) + (2 x 1,00,000) + (5 x 10,000) + (6 x 1,000) + (8 x 100) + (7 x 10) + (8 x 1)
3 x 1,00,000,000 = 3 hundred- millions = 3,00,000,000

Write the number in two other forms.

Question 6.
(8 × 100,000) + (4 × 1,000) + (6 × 1) =
__________

Answer:
80,4006
Eight Hundred Four Thousand Six

Explanation:
(8 × 100,000) + (4 × 1,000) + (6 × 1) = 800,000 + 4,000 + 6 = 80,4006

Question 7.
seven million, twenty thousand, thirty-two
__________

Answer:
7,020,032
Seven Million Twenty Thousand Thirty-Two

Explanation:
seven million = 7,000,000
twenty thousand = 20,000
thirty-two = 32

On Your Own

Write the value of the underlined digit.

Question 8.
849,567,043
__________

Answer:
40,000,000

Explanation:
(8 x 1,00,000,000) + (4 x 10,000,000) + (9 x 1,000,000) + (5 x 1,00,000) + (6 x 10,000) + (7 x 1,000) + (0 x 100) + (4 x 10) + (3 x 1)
4 x 10,000,000 = 4 ten- millions = 40,000,000

Question 9.
9,422,850
__________

Answer:
4,00,000

Explanation:
(9 x 1,000,000) + (4 x 1,00,000) + (2 x 10,000) + (2 x 1,000) + (8 x 100) + (5 x 10) + (0 x 1)
4 x 1,00,000 = 4 Hundred Thousand = 4,00,000

Question 10.
96,283
__________

Answer:
90,000

Explanation:
(9 x 10,000) + (6 x 1,000) + (2 x 100) + (8 x 10) + (3 x 1)
9 x 10,000 = 9 ten-thousands = 90,000

Question 11.
498,354,021
__________

Answer:
4,00,000,000

Explanation:
(4 x 1,00,000,000) + (9 x 10,000,000) + (8 x 1,000,000) + (3 x 1,00,000) + (5 x 10,000) + (4 x 1,000) + (0 x 100) + (2 x 10) + (1 x 1)
4 x 1,00,000,000 = Four Hundred Million = 4,00,000,000

Question 12.
791,350
__________

Answer:
300

Explanation:
(7 x 1,00,000) + (9 x 10,000) + (1 x 1,000) + (3 x 100) + (5 x 10) + (0 x 1)
3 x 100 = 3 hundred = 300

Question 13.
27,911,534
__________

Answer:
7,000,000

Explanation:
(2 x 10,000,000) + (7 x 1,000,000) + (9 x 1,00,000) + (1 x 10,000) + (1 x 1,000) + (5 x 100) + (3 x 10) + (4 x 1)
7 x 1,000,000 = Seven Million = 7,000,000

Question 14.
105,980,774
__________

Answer:
80,000

Explanation:
(1 x 1,00,000,000) + (0 x 10,000,000) + (5 x 1,000,000) + (9 x 1,00,000) + (8 x 10,000) + (0 x 1,000) + (7 x 100) + (7 x 10) + (4 x 1)
8 x 10,000 = 8 ten-thousand = 80,000

Question 15.
8,265,178
__________

Answer:
5,000

Explanation:
(8 x 1,000,000) + (2 x 1,00,000) + (6 x 10,000) + (5 x 1,000) + (1 x 100) + (7 x 10) + (8 x 1)
5 x 1,000 = 5 one-thousand = 5,000

Write the number in two other forms.

Question 16.
345,000
Type below:
__________

Answer:
Three Hundred Forty-Five Thousand
(3 x 1,00,000) + (4 x 10,000) + (5 x 1,000) + (0 x 100) + (0 x 10) + (0 x 1)

Question 17.
119,000,003
Type below:
__________

Answer:
One Hundred Nineteen Million Three
(1 x 100,000,000) + (1 x 10,000,000) + (9 x 1,000,000) + (0 x 1,00,000) + (0 x 10,000) + (0 x 1,000) + (0 x 100) + (0 x 10) + (3 x 1)

Place Value of Whole Numbers – Problem Solving – Page No. 12

Use the table for 18–19.

Question 18.
Which planet is about 10 times as far as Earth is from the Sun?
Go Math Grade 5 Answer Key Chapter 1 Place Value, Multiplication, and Expressions Place Value, Multiplication, and Expressions; Place Value of Whole Numbers img 3
__________

Answer:
Saturn

Explanation:
Saturn = 1,427,000/10 = 142,700 which is 10 times as far as Earth

Question 19.
Which planet is about \(\frac{1}{10}\) of the distance Uranus is from the Sun?
__________

Answer:
Mars

Explanation:
Mars = 227,900
\(\frac{1}{10}\) x 2,871,000 = 287,100
Which planet is about \(\frac{1}{10}\) of the distance Uranus is from the Sun

Question 20.
What’s the Error? Matt wrote the number four million, three hundred five thousand, seven hundred sixty-two as 4,350,762. Describe and correct his error.
Type below:
__________

Answer:
Matt switched 2 digits in the thousands period: 4,305,762

Question 21.
Explain how you know that the values of the digit 5 in the numbers 150,000 and 100,500 are not the same.
Type below:
__________

Answer:
In 150,000, the digit 5 is in the ten-thousands place, So, its value is 50,000; in 100,500, the digit 5 is in the hundreds place. So, its value is 500.

Question 22.
Test Prep In the number 869,653,214, which describes how the digit 6 in the ten-millions place compares to the digit 6 in the hundred-thousands place?
Options:
A. 10 times as much as
B. 100 times as much as
C. 1,000 times as much as
D.\(\frac{1}{10}\) of

Answer:
B. 100 times as much as

Explanation:
869,653,214
(8 x 100,000,000) + (6 x 10,000,000) + (9 x 1,000,000) + (6 x 1,00,000) + (5 x 10,000) + (3 x 1,000) + (2 x 100) + (1 x 10) + (4 x 1)
6 x 10,000,000 = 60,000,000
6 x 1,00,000 = 6,00,000
60,000,000/6,00,000 = 100

Properties – Share and Show – Page No. 15

Use properties to find 4 × 23 × 25.

Question 1.
23 × × 25 ________ Property of Multiplication
23 × ( × ) ________ Property of Multiplication
23 ×
__________
____

Answer:
23 x 4 x 25; Commutative Property of Multiplication
23 x (4 x 25); Associative Property of Multiplication
23 x 100
2,300

Use properties to find the sum or product.

Question 2.
89 + 27 + 11 = ____

Answer:
89 + (27 + 11); Associative Property of Addition
89 + 38
127

Question 3.
9 × 52 = ____

Answer:
468

Explanation:
9 x 52
Write 52 = (50 + 2)
9 x (50 + 2)
(9 x 50) + (9 x 2); Distributive Property of Multiplication
450 + 18
468

Question 4.
107 + 0 + 39 + 13 = ____

Answer:
107 + 0 + 39 + 13
(107 + 0) + (39 + 13); Associative Property of Addition
107 + 0 = 107; Identity Property of Addition
107 + 52 = 159

Complete the equation, and tell which property you used.

Question 5.
9 × (30 + 7) = (9 × ____) + (9 × 7)

Answer:
9 × (30 + 7) = (9 ×30) + (9 × 7)
Distributive Property of Multiplication

Explanation:
9 x (30 + 7)
(9 x 30) + (9 x 7); Distributive Property of Multiplication
270 + 63 = 333

Question 6.
0 + ____ = 47

Answer:
47; Identity Property of Addition

Explanation:
0 + 47 = 47; Identity Property of Addition

Question 6.
Describe how you can use properties to solve problems more easily.
Type below:
__________

Answer:
Using Properties of Addition and Properties of Multiplication, we can solve problems more easily. Simplifying problems is easy with the properties.

On Your Own

Practice: Copy and Solve Use properties to find the sum or product.

Question 7.
3 × 78 = ____

Answer:
234, Associative Property of Multiplication

Explanation:
Write 78 as 6 x 13
3 x 6 x 13
(3 x 6) x 13; Associative Property of Multiplication
18 x 13 = 234

Question 8.
4 × 60 × 5 = ____

Answer:
1,200; Associative Property of Multiplication

Explanation:
4 x 60 x 5
4 x (60 x 5); Associative Property of Multiplication
4 x 300 = 1,200

Question 9.
21 + 25 + 39 + 5 = ____

Answer:
90; Associative Property of Addition

Explanation:
(21 + 25) + (39 + 5); Associative Property of Addition
46 + 44 = 90

Complete the equation, and tell which property you used.

Question 10.
11 + (19 + 6) = (11 + ____) + 6

Answer:
11 + (19 + 6) = (11 + 19) + 6; Associative Property of Addition

Question 11.
25 + 14 = ____ + 25

Answer:
25 + 14 = 14 + 25; Commutative Property of Addition

Question 12.
Show how you can use the Distributive Property to rewrite and find (32 × 6) + (32 × 4).
____

Answer:
(32 × 6) + (32 × 4) = 32 x (6 + 4); Distributive Property

Properties – Problem Solving – Page No. 16

Question 13.
Three friends’ meals at a restaurant cost $13, $14, and $11. Use parentheses to write two different expressions to show how much the friends spent in all. Which property does your pair of expressions demonstrate?
$ ____

Answer:
$38; Associative Law of Addition

Explanation:
Three friends’ meals at a restaurant cost $13, $14, and $11.
Friends spent in all = $13 + $14 + $11
$13 + ($14 + $11) = ($13 + $14) + $11 Associative Law of Addition

Question 14.
Jacob is designing an aquarium for a doctor’s office. He plans to buy 6 red blond guppies, 1 blue neon guppy, and 1 yellow guppy. The table shows the price list for the guppies. How much will the guppies for the aquarium cost?
Go Math Grade 5 Answer Key Chapter 1 Place Value, Multiplication, and Expressions Place Value, Multiplication, and Expressions; Properties img 4
$ ____

Answer:
$162

Explanation:
Jacob is designing an aquarium for a doctor’s office. He plans to buy 6 red blond guppies, 1 blue neon guppy, and 1 yellow guppy.
(6 x $22) + (1 x $11) + (1 x $19) = $132 + $11 + $19 = $162

Question 15.
Sylvia bought 8 tickets to a concert. Each ticket costs $18. To find the total cost in dollars, she added the product 8 × 10 to the product 8 × 8, for a total of 144. Which property did Sylvia use?
i. Distributive Property
ii. Associative Property

Answer:
i. Distributive Property

Explanation:
Sylvia bought 8 tickets to a concert. Each ticket costs $18.
To find the total cost in dollars = 8 x $18
Using Distributive Property
(8 × 10) + (8 × 8) = 8 x (10 + 8) = 144.

Question 16.
Sense or Nonsense? Julie wrote (15 – 6) – 3 = 15 – (6 – 3). Is Julie’s equation sense or nonsense? Do you think the Associative Property works for subtraction? Explain.
__________

Answer:
Nonsense;
(15 – 6) – 3 = 9 – 3 = 6.
15 – (6 – 3) = 15 – 3 = 12
6 not equal to 12.
So, Associative Property does not work for subtraction

Question 17.
Test Prep Canoes rent for $29 per day. Which expression can be used to find the cost in dollars of renting 6 canoes for a day?
Options:
A. (6 + 20) + (6 + 9)
B. (6 × 20) + (6 × 9)
C. (6 + 20) × (6 + 9)
D. (6 × 20) × (6 × 9)

Answer:
B. (6 × 20) + (6 × 9)

Explanation:
Canoes rent for $29 per day. For renting 6 canoes for a day, 6 x $29
6 x $(20 + 9) = (6 x 20) + (6 x 9)

Powers of 10 and Exponents – Share and Show – Page No. 18

Write in exponent form and word form.

Question 1.
10 × 10
Exponent form:
Word form:
Type below:
__________

Answer:
Exponent form: 102
Word form: the second power of ten

Explanation:
10 × 10
Base = 10;
Exponent = 2;
Exponent Form: 102
Word Form: the second power of ten

Question 2.
10 × 10 × 10 × 10
Exponent form:
Word form:
Type below:
__________

Answer:
Exponent Form: 104
Word Form: the fourth power of ten

Explanation:
10 × 10 × 10 × 10
Base = 10;
Exponent = 4;
Exponent Form: 104
Word Form: the fourth power of ten

Find the value.

Question 3.
102 = ____

Answer:
100

Explanation:
100 = 1;
101 = 1 x 10 = 10;
102 = 10 x 10 = 100;

Question 4.
4 × 102 = ____

Answer:
400

Explanation:
4 × 102 =
100 = 1;
101 = 1 x 10 = 10;
102 = 10 x 10 = 100;
4 x 100 = 400

Question 5.
7 × 102 = ____

Answer:
700

Explanation:
7 × 102 =
100 = 1;
101 = 1 x 10 = 10;
102 = 10 x 10 = 100;
7 x 100 = 700

Powers of 10 and Exponents – On Your Own – Page No. 19

Write in exponent form and word form.

Question 6.
10 × 10 × 10
exponent form:
word form:
Type below:
__________

Answer:
Exponent form: 103
Word form: the third power of ten

Explanation:
10 × 10 × 10
Base = 10;
Exponent = 3;
Exponent Form: 103
Word Form: the third power of ten

Question 7.
10 × 10 × 10 × 10 × 10
exponent form:
word form:
Type below:
__________

Answer:
Exponent form: 105
Word form: the fifth power of ten

Explanation:
10 × 10 × 10 × 10 × 10
Base = 10;
Exponent = 5;
Exponent Form: 105
Word Form: the fifth power of ten

Find the value.

Question 8.
104 = ____

Answer:
10,000

Explanation:
100 = 1;
101 = 1 x 10 = 10;
102 = 10 x 10 = 100;
103 = 10 x 10 x 10 = 1,000;
104 = 10 x 10 x 10 x 10 = 10,000;

Question 9.
2 × 103 = ____

Answer:
2,000

Explanation:
2 × 103 =
100 = 1;
101 = 1 x 10 = 10;
102 = 10 x 10 = 100;
103 = 10 x 10 x 10 = 1,000;
2 x 1,000 = 2,000

Question 10.
6 × 104 = ____

Answer:
60,000

Explanation:
6 × 104
100 = 1;
101 = 1 x 10 = 10;
102 = 10 x 10 = 100;
103 = 10 x 10 x 10 = 1,000;
104 = 10 x 10 x 10 x 10 = 10,000;
6 x 10,000 = 60,000

Complete the pattern.

Question 11.
7 × 100 = 7 × 1 = _______
7 × 101 = 7 × 10 = _______
7 × 102 = 7 × 10 × 10 = _______
7 × 103 = 7 × 10 × 10 × 10 = _______
7 × 104 = 7 × 10 × 10 × 10 × 10 = _______

Answer:
7 × 100 = 7 × 1 = 7
7 × 101 = 7 × 10 = 70
7 × 102 = 7 × 10 × 10 = 7 x 100 = 700
7 × 103 = 7 × 10 × 10 × 10 = 7 x 1,000 = 7,000
7 × 104 = 7 × 10 × 10 × 10 × 10 = 7 x 10,000 = 70,000

Explanation:
100 = 1;
101 = 1 x 10 = 10;
102 = 10 x 10 = 100;
103 = 10 x 10 x 10 = 1,000;
104 = 10 x 10 x 10 x 10 = 10,000;

Question 12.
9 × 100 = _______ = 9
9 × 101 = _______ = 90
9 × 102 = _______ = 900
9 × 103 = _______ = 9,000
9 × 104 = _______ = 90,000

Answer:
9 × 100 = 9 x 1 = 9
9 × 101 = 9 x 10 = 90
9 × 102 = 9 x 10 x 10 = 900
9 × 103 = 9 x 10 x 10 x 10= 9,000
9 × 104 = 9 x 10 x 10 x 10 x 10 = 90,000

Explanation:
100 = 1;
101 = 1 x 10 = 10;
102 = 10 x 10 = 100;
103 = 10 x 10 x 10 = 1,000;
104 = 10 x 10 x 10 x 10 = 10,000;

Question 13.
12 × 100 = 12 × 1 = _______
12 × 101 = 12 × 10 = _______
12 × 102 = 12 × 10 × 10 = _______
12 × 103 = 12 × 10 × 10 × 10 _______
12 × 104 = 12 × 10 × 10 × 10 × 10 _______

Answer:
12 × 100 = 12 × 1 = 12
12 × 101 = 12 × 10 = 120
12 × 102 = 12 × 10 × 10 = 1,200
12 × 103 = 12 × 10 × 10 × 10 = 12,000
12 × 104 = 12 × 10 × 10 × 10 × 10 = 120,000

Explanation:
100 = 1;
101 = 1 x 10 = 10;
102 = 10 x 10 = 100;
103 = 10 x 10 x 10 = 1,000;
104 = 10 x 10 x 10 x 10 = 10,000;

Question 14.
103 = 10 × 10n What is the value of n?
Think: 103 = 10 × () × (), or 10 × ()
The value of n is ……..
n = ______

Answer:
2

Explanation:
103 = 10 × 10n
103 = 10 x 10 x 10 = 10 x 102
The value of n is 2

Question 15.
Explain how to write 50,000 using exponents.
Type below:
__________

Answer:
5 x 104

Explanation:
5 x 10,000
100 = 1;
101 = 1 x 10 = 10;
102 = 10 x 10 = 100;
103 = 10 x 10 x 10 = 1,000;
104 = 10 x 10 x 10 x 10 = 10,000;
10,000 = 10 x 10 x 10 x 10 = 104
5 x 104

Powers of 10 and Exponents – UNLOCK the Problem – Page No. 20

Question 16.
Lake Superior is the largest of the Great Lakes. It covers a surface area of about 30,000 square miles. How can you show the estimated area of Lake Superior as a whole number multiplied by a power of ten?
Go Math Grade 5 Answer Key Chapter 1 Place Value, Multiplication, and Expressions Place Value, Multiplication, and Expressions; Powers of 10 and Exponents img 5
a. What are you asked to find?
Options:
A. 3 × 102 sq mi
B. 3 × 103 sq mi
C. 3 × 104 sq mi
D. 3 × 105 sq mi

Answer:
C. 3 × 104 sq mi

Explanation:
Lake Superior is the largest of the Great Lakes. It covers a surface area of about 30,000 square miles.
3 x 10,000
100 = 1;
101 = 1 x 10 = 10;
102 = 10 x 10 = 100;
103 = 10 x 10 x 10 = 1,000;
104 = 10 x 10 x 10 x 10 = 10,000;
10,000 = 10 x 10 x 10 x 10 = 104
3 x 104

Question 16.
b. How can you use a pattern to find the answer?
Type below:
__________

Answer:
3 x 10,000
100 = 1;
101 = 1 x 10 = 10;
102 = 10 x 10 = 100;
103 = 10 x 10 x 10 = 1,000;
104 = 10 x 10 x 10 x 10 = 10,000;
10,000 = 10 x 10 x 10 x 10 = 104
3 x 104

Question 16.
c. Write a pattern using the whole number 3 and powers of ten.
3 × 101 = 3 × 10   =
3 × 102 =              =
3 × 103 =              =
3 × 104 =              =
Type below:
__________

Answer:
3 × 101 = 3 × 10   =
3 × 102 = 3 x 10 x 10 = 300
3 × 103 = 3 x 10 x 10 x 10 = 3,000
3 × 104 = 3 x 10 x 10 x 10 x 10 = 30,000

Question 16.
d. Fill in the correct answer choice above.
Type below:
__________

Answer:
3 × 104 = 3 x 10 x 10 x 10 x 10 = 30,000

Question 17.
The Earth’s diameter through the equator is about 8,000 miles. What is the Earth’s estimated diameter written as a whole number multiplied by a power of ten?
Options:
A. 8 × 101 miles
B. 8 × 102 miles
C. 8 × 103 miles
D. 8 × 104 miles

Answer:
C. 8 × 103 miles

Explanation:
The Earth’s diameter through the equator is about 8,000 miles.
8 x 1,000
100 = 1;
101 = 1 x 10 = 10;
102 = 10 x 10 = 100;
103 = 10 x 10 x 10 = 1,000;
1,000 = 10 x 10 x 10
8 x 1,000 = 8 x 103

Question 18.
The Earth’s circumference around the equator is about 25 × 103 miles. What is the Earth’s estimated circumference written as a whole number?
Options:
A. 250,000 miles
B. 25,000 miles
C. 2,500 miles
D. 250 miles

Answer:
B. 25,000 miles

Explanation:
The Earth’s circumference around the equator is about 25 × 103 miles.
25 × 103 miles;
100 = 1;
101 = 1 x 10 = 10;
102 = 10 x 10 = 100;
103 = 10 x 10 x 10 = 1,000;
25 x 1,000 = 25,000 miles

Multiplication Patterns – Share and Show – Page No. 22

Use mental math and a pattern to find the product.

Question 1.
• What basic fact can you use to help you find 30×4,000?
30 × 4,000 = ____

Answer:
3 x 4 = 12

Explanation:
30 × 4,000
The basic fact is 3 x 4 = 12

Use mental math to complete the pattern.

Question 2.
1 × 1 = 1
1 × 101 = _______
1 × 102 = _______
1 × 103 = _______

Answer:
1 × 1 = 1
1 × 101 = 10
1 × 102 = 100
1 × 103 = 1,000

Explanation:
1 × 1 = 1
1 × 101 = 1 x 10 = 10
1 × 102 = 1 x 10 x 10 = 100
1 × 103 = 1 x 10 x 10 x 10 = 1,000

Question 3.
7 × 8 = 56
(7 × 8) × 101 = _______
(7 × 8) × 102 = _______
(7 × 8) × 106 = _______

Answer:
7 × 8 = 56
(7 × 8) × 101 = 560
(7 × 8) × 102 = 5,600
(7 × 8) × 106 = 56,000,000

Explanation:
7 × 8 = 56
(7 × 8) × 101 = 56 x 10 = 560
(7 × 8) × 102 = 56 x 10 x 10 = 5,600
(7 × 8) × 106 = 56 x 10 x 10 x 10 x 10 x 10 x 10 = 56,000,000

Question 4.
6 × 5 = _______
6 × 5 × _______ = 300
6 × 5 × _______ = 3000
6 × 5 × _______ = 30,000

Answer:
6 × 5 = 30
6 × 5 × 101 = 300
6 × 5 × 103 = 3000
6 × 5 × 104 = 30,000

Explanation:
6 × 5 = 30
6 × 5 × 10 = 300
6 × 5 × 10 x 10 x 10 = 3000
6 × 5 × 10 x 10 x 10 x 10 = 30,000

On Your Own

Use mental math to complete the pattern.

Question 5.
9 × 5 = 45
(9 × 5) × 101 = _______
(9 × 5) × 102 = _______
(9 × 5) × 103 = _______

Answer:
9 × 5 = 45
(9 × 5) × 101 = 450
(9 × 5) × 102 = 4,500
(9 × 5) × 103 = 45,000

Explanation:
9 × 5 = 45
(9 × 5) × 101 = 45 x 10 = 450
(9 × 5) × 102 = 45 x 10 x 10 = 4,500
(9 × 5) × 103 = 45 x 10 x 10 x 10 = 45,000

Question 6.
3 × 7 = 21
(3 × 7) × 101 = _______
(3 × 7) × 102 = _______
(3 × 7) × 103 = _______

Answer:
3 × 7 = 21
(3 × 7) × 101 = 210
(3 × 7) × 102 = 2,100
(3 × 7) × 103 = 21,000

Explanation:
3 × 7 = 21
(3 × 7) × 101 = 21 x 10 = 210
(3 × 7) × 102 = 21 x 10 x 10 = 2,100
(3 × 7) × 103 = 21 x 10 x 10 x 10 = 21,000

Question 7.
5 × 4 = _______
(5 × 4) × _______ = 200
(5 × 4) × _______ = 2,000
(5 × 4) × _______ = 20,000

Answer:
5 × 4 = 20
(5 × 4) × 101 = 200
(5 × 4) × 102 = 2,000
(5 × 4) × 103 = 20,000

Explanation:
5 × 4 = 20
(5 × 4) × 10 = 200
(5 × 4) × 10 x 10 = 2,000
(5 × 4) × 10 x 10 x 10 = 20,000

Question 8.
5 × 7 = _______
(5 × 7) × _______ = 350
(5 × 7) × _______ = 3,500
(5 × 7) × _______ = 35,000

Answer:
5 × 7 = 35
(5 × 7) × 101 = 350
(5 × 7) × 102 = 3,500
(5 × 7) × 103 = 35,000

Explanation:
5 × 7 = 35
(5 × 7) × 10 = 350
(5 × 7) × 10 x 10 = 3,500
(5 × 7) × 10 x 10 x 10 = 35,000

Question 9.
4 × 2 = 8
(4 × 2) × 101 = _______
(4 × 2) × 102 = _______
(4 × 2) × 103 = _______

Answer:
4 × 2 = 8
(4 × 2) × 101 = 80
(4 × 2) × 102 = 800
(4 × 2) × 103 = 8,000

Explanation:
4 × 2 = 8
(4 × 2) × 101 = 8 x 10 = 80
(4 × 2) × 102 = 8 x 10 x 10 = 800
(4 × 2) × 103 = 8 x 10 x 10 x 10 = 8,000

Question 10.
6 × 7 = 42
(6 × 7) × 101 = _______
(6 × 7) × 102 = _______
(6 × 7) × 103 = _______

Answer:
6 × 7 = 42
(6 × 7) × 101 = 420
(6 × 7) × 102 = 4,200
(6 × 7) × 103 = 42,000

Explanation:
6 × 7 = 42
(6 × 7) × 101 = 42 x 10 = 420
(6 × 7) × 102 = 42 x 10 x 10 = 4,200
(6 × 7) × 103 = 42 x 10 x 10 x 10 = 42,000

Use mental math and a pattern to find the product.

Question 11.
(6 × 6) × 101 = ____

Answer:
(6 × 6) × 101 =  360

Explanation:
6 x 6 =36
(6 × 6) × 101 = 36 x 10 = 360

Question 12.
(7 × 4) × 103 = ____

Answer:
28,000

Explanation:
7 x 4 = 28
(7 × 4) × 101 = 28 x 10 = 280
(7 × 4) × 102 = 28 x 10 x 10 = 2,800
(7 × 4) × 103 = 28 x 10 x 10 x 10 = 28,000

Question 13.
(9 × 8) × 102 = ____

Answer:
7,200

Explanation:
(9 × 8) = 72
(9 × 8) × 101 = 72 x 10 = 720
(9 × 8) × 102 = 72 x 10 x 10 = 7,200

Question 14.
(4 × 3) × 102 = ____

Answer:
1,200

Explanation:
(4 × 3) = 12
(4 × 3) × 101 = 12 x 10 = 120
(4 × 3) × 102 = 12 x 10 x 10 = 1,200

Question 15.
(2 × 5) × 103 = ____

Answer:
10,000

Explanation:
(2 × 5) = 10
(2 × 5) × 101 = 10 x 10 = 100
(2 × 5) × 102 = 10 x 10 x 10 = 1,000
(2 × 5) × 103 = 10 x 10 x 10 x 10 = 10,000

Question 16.
(2 × 8) × 102 = ____

Answer:
1,600

Explanation:
(2 × 8) = 16
(2 × 8) × 101 = 16 x 10 = 160
(2 × 8) × 102 = 16 x 10 x 10 = 1,600

Question 17.
(6 × 5) × 103 = ____

Answer:
30,000

Explanation:
(6 × 5) = 30
(6 × 5) × 101 = 30 x 10 = 300
(6 × 5) × 102 = 30 x 10 x 10 = 3,000
(6 × 5) × 103 = 30 x 10 x 10 x 10 = 30,000

Question 18.
(8 × 8) × 104 = ____

Answer:
640,000

Explanation:
(8 × 8) = 64
(8 × 8) × 101 = 64 x 10 = 640
(8 × 8) × 102 = 64 x 10 x 10 = 6,400
(8 × 8) × 103 = 64 x 10 x 10 x 10 = 64,000
(8 × 8) × 104 = 64 x 10 x 10 x 10 x 10 = 640,000

Question 19.
(7 × 8) × 104 = ____

Answer:
560,000

Explanation:
(7 × 8) = 56
(7 × 8) × 101 = 56 x 10 = 560
(7 × 8) × 102 = 56 x 10 x 10 = 5,600
(7 × 8) × 103 = 56 x 10 x 10 x 10 = 56,000
(7 × 8) × 104 = 56 x 10 x 10 x 10 x 10 = 560,000

Multiplication Patterns – Share and Show – Page No. 23

Use mental math to complete the table.

Question 20.
1 roll = 50 dimes ; Think:50 dimes per roll × 20 rolls =(5 × 2) × (10 × 10)
Go Math Grade 5 Answer Key Chapter 1 Place Value, Multiplication, and Expressions Place Value, Multiplication, and Expressions; Multiplication Patterns img 6
Type below:
__________

Answer:
Grade 5 Chapter 1 Image 4

Explanation:
1 roll = 50 dimes ;
Think:50 dimes per roll × 20 rolls =(5 × 2) × (10 × 10) = 10 x 102
50 dimes per roll × 30 rolls = (5 x 3) x (10 × 10) = 15 x 102
50 dimes per roll × 40 rolls = (5 x 4) x (20 × 10) = 20 x 102
50 dimes per roll × 50 rolls = (5 x 5) x (10 × 10) = 25 x 102
50 dimes per roll × 60 rolls = (5 x 6) x (10 × 10) = 30 x 102
50 dimes per roll × 70 rolls = (5 x 7) x (10 × 10) = 35 x 102
50 dimes per roll × 80 rolls = (5 x 8) x (10 × 10) = 40 x 102
50 dimes per roll × 90 rolls = (5 x 9) x (10 × 10) = 45 x 102
50 dimes per roll × 100 rolls = (5 x 10) x (10 × 10) = 50 x 102

Question 21.
1 roll = 40 quarters ; Think:40 quarters per roll × 20 rolls =(4 × 2) × (10 × 10)
Go Math Grade 5 Answer Key Chapter 1 Place Value, Multiplication, and Expressions Place Value, Multiplication, and Expressions; Multiplication Patterns img 7
Type below:
__________

Answer:
Grade 5 Chapter 1 Image 5

Explanation:
1 roll = 40 quarters ;
Think:40 quarters per roll × 20 rolls =(4 × 2) × (10 × 10) = 8 x 102
40 quarters per roll × 30 rolls =(4 × 3) × (10 × 10) = 12 x 102
40 quarters per roll × 40 rolls =(4 × 4) × (10 × 10) = 16 x 102
40 quarters per roll × 50 rolls =(4 × 5) × (10 × 10) = 20 x 102
40 quarters per roll × 60 rolls =(4 × 6) × (10 × 10) = 24 x 102
40 quarters per roll × 70 rolls =(4 × 7) × (10 × 10) = 28 x 102
40 quarters per roll × 80 rolls =(4 × 8) × (10 × 10) = 32 x 102
40 quarters per roll × 90 rolls =(4 × 9) × (10 × 10) = 36 x 102
40 quarters per roll × 100 rolls =(4 × 10) × (10 × 10) = 40 x 102

Question 22.
Go Math Grade 5 Answer Key Chapter 1 Place Value, Multiplication, and Expressions Place Value, Multiplication, and Expressions; Multiplication Patterns img 8
Type below:
__________

Answer:
Grade 5 Chapter 1 Image 6

Explanation:
80 x 800 = 64 x 103
80 x 6 = (8 x 6) x 10 = 48 x 101
80 x 70 = (8 x 7) x (10 x 10) = 56 x 102
80 x 9,000 = (8 x 9) x (10 x 10 x 10 x 10) = 64 x 104

Question 23.
Go Math Grade 5 Answer Key Chapter 1 Place Value, Multiplication, and Expressions Place Value, Multiplication, and Expressions; Multiplication Patterns img 9
Type below:
__________

Answer:
Grade 5 Chapter 1 Image 7

Explanation:
Given that
90 x 9,000 = (9 x 9) x 10 x 10 x 10 x 10 = 81 x 104
90 x 6 = (9 x 6) x 10 = 54 x 101
90 x 70 = (9 x 7) x (10 x 10) = 63 x 102
90 x 800 = (9 x 8) x (10 x 10 x 10) = 72 x 103

Problem Solving

Use the table for 24–26.

Question 24.
What if you magnified the image of a cluster fly by 9 × 103 ? What would the length appear to be?
Go Math Grade 5 Answer Key Chapter 1 Place Value, Multiplication, and Expressions Place Value, Multiplication, and Expressions; Multiplication Patterns img 10
____ mm

Answer:
9,000 mm

Explanation:
9 × 103  = 9 x 10 x 10 x 10 = 9,000

Question 25.
If you magnified the image of a fire ant by 4 × 103 and a tree hopper by 3 × 103 , which insect would appear longer? How much longer?
____ mm

Answer:
103 mm

Explanation:
fire ant: 4 × 103  = 4 x 10 x 10 x 10 = 4,000 mm
tree hopper: 3 × 103 = 3 x 10 x 10 x 10 = 3,000 mm
4,000 > 3,000.
So, fire ant appears to be longer.
4,000 – 3,000 = 1,000 = 103

Question 26.
John wants to magnify the image of a fire ant and a crab spider so they appear to be the same length. How many times their actual sizes would he need to magnify each image?
Fire ant by _______ times
Crab spider by ______ times

Answer:
Fire ant by 5 times
Crab spider by 4 times

Explanation:
Given that Fire ant = 4 mm
crab spider = 5 mm
So, to make them have the same lengths, multiply fire ant by 5 mm and multiply Crab spider by 4 mm

Multiplication Patterns – Share and Show – Page No. 24

Question 27.
What does the product of any whole-number factor multiplied by 100 always have? Explain.
Type below:
__________

Answer:
The product of any whole number factor multiplied by 100 has two digits which are 0 in ones and tens place.
Example: 2 x 100 = 200

Question 28.
Test Prep How many zeros are in the product (5 × 4) × 104?
Options:
A. 3
B. 4
C. 5
D. 6

Answer:
C. 5

Explanation:
(5 × 4) × 104 = 20 x 104 = 2 x 105
5 zeroes

Use patterns and mental math to solve.

Question 29.
A human body has about 30 times as many platelets as white blood cells. A small sample of blood has 8×103 white blood cells. About how many platelets are in the sample?
______ platelets

Answer:
24 x 104 platelets

Explanation:
Let the number of platelets = s.
s = 30 x 8×103
s = 30 x 8 x 10 x 10 x 10 = (3 x 8) x (10 x 10 x 10 x 10) = 24 x 104

Question 30.
Basophils and monocytes are types of white blood cells. A blood sample has about 5 times as many monocytes as basophils. If there are 60 basophils in the sample, about how many monocytes are there?
______ monocytes

Answer:
3 x 102 monocytes

Explanation:
Let the number of monocytes = S
S = 5 x 60 = 300 = 3 x 100
S = 3 x 102

Question 31.
Lymphocytes and eosinophils are types of white blood cells. A blood sample has about 10 times as many lymphocytes as eosinophils. If there are 2 × 102 eosinophils in the sample, about how many lymphocytes are there?
______ lymphocytes

Answer:
2 × 103 lymphocytes

Explanation:
Lymphocytes and eosinophils are types of white blood cells. A blood sample has about 10 times as many lymphocytes as eosinophils.
There are 2 × 102 eosinophils in the sample
Then, Lymphocytes = 10 x 2 × 102 eosinophils = 2 × 103

Question 32.
An average person has 6 × 102 times as many red bloods cells as white blood cells. A small sample of blood has 7 × 103 white blood cells. About how many red blood cells are in the sample?
______ red blood cells

Answer:
42 x 10 red blood cells

Explanation:
Let the red blood cells = S
S = 7 × 103 x 6 × 102
S = 42 x 10

Mid-Chapter Checkpoint – Vocabulary – Page No. 25

Choose the best term for the box.

Question 1.
A group of three digits separated by commas in a multidigit number is a __
________

Answer:
Period

Question 2.
An __ is the number that tells how many times a base is used as a factor
________

Answer:
exponent

Concepts and Skills

Complete the sentence.

Question 3.
7 is \(\frac{1}{10}\) of ______

Answer:
70

Explanation:
Let the unknown number is S.
7 = \(\frac{1}{10}\) S
S = 7 X 10 = 70

Question 4.
800 is 10 times as much as ______

Answer:
80

Explanation:
Let the unknown number is S.
800 = 10S
S = 800/10 = 80.

Write the value of the underlined digit.

Question 5.
6,581,678
________

Answer:
80,000

Explanation:
(6 x 1,000,000) + (5 x 1,00,000) + (8 x 10,000) + (1 x 1,000) + (6 x 100) + (7 x 10) + (8 x 1)
8 x 10,000 = 80,000

Question 6.
25,634
________

Answer:
600

Explanation:
(2 x 10,000) + (5 x 1,000) + (6 x 100) + (3 x 10) + (4 x 1)
6 x 100 = 600

Question 7.
34,634,803
________

Answer:
4,000,000

Explanation:
(3 x 10,000,000) + (4 x 1,000,000) + (6 x 1,00,000) + (3 x 10,000) + (4 x 1,000) + (8 x 100) + (0 x 10) + (3 x 1)
4 x 1,000,000 = 4,000,000

Question 8.
2,764,835
________

Answer:
700,000

Explanation:
(2 x 1,000,000) + (7 x 1,00,000) + (6 x 10,000) + (4 x 1,000) + (8 x 100) + (3 x 10) + (5 x 1)
7 x 1,00,000 = 700,000

Complete the equation, and tell which property you used.

Question 9.
8 × (14 + 7) = ________ + (8 × 7)

Answer:
8 × (14 + 7) = (8 x 14) + (8 × 7);
Distributive Property of Multiplication

Explanation:
8 × (14 + 7)
(8 x 14) + (8 × 7); Distributive Property of Multiplication

Question 10.
7 + (8 + 12) = ________ + 12

Answer:
7 + (8 + 12) = (7 + 8) + 12
Associative Property of Addition

Find the value.

Question 11.
103 = ______

Answer:
1,000

Explanation:
100 = 1;
101 = 1 x 10 = 10;
102 = 10 x 10 = 100;
103 = 10 x 10 x 10 = 1,000;

Question 12.
6 × 102 = ______

Answer:
600

Explanation:
6 × 102
100 = 1;
101 = 1 x 10 = 10;
102 = 10 x 10 = 100;
6 x 100 = 600

Question 13.
4 × 104 = ______

Answer:
40,000

Explanation:
4 × 104
100 = 1;
101 = 1 x 10 = 10;
102 = 10 x 10 = 100;
103 = 10 x 10 x 10 = 1,000;
104 = 10 x 10 x 10 x 10 = 10,000;
4 x 10,000 = 40,000

Use mental math and a pattern to find the product.

Question 14.
70 × 300 = ______

Answer:
21,000

Explanation:
70 × 300 = (7 x 3) x (10 x 10 x 10) = 21 x 1,000 = 21,000

Question 15.
(3 × 4) × 103 = ______

Answer:
12,000

Explanation:
(3 × 4) × 103
100 = 1;
101 = 1 x 10 = 10;
102 = 10 x 10 = 100;
103 = 10 x 10 x 10 = 1,000;
12 x 1,000 = 12,000

Mid-Chapter Checkpoint – Page No. 26

Fill in the bubble completely to show your answer.

Question 16.
DVDs are on sale for $24 each. Which expression can be used to find the cost in dollars of buying 4 DVDs?
Options:
A. (4 + 20) + (4 + 4)
B. (4 × 20) + (4 × 4)
C (4 + 20) × (4 + 4)
D. (4 × 20) × (4 × 4)

Answer:
B. (4 × 20) + (4 × 4)

Explanation:
24 can be written as 25 – 1
4 x 24 = 4 x (20 + 4) = (4 x 20) + (4 x 4)

Question 17.
The Muffin Shop chain of bakeries sold 745,305 muffins last year. Which choice shows that number in expanded form?
Options:
A. (7 × 100,000) + (45 × 10,000) + (3 × 100) + (5 × 10)
B. (7 × 100,000) + (4 × 10,000) + (5 × 1,000) + (5 × 10)
C. (7 × 100,000) + (4 × 10,000) + (5 × 1,000) + (3 × 100) + (5 × 1)
D. (7 × 100,000) + (4 × 10,000) + (3 × 100) + (5 × 1)

Answer:
C. (7 × 100,000) + (4 × 10,000) + (5 × 1,000) + (3 × 100) + (5 × 1)

Explanation:
First, we can write 745,305 as:
700,000 + 40, 000 + 5,000 + 300 + 5
(7 x 100,000) + (4 x 10,000) + (5 x 1,000) + (3 x 100) + 5

Question 18.
The soccer field at Mario’s school has an area of 6,000 square meters. How can Mario show the area as a whole number multiplied by a power of ten?
Options:
A. 6 × 104 sq m
B. 6 × 103 sq m
C. 6 × 102 sq m
D. 6 × 101 sq m

Answer:
B. 6 × 103 sq m

Explanation:
6,000 square meters = 6 x 1,000 = 6 x 10 x 10 x 10 = 6 × 103 sq m

Question 19.
Ms. Alonzo ordered 4,000 markers for her store. Only \(\frac{1}{10}\) of them arrived. How many markers did she receive?
Options:
A. 4
B. 40
C. 400
D. 1,400

Answer:
C. 400

Explanation:
Ms. Alonzo ordered 4,000 markers for her store. Only \(\frac{1}{10}\) of them arrived.
4,000 x \(\frac{1}{10}\) = 400

Question 20.
Mark wrote the highest score he made on his new video game as the product of 70 × 6,000. What was his score?
Options:
A. 420
B. 4,200
C. 42,000
D. 420,000

Answer:
D. 420,000

Explanation:
Mark wrote the highest score he made on his new video game as the product of 70 × 6,000.
(7 x 6) x (10 x 10 x 10 x 10) = 42 x 10,000 = 420,000

Multiply by 1-digit numbers – Share and Show – Page No. 29

Complete to find the product.

Question 1.
6 × 796           Estimate: 6 × ___ = ___
Go Math Grade 5 Answer Key Chapter 1 Place Value, Multiplication, and Expressions Place Value, Multiplication, and Expressions; Multiply by 1-digit numbers img 11
______

Answer:
4,776

Explanation:
Go Math Grade 5 Answer Key Chapter 1 Place Value, Multiplication, and Expressions Place Value, Multiplication, and Expressions; Multiply by 1-digit numbers img 11

Estimate. Then find the product.

Question 2.
Estimate: ___
6 0 8
×   8
———-
Estimate: ________
Product: 608 × 8 = ________

Answer:
Estimate: 6,000
Product: 608 × 8 = 4,864

Explanation:
Estimate: 608 is close to 600; 8 is close to 10
600 x 10 = 6,000
608 x 8
Multiply the ones; 8 x 8 = 64. 4 ones and 6 tens. Write the ones and the
regrouped tens.
Multiply the tens; 0 x 8 = 0 + 6 = 6
Multiply the hundreds; 6 x 8 = 48.
So, 4,864 is the product of 608 × 8
Product: 4,864

Question 3.
Estimate: __
5 5 6
×   4
———–
Estimate: ________
Product: 556 × 4 = ________

Answer:
Estimate: 2,780
Product: 556 × 4 = 2,224

Explanation:
Estimate: 556 is close to 550; 4 is close to 5
556 x 5 = 2,780
556 × 4
Multiply the ones; 6 x 4 = 24. 4 ones and 2 tens. Write the ones and the
regrouped tens.
Multiply the tens; 5 x 4 = 20 + 2 = 22; 2 tens and 2 hundreds. Write the tens and regrouped hundreds.
Multiply the hundreds; 5 x 4 = 20; 20 + 2 = 22.
So, 2,224 is the product of 556 × 4
Product: 2,224

Question 4.
Estimate:
1,925
×    7
———–
Estimate: ________
Product: 1,925 × 7 = ________

Answer:
Estimate: 10,000
Product: 1,925 × 7 = 13,475

Explanation:
Estimate: 1,925 is close to 2000; 7 is close to 5
2,000 x 5 = 10,000
1,925 × 7
Multiply the ones; 7 x 5 = 35. 5 ones and 3 tens. Write the ones and the
regrouped tens.
Multiply the tens; 7 x 2 = 14; 14 + 3 = 17; 7 tens and 1 hundreds. Write the tens and regrouped hundreds.
Multiply the hundreds; 7 x 9 = 63; 63 + 1 = 64. 4 hundred and 6 thousand Write the hundreds and regrouped thousands.
Multiply the thousands; 7 x 1 = 7; 7 + 6 = 13
So, 13,475 is the product of 1,925 × 7
Product: 13,475

On Your Own

Estimate. Then find the product.

Question 5.
Estimate:__
7 9 4
×   3
———-
Estimate: ________
Product:794 × 3 = ________

Answer:
Estimate: 800
Product:794 × 3 = 2,382

Explanation:
Estimate: 794 is close to 800 and 3 is close to 1
800 x 1 = 800
794 x 3 = (700 + 90 + 4) x 3 = (700 x 3) + (90 x 3) + (4 x 3) = 2100 + 270 + 12 = 2,382

Question 6.
Estimate:___
8 2 2
×   6
———-
Estimate: ________
822 × 6 = ________

Answer:
Estimate: 4,000
822 × 6 = 4,932

Explanation:
Estimate: 822 is close to 800 and 6 is close to 5
800 x 5 = 4,000
822 × 6 = (800 + 20 + 2) x 6 = (800 x 6) + (20 x 6) + (2 x 6) = 4800 + 120 + 12 = 4,932

Question 7.
Estimate:
3,102
×    5
———–
Estimate: ________
Product: 3,102 × 5 = ________

Answer:
Estimate: 15,500
Product: 3,102 × 5 = 15,510

Explanation:
Estimate: 3,102 is close to 3,100 and 5 is close to 5
3,100 x 5 = 15,500
3,102 x 5 = (3,000 + 100 + 0 + 2) x 5 = (3000 x 5) + (100 x 5) + 0 + (2 x 5) = 15,000 + 500 + 0 + 10 = 15,510

Algebra Solve for the unknown number.

Question 8.
3 9 6
×   6
———
2, 3 6
396 × 6 = 23 ______ 6

Answer:
7

Explanation:
396 x 6 = (300 + 90 + 6) x 6 = (300 x 6) + (90 x 6) + (6 x 6) = 1800 + 540 + 36 = 2376. So, the unknown number is 7

Question 9.
5,1 2
×   8
——–
16
Type below:
__________

Answer:
5127 x 8 = 41,016.
Unknown numbers = 7 and 410

Explanation:
5,127 x 8 = (5000 + 100 + 20 + 7) x 8 = (5000 x 8) + (100 x 8) + (20 x 8) + (7 x 8) = 40000 + 800 + 160 + 56 = 41,016

Question 10.
8, 5 6
×    7
———
60,03
Type below:
__________

Answer:
8,576 x 7 = 60,032

Explanation:
8,576 x 7 = (8000 + 500 + 70 + 6) x 7 = (8000 x 7) + (500 x 7) + (70 x 7) + (6 x 7) = 56000 + 3500 + 490 + 42 = 60,032

Practice: Copy and Solve Estimate. Then find the product.

Question 11.
116 × 3 = _______
Estimate: _______

Answer:
Estimate: 300
116 × 3 = 348

Explanation:
Estimate: 116 is close to 100;
100 x 3 = 300
116 x 3
6 x 3 =18; add ones and regroup tens
3 x 1 = 3; 3 + 1 = 4
3 x 1 = 3
So, 348 is the product

Question 12.
338 × 4 = _______
Estimate: _______

Answer:
338 × 4 = 1,352
Estimate: 1,200

Explanation:
Estimate: 338 is close to 300;
300 x 4 = 1,200
338 × 4
8 x 4 =32; add ones and regroup tens
3 x 4 = 12; 12 + 3 = 15; add tens and regroup hundreds
3 x 4 = 12; 12 + 1 = 13
So, 1352 is the product

Question 13.
6 × 219 = _______
Estimate: _______

Answer:
6 × 219 = 1,314
Estimate: 1200

Explanation:
Estimate: 219 is close to 200
200 x 6 = 1200
6 × 219
6 x 9 = 54; add ones and regroup tens
6 x 1 = 6; 6 + 5 = 11; add tens and regroup hundreds
6 x 2 = 12; 12 + 1 = 13
So, 1,314

Question 14.
7 × 456 = _______
Estimate: _______

Answer:
7 × 456 = 3192
Estimate: 3500

Explanation:
Estimate: 456 is close to 500
500 x 7 = 3500
7 x 456
7 x 6 = 42; add ones and regroup tens
7 x 5 = 35; 35 + 4 = 39; add tens and regroup hundreds
7 x 4 = 28; 28 + 3 = 31
So, 3192

Question 15.
5 × 1,012 = _______
Estimate: _______

Answer:
5 × 1,012 = 5,060
Estimate: 5,000

Explanation:
Estimate: 1,012 is close to 1,000
1,000 x 5 = 5,000
5 × 1,012
5 x 2 = 10; add ones and regroup tens
5 x 1 = 5; 5 + 1 = 6; add tens and regroup hundreds
5 x 0 = 0
5 x 1 = 5
So, 5,060

Question 16.
2,921 × 3 = _______
Estimate: _______

Answer:
2,921 × 3 = 8,763
Estimate: 9,000

Explanation:
Estimate: 2,921 is close to 3,000
3,000 x 3 = 9,000
2,921 × 3
3 x 1 = 3;
3 x 2 = 6;
3 x 9 = 27; add hundreds and regroup thousands
3 x 2 = 6; 6 + 2 = 8
So, 8,763

Question 17.
8,813 × 4 = _______
Estimate: _______

Answer:
8,813 × 4 = 35,252
Estimate: 3,600

Explanation:
Estimate: 8,813 is close to 9,000
9,000 x 4 = 3,600
8,813 × 4
4 x 3 = 12; add ones and regroup tens
4 x 1 = 4; 4 + 1 = 5;
4 x 8 = 32; add hundreds and regroup thousands
4 x 8 = 32; 32 + 3 = 35
So, 35,252

Question 18.
9 × 3,033 = _______
Estimate: _______

Answer:

Explanation:
Estimate: 3,033 is close to 3,000
3,000 x 9 = 27,000
9 × 3,033
9 x 3 = 27; add ones and regroup tens
9 x 3 = 27; 27 +  = 11; add tens and regroup hundreds
6 x 2 = 12; 12 + 1 = 13
So, 1,314

Multiply by 1-digit numbers – Problem Solving – Page No. 30

What’s the Error?

Question 19.
The Plattsville Glee Club is sending 8 of its members to a singing contest in Cincinnati, Ohio. The cost will be $588 per person. How much will it cost for the entire group of 8 students to attend? Both Brian and Jermaine solve the problem. Brian says the answer is $40,074. Jermaine’s answer is $4,604.
Estimate the cost. A reasonable estimate is _
$ ______

Answer:
$4,800

Explanation:
The Plattsville Glee Club is sending 8 of its members to a singing contest in Cincinnati, Ohio. The cost will be $588 per person.
So, for entire group 8 x $588 = $4,704
Jermaine’s answer is correct. Because the $4,604 is close to $4,704
588 is close to 600. So, 600 x 8 = $4,800

Question 19.
Although Jermaine’s answer seems reasonable, neither Brian nor Jermaine solved the problem correctly. Find the errors in Brian’s and Jermaine’s work.
Then, solve the problem correctly.
Go Math Grade 5 Answer Key Chapter 1 Place Value, Multiplication, and Expressions Place Value, Multiplication, and Expressions; Multiply by 1-digit numbers img 12
What error did Brian make? Explain.
Type below:
__________

Answer:
When Brian multiplied the tens, he wrote the total number of tens in the product instead of regrouping, so the place values of his product are incorrect.

Question 19.
What error did Jermaine make? Explain.
Type below:
__________

Answer:
Jermaine regrouped the wrong amount of hundreds. He regrouped the tens as 6 hundred instead of 7 hundred.
$588 x 8 = $4,704

Question 19.
How could you predict that Jermaine’s answer might be incorrect using your estimate?
Type below:
__________

Answer:
I used 600 × 8 to estimate the product; 588 is 12 less than 600. Since 12 × 8 = 96, and 4,604 is almost 200 less than the estimate of 4,800, the answer is probably too low.

Multiply by 2-digit numbers – Share and Show – Page No. 33

Complete to find the product

Question 1.
Go Math Grade 5 Answer Key Chapter 1 Place Value, Multiplication, and Expressions Place Value, Multiplication, and Expressions; Multiply by 2-digit numbers img 13
Type below:
__________

Answer:
2,752

Explanation:
64 x 3 = 192
64 x 40 = 2,560
2,560 + 192 = 2,752

Question 2.
Go Math Grade 5 Answer Key Chapter 1 Place Value, Multiplication, and Expressions Place Value, Multiplication, and Expressions; Multiply by 2-digit numbers img 14
Type below:
__________

Answer:
21,698

Explanation:
571 x 8 = 4,568
571 x 30 = 17,130
17,130 + 4,568 = 21,698

Estimate. Then find the product.

Question 3.
Estimate:____
2 4
× 1 5
———-
Estimate: ________
Product: ________

Answer:
Estimate: 300
Product: 360

Explanation:
2 4 x 15
Estimate: 20 x 15 = 300
24 x 5 = 120
24 x 10 = 240
Product:: 240 + 120 = 360

Question 4.
Estimate:____
3 7
× 6 3
———-
Estimate: ________
Product: ________

Answer:
Estimate: 2,400
Product: 2,331

Explanation:
37 x 63
Estimate: 40 x 60 = 2,400
37 x 3 = 111
37 x 60 = 2220
Product:: 2220 + 111 = 2,331

Question 5.
Estimate:____
3 8 4
× 4 5
———-
Estimate: ________
Product: ________

Answer:
Estimate: 20,000
Product: 17,280

Explanation:
384 x 45
Estimate: 400 x 50 = 20,000
384 x 5 = 1920
384 x 40 = 15,360
Product:: 15,360 + 1920 = 17,280

On Your Own

Estimate. Then find the product.

Question 6.
Estimate:____
2 8
× 2 2
———-
Estimate: ________
Product: ________

Answer:
Estimate: 600
Product: 616

Explanation:
28 x 22
Estimate: 30 x 20 = 600
28 x 2 = 56
28 x 20 = 560
Product:: 56 + 560 = 616

Question 7.
Estimate:____
9 3
× 7 6
———-
Estimate: ________
Product: ________

Answer:
Estimate: 7200
Product: 7,068

Explanation:
93 x 76
Estimate: 90 x 80 = 7200
93 x 6 = 558
93 x 70 = 6,510
Product:: 558 + 6,510 = 7,068

Question 8.
Estimate:____
2 9 5
× 5 1
———-
Estimate: ________
Product: ________

Answer:
Estimate: 15,000
Product: 15,045

Explanation:
295 x 51
Estimate: 300 x 50 = 15,000
295 x 1 = 295
295 x 50 = 14,750
Product:: 295 + 14,750 = 15,045

Practice: Copy and Solve Estimate. Then find the product.

Question 9.
Estimate: ________
54 × 31 = ________

Answer:
Estimate: 1,500
Product: 1,674

Explanation:
54 x 31
Estimate: 50 x 30 = 1,500
54 x 1 = 54
54 x 30 = 1,620
Product:: 54 + 1,620 = 1,674

Question 10.
Estimate: ________
42 × 26 = ________

Answer:
Estimate: 1,200
Product: 1,092

Explanation:
42 x 26
Estimate: 40 x 30 = 1,200
42 x 6 = 252
42 x 20 = 840
Product:: 252 + 840 = 1,092

Question 11.
Estimate: ________
38 × 64 = ________

Answer:
Estimate: 2,400
Product: 2,432

Explanation:
38 × 64
Estimate: 40 x 60 = 2,400
38 x 4 = 152
38 x 60 = 2,280
Product:: 152 + 2,280 = 2,432

Question 12.
Estimate: ________
63 × 16 = ________

Answer:
Estimate: 1,200
Product: 1,008

Explanation:
63 x 16
Estimate: 60 x 20 = 1,200
63 x 6 = 378
63 x 10 = 630
Product:: 378 + 630 = 1,008

Question 13.
Estimate: ________
204 × 41 = ________

Answer:
Estimate: 8,000
Product: 8,364

Explanation:
204 × 41
Estimate: 200 x 40 = 8,000
204 x 1 = 204
204 x 40 = 8,160
Product:: 204 + 8,160 = 8,364

Question 14.
Estimate: ________
534 × 25 = ________

Answer:
Estimate: 15,000
Product: 13,350

Explanation:
534 x 25
Estimate: 500 x 30 = 15,000
534 x 5 = 2,670
534 x 20 = 10,680
Product:: 2,670 + 10,680 = 13,350

Question 15.
Estimate: ________
722 × 39 = ________

Answer:
Estimate: 28,000
Product: 28,158

Explanation:
722 × 39
Estimate: 700 x 40 = 28,000
722 x 9 = 6,498
722 x 30 = 21,660
Product:: 6,498 + 21,660 = 28,158

Question 16.
Estimate: ________
957 × 43 = ________

Answer:
Estimate: 40,000
Product: 44,022

Explanation:
957 × 43
Estimate: 1,000 x 40 = 40,000
957 x 3 = 2,871
957 x 40 = 41,151
Product:: 2,871 + 41,151 = 44,022

Multiply by 2-digit numbers – Problem Solving – Page No. 34

Use the table for 17–20.

Question 17.
How much sleep does a jaguar get in 1 year?
Go Math Grade 5 Answer Key Chapter 1 Place Value, Multiplication, and Expressions Place Value, Multiplication, and Expressions; Multiply by 2-digit numbers img 15
_____ hours

Answer:
4,004 hours

Explanation:
52 weeks x 77 = 4,004 hours

Question 18.
In 1 year, how many more hours of sleep does a giant armadillo get than a platypus?
_____ hours

Answer:
1,508 hours

Explanation:
giant armadillo: 52 x 127 = 6,604
platypus: 52 x 98 = 5,096
6,604 – 5,096 = 1,508

Question 19.
Owl monkeys sleep during the day, waking about 15 minutes after sundown to find food. At midnight, they rest for an hour or two, then continue to feed until sunrise. They live about 27 years. How many hours of sleep does an owl monkey that lives 27 years get in its lifetime?
_____

Answer:
4927.5 days

Explanation:
Given that the time of Owl monkeys sleeps during the day walking about 15 minutes after sundown. Then, at midnight they rest for an hour or two then continue to feed until sunrise. Notice that the above description doesn’t say that they sleep in after sundown. They either eat or rest.
Day time is usually considered from 6 am to 6 pm which is total of 12 hours.
As per the given information, owl monkey sleeps in that period.
Given that owl, monkeys live for 27 years.
So the time for sleeping in their lifetime = 12 hours * 27 years
= (12/24 days) * 27 years
= (1/2 days) * 27 years
= (1/2 days) * (27*365 days)
= (1/2 days) * (9855 days)
= 4927.5 days
Hence final answer is 4927.5 days.

Question 20.
Three-toed sloths move very slowly, using as little energy as possible. They sleep, eat, and even give birth upside down. A baby sloth may cling to its mother for as much as 36 weeks after being born. How much of that time is the sloth asleep?
_____ hours

Answer:
3,636 hours

Explanation:
101 x 36 = 3,636

Question 21.
Test Prep A sloth’s maximum speed on the ground is 15 feet in 1 minute. Even though it would be unlikely for a sloth to stay in motion for more than a few moments, how far would a sloth travel in 45 minutes at that speed?
Options:
A. 60 feet
B. 270 feet
C. 675 feet
D. 6,750 feet

Answer:
C. 675 feet

Explanation:
sloth’s maximum speed on the ground is 15 feet in 1 minute. For 45 minutes, 45 x 15 = 675 feet

Relate Multiplication to Division – Share and Show – Page No. 37

Question 1.
Brad has 72 toy cars that he puts into 4 equal groups. How many cars does Brad have in each group? Use the array to show your answer.
4 × = 72         72 ÷ 4 =
Go Math Grade 5 Answer Key Chapter 1 Place Value, Multiplication, and Expressions Place Value, Multiplication, and Expressions; Relate Multiplication to Division img 16
4 × ________ = 72
72 ÷ 4 = ________

Answer:
Grade 5 Chapter 1 Image 8

Explanation:
Brad has 18 toy cars in each group.
Because 72÷4 =18
4 × 18 = 72
4 x (9 + 9) = (4 x 9) + (4 x 9)

Use multiplication and the Distributive Property to find the quotient.

Question 2.
108 ÷ 6 = _____

Answer:
18

Explanation:
108 ÷ 6
6 x 18 = 6 x (9 + 9) = (6 x 9) + (6 x 9) = 54 + 54 = 108
(36 + 72) ÷ 6 = (36 ÷ 6) + (72÷ 6) = 6 + 12 = 18

Question 3.
84 ÷ 6 = _____

Answer:
14

Explanation:
84 ÷ 6
(42 + 42) ÷ 6 = (42÷ 6) + (42÷ 6) = 7 + 7 = 14
6 x 14 = 6 x (7 + 7) = (6 x 7) + (6 x 7) = 42 + 42 = 84

Question 4.
184 ÷ 8 = _____

Answer:
23

Explanation:
184 ÷ 8
(92 + 92) ÷ 8 = (92÷ 8) + (92÷ 8) = 11.5 + 11.5 = 23
8 x 23 = 8 x (11 + 12) = (8 x 11) + (8 x 12) = 88 + 96 = 184

On Your Own

Use multiplication and the Distributive Property to find the quotient.

Question 5.
60 ÷ 4 = _____

Answer:
15

Explanation:
60 ÷ 4
(20 + 40) ÷ 4 = (20 ÷ 4) + (40 ÷ 4) = 5 + 10 = 15
4 x 15 = 4 x (7+ 8) = (4 x 7) + (4 x 8) = 28 + 32 = 60

Question 6.
144 ÷ 6 = _____

Answer:
24

Explanation:
144 ÷ 6
(72 + 72) ÷ 6 = (72 ÷ 6) + (72 ÷ 6) = 12 + 12 = 24
6 x 24 = 6 x ( 12 + 12) = (6 x 12) + (6 x 12) = 72 + 72 = 144

Question 7.
252 ÷ 9 = _____

Answer:
28

Explanation:
252 ÷ 9
(126 + 126) ÷ 9 = (126 ÷ 9) + (126 ÷ 9) = 14 + 14 = 28
6 x 28 = 6 x ( 14 + 14) = (6 x 14) + (6 x 14) = 126 + 126 = 252

Find each quotient. Then compare. Write <, > or =.

Question 8.
51 ÷ 3 _____ 68 ÷ 4

Answer:
51 ÷ 3 = 68 ÷ 4

Explanation:
51 ÷ 3 = 17
68 ÷ 4 = 17
17 = 17.

Question 9.
252 ÷ 6 _____ 135 ÷ 3

Answer:
252 ÷ 6 < 135 ÷ 3

Explanation:
252 ÷ 6 = 42
135 ÷ 3 = 45
42 < 45

Question 10.
110 ÷ 5 _____ 133 ÷ 7

Answer:
110 ÷ 5 > 133 ÷ 7

Explanation:
110 ÷ 5 = 22
133 ÷ 7 = 19
22 > 19

Relate Multiplication to Division – Problem Solving – Page No. 38

Use the table to solve 11–13.

Question 11.
A group of 6 friends share a bag of the 45-millimeter bouncy balls equally among them. How many does each friend get?
Go Math Grade 5 Answer Key Chapter 1 Place Value, Multiplication, and Expressions Place Value, Multiplication, and Expressions; Relate Multiplication to Division img 17
____ balls

Answer:
7 balls

Explanation:
A group of 6 friends shares a bag of the 45-millimeter bouncy balls equally among them.
45/6 = 7 balls and 3 balls remained

Question 12.
Mr. Henderson has 2 bouncy-ball vending machines. He buys one bag of the 27-millimeter balls and one bag of the 40-millimeter balls. He puts an equal number of each in the 2 machines. How many bouncy balls does he put in each machine?
________ balls of the 27-millimeter .
________ balls of the 40-millimeter .
________ total balls

Answer:
51 balls of the 27-millimeter
29 balls of the 40-millimeter
80 total balls

Explanation:
There’s 51 27 mm, 29 40 mm

Question 13.
Lindsey buys a bag of each size of bouncy ball. She wants to put the same number of each size of bouncy ball into 5 party-favor bags. How many of each size of bouncy ball will she put in a bag?
________ balls of the 27-millimeter
________ balls of the 40-millimeter
________ balls of the 45-millimeter

Answer:
34 balls of the 27-millimeter
12 balls of the 40-millimeter
34 balls of the 45-millimeter

Explanation:
There’s 34 27 mm, 12 40 mm, and 34 45 mm bouncy balls

Question 14.
What’s the Error? Sandy writes (4 × 30) + (4 × 2) and says the quotient for 128 ÷ 4 is 8. Is she correct? Explain.
128 ÷ 4 = ____

Answer:
Sandy’s answer is incorrect.
128 ÷ 4 = 32

Explanation:
(4 × 30) + (4 × 2) = 120 + 8 = 128
128 ÷ 4 = 32.
(4 × 30) + (4 × 2) = 4 x (30 + 2) = 4 x 32 = 128.
128 ÷ 4 = 32

Question 15.
Test Prep Which of the following can be used to find 150 ÷ 6?
Options:
A. (6 × 20) + (6 × 5)
B. (6 × 10) + (6 × 5)
C. (2 × 75) + (2 × 3)
D. (6 × 15) + (6 × 5)

Answer:
A. (6 × 20) + (6 × 5)

Explanation:
150 ÷ 6 = 25
25 x 6 = 150
6 x 25 = 6 x (20 + 5) = (6 × 20) + (6 × 5)

Problem Solving Multiplication and Division – Share and Show – Page No. 41

Question 1.
To make concrete mix, Monica pours 34 pounds of cement, 68 pounds of sand, 14 pounds of small pebbles, and 19 pounds of large pebbles into a large wheelbarrow. If she pours the mixture into 9 equalsize bags, how much will each bag weigh?
First, find the total weight of the mixture.
____ pounds

Answer:
135 pounds

Explanation:
34 + 68 + 14 + 19 = 135

Question 1.
Then, divide the total by the number of bags. Break the total into two simpler numbers to make the division easier, if necessary.
Finally, find the quotient and solve the problem.
So, each bag will weigh _ pounds.
____ pounds

Answer:
15 pounds

Explanation:
now, calculate 135/9 = 15 pounds.

Question 2.
What if Monica pours the mixture into 5 equal-size bags? How much will each bag weigh?
____ pounds

Answer:
27 pounds

Explanation:
135/5 = 27

Question 3.
Taylor is building doghouses to sell. Each doghouse requires 3 full sheets of plywood which Taylor cuts into new shapes. The plywood is shipped in bundles of 14 full sheets. How many doghouses can Taylor make from 12 bundles of plywood?
____

Answer:
56 doghouses

Explanation:
Taylor is building doghouses to sell. Each doghouse requires 3 full sheets of plywood which Taylor cuts into new shapes. The plywood is shipped in bundles of 14 full sheets.
(12 x 14)/3 = 168/3 = 56 doghouses

Question 4.
Eileen is planting a garden. She has seeds for 60 tomato plants, 55 sweet corn plants, and 21 cucumber plants. She plants them in 8 rows, with the same number of plants in each row. How many seeds are planted in each row?
____ seeds / row

Answer:
17 seeds

Explanation:
Eileen is planting a garden. She has seeds for 60 tomato plants, 55 sweet corn plants, and 21 cucumber plants. She plants them in 8 rows, with the same number of plants in each row.
60 + 55 + 21 = 136
136/8 = 17

Problem Solving Multiplication and Division – On Your Own – Page No. 42

Question 5.
Starting on day 1 with 1 jumping jack, Keith doubles the number of jumping jacks he does every day. How many jumping jacks will Keith do on day 10?
____  jumping jack

Answer:
512 jumping jacks

Explanation:
She doubled the number so you’re supposed to multiply by 2.
On day 1, 1 jumping pack;
On day 2, 2 jumping packs;
On day 3, 2 x 2 = 4 jumping packs;
On day 4, 2 x 2 x 2 = 8 jumping packs;
On day 5, 2 x 2 x 2 x 2 = 16 jumping packs;
On day 6, 2 x 2 x 2 x 2 x 2 = 32 jumping packs;
On day 7, 2 x 2 x 2 x 2 x 2 x 2 = 64 jumping packs;
On day 8, 2 x 2 x 2 x 2 x 2 x 2 x 2 = 128 jumping packs;
On day 9, 2 x 2 x 2 x 2 x 2 x 2 x 2 x 2 = 256 jumping packs;
On day 10, 2 x 2 x 2 x 2 x 2 x 2 x 2 x 2 x 2 = 512 jumping packs;
512 jumping jacks

Question 6.
Starting in the blue square, in how many different ways can you draw a line that passes through every square without picking up your pencil or crossing a line you’ve already drawn? Show the ways.
Go Math Grade 5 Answer Key Chapter 1 Place Value, Multiplication, and Expressions Place Value, Multiplication, and Expressions; Problem Solving Multiplication and Division img 18
____ ways

Answer:
8 ways

Explanation:
Grade 5 Chapter 1 Image 9
8 ways

Question 7.
On April 11, Millie bought a lawn mower with a 50-day guarantee. If the guarantee begins on the date of purchase, what is the first day on which the mower will no longer be guaranteed?
__________

Answer:
May 31

Explanation:
The guarantee begins on April 11. April has 30 days. So, we have 20 days of the guarantee in April.
May has 31 days. So, we have 30 days of the guarantee in May.
Therefore, the last day of the guarantee is May 31.

Question 8.
A classroom bulletin board is 7 feet by 4 feet. If there is a picture of a student every 6 inches along the edge, including one in each corner, how many pictures are on the bulletin board?
____ pictures

Answer:
40 pictures

Explanation:
2 pictures per foot, so that’s 14 pics along the top and bottom (counting the corners), and
6 pictures along each side (not counting the corners)
14+14+6+6 = 40

Question 9.
Dave wants to make a stone walkway. The rectangular walkway is 4 feet wide and 12 feet long. Each 2 foot by 2 foot stone covers an area of 4 square feet. How many stones will Dave need to make his walkway?
____ stones

Answer:
12 stones

Explanation:
Area of walkway = 4 x 12 = 48 square feet
Area of 1 stone = 2 x 2 = 4 square feet
48/4 = 12 stones.

Question 10.
Test Prep Dee has 112 minutes of recording time. How many 4-minute songs can she record?
Options:
A. 28
B. 27
C. 18
D. 17

Answer:
A. 28

Explanation:
Dee has 112 minutes of recording time. 112/4 = 28

Numerical Expressions – Share and Show – Page No. 45

Circle the expression that matches the words.

Question 1.
Teri had 18 worms. She gave 4 worms to Susie and 3 worms to Jamie.
(18 – 4) + 3         18 – (4 + 3)
__________

Answer:
18 – (4 + 3)

Explanation:
Teri had 18 worms. She gave 4 worms.
18 – 4 = 14.
3 worms to Jamie
14 – 3 = 11.
(18 – 4) + 3 = 14 + 3 = 17 not equal to 11;
18 – (4 + 3) = 18 – 7 = 11 = 11

Question 2.
Rick had $8. He then worked 4 hours for $5 each hour.
$8 + (4 × $5)       ($8 + 4) × $5
__________

Answer:
$8 + (4 × $5)

Explanation:
Rick had $8. He then worked 4 hours for $5 each hour = 4 x $5 = $20;
$20 +$8 =$28.
$8 + (4 × $5) = 8 + 20 = $28 = $28.

Write an expression to match the words.

Question 3.
Greg drives 26 miles on Monday and 90 miles on Tuesday.
Type below:
__________

Answer:
26 + 90 = 116 miles

Explanation:
Greg drives 26 miles on Monday and 90 miles on Tuesday. 26 + 90 = 116 miles

Question 4.
Lynda has 27 fewer fish than Jack. Jack has 80 fish.
Type below:
__________

Answer:
80 – 27 = 53

Explanation:
Lynda has 27 fewer fish than Jack. Jack has 80 fish. 80 – 27 = 53

Write words to match the expression.

Question 5.
34 – 17
Type below:
__________

Answer:
Anna has 17 fewer apples than Jack. Jack has 34 apples.

Explanation:
34 – 17 = 17.
Anna has 17 fewer apples than Jack. Jack has 34 apples.

Question 6.
6 × (12 – 4)
Type below:
__________

Answer:
Teri had 12 worms. She gave 4 worms to Susie. She sell remaining worms for $6 each.

Explanation:
Teri had 12 worms. She gave 4 worms to Susie. She sell remaining worms for $6 each.

On Your Own

Write an expression to match the words.

Question 7.
José shared 12 party favors equally among 6 friends.
Type below:
__________

Answer:
12/6 = 2

Explanation:
José shared 12 party favors equally among 6 friends. 12/6 = 2

Question 8.
Braden has 14 baseball cards. He finds 5 more baseball cards.
Type below:
__________

Answer:
14 + 5 = 19

Explanation:
Braden has 14 baseball cards. He finds 5 more baseball cards. 14 + 5 = 19

Question 9.
Isabelle bought 12 bottles of water at $2 each.
Type below:
__________

Answer:
12 x $2 = $24

Explanation:
Isabelle bought 12 bottles of water at $2 each. 12 x $2 = $24

Question 10.
Monique had $20. She spent $5 on lunch and $10 at the bookstore.
Type below:
__________

Answer:
20 – (5 + 10)

Explanation:
Monique had $20. She spent $5 on lunch and $10 at the bookstore. 20 – (5 + 10)

Write words to match the expression.

Question 11.
36 ÷ 9
Type below:
__________

Answer:
Anna shared 36 apples to 9 friends.

Question 12.
35 – (16 + 11)
Type below:
__________

Answer:
Monique had 35 balls. She gave 16 to her one frined and 11 to other friend.

Draw a line to match the expression with the words.

Question 13.
Fred catches 25 fish. Then he
releases 10 fish and catches 8 more.     •        •3 × (15 – 6)

Nick has 25 pens. He gives 10 pens to
one friend and 8 pens to another friend.  •     •15 – 6

Jan catches 15 fish and lets 6 fish go.       •     •25 – (10 + 8)

Libby catches 15 fish and lets 6 fish go
for three days in a row.                             •     •(25 – 10) + 8
Type below:
__________

Answer:
Fred catches 25 fish. Then he releases 10 fish and catches 8 more = (25 – 10) + 8
Nick has 25 pens. He gives 10 pens to one friend and 8 pens to another friend = 25 – (10 + 8)
Jan catches 15 fish and lets 6 fish go = 15 – 6
Libby catches 15 fish and lets 6 fish go for three days in a row = 3 × (15 – 6)

Numerical Expressions – Problem Solving – Page No. 46

Use the rule and the table for 14.
Go Math Grade 5 Answer Key Chapter 1 Place Value, Multiplication, and Expressions Place Value, Multiplication, and Expressions; Numerical Expressions img 19

Question 14.
Write a numerical expression to represent the total number of lemon tetras that could be in a 20-gallon aquarium.
_____ lemon tetras

Answer:
2 lemon tetras

Explanation:
From the given data, increase each inch of lenth, 1 gallon of water increases.
2 + 3 + 5 + 3 + 5 = 18 inches. So, 18 gallon of water available in aquarium. Add 1 lemon tetra to get the 20 gallon of water. So, in total there are 2 lemon tetra available.

Question 15.
Write a word problem for an expression that is three times as great as (15 + 7). Then write the expression.
Type below:
__________

Answer:
3 x (15 + 7)

Explanation:
Given that that is three times as great as (15 + 7)

Question 16.
What’s the Question? Lu has 3 swordtails in her aquarium. She buys 2 more swordtails.
Type below:
__________

Answer:
3 + 2 = 5

Explanation:
Lu has 3 swordtails in her aquarium. She buys 2 more swordtails. So, 3 + 2 = 5 swordtails

Question 17.
Tammy gives 45 stamps to her 9 friends. She shares them equally among her friends. Write an expression to match the words. How many stamps does each friend get?
_____ stamps

Answer:
45/9 = 5 stamps

Explanation:
Tammy gives 45 stamps to her 9 friends. 45/9 = 5.

Question 18.
Test Prep Josh has 3 fish in each of 5 buckets. Then he releases 4 fish. Which expression matches the words?
Options:
a. (3 × 4) – 5
b. (5 × 4) – 3
c. (5 × 3) – 4
d. (5 – 3) × 4

Answer:
c. (5 × 3) – 4

Explanation:
Josh has 3 fish in each of 5 buckets. 3 x 5.
Then he releases 4 fish. (3 x 5) – 4

Evaluate Numerical Expressions – Share and Show – Page No. 49

Evaluate the numerical expression.

Question 1.
10 + 36 ÷ 9
_____

Answer:
14

Explanation:
10 + 36 ÷ 9 = 10 + (36 ÷ 9) = 10 + 4 = 14

Question 2.
10 + (25 – 10) ÷ 5
_____

Answer:
13

Explanation:
10 + (25 – 10) ÷ 5 = 10 + 15 ÷ 5 = 10 + (15 ÷ 5) = 10 + 3 = 13

Question 3.
9 – (3 × 2) + 8
_____

Answer:
11

Explanation:
9 – (3 × 2) + 8 = 9 – 6 + 8 = 3 + 8 = 11

On Your Own

Evaluate the numerical expression.

Question 4.
(4 + 49) – 4 × 10
_____

Answer:
13

Explanation:
(4 + 49) – 4 × 10 = 53 – 4 x 10 = 53 – (4 x 10) = 53 – 40 = 13

Question 5.
5 + 17 – 100 ÷ 5
_____

Answer:

Explanation:
5 + 17 – 100 ÷ 5 = (5 + 17) – (100 ÷ 5) = 22 – 20 = 2

Question 6.
36 – (8 + 5)
_____

Answer:
23

Explanation:
36 – (8 + 5) = 36 – 13 = 23

Question 7.
125 – (68 + 7)
_____

Answer:
50

Explanation:
125 – (68 + 7) = 125 – 75 = 50

Question 8.
(4 × 6) – 12
_____

Answer:
12

Explanation:
(4 × 6) – 12 = 24 – 12 = 12

Question 9.
3 × (22 – 2)
_____

Answer:
60

Explanation:
3 × (22 – 2) = 3 x 20 = 60

Question 10.
23 + (16 – 7)
_____

Answer:
32

Explanation:
23 + (16 – 7) = 23 + 9 = 32

Question 11.
(25 – 4) ÷ 3
_____

Answer:
7

Explanation:
(25 – 4) ÷ 3 = 21 ÷ 3 = 7

Rewrite the expression with parentheses to equal the given value.

Question 12.
100 – 30 ÷ 5
value: 14
Type below:
__________

Answer:
(100 – 30) ÷ 5 = 14

Explanation:
100 – 30 ÷ 5 = (100 – 30) ÷ 5 = 70 ÷ 5 = 14

Question 13.
12 + 17 – 3 × 2
value: 23
Type below:
__________

Answer:
23

Explanation:
(12 + 17) – (3 × 2) = 29 – 6 = 23

Question 14.
9 + 5 ÷ 5 + 2
value: 12
Type below:
__________

Answer:
12

Explanation:
9 + (5 ÷ 5) + 2 = 9 + 1 + 2 = 12

Evaluate Numerical Expressions – UNLOCK the Problem – Page No. 50

Question 15.
A movie theater has 4 groups of seats. The largest group of seats, in the middle, has 20 rows, with 20 seats in each row. There are 2 smaller groups of seats on the sides, each with 20 rows and 6 seats in each row. A group of seats in the back has 5 rows, with 30 seats in each row. How many seats are in the movie theater?
Go Math Grade 5 Answer Key Chapter 1 Place Value, Multiplication, and Expressions Place Value, Multiplication, and Expressions; Evaluate Numerical Expressions img 20
a. What do you need to know?
Type below:
__________

Answer:
We have to know the number of seats in each row to calculate the total number of seats.

Question 15.
b. What operation can you use to find the number of seats in the back group of seats? Write the expression.
Type below:
__________

Answer:
A group of seats in the back has 5 rows with 30 seats in each row. So, to calculate the number of seats, we can use multiplication.

Question 15.
c. What operation can you use to find the number of seats in both groups of side seats? Write the expression.
Type below:
__________

Answer:
2 x (20 x 6) = 2 x 120 = 240

Question 15.
d. What operation can you use to find the number of seats in the middle group? Write the expression.
Type below:
__________

Answer:
20 x 20 = (2 x 2) x (10 x 10) = 4 x 100 = 400

Question 15.
e. Write an expression to represent the total number of seats in the theater.
Type below:
__________

Answer:
(20 x 20) + (2 x 20 x 6) + (5 x 30)

Question 15.
f. How many seats are in the theater? Show the steps you use to solve the problem.
_____ seats

Answer:
(20 x 20) + (2 x 20 x 6) + (5 x 30) = 400 + 240 + 150 = 640 + 150 = 790

Question 16.
Test Prep In the wild, an adult giant panda eats about 30 pounds of food each day. Which expression shows how many pounds of food 6 pandas eat in 3 days?
Options:
a. 3 + (30 × 6)
b. 3 × (30 × 6)
c. (30 × 6) ÷ 3
d. (30 × 6) – 3

Answer:
b. 3 × (30 × 6)

Explanation:
1 panda eats 30 pounds of food each day.
for 3 days, 3 x 30 = 90
1 panda eats 90 pounds of food in 3 days.
6 pandas can eat 90 x 6 = 540 pounds.
3 + (180) = 183 not equal to 540 pounds.
3 x (30 x 6) = 3 x (180) = 540

Question 17.
Test Prep Which expression has a value of 6?
Options:
a. (6 ÷ 3) × 4 + 8
b. 27 – 9 ÷ 3 × ( 4 + 1)
c. (18 + 12) × 6 – 4
d. 71 – 5 × (9 + 4)

Answer:
d. 71 – 5 × (9 + 4)

Explanation:
(6 ÷ 3) × 4 + 8 = 2 x 4 + 8 = 8 + 8 = 16
27 – 9 ÷ 3 × ( 4 + 1) = 27 – (9 ÷ 3) × 5 = 27 – 3 x 5 = 27 – 15 = 12
(18 + 12) × 6 – 4 = 180 – 4 = 176
71 – 5 × (9 + 4) = 71 – (5 x 13) = 71 – 65 = 6

Grouping Symbols – Share and Show – Page No. 53

Evaluate the numerical expression.

Question 1.
12 + [(15 – 5) + (9 – 3)]
12 + [10 + ____]
12 +____
____

Answer:
12 + [(15 – 5) + (9 – 3)]
12 + [10 + 6]
12 + 16
28

Question 2.
5 × [(26 – 4) – (4 + 6)]
____

Answer:
5 × [(26 – 4) – (4 + 6)]
5 x [22 – 10]
5 x 12 = 60

Question 3.
36 ÷ [(18 – 10) – (8 – 6)]
____

Answer:
36 ÷ [(18 – 10) – (8 – 6)]
36 ÷ [8 – 2]
36 ÷ 6 = 6

On Your Own

Evaluate the numerical expression.

Question 4.
4 + [(16 – 4) + (12 – 9)]
____

Answer:
4 + [(16 – 4) + (12 – 9)]
4 + [12 + 3]
4 + 15 = 19

Question 5.
24 – [(10 – 7) + (16 – 9)]
____

Answer:
24 – [(10 – 7) + (16 – 9)]
24 – [3 + 7] = 24 – 10 = 14

Question 6.
16 ÷ [(13 + 7) – (12 + 4)]
____

Answer:
16 ÷ [(13 + 7) – (12 + 4)]
16 ÷ [20 – 16]
16 ÷ 4 = 4

Question 7.
5 × [(7 – 2) + (10 – 8)]
____

Answer:
5 × [(7 – 2) + (10 – 8)]
5 x [5 + 2]
5 x 7 = 35

Question 8.
[(17 + 8) + (29 – 12)] ÷ 6
____

Answer:
[(17 + 8) + (29 – 12)] ÷ 6
[25 + 17] ÷ 6
42 ÷ 6 = 7

Question 9.
[(6 × 7) + (3 × 4)] – 28
____

Answer:
[(6 × 7) + (3 × 4)] – 28
[42 + 12] – 28
54 – 28 = 26

Question 10.
3 × {[(12 – 8) × 2] + [(11 – 9) × 3]}
____

Answer:
3 × {[(12 – 8) × 2] + [(11 – 9) × 3]}
3 x {[4 x 2] + [2 x 3]}
3 x {8 + 6}
3 x 14 = 42

Question 11.
{[(3 × 4) + 18] + [(6 × 7) – 27]} ÷ 5
____

Answer:
{[(3 × 4) + 18] + [(6 × 7) – 27]} ÷ 5
{[12 + 18] + [42 – 27]} ÷ 5
{30 + 15} ÷ 5
45 ÷ 5 = 9

Grouping Symbols – UNLOCK the Problem – Page No. 54

Question 12.
Dan has a flower shop. Each day he displays 24 roses. He gives away 10 and sells the rest. Each day he displays 36 carnations. He gives away 12 and sells the
rest. What expression can you use to find out how many roses and carnations Dan sells in a week?
Go Math Grade 5 Answer Key Chapter 1 Place Value, Multiplication, and Expressions Place Value, Multiplication, and Expressions; Grouping Symbols img 21
a. What information are you given?
Type below:
__________

Answer:
Each day, Dan displays 24 roses (gives away 10 and sells the rest), and he displays 36 carnations (gives away 12 and selles the rest).

Question 12.
b. What are you being asked to do?
Type below:
__________

Answer:
We have to find out how many roses and carnations Dan sells in a week.

Question 12.
c. What expression shows how many roses Dan sells in one day?
Type below:
__________

Answer:
24 – 10 = 14 roses

Question 12.
d. What expression shows how many carnations Dan sells in one day?
Type below:
__________

Answer:
36 – 12 = 24 carnation

Question 12.
e. Write an expression to represent the total number of roses and carnations Dan sells in one day.
Type below:
__________

Answer:
(24 – 10) + (36 – 12) = 14 + 24 = 38 roses and carnations

Question 12.
f. Write the expression that shows how many roses and carnations Dan sells in a week.
Type below:
__________

Answer:
7 x ((24 – 10) + (36 – 12)) = 7 x (14 + 24) = 7 x 38 = 266 roses and carnations

Question 13.
Evaluate the expression to find out how many roses and carnations Dan sells in a week.
____ flowers

Answer:
7 x ((24 – 10) + (36 – 12))
7 x (14 + 24)
7 x 38 = 266 roses and carnations

Explanation:

Question 14.
Test Prep Which expression has a value of 4?
Options:
a. [(4 × 5) + (9 + 7)] + 9
b. [(4 × 5) + (9 + 7)] ÷ 9
c. [(4 × 5) – (9 + 7)] × 9
d. [(4 + 5) + (9 + 7)] – 9

Answer:
b. [(4 × 5) + (9 + 7)] ÷ 9

Explanation:
[(4 × 5) + (9 + 7)] + 9 = [20 + 16] + 9 = 36 + 9 = 45
[(4 × 5) + (9 + 7)] ÷ 9 = [20 + 16] ÷ 9 = 36 ÷ 9 = 4

Chapter Review/Test – Vocabulary – Page No. 55

Go Math Grade 5 Answer Key Chapter 1 Place Value, Multiplication, and Expressions Chapter Review/Test img 22

Question 1.
The ____ states that multiplying a sum by a number is the same as multiplying each addend in the sum by the number and then adding the products.
__________

Answer:
Distributive Property

Concepts and Skills

Complete the sentence.

Question 2.
7,000 is 10 times as much as _____

Answer:
700

Explanation:
Let the unknown number is S.
7,000 = 10S
S = 7,000/10 = 700.
7,000 is 10 times as much as 700.

Question 3.
50 is \(\frac{1}{10}\) of _____

Answer:
500

Explanation:
Let the unknown number is S.
50 = \(\frac{1}{10}\) S
S = 50 X 10 = 500

Complete the equation, and tell which property you used.

Question 4.
4 × (12 + 14) =
Type below:
________

Answer:
104; Distributive Property of Multiplication

Explanation:
4 × (12 + 14) = (4 x 12) + (4 x 14) = 48 + 56 = 104; Distributive Property of Multiplication

Question 5.
45 + 16 =__ + 45
Type below:
Type below:
________

Answer:
45 + 16 = 16 + 45; Commutative Property of Addition

Explanation:
45 + 16 = 16 + 45; Commutative Property of Addition

Find the value.

Question 6.
102 = _____

Answer:
100

Explanation:
100 = 1;
101 = 1 x 10 = 10;
102 = 10 x 10 = 100;

Question 7.
3 × 104 = _____

Answer:
30,000

Explanation:
3 × 104
100 = 1;
101 = 1 x 10 = 10;
102 = 10 x 10 = 100;
103 = 10 x 10 x 10 = 1,000;
104 = 10 x 10 x 10 x 10 = 10,000;
3 x 10,000 = 30,000

Question 8.
8 × 103 = _____

Answer:
8,000

Explanation:
8 × 103
100 = 1;
101 = 1 x 10 = 10;
102 = 10 x 10 = 100;
103 = 10 x 10 x 10 = 1,000;
8 x 1,000 = 8,000

Estimate. Then find the product.

Question 9.
Estimate:
5 7 9
×   6
———-
Estimate: _____ Product: _____

Answer:
Estimate: 3,600
Product: 3,474

Explanation:
Estimate: 579 is close to 600;
600 x 6 = 3,600
579 x 6
6 x 9 =54; add ones and regroup tens
6 x 7 = 42; 42 + 5 = 47; add tens and regroup hundreds
6 x 5 = 30; 30 + 4 = 34
So, 3,474 is the product

Question 10.
Estimate:
7,316
×    6
———-
Estimate: _____ Product: _____

Answer:
Estimate: 42,000
Product: 43,986

Explanation:
Estimate: 7,316 is close to 7,000;
7,000 x 6 = 42,000
7,316 x 6
6 x 6 = 36; add ones and regroup tens
6 x 1 = 6; 6 + 3 = 18; add tens and regroup hundreds
6 x 3 = 18; 18 + 1 = 19; add hundreds and regroup thousads
6 x 7 = 42; 42 + 1 = 43
So, 43,986 is the product

Question 11.
Estimate:
4 3 6
× 3 2
———-
Estimate: _____ Product: _____

Answer:
Estimate: 12,000
Product: 13,952

Explanation:
Estimate: 436 is close to 400; 32 is close to 30
400 x 30 = 12,000
436 x 32;
436 x 2 = 872
436 x 30 = 13,080
13,080 + 872 = 13,952

Use multiplication and the Distributive Property to find the quotient.

Question 12.
54 ÷ 3 = _____

Answer:
18

Explanation:
54 ÷ 3
(27 + 27) ÷ 3 = (27 ÷ 3) + (27 ÷ 3) = 9 + 9 = 18
3 x 18 = 3 x (9+ 9) = (3 x 9) + (3 x 9) = 27 + 27 = 54

Question 13.
90 ÷ 5 = _____

Answer:
18

Explanation:
90 ÷ 5
(45 + 45) ÷ 5 = (45 ÷ 5) + (45 ÷ 5) = 9 + 9 = 18
5 x 18 = 5 x (9+ 9) = (5 x 9) + (5 x 9) = 45 + 45 = 90

Question 14.
96 ÷ 6 = _____

Answer:

Explanation:
96 ÷ 6
(48 + 48) ÷ 6 = (48 ÷ 6) + (48 ÷ 6) = 8 + 8 = 16
6 x 16 = 6 x (8 + 8) = (6 x 8) + (6 x 8) = 48 + 48 = 96

Evaluate the numerical expression.

Question 15.
42 − (9 + 6) = _____

Answer:
42 − (9 + 6)
42 – 15
27

Question 16.
15 + (22 − 4) ÷ 6 = _____

Answer:
15 + (22 − 4) ÷ 6
15 + (18 ÷ 6)
15 + 3
18

Question 17.
6 × [(5 × 7) − (7 + 8)] = _____

Answer:
6 × [(5 × 7) − (7 + 8)]
6 x [35 – 15]
6 x [20]
120

Chapter Review/Test – Page No. 56

Fill in the bubble completely to show your answer.

Question 18.
Erica’s high score on her new video game is 30,000 points. Maria’s high score is \(\frac{1}{10}\) of Erica’s. How many points did Maria score?
Options:
A. 30
B. 300
C. 3,000
D. 30,000

Answer:
C. 3,000

Explanation:
Erica’s high score on her new video game is 30,000 points. Maria’s high score is \(\frac{1}{10}\) of Erica’s.
30,000 x \(\frac{1}{10}\) = 3,000

Question 19.
Rich makes $35 a week mowing lawns in his neighborhood. Which expression can be used to show how much money he makes in 8 weeks?
Options:
A. (8 × 30) + (8 × 5)
B. (8 × 30) + (8 × 5)
C. (8 + 30) × (8 + 5)
D. (8 × 30) × (8 × 5)

Answer:
B. (8 × 30) + (8 × 5)

Explanation:
Rich makes $35 a week mowing lawns in his neighborhood.
8 x $35 = 8 x (30 + 5) = (8 x 30) + (8 x 5)

Question 20.
Mr. Rodriguez bought a supply of 20 reams of printer paper. Each ream contains 500 sheets of paper. How many sheets of printer paper are there?
Options:
A. 1,000
B. 5,000
C. 10,000
D. 100,000

Answer:
C. 10,000

Explanation:
Mr. Rodriguez bought a supply of 20 reams of printer paper. Each ream contains 500 sheets of paper.
500 x 20 = 10,000

Question 21.
Harvester ants are common in the southwestern United States. A single harvester ant colony may have as many as 90,000 members. What is that number written as a whole number multiplied by a power of ten?
Options:
A. 9 × 104
B. 9 × 103
C. 9 × 102
D. 9 × 101

Answer:
A. 9 × 104

Explanation:
Harvester ants are common in the southwestern United States. A single harvester ant colony may have as many as 90,000 members.
90,000
100 = 1;
101 = 1 x 10 = 10;
102 = 10 x 10 = 100;
103 = 10 x 10 x 10 = 1,000;
104 = 10 x 10 x 10 x 10 = 10,000;
9 x 10,000 = 90,000 = 9 x 104

Chapter Review/Test – Page No. 57

Fill in the bubble completely to show your answer.

Question 22.
Megan used the following expression to find the quotient of a division problem.
(4 × 12) + (4 × 6)
What was the division problem and the quotient?
Options:
A. 24 ÷ 4 = 6
B. 48 ÷ 4 = 12
C. 64 ÷ 4 = 16
D. 72 ÷ 4 = 18

Answer:
D. 72 ÷ 4 = 18

Explanation:
Megan used (4 × 12) + (4 × 6)
4 x (12 + 6)
4 x 18 = 72
72 ÷ 4 = 18

Question 23.
It is 1,325 feet from Kinsey’s house to her school. Kinsey walks to school each morning and gets a ride home each afternoon. How many feet does Kinsey walk to school in 5 days?
Options:
A. 6,725 feet
B. 6,625 feet
C. 6,525 feet
D. 5,625 feet

Answer:
B. 6,625 feet

Explanation:
It is 1,325 feet from Kinsey’s house to her school. Kinsey walks to school each morning and gets a ride home each afternoon.
1,325 feet x 5 = 6,625 feet

Question 24.
An adult elephant eats about 300 pounds of food each day. Which expression shows about how many pounds of food a herd of 12 elephants eats in 5 days?
Options:
A. 5 + (300 × 12)
B. 5 × (300 × 12)
C. (300 × 12) ÷ 5
D. (300 × 12) − 5

Answer:
B. 5 × (300 × 12)

Explanation:
An adult elephant eats about 300 pounds of food each day.
(300 x 12)
In 5 days, 5 × (300 × 12)

Question 25.
Carla can type 265 characters a minute on her computer keyboard. At that rate, how many characters can she type in 15 minutes?
Options:
A. 2,975
B. 3,875
C. 3,905
D. 3,975

Answer:
D. 3,975

Explanation:
Carla can type 265 characters a minute on her computer keyboard.
In 15 minutes, 265 x 15 = 3,975

Chapter Review/Test – Page No. 58

Constructed Response

Question 26.
Donavan copied the problem below from the board. He missed one of the numbers needed to show his work. What number is missing in his work? Explain how you found the missing number
Go Math Grade 5 Answer Key Chapter 1 Place Value, Multiplication, and Expressions Chapter Review/Test img 23
17 × 5 = ( ______ +7) × 5
= ( ______ × 5) + (7 × 5)

Answer:
Missing number is 10. Using the Distributive Property, the sum of the two number within the () has to be equal to the other factor 17.
10 + 7 = 17
17 × 5 = ( 10 +7) × 5
= ( 10 × 5) + (7 × 5)

Performance Task

Question 27.
Drew’s weekly allowance is $8.00. His friend Jan’s weekly allowance is $10. Drew spends $3 a week and Jan spends $4 a week.
A. Write two expressions to show how much money each person has at the end of the week. Use parentheses.
Drew has __ .
Jan has __ .
Type below:
__________

Answer:
Drew has ($8 – $3) .
Jan has ($10 – $4).

Question 27.
B. Drew and Jan decide that they want to put their money together to buy a video game. Write an expression that shows how much they can save each week. Explain.
Type below:
__________

Answer:
(8 – 3) + (10 – 4); Add the amount of money Drew and Jan have at the end of the week. This equals the amount they can save a week.

Question 27.
C. The video game Drew and Jan want to buy costs $55. Write an expression to show how many weeks it will take them to save enough to buy the video game. Use parentheses and brackets in your expression. Then evaluate the expression.
_____ weeks

Answer:
55 ÷ [(8 – 3) +(10 – 4)]
55 ÷ [5 + 6]
55 ÷ 11
5
It will take them 5 weeks to save the money from the game.

Conclusion:

You need not look everywhere to seek assistance regarding Go Math Grade 5 Ch 1. Simply access Go Math Grade 5 Answer Key Chapter 1 Place Value, Multiplication, and Expressions and know how to approach while solving different problems on the related concepts.

Go Math Grade 3 Answer Key Chapter 1 Addition and Subtraction within 1,000

go-math-grade-3-chapter-1-addition-and-subtraction-within-1-000-answer-key

Become a master in maths taking the help of Go Math Grade 3 Answer Key Chapter 1 Addition and Subtraction within 1000. Begin your preparation and learn all the fundamental topics in it. Solve all the Problems in Chapter 1 easily from here and understand the concept behind them. Download Go Math Grade 3 Answer Key Chapter 1 Addition and Subtraction within 1,000 free of cost and learn the fundamentals easily.

Go Math Grade 3 Answer Key Chapter 1 Addition and Subtraction within 1,000

You can make the most out of the Grade 3 Go Math Solutions Key Chapter 1 through the quick links available. Simply tap on the topic you want and learn various questions involved in it easily. There are different models in addition and subtraction. We have listed all of them by considering enough examples and explained every problem step by step.

Lesson 1: Number Patterns 

Lesson 2: Round to the Nearest Ten

Lesson 3: Estimate Sums

Lesson 4: Mental Math Strategies for Addition

Lesson 5: Use Properties to Add

Lesson 6: Use the Break Apart Strategy to Add

Lesson 7: Use Place Value to Add

Mid Chapter Check Point

Lesson 8: Estimate Differences

Lesson 9: Mental Math Strategies for Subtraction

Lesson 10: Use Place Value to Subtract

Lesson 11: Combine Place Values to Subtract

Lesson 12: Problem Solving • Model Addition and Subtraction

Review/Test

Number Patterns – Page No. 9

Find the sum. Then use the Commutative Property of Addition to write the related addition sentence.
Question 1:
9 + 2 =  11 

Answer:

What is the commutative property of addition?
To “commute” means to move around or travel.
According to the commutative property of addition, changing the order of the numbers we are adding, does not change the sum.
if you are adding nine and two together, the commutative property of addition says that you will get the same answer whether you are adding
9 + 2 or 2 + 9.
2 + 9 =  11 

Question 2:
4 + 7 = 
    +      =  11

Answer:

If you are adding four and seven together, the commutative property of addition says that you will get the same answer whether you are adding 4 + 7 or 7 + 4.
4 + 7 =  11
7  +  4  =  11

Question 3:
3 + 6 =
     +      =  11

Answer:

According to the commutative property of addition, changing the order of the numbers we are adding, does not change the sum.
There will be no change in the sum whether you add 3 + 6 or 6 + 3.
3 + 6 =  9
 6  +  3  =  9

Question 4:
3 + 10 =
     +      =  11

Answer:

According to the commutative property of addition, changing the order of the numbers we are adding, does not change the sum.
There will be no change in the sum whether you add 3 + 10 or 10 + 3 = 13.
3 + 10 =  13
 10  +  3  =  13

Question 5:
6 + 7 =
     +      =  13

Answer:

According to the commutative property of addition, changing the order of the numbers we are adding, does not change the sum.
There will be no change in the sum whether you add 6 + 7 or 7 + 6 = 13.
6 + 7 =  13
 7  +  6  =  13

Question 6:
7 + 5 =
     +      =  12

Answer:

If you are adding five and seven together, the commutative property of addition says that you will get the same answer whether you are adding 7 + 5 or 5 + 7 = 12
7 + 5 =  12
 5  +  7  =  12

Question 7:
8 + 9 =
     +      =  17

Answer:

If you are adding eight and nine together, the commutative property of addition says that you will get the same answer whether you are adding 8 + 9 or 9 + 8.
8 + 9 =  17
 9  +  8  =  17

Question 8:
0 + 4 =
     +      =  4

Answer:

According to the commutative property of addition, changing the order of the numbers we are adding, does not change the sum.
0 + 4 =  4
 4  +  0  =  4

Question 9:
9 + 6 =
     +      =  15

Answer:

According to the commutative property of addition, changing the order of the numbers we are adding, does not change the sum.
There will be no change in the sum whether you add 9 + 6 or 6 + 9 = 15
9 + 6 =  15
 6  +  9  =  15

Is the sum even or odd? Write even or odd.

Even Numbers:

Any integer that can be divided exactly by 2 is an even number. The last digit is 0, 2, 4, 6 or 8
Example: −24, 0, 6 and 38 are all even numbers

Odd Numbers:

Any integer that cannot be divided exactly by 2 is an odd number. The last digit is 1, 3, 5, 7 or 9
Example: −3, 1, 7 and 35 are all odd numbers
Odd numbers are in between the even numbers.

Question 10:
5 + 2

Answer:

The sum of two odd numbers is an odd number.
5 + 2 = 7.
∴ 7 is an odd number.

Question 11:
6 + 4

Answer:

The sum of two even numbers is always an even number.
6 + 4 = 10.
∴ 10 is an even number.

Question 12:
1 + 0

Answer:
The Sum of any number with zero is always the same number.
1 + 0 = 1.
∴ 1 is an odd number.

Question 13:
5 + 5

Answer:
Any integer that can be divided exactly by 2 is an even number.
5 + 5 = 10.
∴ 10 is an even number.

Question 14:
3 + 8

Answer:

The sum of an even and odd number is an odd number.
3 + 8 = 11.
∴ 11 is an odd number.

Question 15:
7 + 7

Answer:

7 + 7 = 14.
∴ 14 is an even number.

Question 16:
Ada writes 10 + 8 = 18 on the board. Maria wants to use the Commutative Property of Addition to rewrite Ada’s addition sentence. What number sentence should Maria write?

Answer:

According to the commutative property of addition, changing the order of the numbers we are adding, does not change the sum.
If you are adding ten and eight together, the commutative property of addition says that you will get the same answer whether you are adding 10 + 8 or 8 + 10.
Maria should write 8 + 10 =18.

Question 17:
Jackson says he has an odd number of model cars. He has 6 cars on one shelf and 8 cars on another shelf. Is Jackson correct? Explain.

Answer:

Jackson has 6 cars on one shelf and 8 cars on another shelf, hence the total number of cars Jackson has = sum of the cars on both shelves = 6 + 8 = 14.
14 is an even number ends with ‘4’ in the last digit.
Given Statement is False, Jackson has an even number of model cars i.e. 14.

Number Patterns Lesson Check Page No 10

Question 1
Marvella says that the sum of her addends is odd. Which of the following could be Marvella’s addition problem?

Options:
(a) 5 + 3
(b) 9 + 7
(c) 2 + 8
(d) 5 + 6

Answer:

(a) 5 + 3 = 8 (Even Number)
(b) 9 + 7 = 16 (Even Number)
(c) 2 + 8 = 10 (Even Number)
(d) 5 + 6 = 11 (Odd Number)

Option (d) is Correct.

Question 2
Which number sentence shows the Commutative Property of Addition?
3 + 9 = 12

(a) 12 – 9 = 3
(b) 12 = 8 + 4
(c) 9 + 3 = 12
(d) 12 – 3 = 9

Answer:

3 + 9 = 12 and 9 + 3 = 12 are commutative.
Option (c) is correct.

Spiral Review
Question 3
Amber has 2 quarters, a dime, and 3 pennies. How much money does Amber have?

(a) 53 ¢
(b) 58 ¢
(c) 63 ¢
(d) 68 ¢

Answer:

Amber has
2 quarters = 2 x 25 = 50 ¢
1 dime = 10 ¢
3 pennies = 3 ¢
∴ Money does Amber have = 2 quarters + 1 dime + 3 pennies
= 50 ¢ + 10 ¢ + 3 ¢ = 63 ¢
Option (c) is correct.

Question 4
Josh estimates the height of his desk. Which is the best estimate?

(a) 1 foot
(b) 2 feet
(c) 5 feet
(d) 9 feet

Answer:

So, from the given choices, 2 feet or approximately 24 inches would be the most viable answer because most standard desks have a height around 28 inches to 30 inches. 1 foot is too low, while 5 feet and 9 feet are too high. Therefore, 3 to 4 feet would be the most perfect answer but since we have a limited number of options, the nearest reasonable estimate value would be 2 feet. The original numbers from a problem does not exceed in a reasonable estimate.

Option (b) is correct.

Use the bar graph for 5–6.

Go Math Grade 3 Answer Key Chapter 1 Number Patterns Page 10

Question 5
Who read the most books?

(a) Alicia
(b) Bob
(c) Juan
(d) Maria

Answer:

The number of Books Read:
Juan – 6
Bob – 2
Maria – 4
Alicia – 5
Juan Read the most books.
Option (c) is correct.

Question 6
Who read 3 more books than Bob?

(a) Alicia
(b) Juan
(c) Maria
(d) no one

Answer:

Alicia read 3 books.
Option (a) is correct.

Lesson 2: Round to the Nearest Ten Page 15

Round to the Nearest Ten or Hundred

Locate and label 739 on the number line.
Round to the nearest hundred.

Go Math Grade 3 Chapter 1 Round to the Nearest Ten or Hundred Page 15 Answer Key

Question 1

(i) 739 is between __ and __

Answer:

739 is between 700 and 800.

Question 2

739 is closer to       than it is to      .

Answer:

739 is closer to  700 than it is to  800.

Round to the nearest ten and hundred.

Round to the nearest ten

Rounding Numbers to the nearest 10 means finding which 10 they are nearest to. For example, 68 rounded to the nearest 10 is 70.

Rule for rounding to the nearest 10

Look at the number in the one’s place and…

Rule for rounding to the nearest 10

Work through the examples below that show rounding to the nearest 10.

Rounding numbers to the nearest 10

Round to the nearest hundred

Rounding numbers to the nearest 100 means finding which 100 they are nearest to. For example, 680 rounded to the nearest 100 is 700.

Rule for rounding to the nearest 100

Look at the number in the tens’ place and…

Rule for rounding to the nearest 10

Work through the examples below that show rounding to the nearest 100.

Rounding numbers to the nearest 100

Question 3
739 rounded to the nearest hundred is

Answer:

Let’s round 739 to the nearest 100.
The nearest 100’s on both sides of 739 are 700 and 800.
700 is the nearest 100 to 739.
∴ 739 rounded to the nearest hundred is ‘700’

Question 4
363
Round to nearest ten: 
Round to nearest hundred:      

Answer:

(i) Let’s round 363 to the nearest 10.

The nearest 10’s on both sides of 363 are 360 and 370.
360 is the nearest 10 to 363.
∴ 363 rounded to the nearest ten is ‘360’
Round to nearest ten: 360

(ii) Let’s round 363 to the nearest 100.

The nearest 100’s on both sides of 363 are 300 and 400.
400 is the nearest 100 to 363.
∴ 363 rounded to the nearest hundred is ‘400’
Round to nearest hundred: 400

Question 5
829
Round to nearest ten:     
Round to nearest hundred:      

Answer:

(i) Lets round 829 to the nearest 10.
The nearest 10’s on both sides of 829 is 820 and 830.
Round to nearest ten: 830

(ii) Let’s round 829 to the nearest 100
The nearest 100’s on both sides of 829 is 800 and 900.
The number rounded to 829 nearest to 100 is 800.
Round to nearest hundred: 800

Question 6
572
Round to nearest ten:     
Round to nearest hundred:      

Answer:

(i) Lets round 572 to the nearest 10.
The nearest 10’s on both sides of 572 is 560 and 570.
The number rounded to 572 is 570.
Round to nearest ten: 570

(ii) Let’s round 572 to the nearest 100
The nearest 100’s on both sides of 572 is 500 and 600.
The number rounded to 572 nearest to 100
Round to nearest hundred: 600

Question 7
209
Round to nearest ten: 
Round to nearest hundred:      

Answer:

(i) Lets round 209 to the nearest 10.
The nearest 10’s on both sides of 209 is 200 and 210.
The number rounded to Round to nearest ten: 210

(ii) Let’s round 209 to the nearest 100
The nearest 100’s on both sides of 209 are 200 and 300.
The number rounded to 209 nearest to 100 is 200.
Round to nearest hundred: 200

Question 8
663
Round to nearest ten: 
Round to nearest hundred:      

Answer:

(i) Lets round 663 to the nearest 10.
The nearest 10’s on both sides of 663 are 660 and 670.
Round to nearest ten: 660

(ii) Let’s round 663 to the nearest 100
The nearest 100’s on both sides of 663 are 600 and 700.
The number rounded to 663 nearest to 100 is 700.
Round to nearest hundred: 700

Question 9
949
Round to nearest ten: 
Round to nearest hundred:      

Answer:

(i) Lets round 949 to the nearest 10.
The nearest 10’s on both sides of 949 is 940 and 950.
Round to nearest ten: 950

(ii) Let’s round 949 to the nearest 100
The nearest 100’s on both sides of 949 are 900 and 1000.
The number rounded to 949 nearest to 100 is 900.
Round to nearest hundred: 900

Question 10
762
Round to nearest ten: 
Round to nearest hundred:      

Answer:

(i) Lets round 762 to the nearest 10.
The nearest 10’s on both sides of 762 is 760 and 770.
Round to nearest ten:  760

(ii) Let’s round 762 to the nearest 100
The nearest 100’s on both sides of 762 are 700 and 800.
The number rounded to 762 nearest to 100 is 800.
Round to nearest hundred:  800 

Question 11
399
Round to nearest ten: 
Round to nearest hundred:      

Answer:

(i) Lets round 399 to the nearest 10.
The nearest 10’s on both sides of 399 is 390 and 400.
Round to nearest ten: 400

(ii) Let’s round 399 to the nearest 100
The nearest 100’s on both sides of 399 are 300 and 400.
The number rounded to 399 nearest to 100 is 400
Round to nearest hundred: 400

Question 12
402
Round to nearest ten: 
Round to nearest hundred:      

Answer:

(i) Lets round 402 to the nearest 10.
The nearest 10’s on both sides of 402 is 400 and 410.
Round to nearest ten: 400

(ii) Let’s round 402 to the nearest 100
The nearest 100’s on both sides of 402 are 400 and 500.
The number rounded to 402 nearest to 100 is 400.
Round to nearest hundred: 400

Problem Solving

Question 13
The baby elephant weighs 435 pounds. What is its weight rounded to the nearest hundred pounds?
          pounds

Answer:

Let’s round 435 to the nearest hundred pounds.
The nearest 100’s on both sides of 435 are 400 and 500.
400 is the nearest 100 to 435.
∴ 435 rounded to the nearest hundred pounds is ‘400’
Round to nearest hundred: 400 pounds

Question 14
Jayce sold 218 cups of lemonade at his lemonade stand. What is 218 rounded to the nearest ten?
          cups

Answer:

Let’s round 218 to the nearest 10.
The nearest 10’s on both sides of 218 are 210 and 220.
218 is the nearest 10 to 220.
∴ 218 rounded to the nearest ten is ‘220’
Round to nearest ten: 220

Lesson 2: Round to the Nearest Ten Lesson Check Page No 16

Lesson Check

Question 1
One day, 758 people visited the Monkey House at the zoo. What is 758 rounded to the nearest hundred?

(a) 700
(b) 760
(c) 800
(d) 860

Answer:
Let’s round 758 to the nearest hundred.
The nearest hundred on both sides is 700 and 800.
∴ 758 rounded to the nearest hundred is 800.
So, the answer is option C.

Question 2
Sami ordered 132 dresses for her store. What is 132 rounded to the nearest ten?

(a) 100
(b) 130
(c) 140
(d) 200

Answer:
Let’s round 132 to the nearest 10.
The nearest 10’s on both sides of 132 are 130 and 140.
∴ 132 rounded to the nearest ten is ‘130’
132 rounded to the nearest ten: 130
Option B is the correct answer.

Spiral Review
Question 3
Which describes the number sentence?
6 + 0 = 6

(a) Commutative Property of Addition
(b) Identity Property of Addition
(c) even + odd = odd
(d) odd + odd = odd

Answer:
The Identity Property of Zero, also called the Additive Identity Property, states that if you add 0 to any number, the result will be that number. Likewise, if you subtract 0 from any number, the result will be that number.
6 + 0 = 6
∴ Option B is the correct answer.

Question 4
Which has an even sum?

(a) 7 + 4
(b) 2 + 6
(c) 5 + 4
(d) 3 + 2

Answer:
The sum of even numbers is always even.
(a) 7 + 4 = 11 is odd number
(b) 2 + 6 = 8 is even number
(c) 5 + 4 = 9 is odd number
(d) 3 + 2 = 5 is odd number
So, the answer is option (b)

Question 5
What name describes this shape?

HMH Go Math Answer Key Grade 3 Chapter 1 image_1

(a) cone
(b) cube
(c) rectangle
(d) triangle

Answer: Triangle
∴ Option D is the correct answer.

Question 6
What word describes the equal shares of the shape?

Go Math Answer Key Grade 3 Chapter 1 image_2

(a) wholes
(b) thirds
(c) halves
(d) fourths

Answer:
The rectangle is divided into 4 equal rectangles.
So, the name for the equal shapes is fourths.
∴ The answer is option D.

Estimate Sums Page No – 21

Compatible Numbers:

Compatible numbers are the numbers that are easy to compute mentally and are close to the real numbers.

Use rounding or compatible numbers to estimate the sum.

Question 1
198 + 727 =         

Answer:

200 +725 = 925

Explanation:

Step 1:
First round 198 to the nearest hundred.
The number rounded to 198 nearest hundred is 200.
Write zeros for the tens and ones digit.

Step 2:
Write the number closer to 727.
The number closer to 727 is 725.

Step 3:
Now find the sum of the rounded numbers.
200 + 725 = 925

Question 2
87 + 34

Estimate:

         +         =        

Answer: 90 + 30 = 120

Explanation:

Step 1:
First round 87 to the nearest ten.
The number rounded to 87 nearest ten is 90.
Write zeros for the ones digit.

Step 2:
Write the number closer to 34.
The number closer to 34 is 30.

Step 3:
Now find the sum of the rounded numbers.
90 + 30 = 120

Question 3
222 + 203

Estimate:

         +         =        

Answer: 200 +200 = 400

Explanation:

Step 1:
First round 222 to the nearest hundred.
The number rounded to 222 nearest hundred is 200.
Write zeros for the tens and ones digit.

Step 2:
Write the number closer to 203.
The number closer to 203 is 200.

Step 3:
Now find the sum of the rounded numbers.
200 + 200 = 400
The estimated sum of 222 + 203 is 400.

Question 4
52 + 39

Estimate:

         +         =        

Answer: 50 + 40 = 90

Explanation:

Step 1:
First round 52 to the nearest ten.
The number rounded to 52 nearest ten is 50.
Write zeros for the ones digit.

Step 2:
Write the number closer to 39.
The number closer to 39 is 40.

Step 3:
Now find the sum of the rounded numbers.
50 + 40 = 90

Question 5
256 + 321

Estimate:

         +         =        

Answer: 250 + 325 = 575

Explanation:

Step 1:
First round 256 to the nearest ten.
The number rounded to 256 nearest ten is 250.
Write zeros for the ones digit.

Step 2:
Write the number closer to 321.
The number closer to 321 is 325.

Step 3:
Now find the sum of the rounded numbers.
250 + 325 = 575
The estimated sum of 256 + 321 is 575.

Question 6
302 + 412

Estimate:

         +         =        

Answer: 300 + 400 = 700

Step 1:
First round 302 to the nearest ten.
The number rounded to 302 nearest ten is 300.
Write zeros for the ones digit.

Step 2:
Write the number closer to 412.
The number closer to 412 is 400.

Step 3:
Now find the sum of the rounded numbers.
300 + 400 = 700

Question 7
519 + 124

Estimate:

         +         =        

Answer: 500 + 100 = 600

Explanation:

Step 1:
First round 519 to the nearest hundred.
The number rounded to 519 nearest hundred is 500.
Write zeros for the tens and ones digit.

Step 2:
Write the number closer to 124.
The number closer to 124 is 100.

Step 3:
Now find the sum of the rounded numbers.
500 + 100 = 600
The estimated sum is 600.

Question 8
790 + 112

Estimate:

         +         =        

Answer: 800 + 100 = 900

Explanation:

Step 1:
First round 790 to the nearest hundred.
The number rounded to 790 nearest hundred is 800.
Write zeros for the tens and ones digit.

Step 2:
Write the number closer to 112.
The number closer to 112 is 100.

Step 3:
Now find the sum of the rounded numbers.
800 + 100 = 900
The estimated sum of 790 + 112 is 900.

Question 9
547 + 326

Estimate:

         +         =        

Answer: 550 + 325 = 875

Explanation:

Step 1:
First round 547 to the nearest ten.
The number rounded to 547 nearest ten is 550.
Write zeros for the ones digit.

Step 2:
Write the number closer to 326.
The number closer to 326 is 325.

Step 3:
Now find the sum of the rounded numbers.
550 + 325 = 875

Question 10
325 + 458

Estimate:

         +         =        

Answer: 325 + 500 = 825

Explanation:

First round 458 to the nearest hundred.
The number rounded to 458 nearest hundred is 500.
Write zeros for the tens and ones digit.
Now add 325 and 500,
You get, 325 + 500 = 825

Question 11
620 + 107

Estimate:

         +         =        

Answer: 600 + 100 = 700

Explanation:

The number closer to 620 is 600.
And the number closer to 107 is 100.
600 + 100 = 700
Now the estimated sum of 620 + 107 = 700

Problem Solving
Question 12
Stephanie read 72 pages on Sunday and 83 pages on Monday. About how many pages did Stephanie read during the two days?

About         pages

Answer: 150 pages

Explanation:

Stephanie read 72 pages on Sunday and 83 pages on Monday.
The number rounded to 72 is 70 and the number rounded to 83 is 80.
To know how many pages he read in two days we need to add the number of pages he read.
70 + 80 = 150 pages.
Therefore Stephanie read about 150 pages in 2 days.

Question 13
Matt biked 345 miles last month. This month he has biked 107 miles. Altogether, about how many miles has Matt biked last month and this month?

About         miles

Answer: About 450 miles

Explanation:

Matt biked 345 miles last month.
This month he has biked 107 miles.
The number rounded to 345 is 350.
And the number closer to 107 is 100.
Now add number of miles he rides in last month and this month.
350 + 100 = 450 miles.

Estimate Sums Lesson Check – Page No – 22

Lesson Check
Question 1
The McBrides drove 317 miles on one day and 289 on the next day. What is the best estimate of the number of miles the McBrides drove in all during the two days?

(a) 100
(b) 400
(c) 500
(d) 600

Answer: 600

Explanation:

The McBrides drove 317 miles on one day and 289 on the next day.
First, round 317 to the nearest hundred.
The number rounded to 317 nearest hundred is 300.
Write zeros for the tens and ones digit.
Next round 289 to the nearest hundred.
The number rounded to 289 nearest hundred is 300.
Write zeros for the tens and ones digit.
300 +300 = 600.
Option D is the correct answer.

Question 2
Ryan counted 63 birds in his backyard last week. This week, he counted 71 birds in his backyard. About how many birds did Ryan count in all?

(a) about 70
(b) about 100
(c) about 130
(d) about 200

Answer: about 130

Explanation:

Ryan counted 63 birds in his backyard last week. This week, he counted 71 birds in his backyard.
The number closer to 63 is 60.
The number closer to 71 is 70.
Now add 60 and 70 we get 130.
Therefore Ryan count about 130 birds.
So, the correct answer is option C.

Spiral Review
Question 3
What name describes this shape?

Go Math Grade 3 Chapter 1 Round to the Nearest Ten or Hundred Page 22 What name describes this shape

(a) cone
(b) cube
(c) quadrilateral
(d) square

Answer: cube

A cube is a three-dimensional solid object bounded by six square faces, facets or sides, with three meeting at each vertex. The cube is the only regular hexahedron and is one of the five Platonic solids. It has 6 faces, 12 edges, and 8 vertices.

Question 4
Which has an odd sum?

(a) 9 + 9
(b) 5 + 3
(c) 6 + 7
(d) 2 + 8

Answer: 6 + 7

Explanation:

(a) 9 + 9 = 18 even number
(b) 5 + 3 = 8 even number
(c) 6 + 7 = 13 odd number
(d) 2 + 8 = 10 even number
So, the answer is option C.

Question 5
What is 503 rounded to the nearest hundred?

(a) 500
(b) 510
(c) 600
(d) 610

Answer: 500

The number 503 rounded to the nearest hundred is 500.
So, the correct answer is option A.

Question 6
What is 645 rounded to the nearest ten?

(a) 600
(b) 640
(c) 650
(d) 700

Answer: 650

645 rounded to the nearest ten is 650.
So, the correct answer is option C.

Mental Math Strategies for Addition Page No – 27

Count by tens and ones to find the sum.

Use the number line to show your thinking.

Question 1

Go Math Grade 3 Chapter 1 Count by tens and ones to find the sum Question 1

Answer: 29 + 14 = 43

Question 2

Go Math Grade 3 Chapter 1 Count by tens and ones to find the sum Question 2

36 + 28 =

Answer: 36 + 28 = 64

Question 3

Go Math Grade 3 Chapter 1 Count by tens and ones to find the sum Question 2

45 + 26 =

Answer: 45 + 26 = 71

Question 4

Go Math Grade 3 Chapter 1 Count by tens and ones to find the sum Question 2

52 + 34 =

Answer: 52 + 34 = 86

Use mental math to find the sum.

Draw or describe the strategy you use.

Question 5
52 + 19 =

Answer: 52 + 19 = 71

I Used friendly numbers.
Subtract 2 from 52.
52 – 2 = 50
Then add 2 to 19.
19 + 2 = 21
Add 50 and 21
50 + 21 = 71

Question 6
122 + 306 =

Answer: 122 + 306 = 428

I Used compatible numbers
122 = 120 + 2
306 = 300 + 6
120 + 300 = 420
2 + 6 = 8
420 + 8 = 428

Problem Solving
Question 7
Shelley spent 17 minutes washing the dishes. She spent 38 minutes cleaning her room. Explain how you can use mental math to find how long Shelley spent on the two tasks

        minutes

Answer: 20 + 35 = 55 minutes

Explanation:

Shelley spent 17 minutes washing the dishes.
She spent 38 minutes cleaning her room.
First, make friendly numbers.
Add 3 to 17 to make it easy for addition
17 + 3 = 20
Next, subtract 3 from 38.
38 – 3 = 35
Now add both, 35 + 20 = 55
Shelly spent 55 minutes on the two tasks.

Question 8
It took Marty 42 minutes to write a book report. Then he spent 18 minutes correcting his report. Explain how you can use mental math to find how long Marty spent on his book report.

        minutes

Answer: 50 + 10 = 60 minutes

Explanation:

It took Marty 42 minutes to write a book report.
Then he spent 18 minutes correcting his report.
Make a friendly number
Subtract 2 from 42
42 – 2 = 40 minutes
Now add 2 to 18 minutes
18 + 2 = 20 minutes
Now add both, 20 + 40 = 60 minutes
Therefore Marty spent 60 minutes on his book report

Mental Math Strategies for Addition Page No – 28

Lesson Check
Question 1
Sylvia spent 36¢ for a pencil and 55¢ for a notepad. Use mental math to find how much she spent in all.

(a) 80¢
(b) 81¢
(c) 90¢
(d) 91¢

Answer: 91¢

Explanation:

Sylvia spent 36¢ for a pencil and 55¢ for a notepad.
Step 1:
Make a friendly number
Add 36¢ and 55¢
36¢ + 55¢ = 91¢
So, the correct answer is option D.

Question 2
Will spent 24 minutes putting together a model plane. Then he spent 48 minutes painting the model. How long did Will spend working on the model plane?

(a) 62 minutes
(b) 68 minutes
(c) 72 minutes
(d) 81 minutes

Answer: 72 minutes

Explanation:

Will spent 24 minutes putting together a model plane. Then he spent 48 minutes painting the model.
Add 24 and 48
24 + 48 = 72 minutes
Option C is the correct answer.

Spiral Review

Question 3
What name describes this shape?

Go Math Grade 3 Chapter 1 What name describes this shape

(a) hexagon
(b) pentagon
(c) quadrilateral
(d) triangle

Answer: pentagon

Explanation:

From the figure, we can observe that there are 5 sides. A pentagon is a 5-sided Polygon
So, the correct answer is option B.

Question 4
What word describes the equal shares of the shape?

Go Math Grade 3 Chapter 1 What word describes the equal shares of the shape

(a) fourths
(b) halves
(c) sixths
(d) thirds

Answer: fourths

Explanation:

The circle is divided into 4 equal parts. The name for the equal shares of circle is fourths.

Question 5
Tammy wrote an addition problem that has an odd sum. Which could be Tammy’s addition problem?

(a) 2 + 6
(b) 3 + 5
(c) 5 + 6
(d) 7 + 7

Answer: 5 + 6

Explanation:

(a) 2 + 6 = 8 even number
(b) 3 + 5 = 8 even number
(c) 5 + 6 = 11 odd number
(d) 7 + 7 = 14 even number
11 is an odd number. So, the correct answer is option C.

Question 6
Greg counted 83 cars and 38 trucks in the mall parking lot. Which is the best estimate of the total number of cars and trucks Greg counted?

(a) 100
(b) 110
(c) 120
(d) 130

Answer: 120

Explanation:

Greg counted 83 cars and 38 trucks in the mall parking lot.
The number closer to 83 is 80.
And the number closer to 38 is 40.
80 + 40 = 120.
So, the correct answer is option C.

Use Properties to Add Page No 33

Use addition properties and strategies to find the sum.

Question 1
Go Math Grade 3 Chapter 1 Use Properties of Add Question 1

Question 2
27 + 68 + 43 =

Answer: 138

Explanation:

Step 1:
Line up the numbers by place value.
27
68
+43

Step 2:
Group the ones to make them easy to add.
Make a 10
27
68
+43

7 + 3 = 10
1 will be carried to tens place.
8 will be in the ones place.

Step 3:
Group the tens to make them easy to add.
27
68
+43
6 + 4 = 10
10 + 3 = 13

27 + 68 + 43 = 138

Question 3
42 + 36 + 18 =

Answer: 96

Explanation:

Step 1:
Line up the numbers by place value.

42
36
+18

8 + 2 = 10
1 will be carried to the tens place
6 will be in the ones place.

Step 2:
Group the ones to make them easy to add.
Make a 10
42
36
+18

40 + 30 + 10 + 10 = 90

Step 3:
Group the tens to make them easy to add.
90 + 6 = 96

Question 4
74 + 35 + 16 + 45 =

Answer: 170

Explanation:

Step 1:
Line up the numbers by place value.
74
35
16
+45

Step 2:
Group the ones to make them easy to add.
Make a 10
74
35
16
+45

6 + 4 = 10
5 + 5 = 10
10 + 10 = 20
2 will be carried to tens place
0 will be in the ones place.

Step 3:
Group the tens to make them easy to add.
70 + 30 + 10 + 40 +20 = 170

Question 5
41 + 26 + 149 =

Answer: 216

Explanation:

Step 1:
Line up the numbers by place value.

149
41
+26

Step 2:
Group the ones to make them easy to add.
Make a 10

149
41
+26

9 + 1 = 10
1 will be carried to tens place.
6 will be in the ones place.

Step 3:
Group the tens to make them easy to add.
140 + 40 + 20 + 10 = 210
210 + 6 = 216

Question 6
52 + 64 + 28 + 44 =

Answer: 188

Explanation:

Step 1:
Line up the numbers by place value.
52
64
28
+44

Step 2:
Group the ones to make them easy to add.
Make a 10

52
64
28
+44

8 + 2 = 10
4 + 4 = 8
1 will be carried to the tens place.
8 will be in the ones place.

Step 3:
Group the tens to make them easy to add.
50 + 60 + 20 + 40 + 10 = 180
180 + 8 = 188

Problem Solving
Question 7
A pet shelter has 26 dogs, 37 cats, and 14 gerbils. How many of these animals are in the pet shelter in all?

        animals

Answer: 77 animals

Explanation:

Given that, A pet shelter has 26 dogs, 37 cats, and 14 gerbils.
Add the total number of pets
26 + 37 + 14 = 77 animals.

Question 8
The pet shelter bought 85 pounds of dog food, 50 pounds of cat food, and 15 pounds of gerbil food. How many pounds of animal food did the pet shelter buy?

        pounds

Answer: 150 pounds

Explanation:

Step 1:
Line up the numbers by place value.

85
50
+15

Step 2:
Group the ones to make them easy to add.
Make a 10

85
50
+15

5 + 5 = 10

Step 3:
Group the tens to make them easy to add.
80 + 50 + 10 + 10 = 150

Use Properties to Add Page No 34

Lesson Check
Question 1
At summer camp there are 52 boys, 47 girls, and 18 adults. How many people are at summer camp?

(a) 97
(b) 107
(c) 117
(d) 127

Answer: 117

Explanation:

At summer camp there are 52 boys, 47 girls, and 18 adults.
57
47
+18

52 + 47 + 18 = 117
Therefore 117 people are at summer camp.
The correct answer is option C.

Question 2
At camp, 32 children are swimming, 25 are fishing, and 28 are canoeing. How many children are swimming, fishing, or canoeing?

(a) 75
(b) 85
(c) 95
(d) 105

Answer: 85

Explanation:

At camp, 32 children are swimming, 25 are fishing, and 28 are canoeing.
32
25
+28

Make a group of 10.

32
25
+28

8 + 2 = 10
1 will be carried to the tens place.
5 will be in the ones place.
30 + 20 + 20 + 10 = 80
80 + 5 = 85
The correct answer is option B.

Spiral Review
Question 3
Four students estimated the width of the door to their classroom. Who made the best estimate?

(a) Ted: 1 foot
(b) Hank: 3 feet
(c) Ann: 10 feet
(d) Maria: 15 feet

Answer: Hank: 3 feet

Question 4
Four students estimated the height of the door to their classroom. Who made the best estimate?

(a) Larry: 1 meter
(b) Garth: 2 meters
(c) Ida: 14 meters
(d) Jill: 20 meters

Answer: Garth: 2 meters

Question 5
Jeff’s dog weighs 76 pounds. What is the dog’s weight rounded to the nearest ten pounds?

(a) 70 pounds
(b) 80 pounds
(c) 90 pounds
(d) 100 pounds

Answer: 80 pounds

Explanation:

Jeff’s dog weighs 76 pounds.
76 rounded to the nearest ten is 80.
The correct answer is option B.

Question 6
Ms. Kirk drove 164 miles in the morning and 219 miles in the afternoon. Which is the best estimate of the total number of miles she drove that day?

(a) 100 miles
(b) 200 miles
(c) 400 miles
(d) 500 miles

Answer: 400 miles

Explanation:

Ms. Kirk drove 164 miles in the morning and 219 miles in the afternoon.
The number closer to 164 is 200.
The number closer to 219 is 200.
Now add the total number of mile
200 + 200 = 400 miles.
The correct answer is option C.

Use the Break Apart Strategy to Add Page No 39

Estimate. Then use the break apart strategy to find the sum.

Question 1

Question 2
518 + 372

Estimate: 900

Sum:
518 = 500 + 10 + 8
+372 = 300 + 70 + 2
890     800 + 80 + 10

Question 3
473 + 123

Estimate: 600

Sum:
473 = 400 + 70 + 3
123 = 100 + 20 + 3
596 = 500 + 90 + 6

Question 4
208 + 569

Estimate: 800

Sum:
208 = 200 + 00 + 8
569 = 500 + 60 + 9
777 = 700 + 70 + 7

Question 5
731 + 207

Estimate: 900

Sum:
731 = 700 + 30 + 1
207 = 200 + 00 + 7
938 = 900 + 30 + 8

Question 6
495 + 254

Estimate: 800

Sum:
495 = 400 + 90 + 5
254 = 200 + 50 + 4
749 = 700 + 40 + 9

Problem Solving
Use the table for 7–8.

Go Math Grade 3 Chapter 1 Problem Solving

Question 7
Laura is making a building using Set A and Set C. How many blocks can she use in her building?

        blocks

Answer: 410 blocks

Add set A and Set C
165 + 245 = 410 blocks

165 = 100 + 60 +5
245 = 200 + 40 + 5
410 = 300 + 100 + 10
She can use 410 blocks in her building.

Question 8
Clark is making a building using Set B and Set C. How many blocks can he use in his building?

        blocks

Answer: 433 blocks

Add Set B and Set C
188 + 245 =

188 = 100 + 80 + 8
245 = 200 + 40 + 5
433 = 300 + 120 + 13
He can use 433 blocks in his building.

Use the Break Apart Strategy to Add Page No 40

Lesson Check
Question 1
Arthur read two books last week. One book has 216 pages. The other book has 327 pages. Altogether, how many pages are in the two books?

(a) 533
(b) 543
(c) 633
(d) 643

Answer: 543

Explanation:

Add 216 and 327
216 = 200 + 10 + 6
327 = 300 + 20 + 7
543 = 500 + 30 + 13

So, the correct answer is option B.

Question 2
One skeleton in a museum has 189 bones. Another skeleton has 232 bones. How many bones in all are in the two skeletons?

(a) 311
(b) 312
(c) 411
(d) 421

Answer: 421

Explanation:

Add 189 and 232
189 = 100 + 80 + 9
232 = 200 + 30 + 2
421 = 300 + 110 + 11
Thus the answer is option D.

Spiral Review
Question 3
Culver has 1 quarter, 3 dimes, and a penny. How much money does he have?

(a) 41¢
(b) 55¢
(c) 56¢
(d) 86¢

Answer: 56¢

Explanation:

1 quarter = $0.25
1 dime = $0.10
3 dimes = $0.10 × 3 = $0.30
1 penny = $0.01
Add $0.25 + $0.30 + $0.01 = $0.56 = 56 cents
Thus the correct answer is option C.

Question 4
Felicia has 34 quarters, 25 dimes, and 36 pennies. How many coins does Felicia have?

(a) 75
(b) 85
(c) 95
(d) 105

Answer: 95

Explanation:

1 quarter = $0.25
34 quarters = $0.25 × 34 = $8.5
25 dimes = $0.10 × 25 = $2.5
36 pennies = $0.01 × 36 = 0.36
Option C is the correct answer.

Question 5
Jonas wrote 9 + 8 = 17. Which number sentence shows the Commutative Property of Addition?

(a) 9 + 0 = 9
(b) 8 + 9 = 17
(c) 17 – 9 = 8
(d) 17 – 8 = 9

Answer: 8 + 9 = 17

According to the commutative property of addition, changing the order of the numbers we are adding, does not change the sum.
So, the correct answer is option B.

Question 6
At Kennedy School there are 37 girls and 36 boys in the third grade. How many students are in the third grade at Kennedy School?

(a) 63
(b) 73
(c) 81
(d) 91

Answer: 73

Explanation:

Given that,
At Kennedy School there are 37 girls and 36 boys in the third grade.
Add number of girls and boys = 37 + 36 = 73
Therefore the correct answer is option B.

Use Place Value to Add Page No 45

Estimate. Then find the sum.

Question 1
Estimate: 600

324 + 285 = 609

324
285
609

Question 2
519  + 347

Estimate: 500 + 300 = 800

Sum: 519 + 347
519
347
866

Question 3
323 + 151

Estimate: 323 + 151 = 325 + 150= 475

Sum:

323
151
474

Question 4
169 + 354

Estimate: 150 + 350 = 500

Sum:

169
354
523

Question 5
148 + 285

Estimate: 150 + 300 = 450

Sum: 148 + 285 = 433

148
285
433

Question 6
270 + 453

Estimate: 300 + 450 = 750

Sum: 270 + 453 = 723

270
453
723

Question 7
275 + 116

Estimate: 275 + 100 = 375

Sum:

275
116
391

Question 8
157 + 141

Estimate: 150 + 150 = 300

Sum:

157
141
298

Question 9
127 + 290

Estimate: 100 + 300 = 400

Sum:

127
290
417

Question 10
258 + 565

Estimate: 250 + 550 = 800

Sum:

258
565
823

Question 11
311 + 298

Estimate: 300 + 300 = 600

Sum:

311
298
609

Question 12
534 + 256

Estimate: 550 + 250 = 800

Sum:

534
256
790

Problem Solving
Question 13
Mark has 215 baseball cards. Emily has 454 baseball cards. How many baseball cards do Mark and Emily have altogether?

        cards

Answer: 669 baseball cards.

Explanation:

Given,
Mark has 215 baseball cards.
Emily has 454 baseball cards.
Total number of baseball cards = 215 + 454 = 669
Therefore there are 669 baseball cards.

Question 14
Jason has 330 pennies. Richie has 268 pennies. Rachel has 381 pennies. Which two students have more than 700 pennies combined?

Answer: Jason and Rachel

Explanation:

Jason has 330 pennies. Richie has 268 pennies. Rachel has 381 pennies.
The rounded number of 330 is 300.
The number closer to 268 is 300
The number rounded to 381 is 400.
You will get 700 when to combine the pennies of Jason and Rachel

Lesson Check Page No 46

Question 1
There are 167 students in the third grade. The same number of students is in the fourth grade. How many third graders and fourth graders are there in all?

(a) 224
(b) 234
(c) 324
(d) 334

Answer: 334

Explanation:

Given that there are 167 students in the third grade.
The same number of students is in the fourth grade.
That means there are 167 students in the fourth grade.
To find the total number of students in third grade and fourth grade
You need to add 167 and 167
167 + 167 = 334.
Thus the correct answer is option D.

Question 2
Jamal read a book with 128 pages. Then he read a book with 179 pages. How many pages did Jamal read in all?

(a) 397
(b) 307
(c) 297
(d) 207

Answer: 307

Explanation:

Jamal read a book with 128 pages. Then he read a book with 179 pages.
128 + 179 = 307
So, the answer is option B.

Spiral Review
Question 3
Adam travels 248 miles on Monday. He travels 167 miles on Tuesday. Which is the best estimate for the total number of miles Adam travels?

(a) 200
(b) 300
(c) 400
(d) 500

Answer: 400

Explanation:

Adam travels 248 miles on Monday. He travels 167 miles on Tuesday.
The number closer to 248 is 200
And the number closer to 167 is 200.
200 + 200 = 400
Thus the estimated number of miles Adam travels is 400.

Question 4
Wes made $14, $62, $40, and $36 mowing lawns. How much did he make in all mowing lawns?

(a) $116
(b) $152
(c) $166
(d) $188

Answer: $152

Explanation:

Add
14
62
40
+36
152
Thus the correct answer is option B.

Question 5
There are 24 students in Mrs. Cole’s class and 19 students in Mr. Garmen’s class. How many students in all are in the two classes?

(a) 43
(b) 40
(c) 33
(d) 5

Answer: 43

Add 24 and 19
24 + 19 = 43
Thus the correct answer is option A.

Question 6
There were 475 children at the baseball game on Sunday. What is 475 rounded to the nearest ten?

(a) 400
(b) 470
(c) 480
(d) 500

Answer: 480

Explanation:

There were 475 children at the baseball game on Sunday.
475 rounded to the nearest ten is 480.
So, the answer is option C.

Mid Chapter Check Point – Vocabulary Page No 47

Choose the best term from the box.

Go Math Grade 3 Chapter 1 Choose the best term from the box.

Question 1
A ________ is an ordered set of numbers or objects in which the order helps you predict what comes next.

Answer: Pattern is an ordered set of numbers or objects in which the order helps you predict what comes next.

Question 2
The _________ states that when you add zero to any number, the sum is that number.

Answer: Identity property of Addition states that when you add zero to any number, the sum is that number

Concepts and Skills
Is the sum even or odd? Write even or odd.

Question 3
8 + 3

Answer: 8 + 3 = 11 is an odd number.

Question 4
9 + 7

Answer: 9 + 7 = 16 is an even number

Question 5
4 + 6

Answer: 4 + 6 = 10 is an even number

Use rounding or compatible numbers to estimate the sum.

Question 6
56+32

Estimate:

         +         =        

Answer:
The number which is compatible to 56 is 50.
The number compatible to 32 is 25
50
25
75

50 + 25 = 75

Question 7
271+425

Estimate:

         +         =        

Answer:

The number close to 271 is 275
425 will be the same.

275
425
700
425 + 275 = 700

Question 8
328+127

Estimate:

         +         =        

Answer:

The number closer to 328 is 325
The number closer to 127 is 125
325 + 125 = 450

Use mental math to find the sum.

Question 9
46 + 14 =

Answer: 60

Explanation:

Break apart the addends to make them compatible
46 = 40 + 6
14 = 10 + 4
Now add both
40 + 6
10 + 4
50 + 10 = 60
46 + 14 = 60

Question 10
39 + 243 =

Answer: 282

Explanation:

Break apart the addends to make them compatible
39 = 35 + 4
243 = 240 + 3
Now add
240 + 3
  35 + 4
275 + 7

275 + 7 = 282
39 + 243 = 282

Question 11
326 + 402 =

Answer: 728

Explanation:

Break apart the addends to make them compatible.
326 = 325 + 1
402 = 400 + 2
Now add

325 + 1
400 + 2
725 + 3 = 728
326 + 402 = 728

Estimate. Then find the sum.

Question 12
356+442
Estimate: 800
Sum: 798

Answer:

356
442
798
The sum of 356 and 442 is 798
The number close to 798 is 800.
Therefore the estimated sum is 800.

Question 13
164+230
Estimate: 400
Sum: 394

Answer:

230
164
394

The sum of 230 and 164 is 394
The number 394 rounded to the nearest hundred is 400.
Thus the estimated sum is 400.

Question 14
545+139
Estimate: 700
Sum: 684

Answer:

545
139
684

The sum of 545 and 139 is 684.
684 rounded to the nearest hundred is 700.
So, the estimated difference is 700.

Question 15
437+184
Estimate: 600
Sum: 621

Answer:

437
184
621

The sum of 437 and 184 is 621
621 rounded to the nearest hundred is 600.
The estimated sum is 600.

Mid Chapter Check Point – Vocabulary Page No 48

Question 16
Nancy planted 77 daisies, 48 roses, and 39 tulips. About how many roses and tulips did she plant?

about         roses and tulips

Answer: 90 roses and tulips

Explanation:

Given that, Nancy planted 77 daisies, 48 roses, and 39 tulips.
To know how many roses and tulips did she plant
We have to add a number of roses and a number of tulips.
48 and 39.
The number closer to 48 is 50.
And the number closer to 39 is 40.
So, the estimated sum is 90.

Question 17
Tomas collected 139 cans for recycling on Monday, and twice that number on Tuesday. How many cans did he collect on Tuesday?

        cans

Answer: 278 cans

Explanation:

Tomas collected 139 cans for recycling on Monday, and twice that number on Tuesday.
Twice is nothing but double.
139 + 139 = 278
Therefore Tomas collected 278 cans on Tuesday.

Question 18
There are 294 boys and 332 girls in the Hill School. How many students are in the school?

        students

Answer: 626 students

Explanation:

Given,
There are 294 boys and 332 girls in the Hill School.
To find the total number of students in the students
We need to add the total number of boys and number of girls = 294 + 332
294
332
626
So, there are 626 students in the class.

Question 19
Monday’s art group made 25 paper models. Tuesday’s group made 32 paper models. Wednesday’s group made 15 paper models. How many paper models did the groups make?

        paper models

Answer: 72 paper models

Explanation:

Monday’s art group made 25 paper models.
Tuesday’s group made 32 paper models.
Wednesday’s group made 15 paper models.
Add 25, 32 and 15
25
32
15
72

Estimate Differences Page No 53

Use rounding or compatible numbers to estimate the difference.

Question 1
40 – 13 = 
40 – 10
Estimate: 30

Question 2
762 – 332

Estimate:

         –          =        

Estimate: 500

The number closer to 762 is 800
The number closer to 332 is 300
The difference between 800 and 300 is 500

Question 3
823 – 242

Estimate:

         –          =        

Estimate: 550

The number 823 rounded to the nearest hundred is 800.
The number closer to 242 is 250
800
-250
550

Question 4
98 – 49

Estimate:

         –          =        

Estimate: 50

The number closer to 98 is 100
The round number of 49 is 50.
100
-50
50

Question 5
287 – 162

Estimate:

         –          =        

Estimate: 100

282 rounded to the nearest hundred is 300
162 rounded to the nearest hundred is 200
300
-200
100

Question 6
359 – 224

Estimate:

         –          =        

Estimate: 125

The number closer to 359 is 350
The number closer to 224 is 225
350
-225
125

Question 7
68 – 31

Estimate:

         –          =        

Estimate: 40
The round number of 68 is 70
The number closer to 31 is 30
70
-30
40

Question 8
476 – 155

Estimate:

         –          =        

Estimate: 325

The number closer to 476 is 475
The number closer to 155 is 150
475
-150
325

Question 9
622 – 307

Estimate:

         –          =        

Estimate: 300

622 nearest to the hundred is 600
307 nearest to the hundred is 300
600
-300
300

Question 10
771 – 531

Estimate:

         +         =        

Estimate: 225

The number closer to 771 is 775
531 nearest to ten is 550
775
550
225

Question 11
299 – 61

Estimate:

         +         =        

Estimate: 240

The number closer to 299 is 300
The number closer to 61 is 60
300
-60
240

Problem Solving

Question 12
Ben has a collection of 812 stamps. He gives his brother 345 stamps. About how many stamps does Ben have left?

About         stamps

Answer: About 450 stamps

Explanation:

Ben has a collection of 812 stamps. He gives his brother 345 stamps.
812 to the nearest hundred is 800
345 to the nearest ten is 350
800
-350
450
Thus about 450 stamps are left.

Question 13
Danika is making necklaces. She has 512 silver beads and 278 blue beads. About how many more silver than blue beads does Danika have?

About         more silver than blue beads

Answer: About 200 more silver than blue beads

Explanation:

Given,
Danika is making necklaces.
She has 512 silver beads and 278 blue beads.
The number closer to 512 is 500
278 to the nearest hundred is 300
The difference between 500 and 300 is 200.
Therefore Danika has about 200 more silver than blue beads.

Lesson Check Page No 54

Question 1
Jorge has 708 baseball cards and 394 basketball cards. About how many more baseball cards than basketball cards does Jorge have?

(a) about 200
(b) about 300
(c) about 400
(d) about 500

Answer: about 300

Explanation:

Jorge has 708 baseball cards and 394 basketball cards.
The number closer to 708 is 700.
The number closer to 394 is 400
700
-400
300
So, the correct answer is option A.

Question 2
Danika is making necklaces. She has 512 silver beads and 278 blue beads. About how many more silver than blue beads does Danika have?

(a) about 200
(b) about 300
(c) about 400
(d) about 800

Answer: about 200

Explanation:

Danika is making necklaces. She has 512 silver beads and 278 blue beads.
The number closer to 512 is 500
278 to the nearest hundred is 300
The difference between 500 and 300 is 200.
Therefore Danika has about 200 more silver than blue beads.
So, the correct answer is option A.

Spiral Review
Question 3
A store manager ordered 402 baseball caps and 122 ski caps. Which is the best estimate of the total number of caps the manager ordered?

(a) 300
(b) 500
(c) 600
(d) 700

Answer: 500

Explanation:

A store manager ordered 402 baseball caps and 122 ski caps.
To find the best estimate of the total number of caps the manager ordered
We have to add baseball caps and ski caps.
The number closer to 402 is 400
The number closer to 122 is 100.
400 + 100 = 500
So, the correct answer is option B.

Question 4
Autumn collected 129 seashells at the beach. What is 129 rounded to the nearest ten?

(a) 100
(b) 120
(c) 130
(d) 200

Answer: 130

Explanation:

Autumn collected 129 seashells at the beach.
129 rounded to the nearest ten is 130
So, the correct answer is option C.

Question 5
Find the sum.

585 + 346

(a) 239
(b) 821
(c) 900
(d) 931

Answer: 931

585
+346
931
The correct answer is option D.

Question 6
Julie made $22, $55, $38, and $25 babysitting. How much did she make in all babysitting?

(a) $102
(b) $115
(c) $140
(d) $165

Answer: $140

Explanation:

Julie made $22, $55, $38, and $25 babysitting.
Put all the numbers in the order
22
55
38
+25
140
So, the correct answer is option C.

Mental Math Strategies for Subtraction Page No – 59

Use mental math to find the difference.
Draw or describe the strategy you use.
Question 1:
74 – 39 = 35

Go Math Grade 3 Chapter 1 Mental Math Strategies for Subtraction

Question 2
93 – 28 =

Answer: 65

I use friendly numbers.
Add 2 to the 93.
93 + 2 = 95
Add 2 to 28
28 + 2 = 30
95 – 30 = 65

Question 3
51 – 9 =

Answer: 42
I used friendly numbers to subtract 9 from 51.
Now add 1 to 9
9 + 1 = 10
Now subtract 10 from 51
51 – 10 = 41
Now add 1 to 41
41 + 1 = 42

Question 4
76 – 23 =

Answer:

I used friendly numbers
Subtract 1 from 76
76 – 1 = 75
75 – 23 = 52
Now add 1 to 52
52 + 1 = 53.
76 – 23 = 53

Question 5
357 – 214 =

Answer:

I use break apart strategy.
300 – 200 = 100
50 – 10 = 40
7 – 4 = 3
100 + 40 + 3 = 143
357 – 214 = 143

Question 6
285 – 99 =

Answer:

I used friendly numbers.
The number close to 99 is 100
285 – 100 = 185
Now add 1 to 185
185 + 1 = 186
285 – 99 = 186

Problem Solving
Question 7
Ruby has 78 books. Thirty-one of the books are on shelves. The rest are still packed in boxes. How many of Ruby’s books are still in boxes?

        books

Answer: 47 books

Explanation:

Ruby has 78 books. Thirty-one of the books are on shelves. The rest are still packed in boxes.
To know the remaining books in the boxes.
Subtract 31 from 78.
78 – 31
70 – 30 = 40
8 – 1 = 7
40 + 7 = 47 books
Therefore 47 of Ruby’s books are still in boxes.

Question 8
Kyle has 130 pins in his collection. He has 76 of the pins displayed on his wall. The rest are in a drawer. How many of Kyle’s pins are in a drawer?

        pins

Answer: 54 pins

Explanation:

Kyle has 130 pins in his collection.
He has 76 of the pins displayed on his wall. The rest are in a drawer.
130 – 76 = 54
Thus there are 54 pins in a drawer.

Lesson Check Page No – 60

Question 1
One day, a baker made 54 fruit pies. At the end of the day, only 9 of the pies were NOT sold. How many pies were sold that day?

(a) 43
(b) 45
(c) 63
(d) 65

Answer: 45

Explanation:

Given,
One day, a baker made 54 fruit pies.
At the end of the day, only 9 of the pies were NOT sold.
Number of pies sold that day = x
x + 9 = 54
x = 54 – 9 = 45
x = 45
Therefore the number of pies sold that day = 45
So, the correct answer is option B.

Question 2
George’s father bought a 50-pound bag of wild bird seed. At the end of two weeks, 36 pounds of seed were left in the bag. How many pounds of seed had been used?

(a) 14 pounds
(b) 24 pounds
(c) 26 pounds
(d) 86 pounds

Answer: 14 pounds

Explanation:

George’s father bought a 50-pound bag of wild bird seed.
At the end of two weeks, 36 pounds of seed were left in the bag.
Number of pounds used = x
x + 36 = 50
x = 50 – 36
x = 14
Therefore George’s father used 14 pounds.
The correct answer is option A.

Spiral Review
Question 3
For a party, Shaun blew up 36 red balloons, 28 white balloons, and 24 blue balloons. How many balloons did he blow up in all?

(a) 78
(b) 81
(c) 87
(d) 88

Answer: 88

Explanation:

For a party, Shaun blew up 36 red balloons, 28 white balloons, and 24 blue balloons.
Total number of balloons = 36 + 28 + 24
36
28
+24
88
So, the answer is option D.

Question 4
Tiffany has read 115 pages of her book. She has 152 pages left to read. How many pages are in the book?

(a) 37
(b) 267
(c) 277
(d) 367

Answer: 267

Explanation:;

Tiffany has read 115 pages of her book.
She has 152 pages left to read.
Total number of pages = 152 + 115
152
+115
267

Question 5
The flower shop had 568 flowers on Monday. By Tuesday, the shop had 159 flowers left. About how many flowers had been sold?

(a) about 200
(b) about 300
(c) about 400
(d) about 500

Answer: about 400

Explanation:

The flower shop had 568 flowers on Monday.
By Tuesday, the shop had 159 flowers left.
The number closer to 568 is 600.
The number closer to 159 is 200
Subtract 200 from 600.
600 – 200 = 400
The correct answer is option C.

Question 6
There are 383 books in one section of the school library. Of the books, 165 are fiction books. Which is the best estimate of the number of books in that section that are NOT fiction?

(a) about 200
(b) about 300
(c) about 400
(d) about 500

Answer: about 200

Explanation:

There are 383 books in one section of the school library.
Of the books, 165 are fiction books.
383 to the nearest hundred is 400.
165 to the nearest hundred is 200
400 – 200 = 200
So, the correct answer is option A.

Use Place Value to Subtract Page No 65

Estimate. Then find the difference.

Question 1
Estimate: 500

585 – 119

Subtract 119 from 585

585
119
466
585 – 119 = 466

Question 2
738 – 227

Estimate: 500

Difference: 511

Subtract 227 from 738
738
227
511
The estimated difference of 511 is 500.
738 – 227 = 511

Question 3
651 – 376
Estimate: 300
Difference: 275

Subtract 376 from 651
651
376
275
651 – 376 = 275
The estimated difference is 300

Question 4
815 – 281
Estimate: 500 
Difference: 534

Subtract 281 from 815

815
281
534
815 – 281 = 534
The estimated difference is 500

Question 5
487 – 290

Estimate: 200
Difference: 197

487
290
197
487 – 290 = 197
The estimated difference is 200.

Question 6
936 – 329

Estimate: 600
Difference: 607

936
329
607
936 – 329 = 607
The estimated difference is 600.

Question 7
270 – 128

Estimate: 140
Difference: 142

Subtract 128 from 270
270
128
142
270 – 128 = 142
The estimated difference is 140.

Question 8
364 – 177

Estimate: 200
Difference: 187

Subtract 177 from 364
364
177
187
364 – 177 = 187
The estimated difference is 200.

Question 9
627 – 253

Estimate: 400
Difference: 374

Subtract 253 from 627
627
253
374
627 – 253 = 374
The estimated difference is 374

Question 10
862 – 419

Estimate: 450
Difference: 443

Subtract 419 from 862
862
419
443
862 – 419 = 443
The estimated difference is 450.

Question 11
726 – 148

Estimate: 550
Difference: 578

Subtract 148 from 726
726
148
578
726 – 148 = 578
The estimated difference is 550.

Question 12
543 – 358

Estimate: 200
Difference: 185

Subtract 358 from 543
543
358
185
543 – 358 = 185
The estimated difference is 200.

Problem Solving
Question 13
Mrs. Cohen has 427 buttons. She uses 195 buttons to make puppets. How many buttons does Mrs. Cohen have left?

        buttons

Answer: 232 buttons

Explanation:

Mrs. Cohen has 427 buttons.
She uses 195 buttons to make puppets.
To find how many buttons left, we have to subtract number of buttons she used to make puppets from the total number of buttons.
427 – 195 = 232
Therefore 232 buttons are left.

Question 14
There were 625 ears of corn and 247 tomatoes sold at a farm stand. How many more ears of corn were sold than tomatoes?

        more ears

Answer: 378 more ears of corn

Explanation:

There were 625 ears of corn and 247 tomatoes sold at a farm stand.
To know number of ears of corn were sold than tomatoes we have to subtract 247 from 625
625
-247
378
Thus 378 more ears of corn were sold than tomatoes.

Use Place Value to Subtract Page No 66

Question 1
On Saturday, 453 people go to a school play. On Sunday, 294 people go to the play. How many more people go to the play on Saturday?

(a) 159
(b) 169
(c) 259
(d) 747

Answer: 159

Explanation:

On Saturday, 453 people go to a school play. On Sunday, 294 people go to the play.
To find how many more people go to the play on Saturday
We need to subtract number of people go to the play on Sunday from the number of people go to the play on Saturday
= 453 – 294 = 159
159 more people go to the play on Saturday.
Thus the correct answer is option A.

Question 2
Corey has 510 marbles. He fills one jar with 165 marbles. How many of Corey’s marbles are NOT in the jar?

(a) 675
(b) 455
(c) 350
(d) 345

Answer: 345

Explanation:

Corey has 510 marbles.
He fills one jar with 165 marbles.
Let the number of Corey’s marbles are NOT in the jar be x
x + 165 = 510
x = 510 – 165
x = 345
Therefore 345 marbles are NOT in the jar.
The correct answer is option D.

Spiral Review
Question 3
Pattie brought 64 peppers to sell at the farmers’ market. There were 12 peppers left at the end of the day. How many peppers did Pattie sell?

(a) 50
(b) 52
(c) 62
(d) 78

Answer: 52

Explanation:

Pattie brought 64 peppers to sell at the farmers’ market.
There were 12 peppers left at the end of the day.
To find number of peppers did Pattie sell
Subtract 12 from 64
64 – 12 = 52
The correct answer is option B.

Question 4
An airplane flies 617 miles in the morning. Then it flies 385 miles in the afternoon. About how many more miles does the airplane fly in the morning?

(a) about 100 miles
(b) about 200 miles
(c) about 300 miles
(d) about 900 miles

Answer: about 200 miles

Explanation:

An airplane flies 617 miles in the morning.
Then it flies 385 miles in the afternoon.
Here we have to use the concept of estimated difference.
The number closer to 617 is 600
The number closer to 385 is 400
600 – 400 = 200
About 200 miles airplane fly in the morning.
So, the correct answer is option B.

Question 5
What is the unknown number?

(■ + 4) + 59 = 70

(a) 4
(b) 6
(c) 7
(d) 8

Answer: 7

Explanation:

Let ■ be the unknown number
(■ + 4) + 59 = 70
(■ + 4) = 70 – 59
(■ + 4) = 11
(■ = 11 – 4
■ = 7
Thus the correct answer is option C.

Question 6
Dexter has 128 shells. He needs 283 more shells for his art project. How many shells will Dexter use for his art project?

(a) 155
(b) 165
(c) 401
(d) 411

Answer: 411

Explanation:

Dexter has 128 shells. He needs 283 more shells for his art project.
To know the total number of shells that Dexter used for his art project
you need to add 128 and 283
283 + 128 = 411
So, the correct answer is option D.

Combine Place Values to Subtract Page No – 71

Estimate. Then find the difference.

Question 1
Estimate: 200

476 – 269

476
-269
207
The estimated difference is 200.

Question 2
615 – 342

Estimate: 300
Difference: 273

615
-342
273
The difference between 615 and 342 is 273
The estimated difference is 300.

Question 3
508 – 113

Estimate: 400
Difference: 395

508
-113
395
The difference between 508 and 113 is 395
The estimated difference is 400

Question 4
716 – 229

Estimate: 500
Difference: 487

716
229
487
The number closer to 487 is 500.
The difference is 487.

Question 5
700 – 326

Estimate: 400
Difference: 374

700
326
374
The number closer to 374 is 400.
The difference is 374.

Question 6
325 – 179

Estimate: 100
Difference: 146

325
179
146
The number closer to 146 is 100
The difference is 146.

Question 7
935 – 813

Estimate: 100
Difference: 122

935
813
122
The number closer to 122 is 100.
The difference is 122.

Question 8
358 – 292

Estimate: 50
Difference: 66

358
292
66
The number closer to 66 is 50.
The difference is 66.

Question 9
826 – 617

Estimate: 200
Difference: 209

826
617
209
The number closer to 209 is 200.
The difference is 209.

Question 10
900 – 158

Estimate: 750
Difference: 742

900
158
742
The number closer to 742 is 750.
The difference is 742

Question 11
607 – 568

Estimate: 40
Difference: 39

607
568
39
The number closer to 39 is 40.
The difference is 40.

Question 12
973 – 869

Estimate: 100

Difference: 104

973
869
104
The number closer to 104 is 100.
The difference is 104.

Problem Solving
Question 13
Bev scored 540 points. This was 158 points more than Ike scored. How many points did Ike score?

      points

Answer: 382 points

Explanation:

Bev scored 540 points. This was 158 points more than Ike scored.
Let the number of points Ike scored = x
x + 158 = 540
x = 540 – 158
x = 382
Therefore the points that Ike scored is 382.

Question 14
A youth group earned $285 washing cars. The group’s expenses were $79. How much profit did the group make washing cars?

$       profit

Answer: $206

Explanation:

A youth group earned $285 washing cars.
The group’s expenses were $79.
To find how much profit did the group make washing cars.
Subtract 79 from 285
285 – 79
285
-79
206
The group makes $206 profit by washing cars.

Lesson 11: Combine Place Values to Subtract Page No 72

Question 1
A television program lasts for 120 minutes. Of that time, 36 minutes are taken up by commercials. What is the length of the actual program without the commercials?

(a) 84 minutes
(b) 94 minutes
(c) 104 minutes
(d) 156 minutes

Answer: 84 minutes

Explanation:

A television program lasts for 120 minutes.
Of that time, 36 minutes are taken up by commercials.
To find the length of the actual program without the commercials
Subtract 36 minutes from 120 minutes
120
-36
84
Thus the length of the actual program without the commercials is 84 minutes.
The correct answer is option A.

Question 2
Syd spent 215 minutes at the library. Of that time, he spent 120 minutes on the computer. How much of his time at the library did Sid NOT spend on the computer?

(a) 85 minutes
(b) 95 minutes
(c) 105 minutes
(d) 335 minutes

Answer: 95 minutes

Explanation:

Syd spent 215 minutes at the library.
Of that time, he spent 120 minutes on the computer.
To find How much of his time at the library did Sid NOT spend on the computer
We have to subtract the time he spent on the computer from the total time he spent at the library.
i.e., 215 – 120 = 95 minutes
So, the correct answer is option B.

Spiral Review
Question 3
Xavier’s older brother has 568 songs on his music player. To the nearest hundred, about how many songs are on the music player?

(a) 500
(b) 600
(c) 700
(d) 800

Answer: 600

Explanation:

Xavier’s older brother has 568 songs on his music player.
568 to the nearest hundred is 600.
Thus the correct answer is option B.

Question 4
The students traveled to the zoo in 3 buses. One bus had 47 students. The second bus had 38 students. The third bus had 43 students. How many students in all were on the three buses?

(a) 108
(b) 118
(c) 128
(d) 138

Answer: 128

Explanation:

The students traveled to the zoo in 3 buses.
One bus had 47 students.
The second bus had 38 students.
The third bus had 43 students.
Total number of students in all three buses = x
x = 47 + 38 + 43
x = 128 students.
So, the correct answer is option C.

Question 5
Callie has 83 postcards in her collection. Of the postcards, 24 are from Canada. The rest of the postcards are from the United States. How many of the postcards are from the United States?

(a) 58
(b) 59
(c) 61
(d) 69

Answer: 59

Explanation:

Callie has 83 postcards in her collection.
Of the postcards, 24 are from Canada.
The rest of the postcards are from the United States.
Subtract 24 from 83 we get the number of postcards is from the United States.
83 – 24 = 59
So, the correct answer is option B.

Question 6
There were 475 seats set up for the school play. At one performance, 189 of the seats were empty. How many seats were filled at that performance?

(a) 286
(b) 296
(c) 314
(d) 396

Answer: 286

Explanation:

There were 475 seats set up for the school play.
At one performance, 189 of the seats were empty.
Let the Number of seats were filled at that performance = x
x + 189 = 475
x = 475 – 189
x = 286
Thus the correct answer is option A.

Problem Solving • Model Addition and Subtraction Page No – 77

Use the bar model to solve the problem.

Question 1
Elena went bowling. Elena’s score in the first game was 127. She scored 16 more points in the second game than in the first game. What was her total score?

Go Math Grade 3 Chapter 1 Model Addition and Substraction Problem Solving Question 1

Question 2
Mike’s Music sold 287 CDs on the first day of a 2-day sale. The store sold 96 more CDs on the second day than on the first day. How many CDs in all were sold during the 2-day sale?

Go Math Grade 3 Chapter 1 Model Addition and Substraction Problem Solving Question 2

      CDs

Answer: 670 CDs

Explanation:

Mike’s Music sold 287 CDs on the first day of a 2-day sale.
The store sold 96 more CDs on the second day than on the first day.
The means Mike’s music sold CDs on the second day = 287 + 96 = 383.
★ = 283 CDs
Total CDs were sold during the 2-day sale = 383 + 287
♦ = 383 + 287 = 670 CDs

Lesson Check Page No – 78

Question 1
Ms. Hinely picked 46 tomatoes from her garden on Friday. On Saturday, she picked 17 tomatoes. How many tomatoes did she pick in all?

(a) 109
(b) 63
(c) 53
(d) 29

Answer: 63

Explanation:

Ms. Hinely picked 46 tomatoes from her garden on Friday.
On Saturday, she picked 17 tomatoes.
First, find how many tomatoes did she pick in all.
46 + 17 = ★
★ = 63
So, the correct answer is option B.

Question 2
Rosa read 57 pages of a book in the morning. She read 13 fewer pages in the afternoon. How many pages did Rosa read in the afternoon?

(a) 44
(b) 60
(c) 70
(d) 83

Answer: 44

Explanation:

Rosa read 57 pages of a book in the morning.
She read 13 fewer pages in the afternoon.
57 – 13 = ♦
♦ = 57 – 13
♦ = 44
Thus the correct answer is option A.

Spiral Review
Question 3
Mike has 57 action figures. Alex has 186 action figures. Which is the best estimate of the number of action figures Mike and Alex have altogether?

(a) 150
(b) 250
(c) 350
(d) 400

Answer: 250

Explanation:

Mike has 57 action figures.
Alex has 186 action figures.
186 – 57 = ★
★ = 186 – 57
★ = 129
Now Add Mike and Alex action figures
♦ = 186 + 57 = 243
The estimated figure of 243 is 250.
Thus the correct answer is option B.

Question 4
There are 500 sheets of paper in the pack Hannah bought. She has used 137 sheets already. How many sheets of paper does Hannah have left?

(a) 363
(b) 463
(c) 400
(d) 637

Answer: 363

Explanation:

There are 500 sheets of paper in the pack Hannah bought. She has used 137 sheets already.
To find how many sheets of paper does Hannah have left
We have to subtract the number of sheets used from the total number of sheets.
500 – 137 = ★
★ = 500 – 137
★ = 363
Therefore 343 sheets are left.
The correct answer is option A.

Question 5
There were 378 visitors to the science museum on Friday. There were 409 visitors on Saturday. How many more people visited the museum on Saturday?

(a) 25
(b) 31
(c) 171
(d) 787

Answer: 31

Explanation:

There were 378 visitors to the science museum on Friday.
There were 409 visitors on Saturday.
To find how many more people visited the museum on Saturday.
Subtract the number of visitors on Friday from the number of visitors on Saturday.
409 – 378 = 31
31 people visited more the museum on Saturday.
So the correct answer is option B.

Question 6
Ravi scores 247 points in a video game. How many more points does he need to score a total of 650?

(a) 897
(b) 430
(c) 417
(d) 403

Answer: 403

Explanation:

Ravi scores 247 points in a video game.
Let x be the points he needs to score a total of 650
x + 247 = 650
x = 650 – 247
x = 403
Thus he needs 403 points to make a score of 650.
The correct answer is option D.

Review/Test – Page No 79

Question 1

For numbers 1a–1d, choose Yes or No to tell whether the sum is even.

a. 5 + 8

(a) yes
(b) no

Answer: No

Explanation:

5 + 8 = 13 is an odd number.
So, the answer is no.

Question 1
b. 9 + 3

(a) yes
(b) no

Answer: Yes

Explanation:

9 + 3 = 12 is an even number.
So, the answer is yes.

Question 1
c. 6 + 7

(a) yes
(b) no

Answer: No

Explanation:

6 + 7 = 13 is an odd number.
So, the answer is no.

Question 1
d. 9 + 5

(a) yes
(b) no

Answer: Yes

Explanation:

9 + 5 = 14 is an even number.
So, the answer is yes.

Question 2
Select the number sentences that show the Commutative Property of Addition. Mark all that apply.

(a) 14 + 8 = 22
(b) 8 + 14 = 14 + 8
(c) 8 + (13 + 1) = (8 + 13) + 1
(d) (5 + 9) + 8 = (9 + 5) + 8

Answer: 8 + 14 = 14 + 8

Explanation:

According to the commutative property of addition, changing the order of the numbers we are adding, does not change the sum.
So, the answer is option B.

Question 3
Select the numbers that round to 300 when rounded to the nearest hundred. Mark all that apply.

(a) 238
(b) 250
(c) 283
(d) 342
(e) 359

Answer: 283

Explanation:
283 rounded to the nearest hundred is 300.
So, the correct answer is option C.

Question 4
There are 486 books in the classroom library. Complete the chart to show 486 rounded to the nearest 10.

Go Math Grade 3 Chapter 1 Model Addition and Substraction Problem Solving Question 4

Answer:

Hundreds Tens Ones
400 90 0

486 rounded to the nearest ten is 490.

Review/Test – Page No – 80

Question 5
Write each number sentence in the box below the better estimate of the sum.

393+225=■ 481+215=■

352+328=■ 309+335=■

Write each number sentence in the box below the better estimate of the sum

Answer:

600 700
393+225 = 618
The estimated sum is 600.309+335= 644
The estimated sum is 600.
481+215= 696
The estimated sum is 700.352+328= 680
The estimated sum is 700.

Explanation:

393+225=■
■ = 618
The number closer to 618 is 600

481+215=■
■ = 696
The number closer to 696 is 700

352+328=■
■ = 680
The number closer to 680 is 700.

309+335=■
■ = 644
The number closer to 644 is 600.

Question 6
Diana sold 336 muffins at the bake sale. Bob sold 287 muffins. Bob estimates that he sold 50 fewer muffins than Diana. How did he estimate? Explain.

Answer:

Diana sold 336 muffins at the bake sale.
Bob sold 287 muffins.
Bob estimates that he sold 50 fewer muffins than Diana.
To know whether his estimation is right or wrong we have to subtract muffins that Bob sold from muffins that Diana sold
336 – 287 = 49
The number closer to 49 is 50.
So, Bob’s estimation is correct.

Question 7
The table shows how many books each class read.

Go Math Grade 3 The table shows how many books each class read

For numbers 7a–7d, select True or False for each statement.

a. Ms. Martin’s class read about 100 more books than Mr. Lopez’s class.

(i) True
(ii) False

Answer: True

Explanation:

Number of books that Mr. Lopez’s class read = 273
Number of books that Ms. Martin’s class read = 402
402
– 273
129
So, the statement Ms. Martin’s class read about 100 more books than Mr. Lopez’s class is true.

Question 7
b. The 3 classes read over 900 books altogether.

(i) True
(ii) False

Answer: True

Explanation:

Number of books that Mr. Lopez’s class read = 273
Number of books that Ms. Martin’s class read = 402
Number of books that Mrs. Wang read = 247
273
402
274
949
Therefore the statement the 3 classes read over 900 books altogether is true.

Question 7
c. Mrs. Wang’s class read about 50 fewer books than Mr. Lopez’s class.

(i) True
(ii) False

Answer: False

Explanation:

Number of books that Mrs. Wang read = 247
Number of books that Mr. Lopez’s class read = 273
273
– 247
26
Thus the statement Mrs. Wang’s class read about 50 fewer books than Mr. Lopez’s class is false.

Question 7
d. Ms. Martin’s and Mrs. Wang’s class read about 700 books.

(i) True
(ii) False

Answer: False

Explanation:

Number of books that Ms. Martin’s class read = 402
Number of books that Mrs. Wang read = 247
402
247
649
Therefore the statement Ms. Martin’s and Mrs. Wang’s class read about 700 books is false.

Review/Test – Page No – 81

Question 8
Janna buys 2 bags of dog food for her dogs. One bag weighs 37 pounds. The other bag weighs 15 pounds. How many pounds do both bags weigh? Explain how you solved the problem.

      pounds

Answer: 52 pounds

Explanation:

Janna buys 2 bags of dog food for her dogs. One bag weighs 37 pounds. The other bag weighs 15 pounds.
I used friendly numbers

37 = 35 + 2
15 = 15 + 0
52 =  50 + 2
The weight of 2 bags is 52 pounds.

Question 9
Choose the property that makes the statement true.

The Go Math grade 3 Chapter 1 answer key review image_1 Property of addition states that you can group addends in different ways and get the same sum.

Answer: The Associative Property of addition states that you can group addends in different ways and get the same sum.

Use the table for 10–12.

Go Math Grade 3 Chapter 1 Problem Solving Use the table for 10–12

Question 10
The table shows the number of sweaters sold online in three months. How many sweaters were sold in January and February?

        sweaters

Answer: 700 sweaters

Explanation:

The number of sweets sold in the month of January = 402
The number of sweets sold in the month of February = 298
First, make the friendly numbers to make the addition easy.|
Subtract 2 from 402 = 402 – 2 = 400
Next add 2 to 298 = 298 + 2 = 300
Now add both
400 + 300 = 700
Therefore 700 sweaters were sold in January and February.

Question 11
How many more sweaters were sold in January than March?

        sweaters

Answer: 231 sweaters

Explanation:

The number of sweets sold in the month of January = 402
The number of sweets sold in the month of March = 171
To find how many more sweaters were sold in January than March, we have subtracted the number of sweaters sold in the march from January
402 – 171 = 231
231 more sweaters were sold in January than March.

Question 12

How many more sweaters were sold in February and March than in January?

        sweaters

Answer: 67 sweaters

Explanation:

The number of sweets sold in the month of January = 402
The number of sweets sold in the month of February = 298
The number of sweets sold in the month of March = 171
Total number of sweaters sold in February and March = 298 + 171 = 469
Now subtract 402 from 469
469 – 402 = 67 sweaters
67 more sweaters were sold in February and March than in January.

Review/Test – Page No – 82

Question 13
Help Dana find the sum.

346 + 421 + 152
For numbers 13a–13d, select Yes or No to tell Dana when to regroup.

a. Regroup the ones.

(a) yes
(b) no

Answer: Yes

Question 13
b. Add the regrouped ten.

(a) yes
(b) no

Answer: No

Question 13
c. Regroup the tens.

(a) yes
(b) no

Answer: Yes

Question 13
d. Add the regrouped hundred.

(a) yes
(b) no

Answer: Yes

Question 14
Alexandra has 78 emails in her inbox. She deletes 47 emails. How many emails are left in her inbox? Draw jumps and label the number line to show your thinking.

Go Math Grade 3 Chapter 1 Alexandra has 78 emails in her inbox. She deletes 47 emails

        emails

Answer: 31 emails

Explanation:

Alexandra has 78 emails in her inbox.
She deletes 47 emails.
Let x be the number of emails left in her inbox
x + 47 = 78
x = 78 – 47
x = 31
Therefore, 31 emails are left in her inbox.

Question 15
Daniel has 402 pieces in a building set. He uses 186 pieces to build a house. How many pieces does he have left? Show your work.

        pieces

Answer: 216 pieces

Explanation:

Daniel has 402 pieces in a building set.
He uses 186 pieces to build a house.
x be the number of pieces he had left
x + 186 = 402
x = 402 – 186
x = 216 pieces
Thus he left 216 pieces to build a house

Review/Test – Page No – 83

Question 16
Luke solves this problem. He says the difference is 214. Explain the mistake Luke made. What is the correct difference?

352−148 =        

Answer: 204

Explanation:

Make friendly numbers to make the subtraction easy.
First subtract 2 from 352 = 350
350
148
202
1 will be borrowed from tens place. So 0 becomes 10.
10 – 8 = 2
4 – 4 = 0
300 – 100 = 200
200 + 2 = 202
Now add 2 to 202 you get 204.

Question 17
Sunnyday Elementary School is having its annual Read-a-thon. The third graders have read 573 books so far. Their goal is to read more than 900 books. What is the least number of books they need to read to reach their goal? Explain.

        books

Answer: 327 books

Explanation:

Sunnyday Elementary School is having its annual Read-a-thon.
The third graders have read 573 books so far. Their goal is to read more than 900 books.
Let the 3rd graders have to read the total number of books = x
x + 573 = 900
x = 900 – 573
x = 327
Thus the least number of books they need to read to reach their goal is 327 books.

Question 18
There are 318 fiction books in the class library. The number of nonfiction books is 47 less than the number of fiction books.

Part A

About how many nonfiction books are there in the class library? Explain.

About         nonfiction books

Answer: About 270 nonfiction books

Explanation:

Given that,
There are 318 fiction books in the class library.
The number of nonfiction books is 47 less than the number of fiction books.
Number of non fictions books = x
x + 47 = 318
x = 318 – 47
x = 271
The number closer to 271 is 270.
So, there are about 270 nonfiction books.

Question 18
Part B

How many fiction and nonfiction books are there in the class library altogether? Show your work.

        total books

Answer: 589

Explanation:

Number of fiction books = 318
Number of nonfiction books = 271
To find the total number of books we need to add both fiction and nonfiction books
= 318 + 271 = 589
There are 589 books in the class library.

Review/Test – Page No – 84

Question 19
Alia used 67 + 38 = 105 to check her subtraction. Which math problem could she be checking? Mark all that apply.

67−38=■
105−67=■
105+38=■
105−38=■

Answer: 105−67= 38; 105−38=67
She can use option B and Option D to check her subtraction.

Question 20
Alex and Erika collect shells. The tables show the kinds of shells they collected.

Go Math Grade 3 Chapter 1 Problem Solving Alex and Erika collect shells. The tables show the kinds of shells they collected.

Part A

Who collected more shells? How many did she collect? About how many more is that? Explain how you solved the problem.

       

Answer: Alex

Alxe’s Shells:
Number of Scallop = 36
Number of Jingle shells = 95
Number of Clam = 115
Now add all the three shells = 36 + 95 + 115 = 246 shells

Erika’s shells:

Number of Scallop = 82
Number of Whelk shells = 28
Number of Clam = 108
Now add all the three shells = 82 + 28 + 108 = 218 shells
Alex collected about 250 shells.

Question 20
Part B

Alex and Erika have the greatest number of what kind of shell? How many shells of that kind do they have? Show your work.

Answer: Clam

The greatest number of shells that Alex and Erika collected are Clam.

Conclusion

In addition to the exercise and homework problems we also provide the solutions for the Extra Practice. So, the students are advised to go through the Go Math Answer Key Grade 3 Chapter 1 Addition and Subtraction within 1,000 Extra Practice to test your math skills in this chapter. You can also your friends to improve their math skills by sharing this link.

Go Math Grade 3 Answer Key Chapter 3 Understand Multiplication Assessment Test

Go Math Grade 3 Answer Key Chapter 3 Understand Multiplication Assessment Test includes all the topics in Chapter 3. 3rd Grade Go Math Solutions Key covers different questions and provides a detailed explanation to all of them. You can test your preparation standard and understand the areas you are lagging and improvise on them.

Enhance your Subject Knowledge by taking the help of the Go Math Grade 3 Answer Key Chapter 3 Understand Multiplication Assessment Test. Get to know the Topics existing in Go Math Ch 3 Understand Multiplication through the quick links present. To help you understand the concepts better we have mentioned detailed solutions for all the Problems in Chapter Test, Performance Test, Cumulative Practice, etc.

Chapter 3: Understand Multiplication Assessment Test

Test – Page 1 – Page No. 31

Question 1.
There are 4 flower beds in Max’s yard. Three rose bushes grow in each flower bed. How many rosebushes are there? Draw circles to model the problem and explain
how to solve it.
_______ rosebushes

Answer: 12 rosebushes

Explanation: 

Go Math Grade 3 Answer Key Chapter 3 Understand Multiplication Assessment Test

In the above figure, each big circle represents a flower bed and three small circles represent rosebushes. By using this model students can count the number of circles or add 3 + 3+ 3+ 3 = 12 or multiply 4 (flower beds) × 3 (rosebushes) = 12 rosebushes

Question 2.
Greta put 6 coins into each of 3 stacks. She wrote this number sentence to represent the total number of coins.
3 × 6 = 18
What is a related number sentence that also represents the total number of coins she has?
Options:
a. 6 × 3 = ■
b. 6 + 3 = ■
c. 3 + 3 + 3 = ■
d 6 × 6 = ■

Answer: a.

Explanation: For the written number sentence by 3 × 6 = 18 Greeta, only option a. 6 × 3 = 18 matches. The remaining options don’t give an answer as option a.

Question 3.
Cecile went fishing for three days at a lake. The first jump on the number line shows how many fish she caught the first day. She caught the same number of fish the next two days.
Go Math Grade 3 Answer Key Chapter 3 Understand Multiplication Assessment Test Test - Page 1 img 1
Write the multiplication sentence that the number line shows.
_____ × _____ = _____

Answer: 3 × 3 = 9

Explanation:
Cecile made 3 jumps, for every jump she caught 3 fishes
Therefore  3 × 3 = 9.

Test – Page 2 – Page No. 32

Question 4.
Ben drew an array to show the number of video games he has.
Write a multiplication sentence for the array.
Go Math Grade 3 Answer Key Chapter 3 Understand Multiplication Assessment Test Test - Page 2 img 2
_____ video games

Answer: 8 × 2 = 16.

Explanation:
In the given array each row contains 8 video games, total it has 2 rows
Therefore multiplication sentence for the array = 8 × 2 = 16.

Question 5.
Julissa makes 4 bracelets. She uses 9 charms on each bracelet.
For numbers 5a–5d, tell if the number sentence could be used to find the number of charms Julissa uses.
a. 4 + 9 = ■
i. yes
ii. no

Answer: No

Explanation: He cannot use  4 + 9 in this sentence as it gives result = 13 which not correct.

Question 5.
b. 3 + 3 + 3 + 3 = ■
i. yes
ii. no

Answer: No

Explanation: He cannot use 3 + 3 + 3 + 3 this sentence as it gives result = 13 which not correct.

Question 5.
c. 9 + 9 + 9 + 9 = ■
i. yes
ii. no

Answer: Yes

Explanation: He can use 9 + 9 + 9 + 9 = 36, which gives no of charms used on 4 bracelets.

Question 5.
d. 4 × 9 = ■
i. yes
ii. no

Answer: Yes

Explanation: He can use 4 × 9 = 36, which gives no of charms used on 4 bracelets.

Question 6.
Edith sorts buttons into 4 groups for her art project. Each group contains 6 buttons. How many buttons does Edith sort? Make a bar model to solve the problem.
Go Math Grade 3 Answer Key Chapter 3 Understand Multiplication Assessment Test Test - Page 2 img 3
_____ buttons

Answer: 24 buttons.

Explanation: Given that Edith sorts buttons into 4 groups, where each group contains 6 buttons.

Go Math Grade 3 Answer Key Chapter 3 Understand Multiplication Assessment Test

From the given info, we drew the above diagram, from the above model total no of buttons = 6+6+6+6 = 24 buttons

Question 7.
Select the number sentences that show the Commutative Property of Multiplication. Mark all that apply.
Options:
a. 5 × 2 = 5 + 5
b. 6 × 0 = 6
c. 7 × 5 = 5 × 7
d. 8 × 1 = 1 × 8
e. 9 × 1 = 9

Answer: c,d.

Explanation:
Commutative Property of Multiplication means a × b = b × a.
Therefore options b and c,  7 × 5 = 5 × 7 and 8 × 1 = 1 × 8 shows the property.

Test – Page 3 – Page No. 33

Question 8.
There are 5 tables in the library. Four students are sitting at each table. How many students are sitting in the library?
_____ students

Answer: 5 × 4 = 20 students

Explanation:
Given that there 5 tables in the library, 4 students are sitting on each table
Therefore total no of students = 5 × 4 = 20 students.

Question 9.
Keisha needs 3 equal lengths of rope for a Field Day activity. The jump on the number line shows the length of one rope in yards. How many yards of rope does Keisha need?
Go Math Grade 3 Answer Key Chapter 3 Understand Multiplication Assessment Test Test - Page 3 img 4
_____ yards

Answer: 15 yards.

Explanation:
Given that there are 3 equal length rope
From the number line length of the one rope = 5 yards
Since he is having 3 ropes, the total length of all the 3 ropes = 5 × 3 = 15 yards.

Question 10.
Anna’s mom makes 3 sandwiches every school day. Each sandwich gets 3 slices of cheese. How many slices of cheese will Anna’s mom need for all the sandwiches she makes on 2 school days?
_____ slices of cheese

Answer: 18 slices of cheese.

Explanation:   No of Sandwiches Anna’s mom make = 3
Each sandwich having 3 slices of cheese, no of cheese in 3 sandwiches = 3 × 3 = 9
Therefore to make 3 sandwiches for 2 days, the number of cheese slices needed = (3 ×3) × 2 = 18.

Question 11.
Angelo stacked 30 cans of soup collected during a food drive.
Go Math Grade 3 Answer Key Chapter 3 Understand Multiplication Assessment Test Test - Page 3 img 5
Select other ways Angelo could arrange the same number of cans. Mark all that apply.
Options:
a. 1 row of 30
b. 5 rows of 6
c. 6 rows of 6
d. 8 rows of 4
e. 10 rows of 3

Answer:  a, b, e.

Explanation: Total no of soup cans = 30, therefore only by arranging as given in options a, b, e, gives us 30 cans.

Test – Page 4 – Page No. 34

Question 12.
Choose the number that makes the sentence true.
The product of any number and Go Math Grade 3 Answer Key Chapter 3 Understand Multiplication Assessment Test Test - Page 4 img 6 is zero.
_____

Answer: Product of any number and zero is zero

Explanation:

The product of any number and Go Math Grade 3 Answer Key Chapter 3 Understand Multiplication Assessment Test is zero.

Question 13.
Ellen made this array to show that 2 × 9 = 18.
Go Math Grade 3 Answer Key Chapter 3 Understand Multiplication Assessment Test Test - Page 4 img 7
Part A
Ellen says that 9 × 2 = 18. Is Ellen correct? Draw an array to explain your answer.
i. yes
ii. no

Answer: Yes

Explanation: Yes, Ellen is correct. 9 × 2 = 18

Go Math Grade 3 Answer Key Chapter 3 Understand Multiplication Assessment Test

Question 13.
Part B
Which number property supports your answer? Explain.
________

Answer: Commutative Property of Multiplication.

Explanation: The given array shows 2 × 9 = 18 and 9 × 2 = 18, which is a Commutative Property of Multiplication, so Ellen is correct.

Question 14.
Abdul has a collection of stamps. He puts the stamps in 2 equal groups. There are 7 stamps in each group. How many stamps does Abdul have? Use the number line to show your work.
Go Math Grade 3 Answer Key Chapter 3 Understand Multiplication Assessment Test Test - Page 4 img 8
_____ stamps

Answer: 14 stamps.

Explanation:
Given that Abdul has 2 equal groups of stamps
Each group contains 7 stamps in it, so total stamps in both groups = 7 × 2 = 14 stamps.

Test – Page 5 – Page No. 35

Question 15.
Hudson and Asher each collect comic books.
Part A
Hudson sorts his comic books into 3 piles. Each group has 7 comic books. How many comic books does he have?
_____ comic books

Answer: 21 comic books

Explanation:
Given that Hudson sorted his books into 3 piles
Each group has 7 comic books
Total no of comic books in 3 piles = 3 × 7 = 21.

Question 15.
Part B
Asher sorts his comic books into 4 piles. Each pile has 2 comic books in it. Write a multiplication sentence to show how many comic books Asher has.
Then find how many comic books Hudson and Asher have.

Answer: 2 × 4 = 8; 21 + 8 = 29 comic books

Explanation:
Given that Asher sorted his comic books into 4 piles
Each group has 2 comic books
Total no of comic books in 3 piles = 2 × 4 = 8
Total no of comic books both Hudson and Asher have = 21 + 8 = 29.

Question 16.
Aiden sees 4 lifeguard towers at the beach. Each tower has 1 lifeguard. Write a multiplication sentence to show the number of lifeguards Aiden sees.
_____ × _____ = _____ lifeguard towers

Answer: 4 lifeguards.

Explanation: 4 × 1 = 4 lifeguards.

Question 17.
Jorge spends 7 minutes completing each of 4 puzzles. He can use 7 × 4 to find the total amount of time he spends on the puzzles.
For numbers 17a–17d, choose Yes or No to show which expressions are equal to 7 × 4 .
a. 7 + 4
i. yes
ii. no

Answer: No

Explanation: No, 7 × 4 = 28, where in 7 + 4 = 11.

Question 17.
b. 7 + 7 + 7 + 7
i. yes
ii. no

Answer: Yes

Explanation: Yes, 7 × 4 = 28, where in 7 + 7 + 7 + 7 = 28.

Question 17.
c. 4 + 4 + 4 + 4 + 4 + 4 + 4
i. yes
ii. no

Answer: Yes

Explanation: Yes, 7 × 4 = 28, where in 4 + 4 + 4 + 4 + 4 + 4 + 4 = 28.

Question 17.
d. 7 + 7 + 7 + 7 + 7 + 7 + 7
i. yes
ii. no

Answer: No

Explanation: No, 7 × 4 = 28, where in 7 + 7 + 7 + 7 + 7 + 7 + 7 = 49.

Test – Page 6 – Page No. 36

Question 18.
Maya buys 3 bags of dried pears. Each bag contains 6 dried pears.
Select the number sentences that show all the dried pears Maya buys. Mark all that apply.
Options:
a. 3 + 3 + 3 = 9
b. 3 + 3 + 3 + 3 + 3 + 3 = 18
c. 6 + 3 = 9
d. 6 + 6 + 6 = 18
e. 3 × 6 = 18
f. 9 + 9 = 18

Answer: b, d, and e

Explanation:
Given that Maya buys 3 bags of dried pears
Each bag contains 6 dried pears = 3 × 6 = 18
Therefore options b, d, and e match the dried pears Maya buys.

Question 19.
Javier is making 6 smoothies. He puts 4 strawberries and 1 banana in each smoothie.
Part A
Write the total number of strawberries and bananas he uses. Write a multiplication sentence for each
______ strawberries            ______ bananas

Answer: 6 × 4 = 24 strawberries, 6 × 1 = 6 bananas.

Explanation:
Given that Javier is making 6 smoothies, uses 4 strawberries and 1 banana in each smoothie
Therefore to make 6 smoothies no of strawberries needed = 6 × 4 = 24 strawberries,
And the number of bananas needed to make 6 smoothies = 6 × 1 = 6 bananas.

Question 19.
Part B
After making 6 smoothies, Javier has 9 strawberries and 4 bananas left. What is the greatest number of smoothies he can make with that fruit if he uses the same recipe for all the smoothies? Draw models and use them to explain.

Answer: At most, he can make 2 more smoothies.

Explanation:

Go Math Grade 3 Answer Key Chapter 3 Understand Multiplication Assessment Test

He can make 4 smoothies with 1 banana each, but only 2 smoothies with 4 strawberries each, so the greatest number of smoothies he can make is 2. There will be 2 bananas and 1 strawberry left.

Summary

We believe the knowledge shed regarding the Go Math Grade 3 Chapter 3 Assessment Test has helped you in your way of preparation. If you need any assistance check out the Go Math Grade 3 Answer Key Chapter 3 Understand Multiplication and resolve your doubts at the moment.

Go Math Grade 3 Answer Key Chapter 9 Compare Fractions Assessment Test

Enhance your skills using the Go Math Grade 3 Answer Key Chapter 9 Compare Fractions Assessment Test. Score better grades in the exam by practicing using the 3rd Grade Go Math Answer Key Ch 9 Compare Fractions Assessment Test. We have listed all the topics of Chapter 9 in our Assessment Test and even provided Solutions to all the problems given.

Students can check how much they understood the topics by solving the Questions in Grade 3 Go Math Chapter 9 Compare Fractions Assessment Test. Get acquainted with the areas you are lagging and improvise on them accordingly.

Chapter 9: Compare Fractions Assessment Test

Test – Page 1 – Page No. 91

Question 1.
Frank and Dwayne weed their gardens that are the same size. Frank’s garden is divided into 6 equal sections. Dwayne’s garden is divided into 4 equal sections. Each boy has weeded 2 sections of his garden.
Write a fraction to describe what part of his garden each boy has weeded. Then tell who weeded a larger area. Explain
_________

Answer: Dwayne’s part 2/4 > Frank’s part 2/6.

Explanation:
Frank and Dwayne has the same size of gardens
Frank’s garden is divided into 6 equal sections
Dwayne’s garden is divided into 4 equal sections
Since each has weeded 2 sections of their gardens, Frank’s part would be = 2/6,
Dwayne’s part would be = 2/4.
Therefore Dwayne’s part 2/4 > Frank’s part 2/6,
Since the garden with a greater number of sections will have a smaller area per section.

Question 2.
Eli, Beth, and Cory are reading the same book for class. Eli read \(\frac{3}{4}\) of his book. Beth read \(\frac{3}{8}\) of her book and Cory read \(\frac{3}{6}\) of his book. For 2a–2d, choose Yes or No to indicate whether the comparisons are correct.
a. \(\frac{3}{4}\) > \(\frac{3}{8}\)
i. yes
ii. no

Answer: Yes.

Explanation: 3/4 > 3/8. Since both are reading the same book and book with a greater number of sections will have a lesser value.

Question 2.
b. \(\frac{3}{6}\) < \(\frac{3}{8}\)
i. yes
ii. no

Answer: No, 3/6 < 3/8.

Explanation: It’s not true, Since both are reading the same book, and books with a greater number of sections will have a lesser value. Therefore 3/6 > 3/8).

Question 2.
c. \(\frac{3}{8}\) = \(\frac{3}{6}\)
i. yes
ii. no

Answer: No

Explanation: 3/8 = 3/6. Both are not equal, since the same book has been divided into different no of parts. Therefore both are not equal.

Question 2.
d. \(\frac{3}{6}\) < \(\frac{3}{4}\)
i. yes
ii. no

Answer: Yes.

Explanation: 3/6 < 3/4. Since both are reading the same book and book with a greater number of sections will have a lesser value.

Question 3.
Mark and Lisa are on the swim team. Mark swims \(\frac{3}{8}\) mile each day. Lisa swims \(\frac{5}{8}\) mile each day. Which statement is correct? Mark all that apply.
Options:
a. Mark swims farther than Lisa each day.
b. Lisa swims the same distance as Mark each day.
c. Lisa swims less than 1 mile each day.
d. Lisa swims farther than Mark each day.

Answer: Both c and d are correct.

Explanation: Lisa swims less than 1 mile while 5/8 mile each day and She swims farther than Mark each day. i.e. 5/8 > 3/8.

Test – Page 2 – Page No. 92

Question 4.
MacKenzie and Cassie used fabric to make costumes for a play. MacKenzie used \(\frac{3}{4}\) yard of fabric and Cassie used \(\frac{5}{6}\) yard. Who used more fabric? Explain the strategy you used to solve the problem.
_________

Answer:
MacKenzie divided fabric into 4 equal parts and used 3/4 of it. So, 1/4 of it is left. Cassie divided the same yard of fabric into 6 equal parts and used 5/6 of it. Here 1/6 part of it is left, by comparing leftover pieces we can conclude 1/4 > 1/6. Since the larger piece was left with MacKenzie, Cassie has used more fabric.

Question 5.
The soccer team practices passing for \(\frac{3}{4}\) hour and shooting for \(\frac{4}{5}\) hour. On which drill does the team spend less time? Explain how you can use the model to find the answer.
Go Math Grade 3 Answer Key Chapter 9 Compare Fractions Assessment Test Test - Page 2 img 1

Answer: From the model, we can understand that team has spent less time on Passing. Since the model for 3/4 is shorter than the model for 4/5.

Question 6.
Andrew bought \(\frac{7}{8}\) pound of mixed nuts. Margaret bought \(\frac{5}{8}\) pound of mixed nuts. Use the fractions and symbols to show which amount is greater.
Go Math Grade 3 Answer Key Chapter 9 Compare Fractions Assessment Test Test - Page 2 img 2

Answer: 5/8 < 7/8 or 7/8 > 5/8

Explanation:

Go Math Grade 3 Answer Key Chapter 9 Compare Fractions Assessment Test

Test – Page 3 – Page No. 93

Question 7.
Mr. Worth opened new jars of 4 different colors of paint for an art project. All of the jars were the same size.
Go Math Grade 3 Answer Key Chapter 9 Compare Fractions Assessment Test Test - Page 3 img 3
Part A Draw lines to show how Mr. Worth could divide one jar of paint into halves, one into thirds, one into fourths, and one into sixths.
Part B Students in his class used an equivalent amount of two paint colors. Use the models to show the amount of paints used. Write two pairs of equivalent fractions to represent the models.

Answer: The below figure represents halves, one-third, one into fourth, one into sixths.

Explanation:

Go Math Grade 3 Answer Key Chapter 9 Compare Fractions Assessment Test
Representing Halves           one-third                      one into fourth           one into sixths
Therefore from the above figure, two pairs of equivalent fractions representing the models are 1/2 = 2/4, 1/3 = 2/6.

Question 8.
Dalton rode his skateboard for \(\frac{3}{4}\) mile. Amelia rode her skateboard for an equal distance. What is an equivalent fraction that describes how far Amelia rode? Use the models to show your work.
Go Math Grade 3 Answer Key Chapter 9 Compare Fractions Assessment Test Test - Page 3 img 4
\(\frac{3}{4}\) = \(\frac{□}{□}\)

Answer: \(\frac{3}{4}\) = \(\frac{6}{8}\).

Explanation: Below two pictures give Dalton and Amelia distance covered while riding a skateboard. Though they both rode the same distance to represent them in similar different fractions. We can divide Dalton’s covered as in the 1st circle and Amelia’s in 2nd Circle. From the pictures, equivalent fractions will be 3/4 for Dalton and 6/8 for Amelia.

Go Math Grade 3 Answer Key Chapter 9 Compare Fractions Assessment Test

Question 9.
Mr. Barrows opens 2 packs of paper. He puts \(\frac{2}{5}\) of a pack in one pile and \(\frac{3}{5}\) of a pack in another pile. Which pile has more paper? Show your work.
\(\frac{□}{□}\) of a pack

Answer: 3/5 of a pack has more papers.

Explanation: The pile with 3/5 of a pack has more papers. Since 2/5 is there in another pack.

Test – Page 4 – Page No. 94

Question 10.
Treyvon watched \(\frac{2}{8}\) a movie. Juan watched \(\frac{2}{6}\) of the same movie. Use >, =, or < to compare the parts that they watched.
\(\frac{2}{8}\) _____ \(\frac{2}{6}\)

Answer: \(\frac{2}{8}\) < \(\frac{2}{6}\) or \(\frac{2}{6}\) > \(\frac{2}{8}\).

Question 11.
Alison used \(\frac{7}{8}\) quart of orange juice and \(\frac{3}{8}\) quart of cranberry juice to make some punch.
For 11a–11d, select True or False for each comparison.
a. \(\frac{7}{8}\) < \(\frac{3}{8}\)
i. True
ii. False

Answer: False.

Explanation: 7/8 is greater than 3/8.

Question 11.
b. \(\frac{7}{8}\) > \(\frac{3}{8}\)
i. True
ii. False

Answer: True.

Explanation: Since both have the same denominators, we can compare their numerators directly. Here 7 is greater than 3, therefore 7/8 is greater than 3/8.

Question 11.
c. \(\frac{3}{8}\) < \(\frac{7}{8}\)
i. True
ii. False

Answer: True.

Explanation: \(\frac{3}{8}\) < \(\frac{7}{8}\)

Question 11.
d. \(\frac{3}{8}\) = \(\frac{7}{8}\)
i. True
ii. False

Answer: False.

Explanation: Both are not equal.

Question 12.
Will, Ann, and Jim are working on their science fair projects. Will has finished \(\frac{1}{4}\) of his project. Ann has finished \(\frac{3}{4}\) of her project, and Jim has finished \(\frac{2}{3}\) of his project.
Part A
Who has finished less of their project, Will or Ann?
Explain how you know.
__________

Answer: Will has finished less compared to Ann.

Explanation: Given that will has finished 1/4 of his project wherein Ann has finished 3/4 of her project. Since 1/4 and 3/4 have the same denominator, so we can compare the numerator directly. Since 1 is less than 3, we can say 1/4 is less than 3/4. Thus Will has finished less part of his project compared to Ann.

Question 12.
Part B
Who has finished less of their project, Ann or Jim?
Explain how you know.
__________

Answer: 2/3 is less than 3/4. Therefore, Jim has finished less part of his project compared to Ann.

Explanation: Given that Jim has finished 2/3 of his project were in Ann has finished 3/4 of her project. Since denominators are not the same to compare first we shall make the same. Therefore multiply Jim’s work (2/3) with (4/4) which gives = 8/12 and Ann’s work (3/4) with (3/3) which gives = 9/12. Now by comparing numerators 8/12 is less than 9/12, which means Jim has finished less part of his project compared to Ann.

Test – Page 5 – Page No. 95

Question 13.
Sarah needs \(\frac{4}{6}\) yard of ribbon to wrap a gift. She has 6 pieces of ribbon with the following lengths. She can cut the piece if it is too long. Mark all of the pieces of ribbon that Sarah could use.
Options:
a. \(\frac{1}{2}\) yard
b. \(\frac{6}{6}\) yard
c. \(\frac{3}{8}\) yard
d. \(\frac{4}{4}\) yard
e. \(\frac{1}{3}\) yard
f. \(\frac{5}{6}\) yard

Answer: Sarah could use 6/6 yard, 4/4 yard and 5/6 yard of ribbons to wrap gift.

Explanation:
Given that Sarah needs a 4/6 yard of ribbon to wrap a gift.
Lengths of available ribbons are 1/2, 6/6, 3/8, 4/4,1/3, and 5/6.
Comparing all the lengths with 4/6 – 6/6 and 5/6 are having the same denominators. So we compare only their numerators, both 6 and 5 are greater than 4. So these two ribbons are more than enough for her gift wrapping. With ribbon length, 4/4 yard – since complete ribbons all the parts of the ribbon are available thus this length also enough to wrap the gift. Remaining 1/2, 3/8, and 1/3 – we shall make their denominators equal to 4/6 to compare
multiply (1/2) with (3/3) = 3/6, 3 is less than 4 in 4/6. So this is not having enough length
multiply (1/3) with (2/2) = 2/6, 2 is also less than 4. So this also not having enough length.
multiply (3/8) with (6/6) = 18/48 and (4/6) with (8/8) = 32/48. Since 18 is less than 32, this ribbon length also won’t be enough to wrap gifts. Thus Sarah could use only 6/6 yard, 4/4 yard, and 5/6 yard of ribbons to wrap the gift.

Question 14.
There are 8 people having breakfast. Each person wants \(\frac{1}{2}\) of an omelet. How many whole omelets are needed? Use the models to show your answer.
Go Math Grade 3 Answer Key Chapter 9 Compare Fractions Assessment Test Test - Page 5 img 5
_____ omelets

Answer: Total they need 4 omelets.

Explanation:
Total no of having omelets are = 8
Each person need 1/2 of omelette, so for 8 persons = (1/2) * 8 = 4.

Question 15.
Michele mixed \(\frac{3}{4}\) cup of flour with \(\frac{1}{2}\) cup of water to make paste for an art project. Compare the fractions. Choose the symbol that makes the statement true.
Go Math Grade 3 Answer Key Chapter 9 Compare Fractions Assessment Test Test - Page 5 img 6
\(\frac{3}{4}\) _____ \(\frac{1}{2}\)

Answer: \(\frac{3}{4}\) > \(\frac{1}{2}\).

Explanation:

Go Math Grade 3 Answer Key Chapter 9 Compare Fractions Assessment Test
Let’s make 3/4 and 1/2 denominators the same (1/2) * (2/2) = 2/4.
Since 3 is greater than 2, 3/4 > 1/2.

Question 16.
Jeff has three boxes that weigh \(\frac{5}{8}\), \(\frac{1}{8}\), and \(\frac{3}{8}\) pound. Write the weights in order from least to greatest.

Answer: \(\frac{1}{8}\), \(\frac{3}{8}\), \(\frac{5}{8}\).

Explanation: Weights in the least to greatest form 1/8, 3/8, 5/8.

Test – Page 6 – Page No. 96

Question 17.
Ben measures the lengths of three insects. Draw a line to match each length on the left to the word on the right that describes its place in the order of lengths.
Go Math Grade 3 Answer Key Chapter 9 Compare Fractions Assessment Test Test - Page 6 img 7

Answer:

Go Math Grade 3 Answer Key Chapter 9 Compare Fractions Assessment Test

Question 18.
Kerri drew a model to show equivalent fractions.
Go Math Grade 3 Answer Key Chapter 9 Compare Fractions Assessment Test Test - Page 6 img 8
Use the model to complete the number sentence.
\(\frac{2}{3}\) = \(\frac{□}{□}\)

Answer: \(\frac{2}{3}\) = \(\frac{4}{6}\).

Explanation: Here from the model four parts are shaded out of 6, which also equal to \(\frac{2}{3}\) by value.

Question 19.
Elaine brought \(\frac{3}{4}\) pound of potato salad to a picnic. Jake brought \(\frac{2}{3}\) pound of macaroni salad. Who brought more salad? Explain the strategy you used to solve the problem.

Answer: Elaine has brought more Salad compared to Jake.

Explanation:
Elaine has brought \(\frac{3}{4}\) pound of potato salad
Jake brought \(\frac{2}{3}\) pound of macaroni salad
Lets compare both parts by making their denominators same.Therefore multiply (3/4) * (3/3) = (9/12) and (2/3) * (4/4) = (8/12)
Since 8 is less than 9,\(\frac{8}{12}\)  < \(\frac{9}{12}\).
Which means \(\frac{2}{3}\) < \(\frac{3}{4}\). Thus Elaine has brought more Salad compared to Jake.

Question 20.
It took Mike \(\frac{2}{6}\) hour to clean his room.
Go Math Grade 3 Answer Key Chapter 9 Compare Fractions Assessment Test Test - Page 6 img 9
What fraction is equivalent to \(\frac{2}{6}\) ?
\(\frac{2}{6}\) = \(\frac{□}{□}\)

Answer: \(\frac{2}{6}\) = \(\frac{1}{3}\).

Final Words

Hope the knowledge shared regarding the Go Math Grade 3 Answer Key Chapter 9 Compare Fractions Assessment Test. To know more such info feel free to reach us via comment box and we will get back to you. Check out Go Math Grade 3 Answer Key Chapter 9 Compare Fractions to know more related topics.

Go Math Grade 3 Answer Key Chapter 6 Understand Division Assessment Test

Test your level of understanding by taking the help of the Go Math Grade 3 Answer Key Chapter 6 Understand Division Assessment Test. Assess your strengths and weaknesses and allot time accordingly using the 3rd Grade Go Math  Assessment Test Answer Key. With consistent practice, you can score better grades in your exam.

Go Math Grade 3 Answer Key Chapter 6 Understand Division Assessment Test contains all the topics of chapter 6. Enhance your subject knowledge using the 3rd Grade Go Math Solutions Key Chapter 6 Assessment Test. Teachers can use these 3rd Grade Go Math Answer Key Ch 6 Understand Division Assessment Test to test students on how much they understand topics. Detailed Solutions are provided for all the concepts and you can refer them to clear your queries.

Chapter 6: Understand Division Assessment Test

Test – Page 1 – Page No. 61

Question 1.
For numbers 1a–1d, select True or False for each equation.
a. 5 ÷ 1 = 5
i. True
ii. False

Answer: True.

Explanation: 5 ÷ 1 = 5, so answer is true.

Question 1.
b. 3 ÷ 3 = 1
i. True
ii. False

Answer: True.

Explanation: 3 ÷ 3 = 1, so the answer is true.

Question 1.
c. 4 ÷ 1 = 1
i. True
ii. False

Answer: False.

Explanation: 4 ÷ 1 = 4, not 1. So the answer is False.

Question 1.
d. 0 ÷ 1 = 0
i. True
ii. False

Answer: True.

Explanation: 0 ÷ 1 = 0, so the answer is true.

Question 2.
Elian is making 36 ounces of punch. He pours the same amount into each of 6 cups. How many ounces of punch does he pour into each cup?
Circle the amount to complete the sentence.
Elian pours Go Math Grade 3 Answer Key Chapter 6 Understand Division Assessment Test Test - Page 1 img 1 ounces of punch into each cup.
_____

Answer: 6 each cup.

Explanation:

Elian pours Go Math Grade 3 Answer Key Chapter 6 Understand Division Assessment Test ounces of punch into each cup.

Question 3.
Gina arranges her 21 puzzles in 7 equal stacks. How many puzzles does Gina put in each stack?
Shade squares to make an array to model the problem.
Go Math Grade 3 Answer Key Chapter 6 Understand Division Assessment Test Test - Page 1 img 2

Answer: 3 puzzles.

Explanation:

Go Math Grade 3 Answer Key Chapter 6 Understand Division Assessment Test

Test – Page 2 – Page No. 62

Question 4.
Philip has 30 pennies that he exchanges for nickels. He exchanges 5 pennies for each nickel. How many nickels does Philip get?
Make equal groups to model the problem.
Go Math Grade 3 Answer Key Chapter 6 Understand Division Assessment Test Test - Page 2 img 3
_____ nickels

Answer: 6 nickels.

Explanation: As Philips has 30 pennies and he exchanges 5 pennies for each nickel, so Philips get 30÷5= 6 nickels.

Go Math Grade 3 Answer Key Chapter 6 Understand Division Assessment Test

Question 5.
A book has 54 pages divided into 6 equal chapters.
How many pages are in each chapter?
Go Math Grade 3 Answer Key Chapter 6 Understand Division Assessment Test Test - Page 2 img 4
_____ pages

Answer: 9 pages.

Explanation: 54÷6= 9 pages.

Question 6.
Select the equations that the array represents.
Mark all that apply.
Go Math Grade 3 Answer Key Chapter 6 Understand Division Assessment Test Test - Page 2 img 5
Options:
a. 2 × 4 = 8
b. 3 × 2 = 6
c. 6 ÷ 3 = 2
d. 2 × 3 = 6
e. 6 ÷ 2 = 3
f. 8 ÷ 2 = 4

Answer: b,c,d,e.

Explanation:
3 × 2 = 6
6 ÷ 3 = 2
2 × 3 = 6
6 ÷ 2 = 3

Test – Page 3 – Page No. 63

Question 7.
The 16 players at tennis camp are divided into 4 equal groups. How many players are in each group?
Part A
Draw jumps on the number line to model the problem.
Go Math Grade 3 Answer Key Chapter 6 Understand Division Assessment Test Test - Page 3 img 6
Type below:
___________

Answer:

Go Math Grade 3 Answer Key Chapter 6 Understand Division Assessment Test

Question 7.
Part B
Write a division equation to represent the model.
_____ ÷ _____ = _____

Answer: 4

Explanation: 16÷4= 4

Question 8.
Max writes a total of 45 lines in his journal. Each journal entry is 9 lines long. How many journal entries does Max make?
Choose a number from the box to complete the sentence.
Go Math Grade 3 Answer Key Chapter 6 Understand Division Assessment Test Test - Page 3 img 7
Max makes _____ journal entries.
_____

Answer: 5

Explanation: Max writes a total of 45 lines in his journal and each journal entry is 9 lines long, so 45÷9= 5 journal entries does Max makes.

Question 9.
A farmer sells baskets that hold 6 peaches each. Etta buys a total of 18 peaches. How many baskets of peaches does Etta buy?
Complete each equation to represent the problem.
6 × _____ = 18;    18 ÷ 6 = _____ ;     _____ baskets

Answer: 3 baskets.

Explanation: As the farmer sells baskets that hold 6 peaches each and Etta buys a total of 18 peaches, the number of baskets of peaches does Etty bought is 18÷6= 3 baskets.
6 × 3 = 18;
18 ÷ 6 = 3

Question 10.
Penn has 12 eggs to use in some recipes. Select a way that he could divide all of the eggs equally among the recipes. Mark all that apply.
Options:
a. 6 eggs in each of 2 recipes
b. 5 eggs in each of 3 recipes
c. 3 eggs in each of 4 recipes
d. 4 eggs in each of 4 recipes
e. 2 eggs in each of 6 recipes
f. 4 eggs in each of 3 recipes

Answer: a,c,e,f.

Explanation:
6×2= 12
3×4= 12
2×6= 12
4×3= 12

Test – Page 4 – Page No. 64

Question 11.
Nancy made this array to model a division equation. Which equation could Nancy have modeled? Mark all that apply.
Go Math Grade 3 Answer Key Chapter 6 Understand Division Assessment Test Test - Page 4 img 8
Options:
a. 63 ÷ 7 = 9
b. 54 ÷ 6 = 9
c. 72 ÷ 9 = 8
d. 63 ÷ 9 = 7

Answer: a,d.

Explanation:
63 ÷ 7 = 9
63 ÷ 9 = 7

Question 12.
Rico went for a bike ride around a 3-mile loop. He rode a total of 12 miles. How many times did Rico ride his bike around the loop?
Go Math Grade 3 Answer Key Chapter 6 Understand Division Assessment Test Test - Page 4 img 9
_____ times

Answer: 4 times.

Explanation: As Rico went for a bike ride around a 3-mile loop and rode a total of 12 miles, so 12÷3= 4 times Rico ride his bike around the loop.

Question 13.
Write the letter for each problem in the box next to its quotient.
Options:
A. 5 ÷ 5 _____ 0
B. 0 ÷ 5 _____ 5
C. 5 ÷ 1 _____ 1

Answer:
5 ÷ 5= 1
0 ÷ 5= 0
5 ÷ 1= 5.

Question 14.
Karson earns $6 each hour babysitting. Last week she earned a total of $48 babysitting. How many hours did Karson babysit last week?
Choose a number from the box.
Go Math Grade 3 Answer Key Chapter 6 Understand Division Assessment Test Test - Page 4 img 10
______ hours

Answer: 8 hours.

Explanation: As Karson earns $6 each hour babysitting and in last week she earned a total of $48 babysitting, so $48÷$6= 8 hours did Karson earn in babysit last week.

Test – Page 5 – Page No. 65

Question 15.
Write the division equation to represent the repeated subtraction.
Go Math Grade 3 Answer Key Chapter 6 Understand Division Assessment Test Test - Page 5 img 11
_____ ÷ _____ = _____

Answer: 3.

Explanation:
15÷5= 3

Question 16.
Write the related facts for the array.
Go Math Grade 3 Answer Key Chapter 6 Understand Division Assessment Test Test - Page 5 img 12

Answer:
5×5= 25
25÷5= 5

Explain why there are not more related facts for the array.

Answer: There are only two equations, as both factors are the same.

Question 17.
Sierra separates 14 animal cards into equal piles with none left over.
Draw a picture to show how Sierra divides the cards and complete the sentence.
Sierra made ______ piles of cards.

Answer: 2 piles of cards.

Explanation: Here we must know how 14 can be divided. So 14 can be divided into two ways, 7×2= 14 and 2×7= 14. So Sierra made 2 piles of cards.

Could Sierra have separated the cards into 4 equal piles with none left over? Explain why or why not.

Answer: No.

Explanation: Sierra cannot separate into 4 equal piles, as 14 cannot be divided into 4 groups of equal size.

Test – Page 6 – Page No. 66

Question 18.
Circle numbers to complete the related facts.
Go Math Grade 3 Answer Key Chapter 6 Understand Division Assessment Test Test - Page 6 img 13 × 6 = 42 42 ÷ Go Math Grade 3 Answer Key Chapter 6 Understand Division Assessment Test Test - Page 6 img 14 = 6
_____ _____

Answer: 7,6.

Explanation:
7×6= 42
42÷7= 6

Question 19.
Use numbers from the box to write related multiplication and division facts.
Go Math Grade 3 Answer Key Chapter 6 Understand Division Assessment Test Test - Page 6 img 15
_____ × _____ = _____
_____ × _____ = _____
_____ ÷ _____ = _____
_____ ÷ _____ = _____

Answer:
8×4= 32
4×8= 32
32÷4= 8
32÷8= 4

Question 20.
Angela plants 24 rosebushes in flowerbeds in her yard. She plants the same number of rosebushes in each of 6 flowerbeds.
How many rosebushes does Angela plant in each flowerbed? Show your work.
Go Math Grade 3 Answer Key Chapter 6 Understand Division Assessment Test Test - Page 6 img 16
______ rosebushes

Answer: 4 rosebushes.

Explanation: As Angela plants 24 rosebushes in flowerbeds in her yard and she plants the same number of rosebushes in each of 6 flowerbeds, so 24÷6= 4 rosebushes.

Go Math Grade 3 Answer Key Chapter 6 Understand Division Assessment Test Go Math Grade 3 Answer Key Chapter 6 Understand Division Assessment Test Go Math Grade 3 Answer Key Chapter 6 Understand Division Assessment Test Go Math Grade 3 Answer Key Chapter 6 Understand Division Assessment Test Go Math Grade 3 Answer Key Chapter 6 Understand Division Assessment Test Go Math Grade 3 Answer Key Chapter 6 Understand Division Assessment Test

Conclusion

Test your Math Skills using the Go Math Grade 3 Chapter 6 Understand Division Assessment Test. Hope, the information shared has shed some light on you. If you have any queries feel free to reach us via comment box and we will resolve them at the earliest. To know more such questions and clear your doubts on the Grade 3 Chapter 6 access Go Math Grade 3 Answer Key Chapter 6 Understand Division and make the most out of them.

Go Math Grade 4 Answer Key Chapter 5 Factors, Multiples, and Patterns

go-math-grade-4-chapter-5-factors-multiples-and-patterns-answer-key

Go Math Grade 4 Answer Key Chapter 5 includes topics like Factors, Common factors, Divisibilities and Review tests, etc. that aid students to solve the homework and assessment tests. Also, it is the best and ultimate guide for exam preparation. You will find every question was explained in a simplistic way so that you are able to understand the concepts easily. Go Math Grade 4 Answer Key Chapter 5 Factors, Multiples, and Patterns pdf links are available here for each and every lesson. So, kickstart your preparation and score good grades in the exams.

Go Math Grade 4 Answer Key Chapter 5 Factors, Multiples, and Patterns

Improve your Problem-Solving Skills utilizing the Go Math Grade 4 Answer Key Chapter 5 Factors, Multiples, and Patterns. Start practicing the question covered in the Go Math 4th grade Solution Key and Cross Check the Solutions of Chapter 5 Factors, Multiples, and Patterns from here. So that you can easily rectify your mistakes and fill up the knowledge gap. Take the help from the direct links available below and solve the problems covered in Go Math Grade 4 Answer Key.

Lesson 1: Model Factors

Lesson 2: Factors and Divisibility

Lesson 3: Problem Solving • Common Factors

Lesson 4: Factors and Multiples

Lesson 5: Prime and Composite Numbers

Lesson 6: Algebra • Number Patterns

Chapter 5 Review/Test


Common Core – Model Factors – Page No. 283

Model Factors

Use tiles to find all the factors of the product.

Record the arrays on grid paper and write the factors shown.

Question 1.
Go Math Grade 4 Answer Key Chapter 5 Factors, Multiples, and Patterns Common Core Model Factors img 1

Question 2.
Write the factors of: 30

Answer:
The Factors Of 30 are: 1,2,3,5,6,10,15,30.

Explanation:
Factors are the numbers which divides the original number completely. Here, we can see the numbers which gives the result as 30 when multiplied together.So the factors of 30 are 1,2,3.5,6,10,15,30.

1×30=30
2×15=30
3×10=30
5×6=30
6×5=30
10×3=30
15×2=30
30×1=30

Question 3.
Write the factors of: 45

Answer: The Factors Of 45 are:1,3,5,9,15,45.

Explanation:
Factors are the numbers which divides the original number completely. Here, we can see the numbers which gives the result as 45 when multiplied together.So the factors of 45 are:1,3,5,9,15,45.

1×45=45
3×15=45
5×9=45
9×5=45
15×3=45
45×1=45

Question 4.
Write the factors of: 19

Answer: The Factors Of 19 are:1,19.

Explanation:
Since 19 is a Prime number that means it is divisible by 1 and itself. So the factors of  19 are 1,19.

1×19=19
19×1=19.

Question 5.
Write the factors of: 40

Answer: The Factors Of 40 are:1,2,4,5,8,10,20,40.

Explanation:Factors are the numbers which divides the original number completely. The Factors Of 40 are:1,2,4,5,8,10,20,40.

1×40=40
2×20=40
4×10=40
5×8=40
8×5=40
10×4=40
20×2=40
40×1=40

Question 6.
Write the factors of: 36

Answer: The Factors Of 36 are:1,2,3,4,6,9,12,18,36.

Explanation:
Factors are the numbers which divides the original number completely. The factors of 36 are:1,2,3,4,6,9,12,18,36.

1×36=36
2×18=36
3×12=36
4×9=36
6×6=36
9×4=36
12×3=36
18×3=36
36×1=36.

Question 7.
Write the factors of: 22

Answer: The Factors Of 22 are:1,2,11,22.

Explanation:
Factors are the numbers which divides the original number completely. The factors of 22 are:1,2,11,22.

1×22=22
2×11=22
11×2=22
22×1=22.

Question 8.
Write the factors of: 4

Answer: The Factors Of 4 are:1,2,4.

Explanation:
Factors are the numbers which divides the original number completely. The Factors Of 4 are:1,2,4.

1×4=4
2×2=4
4×1=4.

Question 9.
Write the factors of: 26

Answer: The Factors Of 26 are:1,2,13,26.

Explanation:
Factors are the numbers which divides the original number completely. Here, we can see the numbers which gives the result as 26 when multiplied together.So the factors of 26 are:1,2,13,26.

1×26=26
2×13=26
13×2=26
26×1=26.

Question 10.
Write the factors of: 49

Answer: The Factors Of 49 are:1,7,49.

Explanation:
Factors are the numbers which divides the original number completely. The Factors Of 49 are:1,7,49.

1×49=49
7×7=49
49×1=49.

Question 11.
Write the factors of: 32

Answer: The Factors Of 32 are:1,2,4,8,16,32.

Explanation:
Factors are the numbers that divide the original number completely. Here, we can see the numbers which give the result as 32 when multiplied together.So the factors of 32 are:1,2,4,8,16,32.

1×32=32
2×16=32
4×8=32
8×4=32
16×2=32
32×1=32.

Question 12.
Write the factors of 23

Answer: The Factors Of 23 are:1,23.

Explanation:
Since 23 is a Prime number that means it is divisible by 1 and itself. So the factors of  23 are 1,23.

1×23=23
23×1=23.

Question 13.
Brooke has to set up 70 chairs in equal rows for the class talent show. But, there is not room for more than 20 rows. What are the possible number of rows that Brooke could set up?

Answer:
Answer is 2,5,7,10,14.

Explanation:
Let the possible no.of rows be X, As there is no room for more than 20 rows so there should not be more than 20 rows.X should be less than or equal to 20(X<=20). As Brooke has 70 chairs to set up in equal rows we will find the factors of 70 and in that we must pick up the numbers which are less than equal to 20.Therefore the factors of 70 are 2,5,7,10,14.

 

Question 14.
Eduardo thinks of a number between 1 and 20 that has exactly 5 factors. What number is he thinking of?

Answer: 16

Explanation: If find factors for 1 to 20 we don’t get exactly 5 factors for any number except 16. So the answer is 16.


Common Core – Factors – Page No. 284

Lesson Check

Question 1.
Which of the following lists all the factors of 24?
Options:
a. 1, 4, 6, 24
b. 1, 3, 8, 24
c. 3, 4, 6, 8
d. 1, 2, 3, 4, 6, 8, 12, 24

Answer: d(1, 2, 3, 4, 6, 8, 12, 24)

Explanation:Factors are the numbers which divides the original number completely. Here, we can see the numbers which gives the result as 24 when multiplied together.So the factors of 24 are:1, 2, 3, 4, 6, 8, 12, 24.

1×24=24
2×12=24
3×8=24
4×6=24
6×4=4
8×3=24
12×2=24
24×1=24

Question 2.
Natalia has 48 tiles. Which of the following shows a factor pair for the number 48?
Options:
a. 4 and 8
b. 6 and 8
c. 2 and 12
d. 3 and 24

Answer: b(6 and 8)

Explanation: 6 and 8 are factor pair for 48 because 6×8=48.

 

Spiral Review

Question 3.
The Pumpkin Patch is open every day. If it sells 2,750 pounds of pumpkins each day, about how many pounds does it sell in 7 days?
Options:
a. 210 pounds
b. 2,100 pounds
c. 14,000 pounds
d. 21,000 pounds

Answer: d

Explanation: Let’s round off 2750 pounds to 3000 pounds. In one day 3000 pounds pumpkins were sold out, and in
7 days?? —- 3000×7= 21,000 pounds.

Question 4.
What is the remainder in the division problem modeled below?
Go Math Grade 4 Answer Key Chapter 5 Factors, Multiples, and Patterns Common Core Factors img 2
Options:
a. 2
b. 3
c. 5
d. 17

Answer: a

Explanation: We can see in the above figure 3 circles with 5 sub circles inside it and a pair of  sub circles. Here total sub circles are (3×5)+2=17. If we divide 17 with 3 then we will get reminder as 2. So answer is 2.

Question 5.
Which number sentence is represented by the following array?
Go Math Grade 4 Answer Key Chapter 5 Factors, Multiples, and Patterns Common Core Factors img 3
Options:
a. 4 × 5 = 20
b. 4 × 4 = 16
c. 5 × 2 = 10
d. 5 × 5 = 25

Answer: a

Explanation: As we can see 4 rows and 5 squares, So 4 × 5 = 20.

Question 6.
Channing jogs 10 miles a week. How many miles will she jog in 52 weeks?
Options:
a. 30 miles
b. 120 miles
c. 200 miles
d. 520 miles

Answer: d

Explanation: No.of weeks = 52. So 1 week = 10 miles, then 52 weeks =?????
52×10=520 miles.


Page No. 287

Question 1.
Is 4 a factor of 28? Draw a model to help.
Think: Can you make a rectangle with 28 squares in 4 equal rows?
Go Math Grade 4 Answer Key Chapter 5 Factors, Multiples, and Patterns img 4
4 ______ a factor of 28.
Type below:
__________

Is 5 a factor of the number? Write yes or no.

Question 2.
27
Answer: No.

Explanation: Factors of 27 are 1,3,9,27. So the answer is No.

Question 3.
30
Answer : Yes.

Explanation: As the last digit is 0 which is divisible 5.

Question 4.
36
Answer: No

Explanation: 36 is not divisible by 5, So the answer is no

Question 5.
53
Answer: No

Explanation: Factors of 53 are 1, 53. So the answer is No.

Is 9 a factor of the number? Write yes or no.

Question 6.
54
Answer: Yes.

Explanation: As 54 is divisible by 9.

Question 7.
63
Answer: Yes.

Explanation: 63 is divisible by 9, So the answer is Yes

Question 8.
67
Answer: No.

Explanation: 67 is a prime number that means it is divisible by 1 and itself. So the answer is No.

Question 9.
93
Answer: No.

Explanation: The factors of 93 are 1,3,31 and 93. So the answer is No.

List all the factor pairs in the table.

Question 10.
Go Math Grade 4 Answer Key Chapter 5 Factors, Multiples, and Patterns img 5
Answer:
1×24=24    1,24
2×12=24     2,12
3×8=24       3,8
4×6=24       4,6

Explanation: Factors of 24.

Question 11.
Go Math Grade 4 Answer Key Chapter 5 Factors, Multiples, and Patterns img 6
Answer:
1×39=39    1,39
3×13=39.   3,13

Explanation: Factors of 39.

Practice: Copy and Solve List all the factor pairs for the number. Make a table to help.

Question 12.
56
Answer:
1×56=56     1,56
2×23=56     2,23
4×14=56      4,14
7×8=56         7,8
8×7=56         8,7

Explanation: Factors of 56.

Question 13.
64
Answer:
1×64=64    1,64
2×32=64    2,32
4×16=64    4,16
8×8=64      8,8

Explanation: Factors of 64 and factor pair for 64 is 8,8.

Page No. 288

Use the table to solve 14–15.
Go Math Grade 4 Answer Key Chapter 5 Factors, Multiples, and Patterns img 7

Question 14.
Dirk bought a set of stamps. The number of stamps in the set he bought is divisible by 2, 3, 5, 6, and 9. Which set is it?
Answer: 90

Explanation: 90 is divisible by all numbers 2,3,5,6, and 9. So the answer is 90.

Question 15.
Geri wants to put 6 stamps on some pages in her stamp book and 9 stamps on other pages. Explain how she could do this with the stamp set for Sweden.

Answer: 10 pages with 6 stamps and 2 pages with 9 stamps.

Explanation: Geri could break 78 into 60+18, As 60 is divisible by 6, and 18 is divisible by 9. Then she could make 10 pages with 6 stamps as 60÷6=10 and 2 pages with 9 stamps as 18÷9=2.

Question 16.
Use Counterexamples George said if 2 and 4 are factors of a number, then 8 is a factor of the number. Is he correct? Explain.

Answer: No

Explanation: Because if we 12 as an example, 2 and 4 are factors of 12 but not 8.

Question 17.
Classify the numbers. Some numbers may belong in more than one box.
Go Math Grade 4 Answer Key Chapter 5 Factors, Multiples, and Patterns img 8
Answer:
Divisible by 5 and 9 — 45
Divisible by 3 and 9 — 27,45,54,72,81
Divisible by 2 and 6 — 54,72,84.

Common Core – Factors and Divisibility – Page No. 289

Is 6 a factor of the number? Write yes or no.

Question 1.
Go Math Grade 4 Answer Key Chapter 5 Factors, Multiples, and Patterns Common Core Factors and Divisibility img 9

Question 2.
56
Answer: No

Explanation: 56 is not divisible by 6. So the answer is No.

Question 3.
42

Answer: Yes

Explanation: Since 42 is divisible by 6.

Question 4.
66
Answer: Yes

Explanation: 66 is divisible by 6.

Is 5 a factor of the number? Write yes or no.

Question 5.
38
Answer: No

Explanation: If the end is 0 or 5 then the number is divisible by 5. As the number is 38 the answer is No

Question 6.
45

Answer: Yes

Explanation: 45 is divisible by 5. So the answer is Yes

Question 7.
60
Answer: Yes

Explanation: 60 is a factor of 5 because 60 is divisible by 5.

Question 8.
39
Answer: No

Explanation: As 39 is not divisible by 5. So the answer is No.

List all the factor pairs

Question 9.
Factors of 12

Answer:
1 × 12 = 12; ( 1 , 12 )
2 × 6 = 12; ( 2, 6 )
3 × 4 = 12; ( 3 , 4 )

Question 10.
Factors of 25

Answer:
1 ×25  = 25; ( 1 , 25 )
5 × 5 = 25; ( 5 , 5 )

Question 11.
List all the factor pairs for 48.

Answer: Factor pairs of 48 are (1,48),(2,24),(3,16),(4,12),(6,8),(12,2),(6,3),(24,2),(48,1).

Explanation: Factor pairs are the pairs when we multiplied both numbers will get the result. Here factor pairs for 48 are
1×48=48     (1,48)
2×24=48      (2,24)
3×16=48      (3,16)
4×12=48      (4,12)
6×8 =48       (6,8)

Problem Solving

Question 12.

Bryson buys a bag of 64 plastic miniature dinosaurs. Could he distribute them equally into six storage containers and not have any left over?

Answer: No

Explanation: 64 is not divisible by 6, So he cannot distribute them equally into six storage containers.

Question 13.
Lori wants to distribute 35 peaches equally into baskets. She will use more than 1 but fewer than 10 baskets. How many baskets does Lori need?

Answer: 5 or 7.

Explanation: First we need to know the factors of 35. The factors of 35 are 1,5,7,35. As Lori uses more than 1 but fewer than 10, the answer is 5 or 7. Lori can distribute 35 peaches equally in 5 or 7 baskets.

Common Core – Factors – Page No. 290

Lesson Check

Question 1.
Which of the following numbers has 9 as a factor?
Options:
a. 28
b. 30
c. 39
d. 45

Answer: d

Explanation: 45 is divisible 9. So the answer is 45.

Question 2.
Which of the following numbers does NOT have 5 as a factor?
Options:
a. 15
b. 28
c. 30
d. 45

Answer: 28

Explanation: 28 is not divisible by 5. So 28 is not a factor of 5.

Spiral Review

Question 3.
Which of the following shows a strategy to use to find 4 × 275?
Options:
a. (4 × 300) + (4 × 25)
b. (4 × 300) – (4 × 25)
c. (4 × 275) – 100
d. (4 × 200) + 75

Answer: b

Explanation: First we must replace 300-25 in the place of 275 then it becomes 4×(300-25), Now we must use the distributive property of multiplication then (4×300)-(4×25). So the answer is b.

Question 4.
Jack broke apart 5 × 216 as (5 × 200) + (5 × 16) to multiply mentally. What strategy did Jack use?
Options:
a. the Commutative Property
b. the Associative Property
c. halving and doubling
d. the Distributive Property

Answer: d

Explanation: Distributive property means if we multiply a sum by a number is same as multiplying each addend by the number and adding the products. This is the strategy Jack used.

Question 5.
Jordan has $55. She earns $67 by doing chores. How much money does Jordan have now?
Options:
a. $122
b. $130
c. $112
d. $12

Answer: a

Explanation: Jordan has $55, she earns by doing chores is $67. So total money is $55+$67=$122.

Question 6.
Trina has 72 collector’s stamps. She puts 43 of the stamps into a stamp book. How many stamps are left?
Options:
a. 29
b. 31
c. 39
d. 115

Answer: a

Explanation: Stamps left are 72-43=29.

Page No. 293

Question 1.
Lucy has 40 bean plants, 32 tomato plants, and 16 pepper plants. She wants to put the plants in rows with only one type of plant in each row. All rows will have the same number of plants. How many plants can Lucy put in each row?
First, read the problem and think about what you need to find. What information will you use? How will you use the information?

Answer: We will find common factors for 40,32 and 16.

Question 1.
Next, make a list. Find the factors for each number in the problem.

Answer:
Factors of 40 are — 1,2,4,5,8,10,20,40
Factors of 32 are — 1,2,4,8,16,32
Factors of 16 are — 1,2,4,8,16

Question 1.
Finally, use the list. Circle the common factors.
So, Lucy can put ___ , ___ , ___ , or ___ plants in each row.

Answer: 1,2,4,8

Explanation: Because 1,2,4,8, are common factors in 40,32,16.

Question 2.
What if Lucy has 64 bean plants instead of 40 bean plants? How many plants can Lucy put in each row?

Answer: 1,2,4,8,16

Explanation: Here we need to find the factors of 64,32 and 16. We get common factors as 1,2,4,8,16.

Question 3.
One common factor of two numbers is 40. Another common factor is 10. If both numbers are less than 100, what are the two numbers?
______ and ______

Answer:  40 and 80.

Explanation: As the next multiple of 40 is 80. So both 40 and 80 are less than 100 and has a common factor as 10.

Question 4.
The sum of two numbers is 136. One number is 51. What is the other number? What are the common factors of these two numbers?

Answer: 85.
Common Factors are 1,17.

Explanation: As 136-51= 85
Factors of 51 are 1,3,17,51
Factors of 85 are 1,5,17,85.

Page No. 294

Question 5.
Analyze A number is called a perfect number if it equals the sum of all of its factors except itself. For instance, 6 is a perfect number because its factors are 1, 2, 3, and 6, and 1 + 2 + 3 = 6. What is the next greater perfect number?

Answer: 28

Explanation: The factors of 28 are 1,2,4,7,14 and 28. If we add 1+2+4+7+14 we will get 28. So 28 is a perfect number.

Question 6.
Sona knits 10 squares a day for 7 days. Can she sew together the squares to make 5 equal-sized blankets? Explain.

Answer: Yes

Explanation: As 10×7= 70 which is a factor of 5.

Question 7.
Julianne earned $296 working at a grocery store last week. She earns $8 per hour. How many hours did Julianne work?
Answer: 37 hours

Explanation: Julianne earned $296 in last week. Per hour she earns $8, So total no.of hours did she worked is
296÷8= 37 hours.

Question 8.
There are 266 students watching a play in the auditorium. There are 10 rows with 20 students in each row and 5 rows with 8 students in each row. How many students are sitting in each of the 2 remaining rows if each of those rows has an equal number of students?

Answer: 13 Students

Explanation: Total number of students is 266. In which 10 rows were filled with 20 students that means 10×20=200 students, and 5 rows were filled with 8 students which means 5×8= 40 students. The total students filled are 240. And to know how many students filled in the remaining 2 rows we need to subtract 266-240=26, As students are filled in 2 rows 26÷2= 13.

Question 9.
Ben is planting a garden with 36 zinnias, 18 marigolds, and 24 petunias. Each row will have only one type of plant. Ben says he can put 9 plants in each row. He listed the common factors of 36, 18 and 24 below to support his reasoning.
36: 1, 2, 3, 4, 6, 9, 12, 18, 36
18: 1, 2, 3, 6, 8, 9, 18
24: 1, 2, 3, 4, 6, 8, 9, 12, 24
Is he correct? Explain your answer. If his reasoning is incorrect, explain how he should have found the answer.

Answer: No

Explanation: The factors of 18 and 24 are incorrect which he listed. And the common factors for 36,24 and 18 are 1,2,3 and 6. So he can put 1,2,3 and 6 plants in a row.

Common Core – Common Factors – Page No. 295

Problem Solving Common Factors

Solve each problem.

Question 1.
Grace is preparing grab bags for her store’s open house. She has 24 candles, 16 pens, and 40 figurines. Each grab bag will have the same number of items, and all the items in a bag will be the same. How many items can Grace put in each bag?
Go Math Grade 4 Answer Key Chapter 5 Factors, Multiples, and Patterns Common Core Common Factors img 10

Question 2.
Simon is making wreaths to sell. He has 60 bows, 36 silk roses, and 48 silk carnations. He wants to put the same number of items on each wreath. All the items on a wreath will be the same type. How many items can Simon put on each wreath?

Answer:1,2,3,4,6 or 12 items Simon puts on each wreath.

Explanation: First we will find the common factors of 36,48,60
factors of 36 are: 1,2,3,4,6,9,12,18,36.
factors of 48 are: 1,2,3,4,6,8,12,16,24,48
factors of 60 are: 1,2,3,4,5,6,10,12,15,20,30,60.
The common factors of 36,48,60 are 1,2,3,4,6,12. So Simon can put 1,2,3,4,6 or 12 items on each wreath.

Question 3.
Justin has 20 pencils, 25 erasers, and 40 paper clips. He organizes them into groups with the same number of items in each group. All the items in a group will be the same type. How many items can he put in each group?

Answer: Justin can put 1 or 5 items in each group.

Explanation: We will find common factors of 20,25,40.
factors of 20 are: 1,2,4,5,10,20.
factors of 25 are: 1,5,25.
factors of 40 are: 1,2,4,5,8,10,20,40
So common factors are 1 and 5.

Question 4.
A food bank has 50 cans of vegetables, 30 loaves of bread, and 100 bottles of water. The volunteers will put the items into boxes. Each box will have the same number of food items and all the items in the box will be the same type. How many items can they put in each box?

Answer: 1,2,5, or 10.

Explanation: 1,2,5 or 10 are the common factors of 30,50 and 100.
factors for 30 are: 1,2,3,5,6,10,15,30
factors for 50 are: 1,2,5,10,25,50
factors of 100 are: 1,2,4,5,10,20,25,50,100
So answer is 1,2,5,10.

Question 5.
A debate competition has participants from three different schools: 15 from James Elementary, 18 from George Washington School, and 12 from the MLK Jr. Academy. All teams must have the same number of students. Each team can have only students from the same school. How many students can be on each team?

Answer: 3

Explanation: Lets find the common factors of 12,15,18
factors of 12 are: 1,2,3,4,6,12
factors of 15 are: 1,3,5,15
factors of 18 are: 1,2,3,6,9,18
3 is the common factor for 12,15,18

Common Core – Common Factors – Page No. 296

Lesson Check

Question 1.
What are all the common factors of 24, 64, and 88?
Options:
a. 1 and 4
b. 1, 4, and 8
c. 1, 4, 8, and 12
d. 1, 4, 8, and 44

Answer: b

Explanation:
factors of 24 are: 1,2,3,4,8,12,24
factors of 64 are: 1,2,4,8,16,32,64
factors of 88 are: 1,2,4,8,11,22,44,88

Question 2.
Which number is NOT a common factor of 15, 45, and 90?
Options:
a. 3
b. 5
c. 10
d. 15

Answer: c

Explanation: As 15 and 45 are not divisible by 10.

Spiral Review

Question 3.
Dan puts $11 of his allowance in his savings account every week. How much money will he have after 15 weeks?
Options:
a. $165
b. $132
c. $110
d. $26

Answer: a

Explanation: Dan puts $11 in his savings account every week, So after 15 weeks it will be 15×11=165.
The total money he will have after 15 weeks is $165.

Question 4.
James is reading a book that is 1,400 pages. He will read the same number of pages each day. If he reads the book in 7 days, how many pages will he read each day?
Options:
a. 20
b. 50
c. 140
d. 200

Answer: d

Explanation: Total no.of.pages is 1400, no.of pages James read each day is 1400÷7= 200

Question 5.
Emma volunteered at an animal shelter for a total of 119 hours over 6 weeks. Which is the best estimate of the number of hours she volunteered each week?
Options:
a. 10 hours
b. 20 hours
c. 120 hours
d. 714 hours

Answer: b

Explanation: Total hours Emma volunteered is 119 hours over 6 weeks, how much she volunteered each week is
119÷6= 19.833 i.e 20 hours. We must round off to the nearest one i.e 20 hours.

Question 6.
Which strategy can be used to multiply 6 × 198 mentally?
Options:
a. 6 × 198 = (6 × 19) + (6 × 8)
b. 6 × 198 = (6 × 200) + (6 × 2)
c. 6 × 198 = (6 × 200) – (6 × 2)
d. 6 × 198 = (6 + 200) × (6 + 2)

Answer: c

Explanation: By Distributive property of multiplication 6×198 can be written as (6 × 200) – (6 × 2).

Page No. 297

Choose the best term from the box.
Go Math Grade 4 Answer Key Chapter 5 Factors, Multiples, and Patterns img 11

Question 1.
A number that is multiplied by another number to find a product is called a

Answer: Factor.

Question 2.
A number is _________ by another number if the quotient is a counting number and the remainder is zero.
Answer: Divisible.

List all the factors from least to greatest.

Question 3.
8
Answer: 1,2,4,8

Question 4.
14
Answer: 1,2,7,14

Is 6 a factor of the number? Write yes or no.

Question 5.
81
Answer: No

Explanation: 81 is not divisible by 6

Question 6.
45
Answer: No

Explanation: 45 is not divisible by 6

Question 7.
42
Answer: Yes

Explanation: 42 is divisible by 6

Question 8.
56

Answer: No.

Explanation: 56 is not divisible by 6

List all the factor pairs in the table.

Question 9.
Go Math Grade 4 Answer Key Chapter 5 Factors, Multiples, and Patterns img 12

Answer:
1×64=64    1,64
2×32=64    2,32
4×16=64    4,16
8×8=64      8,8

Explanation: Factors of 64

Question 10.
Go Math Grade 4 Answer Key Chapter 5 Factors, Multiples, and Patterns img 13

Answer:
1×44=44    1,44
2×22=44    2,22
11×4=44    11,4

List the common factors of the numbers.

Question 11.
9 and 18

Answer: 1,3,9

Explanation:
Factors of 9 are: 1,3,9
Factors of 18 are: 1,2,3,9,18

Question 12.
20 and 50

Answer: 1,2,5,10

Explanation:
Factors of 20 are: 1,2,4,5,10,20
Factors of 50 are: 1,2,5,10,25,50

Page No. 298

Question 13.
Sean places 28 tomato plants in rows. All rows contain the same number of plants. There are between 5 and 12 plants in each row. How many plants are in each row?

Answer: 7 plants.

Explanation: There are 28 tomato plants in a row. To find out how many plants in a row we will find the factors of 28 i.e 1,2,4,7,14,28. As there are between 5 and 12 plants 7 is the only number between 5 and 12. So 7 plants are planted in each row.

Question 14.
Ella bought some key chains and spent a total of $24. Each key chain costs the same whole-dollar amount. She bought between 7 and 11 key chains. How many key chains did Ella buy?

Answer: 8

Explanation: Ella spent a total of $24. To find how many key chains first we will find the factors of 24. Factors of 24 are
1,2,3,4,6,8,12,24. As Ella bought between 7 and 11 key chains 8 is the only number between 7 and 11. So 8 key chains Ella bought.

Question 15.
Sandy has 16 roses, 8 daisies, and 32 tulips. She wants to arrange all the flowers in the bouquets. Each bouquet has the same number of flowers and the same type of flower. What is the greatest number of flowers that could be in a bouquet?

Answer: 2 roses, 1 daisy, and 4 tulips in 8 bouquets.

Explanation: First we must add all the flowers i.e 16+8+32= 56, Now we can divide 56 flowers equally in each bouquet. Like 2 roses, 1 daisy and 4 tulips in 8 bouquets or 8 roses in 2 bouquets, 8 daisies in 1 bouquet and 8 tulips in 4 bouquets.

Question 16.
Amir arranged 9 photos on a bulletin board. He put the photos in rows. Each row contains the same number of photos. How many photos could be in each row?

Answer: 9 photos in a row and 3 photos in 3 rows or 9 photos in 1 row.

Explanation: Factors of 9 are 1,3,9. So Amir can arrange 9 photos in a row and 3 photos in 3 rows or 9 photos in 1 row.

Page No. 301

Question 1.
Multiply to list the next five multiples of 4.
4 , _____ , _____ , _____ , _____ , _____
1 × 4
4 , _____ , _____ , _____ , _____ , _____

Answer:
4     1×4
8     2×4
12   3×4
16   4×4
20   4×5

Explanation: Multiplies of 4

Is the number a factor of 6? Write yes or no.

Question 2.
2

Answer: Yes

Explanation: 6 is divisible by 2. So 2 is the factor of 6.

Question 3.
6

Answer: Yes

Explanation: 6 is divisible by 6.

Question 4.
16

Answer: No

Explanation: 16 is not divisible by 6

Question 5.
18

Answer: Yes

Explanation: 18 is divisible by 6

Is the number a multiple of 6? Write yes or no.

Question 6.
3

Answer: No

Explanation: Multiples of 6 are 6,12,18,24,30, etc.

Question 7.
6

Answer: Yes

Explanation: 1×6= 6. So 6 is multiple of 6.

Question 8.
16

Answer: No

Explanation: Multiples of 6 are 6,12,18,24,30, etc.

Question 9.
18

Answer: Yes

Explanation: Multiples of 6 are 6,12,18,24,30, etc.

Is the number a multiple of 3? Write yes or no.

Question 10.
4

Answer: No

Explanation: Multiples of 3 are 3,6,9,12,15,etc.

Question 11.
8

Answer: No

Explanation: Multiples of 3 are 3,6,9,12,15,etc.

Question 12.
24

Answer: Yes

Explanation: Multiples of 3 are 3,6,9,12,15,etc.

Question 13.
38

Answer: No

Explanation: Multiples of 3 are 3,6,9,12,15,18,21,24,27,30,33,36,39,42,etc.

Question 14.
List the next nine multiples of each number. Find the common multiples.
Multiples of 2: 2, _________________
Multiples of 8: 8, _________________
Common multiples: _________________

Answer: 8,16.

Explanation:
Multiples of 2: 2,4,6,8,10,12,14,16,18,20.
Multiples of 8: 8,16,24,32,40,48,56,64,72,80.
So common multiples are: 8,16

Generalize Algebra Find the unknown number.

Question 15.
12, 24, 36, _____

Answer: 48

Explanation:
12×1= 12
12×2= 24
12×3= 36
12×4= 28

Question 16.
25, 50, 75, 100, ______

Answer: 125

Explanation:
25×1= 25
25×2= 50
25×3= 75
25×4= 100
25×5= 125

Tell whether 20 is a factor or multiple of the number.

Write factor, multiple, or neither.

Question 17.
10

Answer: Multiple

Explanation: 2×10= 20.

Question 18.
20

Answer: Factor and multiple

Explanation:
1×20= 20
20÷1= 20.

Question 19.
30

Answer: Neither

Explanation:
Factors of 30 are:    1,2,3,5,6,10,15,and 30.
Multiples of 30 are: 30,60,90,etc.

Write true or false. Explain.

Question 20.
Every whole number is a multiple of 1.

Answer: True.

Explanation: For every whole number which is multiplied with 1, the result will be that number.

Question 21.
Every whole number is a factor of 1.

Answer: False

Explanation: Not every whole number is a factor of 1.

Question 22.
Julio wears a blue shirt every 3 days. Larry wears a blue shirt every 4 days. On April 12, both Julio and Larry wore a blue shirt. What is the next date that they will both wear a blue shirt?
Go Math Grade 4 Answer Key Chapter 5 Factors, Multiples, and Patterns img 14

Answer: April 24

Explanation:
As Julio wears a blue shirt every 3 days and another shirt in the remaining 4 days, So 4×3 days= 12
Larry wears a blue shirt every 4 days and another shirt in the remaining 3 days, So 3×4 days= 12
12+12= 24. So the next date will be April 24.

Page No. 302

Complete the Venn diagram. Then use it to solve 23–25.
Go Math Grade 4 Answer Key Chapter 5 Factors, Multiples, and Patterns img 15

Question 23.
What multiples of 4 are not factors of 48?

Answer: 20,28,32,36,40,44.

Explanation:
Multiples of 4 are 4,8,12,16,20,24,28,32,36,40,44,48.
Not a factors of 48 are 20,28,32,36,40,44.

Question 24.
What factors of 48 are multiples of 4?

Answer: 4,8,12,16,24,48.

Explanation:
Multiples of 4 are: 4,8,12,16,20,24,28,32,36,40,44,48.
Factors of 48 are: 1,2,4,8,12,16,24,48.

Question 25.
Pose a Problem Look back at Problem 24. Write a similar problem by changing the numbers. Then solve.

Answer: Let’s take factors of 64 are multiples of 8?
8,16,32,64.

Explanation:
Multiples of 8 are: 8,16,24,32,40,48,56,64,72,80
Factors of 64 are: 1,2,4,8,16,32,64

Question 26.
Kia paid $10 for two charms. The price of each charm was a multiple of $2. What are the possible prices of the charms?

Answer: $2,$8 and $4,$6.

Explanation: Since the price was multiple of 2 and Kia paid $10 for two charms, So possible prices are $2+$8=$10
and $4+$6=$10.

Question 27.
Look for Structure The answer is 9, 18, 27, 36, 45. What is the question?

Answer: Write the multiples of 9

Question 28.
How do you know whether a number is a multiple of another number?

Answer: When the number is divisible by the number then that number is multiple of another number.

Explanation: For example, if we take a number i.e 8 which is divisible by 2 and 8 is a multiple of 2.

Question 29.
For numbers 29a–29e, select True or False for each statement.
a. The number 45 is a multiple of 9.
i. True
ii. False

Answer: True

Explanation: As 9×5= 45, So 45 is multiple of 9.

Question 29.
b. The number 4 is a multiple of 16.
i. True
ii. False

Answer: False.

Explanation: As 16 is divisible by 4 and not a multiple of 16.
Multiple of 16 are : 16,32,48,64,80.

Question 29.
c. The number 28 is a multiple of 4.
i. True
ii. False

Answer: True.

Explanation: 4×7=28.

Question 29.
d. The number 4 is a factor of 28.
i. True
ii. False

Answer: True.

Explanation:
Factors of 28 are: 1,2,4,7,14,28.

Question 29.
e. The number 32 is a factor of 8.
i. True
ii. False

Answer:

Explanation:

Common Core – Factors and Multiples – Page No. 303

Factors and Multiples
Is the number a multiple of 8? Write yes or no.

Question 1.
Go Math Grade 4 Answer Key Chapter 5 Factors, Multiples, and Patterns Common Core Factors and Multiples img 16

Question 2.
8
Answer: Yes

Explanation: Since 8×1=8, it is a multiple of 8

Question 3.
20
Answer: No

Explanation: 20 is not a multiple of 8

Question 4.
40
Answer: Yes

Explanation: 8×5=40, So 40 is multiple of 8

List the next nine multiples of each number. Find the common multiples.

Question 5.
Multiples of 4:
Multiples of 7:
Common multiples:

Answer:

Explanation:
Multiples of 4: 4,8,12,16,20,24,28,32,36,40.
Multiples of 7: 7,14,21,28,35,42,49,56,63,70.
Common Multiples: 28,

Question 6.
Multiples of 3:
Multiples of 9:
Common multiples:

Answer: 9,18,45,54,63, etc.

Explanation:
Multiples of 3: 3,6,9,12,15,18,21,24,27,30,33,36,39,42,45,48,51,54,57,60,63.
Multiples of 9: 9,18,27,36,45,54,63,72,81,90.
Common multiples: 9,18,45,54,63, etc.

Question 7.
Multiples of 6:
Multiples of 8:
Common multiples:

Answer: 24,48,72.

Explanation:
Multiples of 6: 6,12,18,24,30,36,42,48,54,60,66,72,78.
Multiples of 8: 8,16,24,32,40,48,56,64,72,80.
Common multiples: 24,48,72.

Tell whether 24 is a factor or multiple of the number. Write factor, multiple, or neither.

Question 8.
6

Answer: Multiple

Explanation: 6×4=24

Question 9.
36

Answer: Neither

Explanation: 36 is not a factor or multiple of 24.

Question 10.
48

Answer: Factor

Explanation: 24×2= 48, So 48 is a factor of 24

Problem Solving

Question 11.
Ken paid $12 for two magazines. The cost of each magazine was a multiple of $3. What are the possible prices of the magazines?

Answer: $3+$9=$12.

Explanation: As each magazine cost was multiple of $3, The possible price for 2 magazines are $3+$9=$12, which is a multiple of 3

Question 12.
Jodie bought some shirts for $6 each. Marge bought some shirts for $8 each. The girls spent the same amount of money on shirts. What is the least amount they could have spent?

Answer: $24

Explanation: As they spent the same amount of money which means the number should be multiple of $6 and $8, So multiples of 6 are: 6,12,18,24,30,36,42 and
multiples of 8 are: 8,16,24,32,40. The least amount they could spend is 24. As 24 is the least common multiple.

Common Core – Factors and Multiples – Page No. 304

Lesson Check

Question 1.
Which list shows numbers that are all multiples of 4?
Options:
a. 2, 4, 6, 8
b. 3, 7, 11, 15, 19
c. 4, 14, 24, 34
d. 4, 8, 12, 16

Answer: d

Explanation: Multiples of 4 are 4,8,12,16.

Question 2.
Which of the following numbers is a common multiple of 5 and 9?
Options:
a. 9
b. 14
c. 36
d. 45

Answer: 45

Explanation: 5×9= 45

Spiral Review

Question 3.
Jenny has 50 square tiles. She arranges the tiles into a rectangular array of 4 rows. How many tiles will be left over?
Options:
a. 0
b. 1
c. 2
d. 4

Answer: 2

Explanation: As Jenny arranges in 4 rows, each row contains 12 tiles. So 12×4= 48. The tiles left are 50-48=2.

Question 4.
Jerome added two numbers. The sum was 83. One of the numbers was 45. What was the other number?
Options:
a. 38
b. 48
c. 42
d. 128

Answer: a

Explanation: The sum of two numbers is 83, in that one number is 45. To find another number we will do subtraction,
i.e 83-45=38.

Question 5.
There are 18 rows of seats in the auditorium. There are 24 seats in each row. How many seats are in the auditorium in all?
Options:
a. 42
b. 108
c. 412
d. 432

Answer: d

Explanation:
No.of rows= 18, each row has 24 seats. So total no.of seats are 18×24= 432.

Question 6.
The population of Riverdale is 6,735. What is the value of the 7 in the number 6,735?
Options:
a. 7
b. 700
c. 735
d. 7,000

Answer: b

Explanation: In 6,735 the 7 is in the Hundreds Place. So the answer is 7.

Page No. 307

Question 1.
Use the grid to model the factors of 18. Tell whether 18 is prime or composite.

Factors of 18: ____ , ____ , ____ , ____ , ____ , ____
Think: 18 has more than two factors.
So, 18 is _________ .

Answer: Composite number.

Explanation: The number which has more than two factors is called composite numbers.
Factors of 18 are: 1,2,3,6,9,18.

Tell whether the number is prime or composite.

Question 2.
11
Think: Does 11 have other factors besides 1 and itself?

Answer: Prime number.

Explanation: A Prime number is a number that is divisible 1 and itself.

Question 3.
73

Answer: Prime number

Explanation: A Prime number is a number that is divisible 1 and itself.

Question 4.
69

Answer: Composite number.

Explanation: The number which has more than two factors is called composite numbers.
Factors of 69 are: 1,3,23,69.

Question 5.
42

Answer: Composite number.

Explanation: The number which has more than two factors is called composite numbers.
Factors of 42 are: 1,2,3,6,7,21,42.

Tell whether the number is prime or composite.

Question 6.
18

Answer: Composite number.

Explanation: The number which has more than two factors is called composite numbers.
Factors of 18 are: 1,2,3,6,9,18.

Question 7.
49

Answer: Composite number.

Explanation: The number which has more than two factors is called composite numbers.
Factors of 49 are 1,7,49.

Question 8.
29

Answer: Prime number.

Explanation: A Prime number is a number that is divisible 1 and itself.

Question 9.
64

Answer: Composite number.

Explanation: The number which has more than two factors is called composite numbers.
Factors of 64 are: 1,2,4,8,32,64.

Question 10.
33

Answer: Composite number.

Explanation: The number which has more than two factors is called composite numbers.
Factors of 33 are: 1,3,11,33.

Question 11.
89

Answer: Prime number.

Explanation: A Prime number is a number that is divisible 1 and itself.

Question 12.
52

Answer: Composite number.

Explanation: The number which has more than two factors is called composite numbers.
Factors of 52 are: 1,2,4,13,26,52.

Question 13.
76

Answer: Composite number.

Explanation: The number which has more than two factors is called composite numbers.
Factors of 76 are: 1,2,4,19,38,76.

Write true or false for each statement. Explain or give an example to support your answer.

Question 14.
Only odd numbers are prime numbers.

Answer: False.

Explanation: Not all odd numbers are prime numbers. For example. 39 is an odd number but not a prime number because it is divisible by 3 and 13.

Question 15.
A composite number cannot have three factors.

Answer: False

Explanation: A Composite number is a number that has more than two factors.
For example. 21 is a composite number and the factors of 21 are 1,3,7,21.

Question 16.
I am a number between 60 and 100. My ones digit is two less than my tens digit. I am a prime number. What number am I?

Answer: 97

Explanation:
Prime numbers between 60 to 100 are 61,67,71,73,79,83,89,97. 97 is the number which ones digit is two less than tens digit.

Question 17.
Name a 2-digit odd number that is prime. Name a 2-digit odd number that is composite.

Answer:
2 digit Prime odd numbers are 11,13,17 etc.
2 digit Composite odd numbers are 15,21,39

Explanation: A Prime number is a number that is divisible 1 and itself.
The number which has more than two factors is called composite numbers.

Question 18.
Choose the words that correctly complete the sentence.
The number 9 is img 18 because it has img 19 two factors.
Type below:
__________

Page No. 308

The Sieve of Eratosthenes
Go Math Grade 4 Answer Key Chapter 5 Factors, Multiples, and Patterns img 20
Eratosthenes was a Greek mathematician who lived more than 2,200 years ago. He invented a method of finding prime numbers, which is now called the Sieve of Eratosthenes.

Question 19.
Follow the steps below to circle all prime numbers less than 100. Then list the prime numbers.
STEP 1
Cross out 1, since 1 is not prime.
STEP 2
Circle 2, since it is prime. Cross out all other multiples of 2.
STEP 3
Circle the next number that is not crossed out. This number is prime. Cross out all the multiples of this number.
STEP 4
Repeat Step 3 until every number is either circled or crossed out.
Go Math Grade 4 Answer Key Chapter 5 Factors, Multiples, and Patterns img 21
So, the prime numbers less than 100 are

Answer: 2,3,5,7,11,13,17,19,23,29,31,37,41,43,47,53,59,61,67,71,73,79,83,89,97.

Explanation: A Prime number is a number that is divisible 1 and itself.

Question 20.
Explain why the multiples of any number other than 1 are not prime numbers.

Answer:

Common Core – Prime and Composite Numbers – Page No. 309

Prime and Composite Numbers

Tell whether the number is prime or composite

Question 1.
Go Math Grade 4 Answer Key Chapter 5 Factors, Multiples, and Patterns Common Core Prime and Composite Numbers img 22

Question 2.
68

Answer: Composite number.

Explanation: The number which has more than two factors is called composite numbers.
Factors of 68 are: 1,2,4,17,34,69.

Question 3.
52

Answer: Composite number.

Explanation: The number which has more than two factors is called composite numbers.
Factors of 52 are: 1,2,4,13,26,52.

Question 4.
63

Answer: Composite number.

Explanation: The number which has more than two factors is called composite numbers.
Factors of 63 are: 1,2,3,7,9,21,63.

Question 5.
75

Answer: Composite number.

Explanation: The number which has more than two factors is called composite numbers.
Factors of 75 are: 1,3,5,15,25,75

Question 6.
31

Answer: Prime number.

Explanation: 31 is a prime number that means it is divisible by 1 and itself.

Question 7.
77

Answer: Composite number.

Explanation: The number which has more than two factors is called composite numbers.
Factors of 77 are: 1,7,11,77.

Question 8.
59

Answer: Prime number

Explanation: 59 is a prime number that means it is divisible by 1 and itself.

Question 9.
87

Answer: Composite Number.

Explanation: The number which has more than two factors is called composite numbers.
Factors of 87 are: 1,3,29,87.

Question 10.
72

Answer: Composite Number.

Explanation: The number which has more than two factors is called composite numbers.
Factors of 72 are 1, 2, 3, 4, 6, 8, 9, 12, 18, 24, 36, 72.

Question 11.
49

Answer: Composite Number.

Explanation: The number which has more than two factors is called composite numbers.
Factors of 49 are 1,7,49.

Question 12.
73

Answer: Prime number.

Explanation: A Prime number is a number that is divisible 1 and itself.

Problem Solving

Question 13.
Kai wrote the number 85 on the board. Is 85 prime or composite?

Answer: Composite number

Explanation: The number which has more than two factors is called composite numbers.
Factors of 85 are 1,5,17,85

Question 14.
Lisa says that 43 is a 2-digit odd number that is composite. Is she correct?

Answer: No

Explanation: 43 is a prime number. A Prime number is a number that is divisible 1 and itself.

Common Core – Prime and Composite Numbers – Page No. 310

Lesson Check

Question 1.
The number 5 is:
Options:
a. prime
b. composite
c. both prime and composite
d. neither prime nor composite

Answer: Prime number.

Explanation: A Prime number is a number that is divisible 1 and itself.

Question 2.
The number 1 is:
Options:
a. prime
b. composite
c. both prime and composite
d. neither prime nor composite

Answer: d

Explanation: A Prime number is a number that is divisible 1 and itself. So prime number should have two divisors but 1 has only one divisor. The number which has more than two factors is called composite numbers. So 1 doesn’t have more than two factors. So 1 is neither Prime nor Composite.

Spiral Review

Question 3.
A recipe for a vegetable dish contains a total of 924 calories. The dish serves 6 people. How many calories are in each serving?
Options:
a. 134 calories
b. 150 calories
c. 154 calories
d. 231 calories

Answer: c

Explanation: Total no.of calories are 924, which serves 6 people. To find each serving we will perform division
924÷6= 154 calories.

Question 4.
A store clerk has 45 shirts to pack in  boxes. Each box holds 6 shirts. What is the fewest boxes the clerk will need to pack all the shirts?
Options:
a. 9
b. 8
c. 7
d. 6

Answer: b

Explanation: As the box holds only 6 shirts, 42 shirts are packed in 7 boxes, and the remaining 3 shirts will be packed in another box. So the total number of boxes is 8.

Question 5.
Which number rounds to 200,000?
Options:
a. 289,005
b. 251,659
c. 152,909
d. 149,889

Answer: c

Explanation: 152,909 is nearest to 200,000.

Question 6.
What is the word form of the number 602,107?
Options:
a. six hundred twenty thousand,seventeen
b. six hundred two thousand, one hundred seven
c. six hundred twenty-one thousand, seventeen
d. six hundred two thousand, one hundred seventy

Answer: b

Page No. 313

Use the rule to write the numbers in the pattern.

Question 1.
Rule: Subtract 10. First term: 100
Go Math Grade 4 Answer Key Chapter 5 Factors, Multiples, and Patterns img 23

Answer: 100,90,80,70,60,..

Explanation:
100
100-10= 90
90-10= 80
80-10= 70
70-10= 60

Use the rule to write the numbers in the pattern.
Describe another pattern in the numbers.

Question 2.
Rule: Multiply by 2. First term: 4
4 , _____ , _____ , _____ , _____ , …….

Answer: 4,8,16,32,64,…

Explanation:
4
4×2= 8
8×2= 16
16×2= 32
32×2= 64

Question 3.
Rule: Skip-count by 6. First term: 12
12 , _____ , _____ , _____ , _____ , …….

Answer: 12,18,24,30,36,…

Explanation:
12
12+6= 18
18+6= 24
24+6= 30
30+6= 36

Use the rule to write the first twelve numbers in the pattern. Describe another pattern in the numbers.

Question 4.
Rule: Add 7. First term: 3

Answer:
3
3+7= 10
10+7= 17
17+7= 34
34+7= 41
41+7= 48
48+7= 55
55+7= 62
62+7= 69
69+7= 76
76+7= 83
83+7= 90.

Explanation: Added 7 to the given term.

Question 5.
5. Rule: Add 2, add 1. First term: 12

Answer: 12,14,15,17,19,21,22,24,25,27,28,30,31.

Explanation:
12
12+2= 14
14+1= 15
15+2= 17
17+1= 19
19+2= 21
21+1= 22
22+2= 24
24+1= 25
25+2= 27
27+1= 28
28+2= 30
30+1= 31

Question 6.
Use Patterns Marcie likes to collect stickers, but she also likes to give them away. Currently, Marcie has 87 stickers in her collection. If Marcie collects 5 new stickers each week and gives away 3 stickers each week, how many stickers will Marcie have in her collection after 5 weeks?
_______ stickers

Answer: 97 stickers

Explanation: Marcie has 87 stickers, in 1st week she collects 5 stickers and gives away 3 stickers, that means
87+5-3= 89
2nd week 89+5-3= 91
3rd week 91+5-3= 93
4th week 93+5-3= 95
5th week 95+5-3= 97.

Page No. 314

Question 7.
John is saving for his trip to see the Alamo. He started with $24 in his savings account. Every week he earns $15 for baby-sitting. Out of that, he spends $8 and saves the rest. John uses the rule add 7 to find out how much money he has at the end of each week. What are the first 8 numbers in the pattern?

Answer: $24, $31, $38, $45, $52, $59, $66, $73.

Explanation:
24
24+7= 31
31+7= 38
38+7= 45
45+7= 52
52+7= 59
59+7= 66
66+7= 73.

Question 8.
Draw a check under the column that describes the number.
Go Math Grade 4 Answer Key Chapter 5 Factors, Multiples, and Patterns img 24

Pose a Problem

Question 9.
An activity at the Math Fair shows two charts.
Go Math Grade 4 Answer Key Chapter 5 Factors, Multiples, and Patterns img 25
Use at least two of the numbers and an operation from the charts to write a pattern problem. Include the first five terms of your pattern in the solution to your problem.
Pose a problem. Solve your problem.
Describe other patterns in the terms you wrote.

Answer:
2+3= 5    Addition.
10-6= 4   Subtraction.
5×2= 10  Multiplication.

Common Core – Number Patterns – Page No. 315

Number Patterns

Use the rule to write the first twelve numbers in the pattern.

Describe another pattern in the numbers.

Question 1.
Rule: Add 8. First term: 5
Go Math Grade 4 Answer Key Chapter 5 Factors, Multiples, and Patterns Common Core Number Patterns img 26

Question 2.
Rule: Subtract 7. First term: 95

Answer: 95,88,81,74,67,60,53,46,39,32,25,118,11.

Explanation: 95
95-7= 88
88-7= 81
81-7= 74
74-7= 67
67-7= 60
60-7= 53
53-7= 46
46-7= 39
39-7= 32
32-7= 25
25-7= 18
18-7= 11

Question 3.
Rule: Add 15, subtract 10. First term: 4

Answer: 4,19,9,24,14,29,19,34,24,39,29,44,34.

Explanation: 4
4+15= 19
19-10= 9
9+15= 24
24-10= 14
14+15= 29
29-10= 19
19+15= 34
34-10= 24
24+15= 39
39-10=29
29+15=44
44-10=34

Question 4.
Rule: Add 1, multiply by 2. First term: 2

Answer: 2,4,5,10,11,22,23,46,47,94,95,190.

Explanation: 2
2+1= 2
2×2= 4
4+1= 5
5×2= 10
10+1= 11
11×2= 22
22+1= 23
23×2= 46
46+1= 47
47×2= 94
94+1= 95
95×2= 190.

Problem Solving

Question 5.
Barb is making a bead necklace. She strings 1 white bead, then 3 blue beads, then 1 white bead, and so on. Write the numbers for the first eight beads that are white. What is the rule for the pattern?

Answer:

Explanation: 1
1+4= 5
5+4= 9
9+4= 13
13+4= 17
17+4= 21
21+4= 25
25+4=29

Question 6.
An artist is arranging tiles in rows to decorate a wall. Each new row has 2 fewer tiles than the row below it. If the first row has 23 tiles, how many tiles will be in the seventh row?

Answer: 11 tiles.

Explanation: 23
23-2= 21
21-2= 19
19-2= 17
17-2= 15
15-2= 13
13-2= 11

Common Core – Number Patterns – Page No. 316

Lesson Check

Question 1.
The rule for a pattern is add 6. The first term is 5. Which of the following numbers is a term in the pattern?
Options:
a. 6
b. 12
c. 17
d. 22

Answer: c

Explanation: 5
5+6= 11
11+6= 17

Question 2.
What are the next two terms in the pattern 3, 6, 5, 10, 9, 18, 17, . . .?
Options:
a. 16, 15
b. 30, 31
c. 33, 34
d. 34, 33

Answer: d

Explanation: 3
3×2= 6
6-1= 5
5×2= 10
10-1= 9
9×2= 18
18-1= 17
17×2= 34
34-1= 33

Spiral Review

Question 3.
To win a game, Roger needs to score 2,000 points. So far, he has scored 837 points. How many more points does Roger need to score?
Options:
a. 1,163 points
b. 1,173 points
c. 1,237 points
d. 2,837 points

Answer: a

Explanation: Roger has scored 837 points, He needs to score 2000 points to win, So to know how much more points do Roger needs we need to subtract i.e 2,000-837= 1,163.

Question 4.
Sue wants to use mental math to find 7 × 53. Which expression could she use?
Options:
a. (7 × 5) + 3
b. (7 × 5) + (7 × 3)
c. (7 × 50) + 3
d. (7 × 50) + (7 × 3)

Answer: d

Explanation: Distributive property means if we multiply a sum by a number is the same as multiplying each addend by the number and adding the products.

Question 5.
Pat listed numbers that all have 15 as a multiple. Which of the following could be Pat’s list?
Options:
a. 1, 3, 5, 15
b. 1, 5, 10, 15
c. 1, 15, 30, 45
d. 15, 115, 215

Answer: a

Explanation:
1×15= 15
3×5= 15
5×3= 15
15×1= 15

Question 6.
Which is a true statement about 7 and 14?
Options:
a. 7 is a multiple of 14.
b. 14 is a factor of 7.
c. 14 is a common multiple of 7 and 14.
d. 21 is a common multiple of 7 and 14.

Answer: c

Explanation:
7×2=14
14×1=14

Review/Test – Page No. 317

Question 1.
List all the factors of the number.
14: ______ , ______ , ______ , ______

Answer: 1,2,7,14

Explanation: Factors are the numbers that divide the original number completely. Here, we can see the numbers which give the result as 14 when multiplied together. So the factors of 14 are 1,2,7,14.

Question 2.
Select the numbers that have a factor of 5. Mark all that apply.
Options:
a. 15
b. 3
c. 45
d. 5
e. 50
f. 31

Answer: a,c,d,e.

Explanation: Factors are the numbers that divide the original number completely.

Question 3.
Jackson was making a poster for his room. He arranged 50 trading cards in the shape of a rectangle on the poster.
For 3a–3e, choose Yes or No to tell whether a possible arrangement of cards is shown.
a. 5 rows of 10 cards
i. yes
ii. no

Answer: Yes

Explanation: 5 rows of 10 cards that means 5×10= 50. So the answer is Yes.

Question 3.
b. 7 rows of 8 cards
i. yes
ii. no

Answer: No

Explanation: 7×8= 56, There will be extra cards. So the answer is No.

Question 3.
c. 25 rows of 2 cards
i. yes
ii. no

Answer: Yes.

Explanation: 25×2=50. So the answer is Yes

Question 3.
d. 50 rows of 1 card
i. yes
ii. no

Answer: Yes

Explanation: 50×1=50. So the answer is Yes.

Question 3.
e. 45 rows of 5 cards
i. yes
ii. no

Answer: No

Explanation: 45×5= 225. Which is not equal to 50. So the answer No.

Question 4.
List all the factor pairs in the table.
Go Math Grade 4 Answer Key Chapter 5 Factors, Multiples, and Patterns Review/Test img 27

Answer:
1×48= 48   1,48
2×24= 48   2,24
3×16= 48   3,16
4×12= 48   4,12
6×8=  48    6,8

Explanation: Factors are the numbers that divide the original number completely. Here, we can see the numbers which give the result as 30 when multiplied together.

Review/Test – Page No. 318

Question 5.
Classify the numbers. Some numbers may belong in more than one box.
Go Math Grade 4 Answer Key Chapter 5 Factors, Multiples, and Patterns Review/Test img 28

Answer:
Divisible by 5 and 9: 90
Divisible by 6 and 9: 54,72,90
Divisible by 2 and 6: 54,72,84,90,96

Question 6.
James works in a flower shop. He will put 36 tulips in vases for a wedding. He must use the same number of tulips in each vase. The number of tulips in each vase must be greater than 1 and less than 10. How many tulips could be in each vase?

Answer: 2, 3, 4, 6, 9.

Explanation:

Question 7.
Brady has a card collection with 64 basketball cards, 32 football cards, and 24 baseball cards. He wants to arrange the cards in equal piles, with only one type of card in each pile. How many cards can he put in each pile? Mark all that apply.
Options:
a. 1
b. 2
c. 3
d. 4
e. 8
f. 32

Answer: a,b,d,e

Explanation:
Factors of 64 are 1,2,4,8,16,32,64.
Factors of 32 are 1,2,4,8,16,32.
Factors of 24 are 1,2,3,4,6,8,12,24.
Common factors are 1,2,4,8.

Question 8.
The Garden Club is designing a garden with 24 cosmos, 32 pansies, and 36 marigolds. Each row will have only one type of plant in each row. Ben says he can put 6 plants in each row. He listed the common factors of 24, 32, and 36 below to support his reasoning.
24: 1, 2, 3, 4, 6, 8, 12, 24
32: 1, 2, 4, 6, 9, 16, 32
36: 1, 2, 3, 4, 6, 8, 12, 18, 36
Is he correct? Explain your answer. If his reasoning is incorrect, explain how he should have found the answer.

Answer: No. He can put 1,2,4 plants in each row

Explanation: The factors of 32 are incorrect. He listed as 6 and 9 are factors of 32 which is wrong and 8 is not a factor of 36.
Factors of 32 are 1,2,4,8,16,32.
Factors of 36 are 1,2,3,4,6,9,18,36.
Common factors of 24,32 and 36 are 1,2,4. So he can put 1,2,4 plants in each row.

Review/Test – Page No. 319

Question 9.
Go Math Grade 4 Answer Key Chapter 5 Factors, Multiples, and Patterns Review/Test img 29
Part A
The museum is hosting a show for July that features the oil paintings by different artists. All artists show the same number of paintings and each will show more than 1 painting. How many artists could be featured in the show?

Answer: 2,3,5,6,10,15

Explanation:
Factors of 30 are 1,2,3,5,6,10,15,30.

Question 9.
Part B
The museum wants to display all the art pieces in rows. Each row has the same number of pieces and the same type of pieces. How many pieces could be in each row? Explain how you found your answer.

Answer: 1,3.

Explanation:
Factors of 30 are 1,2,3,5,6,10,15,30.
Factors of 24 are 1,2,3,4,6,8,12,24
Factors of 21 are 1,3,7,21
Common Factors are 1,3

Question 10.
Charles was skip counting at the Math Club meeting. He started to count by 8s. He said 8, 16, 24, 32, 40, and 48. What number will he say next?

Answer: 56

Explanation: Multiples of 8
8×1= 8
8×2= 16
8×3= 24
8×4= 32
8×5= 40
8×6= 48
8×7= 56.

Question 11.
Jill wrote the number 40. If her rule is add 7, what is the fourth number in Jill’s pattern? How can you check your answer?

Answer: 61

Explanation:
40
40+7= 47
47+7= 54
54+7= 61, And the fourth number is 61

Review/Test – Page No. 320

Question 12.
For numbers 12a–12e, select True or False for each statement.
a. The number 36 is a multiple of 9.
i. True
ii. False

Answer: True

Explanation: 9×4= 36.

Question 12.
b. The number 3 is a multiple of 9.
i. True
ii. False

Answer: False

Explanation: Multiples of 9 are 9,18,27,36,45,54,63, etc.

Question 12.
c. The number 54 is a multiple of 9.
i. True
ii. False

Answer: True

Explanation: 9×6= 54

Question 12.
d. The number 3 is a factor of 9.
i. True
ii. False

Answer: True

Explanation: Factors of 9 are 1,3,9.

Question 12.
e. The number 27 is a factor of 9.
i. True
ii. False

Answer: True

Explanation: Factors of 27 are 1,3,9,27

Question 13.
What multiple of 7 is also a factor of 7?

Answer: 7

Explanation: 7 is both multiple and a factor of 7.

Question 14.
Manny makes dinner using 1 box of pasta and 1 jar of sauce. If pasta is sold in packages of 6 boxes and sauce is sold in packages of 3 jars, what is the least number of dinners that Manny can make without any supplies leftover?

Answer: 6

Manny has 1 box of pasta and 1 jar of sauce and he sold in a package of 6 boxes of pasta and 3 jars of sauce. Let the packages of pasta be 6P and jars of sauce be 3s.
As Manny sold without any leftover 3S=6P,
If we take 1 package of pasta then P=1,
And 3S=6×1, where S= 6/3 which is equal to 2,
So for every package of pasta, we need 2 packages of sauce,
So the minimum purchase is 2 packages of sauce and 1 package of pasta. Since pasta packages are 6 boxes the minimum number of meals is 6.

Question 15.
Serena has several packages of raisins. Each package contains 3 boxes of raisins. Which could be the number of boxes of raisins Serena has? Mark all that apply.
Options:
a. 9
b. 18
c. 23
d. 27
e. 32

Answer: a,b,d

Explanation: Factors of 3.

Question 16.
Choose the words that make the sentence true.
The number 7 is Go Math Grade 4 Answer Key Chapter 5 Factors, Multiples, and Patterns Review/Test img 30 because it has Go Math Grade 4 Answer Key Chapter 5 Factors, Multiples, and Patterns Review/Test img 31 two factors.
The number 7 is _________ because it has
_________ two factors.

Answer: The number 7 is a prime number because it has exactly two factors.

Explanation: A Prime number is a number that is divisible 1 and itself.

Review/Test – Page No. 321

Question 17.
Winnie wrote the following riddle: I am a number between 60 and 100. My ones digit is two less than my tens digit. I am a prime number.
Part A
What number does Winnie’s riddle describe? Explain.

Answer: 97

Explanation: 97 is the number which ones digit is two less than tens digit.

Question 17.
Part B
Winnie’s friend Marco guessed that her riddle was about the number 79. Why can’t 79 be the answer to Winnie’s riddle?
Explain.

Answer: It’s wrong because in Winnie’s riddle ones digit is two less than tens digit. But in 79 ones digit is two greater than tens digit.

Explanation: In 79 ones digit is two greater than tens digit. So Marco guess was incorrect.

Question 18.
Classify the numbers as prime or composite.
Go Math Grade 4 Answer Key Chapter 5 Factors, Multiples, and Patterns Review/Test img 32

Answer: Prime numbers are 37, 71
Composite numbers are 65, 82

Explanation:
A Composite number is a number that has more than two factors.
A Prime number is a number that is divisible 1 and itself.

Question 19.
Erica knits 18 squares on Monday. She knits 7 more squares each day from Tuesday through Thursday. How many squares does Erica knit on Friday?

Answer: 46 squares.

Explanation: 18
18+7= 25
25+7= 32
32+7= 39
39+7= 46.

Question 20.
Use the rule to write the first five terms of the pattern.
Rule: Add 10, subtract 5
First term: 11 ______ ______ ______ ______

Answer: 11,21,16,26,21.

Explanation: 11
11+10= 21
21-5= 16
16+10= 26
26-5= 21

Review/Test – Page No. 322

Question 21.
Elina had 10 tiles to arrange in a rectangular design. She drew a model of the rectangles she could make with the ten tiles.
Go Math Grade 4 Answer Key Chapter 5 Factors, Multiples, and Patterns Review/Test img 33
Part A
How does Elina’s drawing show that the number 10 is a composite number?

Answer: 10 is a composite number because it has more than two factors.

Explanation: The number which has more than two factors is called composite numbers.

Question 21.
Part B
Suppose Elina used 15 tiles to make the rectangular design. How many different rectangles could she make with the 15 tiles? Write a list or draw a picture to show the number and dimensions of the rectangles she could make.

Answer: 2

Explanation: one by 15 tiles and second by 3tiles in a row.

Question 21.
Part Cs
Elina’s friend Luke said that he could make more rectangles with 24 tiles than with Elina’s 10 tiles. Do you agree with Luke? Explain.

Answer: Yes

Explanation: As 24 has more factors than 10.

Page No. 329

Use the model to write an equivalent fraction.

Question 1.
Go Math Grade 4 Answer Key Chapter 5 Factors, Multiples, and Patterns img 34
Go Math Grade 4 Answer Key Chapter 5 Factors, Multiples, and Patterns img 35
\(\frac{1}{5}\) = \(\frac{□}{□}\)

Answer: 1/5= 2/10

Explanation: From the above figure we can see that there are 5 equal parts and in that 1 part is shaded. So the fraction of the shaded part is 1/5.

Question 2.
Go Math Grade 4 Answer Key Chapter 5 Factors, Multiples, and Patterns img 36
Go Math Grade 4 Answer Key Chapter 5 Factors, Multiples, and Patterns img 37
\(\frac{2}{3}\) = \(\frac{□}{□}\)

Answer: 2/3= 6/9

Explanation: From the above figure we can see that there are 3 equal parts and in that 2 part is shaded. So the fraction of the shaded part is 2/3.

Tell whether the fractions are equivalent. Write = or ≠.

Question 3.
\(\frac{1}{6}\) _____ \(\frac{2}{12}\)

Answer: 1/6=2/12

Explanation: The denominator and numerators are equal for both the fractions. So 1/6=2/12 are equal.

Question 4.
\(\frac{2}{5}\) _____ \(\frac{6}{10}\)

Answer: 2/5≠ 6/10

Explanation: The denominator and numerators are not equal for both the fractions.

Question 5.
\(\frac{4}{12}\) _____ \(\frac{1}{3}\)

Answer: 4/12=1/3

Explanation: The denominator and numerators are equal for both the fractions.

Question 6.
\(\frac{5}{8}\) _____ \(\frac{2}{4}\)

Answer: 5/8≠2/4

Explanation: The denominator and numerators are not equal for both the fractions.

Question 7.
\(\frac{5}{6}\) _____ \(\frac{10}{12}\)

Answer: 5/6=10/12

Explanation: The denominator and numerators are equal for both the fractions.

Question 8.
\(\frac{1}{2}\) _____ \(\frac{5}{10}\)

Answer: 1/2=5/10

Explanation: The denominator and numerators are equal for both the fractions.

Question 9.
Manny used 8 tenth-size parts to model \(\frac{8}{10}\). Ana used fewer parts to model an equivalent fraction. How does the size of a part in Ana’s model compare to the size of a tenth-size part? What size part did Ana use?

Answer: Larger than a tenth-size part. And she used the fifth-size part.

Explanation: A part of Ana’s model is larger than a tenth-size part. And she used the fifth-size part.

Question 10.
Use a Concrete Model How many eighth-size parts do you need to model \(\frac{3}{4}\)? Explain.

Answer: 6

Explanation: Let the parts be X, then 1/8×X=3/4. By calculation, we will get X as 6.
So we need 6 parts.

Page No. 330

Question 11.
Ben brought two pizzas to a party. He says that since 14_ of each pizza is left, the same amount of each pizza is left. What is his error?
Go Math Grade 4 Answer Key Chapter 5 Factors, Multiples, and Patterns img 38

Answer: As the size of pizzas is not the same, So 1/4 of leftover pizza is not equal to another.

Question 12.
For numbers 12a–12d, tell whether the fractions are equivalent by selecting the correct symbol.
a. \(\frac{3}{15}\) _____ \(\frac{1}{6}\)

Answer: 3/5≠1/6

Question 12.
b. \(\frac{3}{4}\) _____ \(\frac{16}{20}\)

Answer: 3/4≠16/20

Question 12.
c. \(\frac{2}{3}\) _____ \(\frac{8}{12}\)

Answer: 2/3=8/12

Question 12.
d. \(\frac{4}{5}\) _____ \(\frac{8}{10}\)

Answer: 4/5=8/10.

Go Math Grade 5 Answer Key Chapter 5 Divide Decimals

go-math-grade-5-chapter-5-divide-decimals-answer-key

Looking everywhere for Go Math Grade 5 Answer Key Chapter 5 Divide Decimals? Yes, you have come the right way and we have curated the Go Math Grade 5 Answer Key. Access the Step by Step Solutions provided for the Questions from Chapter Test, Practice Test, Cumulative Practice. Refer to the HMH Go math Grade 5 Chapter 5 Divide Decimals Solution Key to cross-check your Answers. Practice all Grade 5 Go Math Chapter 5 Divide Decimals for free of cost and score well.

Go Math Grade 5 Chapter 5 Divide Decimals Answer Key

You can improve math proficiency by practicing with the Go Math Grade 5 Chapter 5 Answer Key on a regular basis. The Best Guide to Prepare Math is to refer to HMH Go Math Grade 5 Answer Key. Solve as many times as possible and try to cover all the topics in it via quick links out there. Simply click on the topics you want to prepare and check all the Problems in it.

Lesson 1: Algebra • Division Patterns with Decimals

Lesson 2: Investigate • Divide Decimals by Whole Numbers

Lesson 3: Estimate Quotients

Lesson 4: Division of Decimals by Whole Numbers

Mid-Chapter Checkpoint

Lesson 5: Investigate • Decimal Divisions

Lesson 6: Divide Decimals

Lesson 7: Write Zeros in the Dividend

Lesson 8: Problem Solving • Decimal Operations

Chapter 5 Review/Test

Share and Show – Page No. 203

Complete the pattern.

Question 1.
456 ÷ 100 = 456
456 ÷ 101 = 45.6
456 ÷ 102 = 4.56
Think: The dividend is being divided by an increasing power of 10, so the decimal point will move to the left one place for each increasing power of 10.
456 ÷ 103 = _____

Answer:
456 ÷ 103 = 0.456

Explanation:
The dividend is being divided by an increasing power of 10, so the decimal point will move to the left one place for each increasing power of 10.
456 ÷ 100 = 456
456 ÷ 101 = 45.6
456 ÷ 102 = 4.56
456 ÷ 103 = 0.456

Complete the pattern.

Question 2.
225 ÷ 100 = _____
225 ÷ 101 = _____
225 ÷ 102 = _____
225 ÷ 103 = _____

Answer:
225 ÷ 100 = 225
225 ÷ 101 = 22.5
225 ÷ 102 = 2.25
225 ÷ 103 = 0.25

Explanation:
The dividend is being divided by an increasing power of 10, so the decimal point will move to the left one place for each increasing power of 10.
225 ÷ 100 = 225/1 = 225
225 ÷ 101 = 225/10 = 22.5
225 ÷ 102 = 225/100 = 2.25
225 ÷ 103 = 225/1,000 = 0.25

Question 3.
605 ÷ 100 = _____
605 ÷ 101 = _____
605 ÷ 102 = _____
605 ÷ 103 = _____

Answer:
605 ÷ 100 = 605
605 ÷ 101 = 60.5
605 ÷ 102 = 6.05
605 ÷ 103 = 0.605

Explanation:
The dividend is being divided by an increasing power of 10, so the decimal point will move to the left one place for each increasing power of 10.
605 ÷ 100 = 605/1 = 605
605 ÷ 101 = 605/10 = 60.5
605 ÷ 102 = 605/100 = 6.05
605 ÷ 103 = 605/1,000 = 0.605

Question 4.
74.3 ÷ 1 = _____
74.3 ÷ 10 = _____
74.3 ÷ 100 = _____

Answer:
74.3 ÷ 1 = 74.3
74.3 ÷ 10 = 7.43
74.3 ÷ 100 = 0.743

Explanation:
The dividend is being divided by an increasing power of 10, so the decimal point will move to the left one place for each increasing power of 10.
74.3 ÷ 100 = 74.3 ÷ 1 = 74.3
74.3 ÷ 101 = 74.3 ÷ 10 = 7.43
74.3 ÷ 102 = 74.3 ÷ 100 = 0.743

On Your Own

Complete the pattern.

Question 5.
156 ÷ 1 = _____
156 ÷ 10 = _____
156 ÷ 100 = _____
156 ÷ 1,000 = _____

Answer:
156 ÷ 1 = 156
156 ÷ 10 = 15.6
156 ÷ 100 = 1.56
156 ÷ 1,000 = 0.156

Explanation:
The dividend is being divided by an increasing power of 10, so the decimal point will move to the left one place for each increasing power of 10.
156 ÷ 1 = 156
156 ÷ 10 = 15.6
156 ÷ 100 = 1.56
156 ÷ 1,000 = 0.156

Question 6.
32 ÷ 1 = _____
32 ÷ 10 = _____
32 ÷ 100 = _____
32 ÷ 1,000 = _____

Answer:
32 ÷ 1 = 32
32 ÷ 10 = 3.2
32 ÷ 100 = 0.32
32 ÷ 1,000 = 0.032

Explanation:
The dividend is being divided by an increasing power of 10, so the decimal point will move to the left one place for each increasing power of 10.
32 ÷ 1 = 32
32 ÷ 10 = 3.2
32 ÷ 100 = 0.32
32 ÷ 1,000 = 0.032

Question 7.
16 ÷ 100 = _____
16 ÷ 101 = _____
16 ÷ 102 = _____
16 ÷ 103 = _____

Answer:
16 ÷ 100 = 16
16 ÷ 101 = 1.6
16 ÷ 102 = 0.16
16 ÷ 103 = 0.016

Explanation:
The dividend is being divided by an increasing power of 10, so the decimal point will move to the left one place for each increasing power of 10.
16 ÷ 100 = 16
16 ÷ 101 = 1.6
16 ÷ 102 = 0.16
16 ÷ 103 = 0.016

Question 8.
12.7 ÷ 1 = _____
12.7 ÷ 10 = _____
12.7 ÷ 100 = _____
12.7 ÷ 1,000 = _____

Answer:
12.7 ÷ 1 = 12.7
12.7 ÷ 10 = 1.27
12.7 ÷ 100 = 0.127
12.7 ÷ 1,000 = 0.0127

Explanation:
The dividend is being divided by an increasing power of 10, so the decimal point will move to the left one place for each increasing power of 10.
12.7 ÷ 1 = 12.7
12.7 ÷ 10 = 1.27
12.7 ÷ 100 = 0.127
12.7 ÷ 1,000 = 0.0127

Question 9.
92.5 ÷ 100 = _____
92.5 ÷ 101 = _____
92.5 ÷ 102 = _____
92.5 ÷ 103 = _____

Answer:
92.5 ÷ 100 = 92.5
92.5 ÷ 101 = 9.25
92.5 ÷ 102 = 0.925
92.5 ÷ 103 = 0.0925

Explanation:
The dividend is being divided by an increasing power of 10, so the decimal point will move to the left one place for each increasing power of 10.
92.5 ÷ 100 = 92.5
92.5 ÷ 101 = 9.25
92.5 ÷ 102 = 0.925
92.5 ÷ 103 = 0.0925

Question 10.
86.3 ÷ 100 = _____
86.3 ÷ 101 = _____
86.3 ÷ 102 = _____
86.3 ÷ 103 = _____

Answer:
86.3 ÷ 100 = 86.3
86.3 ÷ 101 = 8.63
86.3 ÷ 102 = 0.863
86.3 ÷ 103 = 0.0863

Explanation:
The dividend is being divided by an increasing power of 10, so the decimal point will move to the left one place for each increasing power of 10.
86.3 ÷ 100 = 86.3
86.3 ÷ 101 = 8.63
86.3 ÷ 102 = 0.863
86.3 ÷ 103 = 0.0863

Algebra Find the value of n.

Question 11.
268 ÷ n = 0.268
n = _____

Answer:
n = 1000

Explanation:
268 ÷ n = 0.268
268 = n x 0.268
n = 268 ÷ 0.268
n = 1000

Question 12.
n ÷ 102 = 0.123
n = _____

Answer:
n = 12.3

Explanation:
n ÷ 102 = 0.123
n = 0.123 x 102
n = 0.123 x 100
n = 12.3

Question 13.
n ÷ 101 = 4.6
n = _____

Answer:
n = 46

Explanation:
n ÷ 101 = 4.6
n = 4.6 x 101
n = 4.6 x 10
n = 46

Problem Solving – Page No. 204

Use the table to solve 14–16.
Go Math Grade 5 Answer Key Chapter 5 Divide Decimals img 1

Question 14.
If each muffin contains the same amount of cornmeal, how many kilograms of cornmeal are in each corn muffin?
_____ kilograms

Answer:
0.15 kilograms

Explanation:
There are 1,000 muffins. Cornmeal = 150 Kg
If each muffin contains the same amount of cornmeal, 150 ÷ 1000 = 0.15
0.15 kilograms of cornmeal is in each corn muffin

Question 15.
If each muffin contains the same amount of sugar, how many kilograms of sugar, to the nearest thousandth, are in each corn muffin?
_____ kilograms

Answer:
0.07 kilograms

Explanation:
There are 1,000 muffins. Sugar = 66.7 kilograms
If each muffin contains the same amount of sugar, 66.7 ÷ 1000 = 0.0667.
0.0667 kilograms of sugar is in each corn muffin.
The thousandth digit is 6. 6 > 5
So, 0.07

Question 16.
The bakery decides to make only 100 corn muffins on Tuesday. How many kilograms of sugar will be needed?
_____ kilograms

Answer:
0.007 kilograms

Explanation:
The bakery decides to make only 100 corn muffins on Tuesday.
As 0.07 kilograms required for 1,000 muffins,
for 100 muffins, (100 x 0.07) ÷ 1000 = 0.007

Question 17.
Explain how you know that the quotient 47.3 ÷ 101 is equal to the product 47.3 × 0.1.
Type below:
_________

Answer:
Quotient 47.3 ÷ 101 = 47.3 ÷ 10 = 4.73. The power of 101 = 10.
47.3 × 0.1 = 4.73.
By dividing 10 to a number is equal to the multiplying 0.1 to that number.

Question 18.
Test Prep Ella used 37.2 pounds of apples to make applesauce. She used one-tenth as many pounds of sugar as pounds of apples. How many pounds of sugar did Ella use?
Options:
a. 372 pounds
b. 3.72 pounds
c. 0.372 pound
d. 0.0372 pound

Answer:
b. 3.72 pounds

Explanation:
Ella used 37.2 pounds of apples to make applesauce. She used one-tenth as many pounds of sugar as pounds of apples.
37.2 ÷ 10 = 3.72 pounds

Share and Show – Page No. 207

Use the model to complete the number sentence.

Question 1.
1.6 ÷ 4 =
Go Math Grade 5 Answer Key Chapter 5 Divide Decimals img 2
_____

Answer:
1.6 ÷ 4  = 0.4

Explanation:
1.6 ÷ 4
Share your model among 4 equal groups.
Since 1 whole cannot be shared among 4 groups without regrouping, cut your model apart to show the tenths.
1 ones = 10 tenths
10 + 6 = 16 tenths
There are 16 tenths in 1.6.
Share the tenths equally among the 4 groups.
There are 0 ones and 16 tenths in each group.
Decimal for the amount in each group = 0.4
1.6 ÷ 4  = 0.4

Question 2.
3.42 ÷ 3 =
Go Math Grade 5 Answer Key Chapter 5 Divide Decimals img 3
_____

Answer:
3.42 ÷ 3 = 1.14

Explanation:
3.42 ÷ 3
Share your model among 3 equal groups.
1 whole in each group. So, 3 wholes shared equally in 3 groups. 1 ones
3 ÷ 3 = 1 ones
3 tenths shared equally in 3 groups. 1 tenth has remained. 1 tenth
3 ÷ 3 = 1 tenths
1 tenth = 10 hundredths.
10 + 2 = 12 hundredths.
Share 12 hundredths equally among the 3 groups.
12 hundredths ÷ 3 = 4 hundredths.
Decimal for the amount in each group = 1.14
3.42 ÷ 3 = 1.14

Divide. Use base-ten blocks.

Question 3.
1.8 ÷ 3 = _____

Answer:
1.8 ÷ 3 = 0.6

Explanation:
1.8 ÷ 3
Share your model among 3 equal groups.
Since 1 whole cannot be shared among 3 groups without regrouping, cut your model apart to show the tenths. 0 ones
1 ones = 10 tenths
10 + 8 = 18 tenths
There are 18 tenths in 1.8.
Share the 18 tenths equally among the 3 groups.
18 ÷ 3 = 6
There are 0 ones and 18 tenths in each group.
Decimal for the amount in each group = 0.6
1.8 ÷ 3 = 0.6

Question 4.
3.6 ÷ 4 = _____

Answer:
3.6 ÷ 4 = 0.9

Explanation:
3.6 ÷ 4
Share your model among 4 equal groups.
Since 3 whole cannot be shared among 4 groups without regrouping, cut your model apart to show the tenths. 0 ones
1 ones = 10 tenths
30 + 6 = 36 tenths
There are 36 tenths in 3.6.
Share the 36 tenths equally among the 4 groups.
There are 0 ones and 36 tenths in each group.
36 ÷ 4 = 9
Decimal for the amount in each group = 0.9
3.6 ÷ 4 = 0.9

Question 5.
2.5 ÷ 5 = _____

Answer:
2.5 ÷ 5 = 0.5

Explanation:
2.5 ÷ 5
Share your model among 5 equal groups.
Since 2 whole cannot be shared among 5 groups without regrouping, cut your model apart to show the tenths. 0 ones
1 ones = 10 tenths
20 + 5 = 25 tenths
There are 25 tenths in 2.5.
Share the 25 tenths equally among the 5 groups.
There are 0 ones and 25 tenths in each group.
25 ÷ 5 = 5
Decimal for the amount in each group = 0.5
2.5 ÷ 5 = 0.5

Question 6.
2.4 ÷ 8 = _____

Answer:
2.4 ÷ 8 = 0.3

Explanation:
2.4 ÷ 8
Share your model among 8 equal groups.
Since 2 whole cannot be shared among 8 groups without regrouping, cut your model apart to show the tenths. 0 ones
1 ones = 10 tenths
20 + 4 = 24 tenths
There are 24 tenths in 2.4.
Share the 24 tenths equally among the 8 groups.
There are 0 ones and 24 tenths in each group.
24 ÷ 8 = 3
Decimal for the amount in each group = 0.3
2.4 ÷ 8 = 0.3

Question 7.
3.78 ÷ 3 = _____

Answer:
3.78 ÷ 3 = 1.26

Explanation:
3.78 ÷ 3
Share your model among 3 equal groups.
1 whole in each group. So, 3 wholes shared equally in 3 groups.
3 ÷ 3 = 1 ones
6 tenths shared equally in 3 groups. 1 tenth has remained.
6 ÷ 3 = 2 tenths
1 tenth = 10 hundredths.
10 + 8 = 18 hundredths.
Share 18 hundredths equally among the 3 groups.
18 hundredths ÷ 3 = 6 hundredths.
Decimal for the amount in each group = 1.26
3.78 ÷ 3 = 1.26

Question 8.
1.33 ÷ 7 = _____

Answer:
1.33 ÷ 7 = 0.19

Explanation:
1.33 ÷ 7
Share your model among 7 equal groups.
Since 1 whole cannot be shared among 7 groups without regrouping, cut your model apart to show the tenths. 0 ones
1 ones = 10 tenths
10 + 3 = 13 tenths
There are 13 tenths in 1.3.
Share the 13 tenths equally among the 7 groups.
Share 7 tenths equally among the 3 groups. 6 tenths remained.
7 ÷ 7 = 1 tenths
Since 6 tenths cannot be shared among 7 groups without regrouping, cut your model apart to show the tenths.
1 tenths = 10 hundredths
60 + 3 = 63 hundredths
Share 63 hundredths equally among the 7 groups.
63 ÷ 7 = 9 hundredths
Decimal for the amount in each group = 0.19
1.33 ÷ 7 = 0.19

Question 9.
4.72 ÷ 4 = _____

Answer:
4.72 ÷ 4 = 1.18

Explanation:
4.72 ÷ 4
Share your model among 4 equal groups.
1 whole in each group. So, 4 wholes shared equally in 4 groups. 1 ones
4 ÷ 4 = 1 ones
4 tenths shared equally in 4 groups. 3 tenths have remained.
4 ÷ 4 = 1 tenths
1 tenth = 10 hundredths.
30 + 2 = 32 hundredths.
Share 32 hundredths equally among the 4 groups.
32 hundredths ÷ 4 = 8 hundredths.
Decimal for the amount in each group = 1.18
4.72 ÷ 4 = 1.18

Question 10.
2.52 ÷ 9 = _____

Answer:
2.52 ÷ 9 = 0.28

Explanation:
2.52 ÷ 9
Share your model among 9 equal groups.
Since 2 whole cannot be shared among 9 groups without regrouping, cut your model apart to show the tenths.
9 ÷ 9 = 1 ones
1 ones = 10 tenths
20 + 5 = 25 tenths
There are 25 tenths in 2.5.
Share the 18 tenths equally among the 9 groups. 7 tenths remained.
18 ÷ 9 = 2 tenths
1 tenth = 10 hundredths.
70 + 2 hundredths = 72 hundredths
Share the 72 hundredths equally among the 9 groups.
72 ÷ 9 = 8
Decimal for the amount in each group = 0.28
2.52 ÷ 9 = 0.28

Question 11.
6.25 ÷ 5 = _____

Answer:
6.25 ÷ 5 = 1.25

Explanation:
6.25 ÷ 5
Share your model among 5 equal groups.
1 whole in each group. So, 5 wholes shared equally in 5 groups. 1 whole remained.
5 ÷ 5 = 1 ones 
1 ones = 10 tenths
10 + 2 = 12 tenths
10 tenths shared equally in 5 groups. 2 tenths have remained.
10 ÷ 5 = 2 tenths
1 tenth = 10 hundredths.
20 + 5 = 25 hundredths.
Share 25 hundredths equally among the 5 groups.
25 hundredths ÷ 5 = 5 hundredths.
Decimal for the amount in each group = 1.25
6.25 ÷ 5 = 1.25

Problem Solving – Page No. 208

What’s the Error?

Question 12.
Aida is making banners from a roll of paper that is 4.05 meters long. She will cut the paper into 3 equal lengths. How long will each banner be?
Go Math Grade 5 Answer Key Chapter 5 Divide Decimals img 4
Look how Aida solved the problem.      Solve the problem and correct
Find the error.                                            the error.
Go Math Grade 5 Answer Key Chapter 5 Divide Decimals img 5
So, Aida said that each banner would be _________ meters long, but each banner should be _________ meters long.
Type below:
_________

Answer:
So, Aida said that each banner would be 4.05 meters long, but each banner should be 1.35 meters long.
So, 1 ones, 3 tenths, and 5 hundredths are shared among 3 groups.
But Aida draws only one whole and 5 hundredths among 3 groups.

Explanation:
Aida is making banners from a roll of paper that is 4.05 meters long. She will cut the paper into 3 equal lengths.
4.05 ÷ 3
Share your model among 3 equal groups.
1 whole in each group. So, 3 wholes shared equally in 5 groups. 1 whole remained.
3 ÷ 3 = 1 ones 
1 ones = 10 tenths
10 + 0 = 10 tenths
9 tenths shared equally in 3 groups. 1 tenth remained.
9 ÷ 3 = 3 tenths
1 tenth = 10 hundredths.
10 + 5 = 15 hundredths.
Share 6 hundredths equally among the 3 groups.
15 hundredths ÷ 3 = 5 hundredths.
Decimal for the amount in each group = 1.35
4.05 ÷ 3 = 1.35

Question 12.
• Describe Aida’s error.
Type below:
_________

Answer:
So, Aida said that each banner would be 4.05 meters long, but each banner should be 1.35 meters long.
So, 1 ones, 3 tenths, and 5 hundredths are shared among 3 groups.
But Aida draws only one whole and 5 hundredths among 3 groups.

Question 12.
• What if the roll of paper were 4.35 meters long? How long would each banner be?
Type below:
_________

Answer:
1.45 meters long

Explanation:
If the roll paper were 4.35 meters long,
4.35 ÷ 3
Share your model among 3 equal groups.
1 whole in each group. So, 3 wholes shared equally in 5 groups. 1 whole remained.
3 ÷ 3 = 1 ones 
1 ones = 10 tenths
10 + 3 = 13 tenths
12 tenths shared equally in 3 groups. 1 tenth remained.
12 ÷ 3 = 4 tenths
1 tenth = 10 hundredths.
10 + 5 = 15 hundredths.
Share 6 hundredths equally among the 3 groups.
15 hundredths ÷ 3 = 5 hundredths.
Decimal for the amount in each group = 1.45
4.35 ÷ 3 = 1.45

Share and Show – Page No. 211

Use compatible numbers to estimate the quotient.

Question 1.
28.8 ÷ 9
Type below:
_________

Answer:
About to 3

Explanation:
28.8 is closer to 30
9 is closer to 10
30 ÷ 10 = 3

Question 2.
393.5 ÷ 41
_____ ÷ _____ = _____

Answer:
400 ÷ 40 = 10

Explanation:
Use a whole number greater than the dividend.
393.5 is closer to 400
41 is closer to 40
400 ÷ 40 = 10
Use a whole number less than the dividend.
393.5 is closer to 360
41 is closer to 40
360 ÷ 40 = 9
The quotient should be between 9 and 10.
393.5 is closer to 400.
So, the answer is about 10

Estimate the quotient.

Question 3.
161.7 ÷ 7
_____

Answer:
About 23

Explanation:
161.7 is closer to 161
161 ÷ 7 = 23

Question 4.
17.9 ÷ 9
_____

Answer:
About 2

Explanation:
17.9 is closer to 18
18 ÷ 9 = 2

Question 5.
145.4 ÷ 21
_____

Answer:
about 7

Explanation:
Use a whole number greater than the dividend.
145.4 is closer to 160
21 is closer to 20
160 ÷ 20 = 8
Use a whole number less than the dividend.
145.4 is closer to 140
21 is closer to 20
140 ÷ 20 = 7
The quotient should be between 7 and 8.
145.4 is closer to 140.
So, the answer is about 7

On Your Own

Estimate the quotient.

Question 6.
15.5 ÷ 4
_____

Answer:
About 4

Explanation:
15.5 is closer to 16
16 ÷ 4 = 4

Question 7.
394.8 ÷ 7
_____

Answer:
About 60

Explanation:
394.8 is closer to 420
420 ÷ 7 = 60
About 60

Question 8.
410.5 ÷ 18
_____

Answer:
About 22

Explanation:
410.5 is closer to 396
396 ÷ 18 = 22

Question 9.
72.1 ÷ 7
_____

Answer:
About 10

Explanation:
72.1 is closer to 70
70 ÷ 7 = 10

Question 10.
32.4 ÷ 52
_____

Answer:
About 1

Explanation:
32.4 is closer to 52
52 ÷ 52 = 1

Question 11.
$134.42 ÷ 28
$ _____

Answer:
about 4

Explanation:
Use a whole number greater than the dividend.
$134.42 is closer to 150
28 is closer to 30
150 ÷ 30 = 5
Use a whole number less than the dividend.
$134.42 is closer to 120
28 is closer to 30
120 ÷ 30 = 4
The quotient should be between 4 and 5.
$134.42 is closer to $120.
So, the answer is about 4

Question 12.
21.8 ÷ 4
_____

Answer:
About 5

Explanation:
21.8 is closer to 20
20 ÷ 4 = 5

Question 13.
3.4 ÷ 5
_____

Answer:
About 1

Explanation:
3.4 is closer to 5
5 ÷ 5 = 1
About 1

Question 14.
$759.92 ÷ 42
$ _____

Answer:
About $18

Explanation:
$759.92 is closer to $756
$756 ÷ 42 = 18
About 18

Question 15.
157.5 ÷ 38
_____

Answer:
About 4

Explanation:
157.5 is closer to 152
152 ÷ 38 = 4

Question 16.
379.2 ÷ 6
_____

Answer:
About 63

Explanation:
379.2 is closer to 378
378 ÷ 6 = 63

Question 17.
108.4 ÷ 21
_____

Answer:
About 5

Explanation:
108.4 is closer to 100
21 is closer to 20
100 ÷ 20 = 5

Problem Solving – Page No. 212

Use the table to solve 18–20.
Go Math Grade 5 Answer Key Chapter 5 Divide Decimals img 6

Question 18.
Estimate the average daily snowfall for Alaska’s greatest 7-day snowfall.
_____ inches

Answer:
26 inches

Explanation:
Alaska = 186.9 inches snowfall for 7 days.
For 1 day, 186.9 ÷ 7
186.9 is closer to 182
1825 ÷ 7 = 26 inches

Question 19.
How does the estimate of the average daily snowfall for Wyoming’s greatest 7-day snowfall compare to the estimate of the average daily snowfall for South Dakota’s greatest 7-day snowfall?
Type below:
_________

Answer:
South Dakota’s snowfall is greater than Wyoming’s snowfall.

Explanation:
Wyoming’s = 84.5 inches snowfall for 7 days.
For 1 day, 84.5 ÷ 7
84.5 is closer to 84
84 ÷ 7 = 12 inches
South Dakota’s = 112.7 inches snowfall for 7 days.
For 1 day, 112.7 ÷ 7
112.7 is closer to 112
112 ÷ 7 = 16 inches

Question 20.
The greatest monthly snowfall total in Alaska is 297.9 inches. This happened in February, 1953. Compare the daily average snowfall for February, 1953, with the average daily snowfall for Alaska’s greatest 7-day snowfall. Use estimation.
Type below:
_________

Answer:
The average snowfall of Alaska in February, 1953 is greater than snowfall for Alaska’s greatest 7-day snowfall.

Explanation:
Alaska = 186.9 inches snowfall for 7 days.
For 1 day, 186.9 ÷ 7
186.9 is closer to 182
1825 ÷ 7 = 26 inches
Alaska = 297.9 inches snowfall for 7 days.
For 1 day, 297.9 ÷ 7
297.9 is closer to 294
294 ÷ 7 = 42 inches

Question 21.
What’s the Error? During a 3-hour storm, it snowed 2.5 inches. Jacob said that it snowed an average of about 8 inches per hour.
Type below:
_________

Answer:
During a 3-hour storm, it snowed 2.5 inches.
For 1 hour, 2.5 ÷ 3 = 0.833
Jacob said that it snowed an average of about 8 inches per hour. He missed the remained inches.

Question 22.
Test Prep A plant grew 23.8 inches over 8 weeks. Which is the best estimate of the average number of inches the plant grew each week?
Options:
a. 0.2 inch
b. 0.3 inch
c. 2 inch
d. 3 inch

Answer:
d. 3 inch

Explanation:
A plant grew 23.8 inches over 8 weeks.
23.8 is closer to 24
24 ÷ 8 = 3 inch

Share and Show – Page No. 215

Write the quotient with the decimal point placed correctly.

Question 1.
4.92 ÷ 2 = 246
_____

Answer:
2.46

Explanation:
Divide. 4 ones ÷ 2
Multiply. 2 × 2 ones = 4 ones. Subtract. 4 ones − 4 ones = 0
Check. 4 ones shared among 2 groups as 2 wholes.
Divide. 9 tenths ÷ 2
Multiply. 2 × 4 tenths = 8 tenths. Subtract. 9 tenths − 8 tenths = 1 tenths
Check. 8 tenths shared among 2 groups as 4 tenths. 1 tenth remained.
1 tenth = 10 hundredths.
10 + 2 = 12 hundredths
Divide. 12 hundredths ÷ 2
Multiply. 2 × 6 hundredths = 12 hundredths. Subtract. 12 hundredths − 12 hundredths = 0 hundredths
Check. 12 hundredths shared among 2 groups as 6 hundredths.
So, 2.46

Question 2.
50.16 ÷ 38 = 132
_____

Answer:
1.32

Explanation:
Divide. 50 ones ÷ 38
Multiply. 38 × 1 ones = 38 ones. Subtract. 50 ones − 38 ones = 12 ones
Check. 38 ones shared among 38 groups as 1 whole.
12 one(s) cannot be shared among 38 groups without regrouping.
120 + 1 = 121 tenths
Divide. 121 tenths ÷ 38
Multiply. 38 x 3 tenths = 114 tenths. Subtract. 121 tenths − 114 tenths = 7 tenths
Check. 114 tenths shared among 38 groups as 3 tenths. 7 tenths remained.
1 tenth = 10 hundredths.
70 + 6 = 76 hundredths
Divide. 76 hundredths ÷ 38
Multiply. 38 x 2 hundredths = 76 hundredths. Subtract. 76 hundredths − 76 hundredths = 0 hundredths
Check. 76 hundredths shared among 38 groups as 2 hundredths.
So, 1.32

Divide.

Question 3.
5)\(\overline { 8.65 }\)
_____

Answer:
1.73

Explanation:
8.65 ÷ 5
Divide. 8 ones ÷ 5
Multiply. 5 × 1 ones = 5ones. Subtract. 8 ones − 5 ones = 3 ones
Check. 5 ones shared among 5 groups as 1 whole.
3 one(s) cannot be shared among 5 groups without regrouping.
30 + 6 = 36 tenths
Divide. 36 tenths ÷ 5
Multiply. 5 x 7 tenths = 35 tenths. Subtract. 36 tenths − 35 tenths = 7 tenths
Check. 35 tenths shared among 5 groups as 7 tenths.
1 tenths(s) cannot be shared among 5 groups without regrouping.
1 tenth = 10 hundredths.
10 + 5 = 15 hundredths
Divide. 15 hundredths ÷ 5
Multiply. 5 x 3 hundredths = 15 hundredths. Subtract. 15 hundredths − 15 hundredths = 0 hundredths
Check. 15 hundredths shared among 5 groups as 3 hundredths.
So, 1.73

Question 4.
3)\(\overline { 2.52 }\)
_____

Answer:
0.84

Explanation:
2.52 ÷ 3
2 one(s) cannot be shared among 3 groups without regrouping.
20 + 5 = 25 tenths
Divide. 25 tenths ÷ 3
Multiply. 3 x 8 tenths = 24 tenths. Subtract. 25 tenths − 24 tenths = 1 tenths
Check. 24 tenths shared among 3 groups as 8 tenths.
1 tenths(s) cannot be shared among 3 groups without regrouping.
1 tenth = 10 hundredths.
10 + 2 = 12 hundredths
Divide. 12 hundredths ÷ 3
Multiply. 3 x 4 hundredths = 12 hundredths. Subtract. 12 hundredths − 12 hundredths = 0 hundredths
Check. 12 hundredths shared among 3 groups as 4 hundredths.
So, 0.84

Question 5.
27)\(\overline { 97.2 }\)
_____

Answer:
3.6

Explanation:
97.2 ÷ 27
Divide. 97 ones ÷ 27
Multiply. 27 x 3 ones = 81 ones. Subtract. 97 ones − 81 ones = 16 ones
Check. 81 ones shared among 27 groups as 3 whole.
16 one(s) cannot be shared among 27 groups without regrouping.
160 + 2 = 162 tenths
Divide. 162 tenths ÷ 27
Multiply. 27 x 6 tenths = 162 tenths. Subtract. 162 tenths − 162 tenths = 0 tenths
Check. 162 tenths shared among 27 groups as 6 tenths.
So, 3.6

On Your Own

Divide.

Question 6.
6)\(\overline { 8.94 }\)
_____

Answer:
1.49

Explanation:
8.94 ÷ 6
Divide. 8 ones ÷ 6
Multiply. 6 × 1 ones = 6 ones. Subtract. 8 ones − 6 ones = 2 ones
Check. 6 ones shared among 6 groups as 1 whole.
2 one(s) cannot be shared among 6 groups without regrouping.
20 + 9 = 29 tenths
Divide. 29 tenths ÷ 6
Multiply. 6 x 4 tenths = 24 tenths. Subtract. 29 tenths − 24 tenths = 5 tenths
Check. 24 tenths shared among 6 groups as 4 tenths.
5 tenths(s) cannot be shared among 6 groups without regrouping.
1 tenth = 10 hundredths.
50 + 4 = 54 hundredths
Divide. 54 hundredths ÷ 6
Multiply. 6 x 9 hundredths = 54 hundredths. Subtract. 54 hundredths − 54 hundredths = 0 hundredths
Check. 54 hundredths shared among 6 groups as 54 hundredths.
So, 1.49

Question 7.
5)\(\overline { 3.75 }\)
_____

Answer:
0.75

Explanation:
3.75 ÷ 5
3 one(s) cannot be shared among 5 groups without regrouping.
30 + 7 = 37 tenths
Divide. 37 tenths ÷ 5
Multiply. 5 x 7 tenths = 35 tenths. Subtract. 37 tenths − 35 tenths = 2 tenths
Check. 35 tenths shared among 5 groups as 7 tenths.
2 tenths(s) cannot be shared among 5 groups without regrouping.
1 tenth = 10 hundredths.
20 + 5 = 25 hundredths
Divide. 25 hundredths ÷ 5
Multiply. 5 x 5 hundredths = 25 hundredths. Subtract. 25 hundredths − 25 hundredths = 0 hundredths
Check. 25 hundredths shared among 5 groups as 25 hundredths.
So, 0.75

Question 8.
19)\(\overline { 55.1 }\)
_____

Answer:
2.9

Explanation:
55.1 ÷ 19
Divide. 55 ones ÷ 19
Multiply. 19 × 2 ones = 38 ones. Subtract. 55 ones − 38 ones = 17 ones
Check. 38 ones shared among 19 groups as 2 whole.
17 one(s) cannot be shared among 19 groups without regrouping.
170 + 1 = 171 tenths
Divide. 171 tenths ÷ 19
Multiply. 19 x 9 tenths = 171 tenths. Subtract. 171 tenths − 171 tenths = 0 tenths
Check. 171 tenths shared among 19 groups as 9 tenths.
So, 2.9

Question 9.
23)\(\overline { 52.9 }\)
_____

Answer:
2.3

Explanation:
52.9 ÷ 23
Divide. 52 ones ÷ 23
Multiply. 23 × 2 ones = 46 ones. Subtract. 52 ones − 46 ones = 6 ones
Check. 46 ones shared among 23 groups as 2 whole.
6 one(s) cannot be shared among 23 groups without regrouping.
60 + 9 = 69 tenths
Divide. 69 tenths ÷ 23
Multiply. 23 x 3 tenths = 69 tenths. Subtract. 69 tenths − 69 tenths = 0 tenths
Check. 69 tenths shared among 23 groups as 3 tenths.
So, 2.3

Question 10.
8)\(\overline { $8.24 }\)
$ _____

Answer:

Explanation:
8.24 ÷ 8
Divide. 8 ones ÷ 8
Multiply. 8 × 1 ones = 8 ones. Subtract. 8 ones − 8 ones = 0 ones
Check. 8 ones shared among 8 groups as 1 whole.
2 tenths(s) cannot be shared among 8 groups without regrouping.
1 tenth = 10 hundredths.
20 + 4 = 24 hundredths
Divide. 24 hundredths ÷ 8
Multiply. 8 x 3 hundredths = 24 hundredths. Subtract. 24 hundredths − 24 hundredths = 0 hundredths
Check. 24 hundredths shared among 8 groups as 3 hundredths.
So, 1.03

Question 11.
5)\(\overline { 44.5 }\)
_____

Answer:

Explanation:
44.5 ÷ 5
Divide. 44 ones ÷ 5
Multiply. 5 x 8 ones = 40 ones. Subtract. 44 ones − 40 ones = 4 ones
Check. 40 ones shared among 5 groups as 8 wholes.
4 one(s) cannot be shared among 5 groups without regrouping.
40 + 5 = 45 tenths
Divide. 45 tenths ÷ 5
Multiply. 5 x 9 tenths = 45 tenths. Subtract. 45 tenths − 45 tenths = 0 tenths
Check. 45 tenths shared among 5 groups as 9 tenths.
So, 8.9

Practice: Copy and Solve Divide.

Question 12.
3)\(\overline { $7.71 }\)
$_____

Answer:
2.57

Explanation:
7.71 ÷ 3
Divide. 7 ones ÷ 3
Multiply. 3 x 2 ones = 6 ones. Subtract. 7 ones − 6 ones = 1 ones
Check. 6 ones shared among 3 groups as 2 wholes.
1 one(s) cannot be shared among 3 groups without regrouping.
10 + 7 = 17 tenths
Divide. 17 tenths ÷ 3
Multiply. 3 x 5 tenths = 15 tenths. Subtract. 17 tenths − 15 tenths = 2 tenths
Check. 15 tenths shared among 3 groups as 5 tenths.
2 tenths(s) cannot be shared among 3 groups without regrouping.
1 tenth = 10 hundredths.
20 + 1 = 21 hundredths
Divide. 21 hundredths ÷ 3
Multiply. 3 x 7 hundredths = 21 hundredths. Subtract. 21 hundredths − 21 hundredths = 0 hundredths
Check. 21 hundredths shared among 3 groups as 7 hundredths.
So, 2.57

Question 13.
14)\(\overline { 79.8 }\)
_____

Answer:
5.7

Explanation:
79.8 ÷ 14
Divide. 79 ones ÷ 14
Multiply. 14 x 5 ones = 70 ones. Subtract. 79 ones − 70 ones = 9 ones
Check. 70 ones shared among 14 groups as 5 wholes.
9 one(s) cannot be shared among 14 groups without regrouping.
90 + 8 = 98 tenths
Divide. 98 tenths ÷ 14
Multiply. 14 x 7 tenths = 98 tenths. Subtract. 98 tenths − 98 tenths = 0 tenths
Check. 98 tenths shared among 14 groups as 7 tenths.
So, 5.7

Question 14.
33)\(\overline { 25.41 }\)
_____

Answer:
0.77

Explanation:
25.41 ÷ 33
Divide. 25 ones ÷ 33
25 one(s) cannot be shared among 33 groups without regrouping.
250 + 4 = 254 tenths
Divide. 254 tenths ÷ 33
Multiply. 33 x 7 tenths = 231 tenths. Subtract. 254 tenths − 231 tenths = 23 tenths
Check. 231 tenths shared among 33 groups as 7 tenths.
23 tenths(s) cannot be shared among 33 groups without regrouping.
1 tenth = 10 hundredths.
230 + 1 = 231 hundredths
Divide. 231 hundredths ÷ 33
Multiply. 33 x 7 hundredths= 231 hundredths. Subtract. 231 hundredths− 231 hundredths = 0 hundredths
Check. 231 tenths shared among 33 groups as 7 tenths.
So, 0.77

Question 15.
7)\(\overline { 15.61 }\)
_____

Answer:
2.23

Explanation:
15.61 ÷ 7
Divide. 15 ones ÷ 7
Multiply. 7 x 2 ones =  14 ones. Subtract. 15 ones − 14 ones = 1 ones
Check. 14 ones shared among 7 groups as 2 wholes.
1 one(s) cannot be shared among 7 groups without regrouping.
10 + 6 = 16 tenths
Divide. 16 tenths ÷ 7
Multiply. 7 x 2 tenths = 14 tenths. Subtract. 16 tenths − 14 tenths = 2 tenths
Check. 14 tenths shared among 7 groups as 2 tenths.
2 tenths(s) cannot be shared among 7 groups without regrouping.
1 tenth = 10 hundredths.
20 + 1 = 21 hundredths
Divide. 21 hundredths ÷ 7
Multiply. 7 x 3 hundredths = 21 hundredths. Subtract. 21 hundredths − 21 hundredths = 0 hundredths
Check. 21 hundredths shared among 7 groups as 3 hundredths.
So, 2.23

Question 16.
14)\(\overline { 137.2 }\)
_____

Answer:
9.8

Explanation:
137.2 ÷ 14
Divide. 137 ones ÷ 14
Multiply. 14 x 9 ones =  126 ones. Subtract. 137 ones − 126 ones = 11 ones
Check. 126 ones shared among 14 groups as 9 wholes.
11 one(s) cannot be shared among 14 groups without regrouping.
110 + 2 = 112 tenths
Divide. 112 tenths ÷ 14
Multiply. 14 x 8 tenths = 112 tenths. Subtract. 112 tenths − 112 tenths = 0 tenths
Check. 112 tenths shared among 14 groups as 8 tenths.
So, 9.8

Question 17.
34)\(\overline { 523.6 }\)
_____

Answer:
15.4

Explanation:
523.6 ÷ 34
Divide. 523 ones ÷ 34
Multiply. 34 x 15 ones = 510 ones. Subtract. 523 ones − 510 ones = 13 ones
Check. 510 ones shared among 34 groups as 15 wholes.
13 one(s) cannot be shared among 34 groups without regrouping.
130 + 6 = 136 tenths
Divide. 136 tenths ÷ 34
Multiply. 34 x 4 tenths = 136 tenths. Subtract. 136 tenths − 136 tenths = 0 tenths
Check. 136 tenths shared among 34 groups as 4 tenths.
So, 15.4

Algebra Write the unknown number for each ■.

Question 18.
■ ÷ 5 = 1.21
■ = _____

Answer:
6.05

Explanation:
■ ÷ 5 = 1.21
■ = 1.21 x 5
■ = 6.05

Question 19.
46.8 ÷ 1.2 = ■
■ = _____

Answer:
39

Explanation:
46.8 ÷ 1.2 = ■
39 = ■

Question 20.
34.1 ÷ ■ = 22
■ = _____

Answer:
1.55

Explanation:
34.1 ÷ ■ = 22
■ = 34.1 ÷ 22
■ = 1.55

UNLOCK the Problem – Page No. 216

Question 21.
The standard width of 8 lanes in swimming pools used for competitions is 21.92 meters. The standard width of 9 lanes is 21.96 meters. How much wider is each lane when there are 8 lanes than when there are 9 lanes?
Go Math Grade 5 Answer Key Chapter 5 Divide Decimals img 7
a. What are you asked to find?
Type below:
_________

Answer:
We have to find how much wider is each line When there are 8 lanes than when there are 9 lanes.

Question 21.
b. What operations will you use to solve the problem?
Type below:
_________

Answer:
Division
Subtraction

Question 21.
c. Show the steps you used to solve the problem.
Type below:
_________

Answer:
21.92 ÷ 8 = 2.74 meters
21.96 ÷ 9 = 2.44 meters
2.74 – 2.44 = 0.30 meters

Question 21.
d. Complete the sentences.
Each lane is ______ meters wide when there are 8 lanes.
Each lane is ______ meters wide when there are 9 lanes.
Since ______ – ______ = ______ , the lanes are ______ meter(s) wider when there are 8 lanes than when there are 9 lanes.
Type below:
_________

Answer:
Each lane is 2.74 meters wide when there are 8 lanes.
Each lane is 2.44 meters wide when there are 9 lanes.
Since 2.74 – 2.44 = 0.30, the lanes are 0.30 meter(s) wider when there are 8 lanes than when there are 9 lanes.

Question 21.
e. Fill in the bubble for the correct answer choice.
Options:
a. 0.30 meter
b. 2.44 meters
c. 2.74 meters
d. 22.28 meters

Answer:
a. 0.30 meter

Question 22.
Robert pays $32.04 for 6 student tickets to the basketball game. What is the cost of each student ticket?
Options:
a. $192.24
b. $53.40
c. $26.04
d. $5.34

Answer:
d. $5.34

Explanation:
Robert pays $32.04 for 6 student tickets to the basketball game.
$32.04 ÷ 6 = $5.34

Question 23.
Jasmine uses 14.24 pounds of fruit for 16 servings of fruit salad. If each serving contains the same amount of fruit, how much fruit is in each serving?
Options:
a. 0.089 pound
b. 0.89 pound
c. 1.76 pounds
d. 17.6 pounds

Answer:
b. 0.89 pound

Explanation:
Jasmine uses 14.24 pounds of fruit for 16 servings of fruit salad. If each serving contains the same amount of fruit, 14.24 ÷ 16 = 0.89 pounds

Mid-Chapter Checkpoint – Page No. 217

Concepts and Skills

Question 1.
Explain how the position of the decimal point changes in a quotient as you divide by increasing powers of 10.
Type below:
_________

Answer:
Dividing a number by ten shifts the decimal point of the dividend one place to the left, making that into a smaller number, the desired quotient, which is one-tenth of the dividend.

Question 2.
Explain how you can use base-ten blocks to find 2.16 ÷ 3.
Type below:
_________

Answer:
0.72

Explanation:
2.16 ÷ 3
There are 2 ones, 1 tenths, and 6 hundredths.
2.16 ÷ 3 =
Chapter 5 grade 5 Divide Decimals 217 image 1
2 ones cannont be shared for 3 groups.
So, write 2 ones as 20 tenths.
20 + 1 = 21 tenths.
21 tenths ÷ 3 = 7 tenths.
7 tenths shared amount 3 groups = 21 tenths
6 hundredths ÷ 3 = 2 hundredths.
2 hundredths shared amount 3 groups = 6 hundredths.
0.72

Complete the pattern.

Question 3.
223 ÷ 1 = _______
223 ÷ 10 = _______
223 ÷ 100 = _______
223 ÷ 1000 = _______

Answer:
223 ÷ 1 = 223
223 ÷ 10 = 22.3
223 ÷ 100 = 2.23
223 ÷ 1000 = 0.223

Explanation:
223 ÷ 1 = 223
223 ÷ 10 = 22.3
223 ÷ 100 = 2.23
223 ÷ 1000 = 0.223

Question 4.
61 ÷ 1 = 61
61 ÷ 10 = 6.1
61 ÷ 100 = 0.61
61 ÷ 1,000 = 0.061

Answer:
61 ÷ 1 = 61
61 ÷ 10 = 6.1
61 ÷ 100 = 0.61
61 ÷ 1,000 = 0.061

Explanation:

Question 5.
57.4 ÷ 100 = _______
57.4 ÷ 101 = _______
57.4 ÷ 102 = _______

Answer:
57.4 ÷ 100 = 57.4
57.4 ÷ 101 = 5.74
57.4 ÷ 102 = 0.574

Explanation:
57.4 ÷ 100 = 57.4 ÷ 1 = 57.4
57.4 ÷ 101 = 57.4 ÷ 10 = 5.74
57.4 ÷ 102 = 57.4 ÷ 100 = 0.574

Estimate the quotient.

Question 6.
31.9 ÷ 4 = ______

Answer:
About 8

Explanation:
31.9 is closer to 32
32 ÷ 4 = 8

Question 7.
6.1 ÷ 8 = ______

Answer:
About 8

Explanation:
6.1 is closer to 64
64 ÷ 8 = 8

Question 8.
492.6 ÷ 48 = ______

Answer:
About 10

Explanation:
492.6 is closer to 480
480 ÷ 48 = 10

Divide.

Question 9.
5)\(\overline { 4.35 }\)
_____

Answer:
0.87

Explanation:
4.35 ÷ 5
Divide. 8 ones ÷ 5
4 one(s) cannot be shared among 5 groups without regrouping.
40 + 3 = 43 tenths
Divide. 43 tenths ÷ 5
Multiply. 5 x 8 tenths = 40 tenths. Subtract. 43 tenths − 40 tenths = 3 tenths
Check. 40 tenths shared among 5 groups as 8 tenths.
3 tenths(s) cannot be shared among 5 groups without regrouping.
1 tenth = 10 hundredths.
30 + 5 = 35 hundredths
Divide. 35 hundredths ÷ 5
Multiply. 5 x 7 hundredths = 35 hundredths. Subtract. 35 hundredths − 35 hundredths = 0 hundredths
Check. 35 hundredths shared among 5 groups as 7 hundredths.
So, 0.87

Question 10.
8)\(\overline { 9.92 }\)
_____

Answer:
1.24

Explanation:
9.92 ÷ 8
Divide. 9 ones ÷ 8
Multiply. 8 × 1 ones = 8 ones. Subtract. 9 ones − 8 ones = 1 ones
Check. 8 ones shared among 8 groups as 1 whole.
1 one(s) cannot be shared among 8 groups without regrouping.
10 + 9 = 19 tenths
Divide. 19 tenths ÷ 8
Multiply. 8 x 2 tenths = 16 tenths. Subtract. 19 tenths − 16 tenths = 3 tenths
Check. 16 tenths shared among 8 groups as 2 tenths.
3 tenths(s) cannot be shared among 8 groups without regrouping.
1 tenth = 10 hundredths.
30 + 2 = 32 hundredths
Divide. 32 hundredths ÷ 8
Multiply. 8 x 4 hundredths = 32 hundredths. Subtract. 32 hundredths − 32 hundredths = 0 hundredths
Check. 32 hundredths shared among 8 groups as 4 hundredths.
So, 1.24

Question 11.
61)\(\overline { 207.4 }\)
_____

Answer:
3.4

Explanation:
207.4 ÷ 61
Divide. 207 ones ÷ 61
Multiply. 61 x 3 ones = 183 ones. Subtract. 207 ones − 183 ones = 24 ones
Check. 183 ones shared among 61 groups as 3 wholes.
24 one(s) cannot be shared among 61 groups without regrouping.
240 + 4 = 244 tenths
Divide. 244 tenths ÷ 61
Multiply. 61 x 4 tenths = 244 tenths. Subtract. 244 tenths − 244 tenths = 0 tenths
Check. 244 tenths shared among 61 groups as 4 tenths.
So, 3.4

Mid-Chapter Checkpoint – Page No. 218

Question 12.
The Westside Bakery uses 440 pounds of flour to make 1,000 loaves of bread. Each loaf contains the same amount of flour. How many pounds of flour are used in each loaf of bread?
_____ pounds

Answer:
0.440 pounds

Explanation:
The Westside Bakery uses 440 pounds of flour to make 1,000 loaves of bread. Each loaf contains the same amount of flour.
440/1000 = 0.440 pounds

Question 13.
Elise pays $21.75 for 5 student tickets to the fair. What is the cost of each student ticket?
$ _____

Answer:
$4.35

Explanation:
Elise pays $21.75 for 5 student tickets to the fair.
$21.75/5 = $4.35

Question 14.
Jason has a piece of wire that is 62.4 inches long. He cuts the wire into 3 equal pieces. Estimate the length of 1 piece of wire.
_____ in

Answer:
20.8 inches

Explanation:
Jason has a piece of wire that is 62.4 inches long. He cuts the wire into 3 equal pieces.
62.4/3 = 20.8 inches

Question 15.
Elizabeth uses 23.25 ounces of granola and 10.5 ounces of raisins for 15 servings of trail mix. If each serving contains the same amount of trail mix, how much trail mix is in each serving?
_____ ounces

Answer:
2.25 ounces

Explanation:
Elizabeth uses 23.25 ounces of granola and 10.5 ounces of raisins for 15 servings of trail mix.
23.25 + 10.5 = 33.75
33.75/15 = 2.25 ounces

Share and Show – Page No. 221

Use the model to complete the number sentence.

Question 1.
1.2 ÷ 0.3 =
Go Math Grade 5 Answer Key Chapter 5 Divide Decimals img 8
_____

Answer:
1.2 ÷ 0.3 = 4

Explanation:
There are 4 groups of 3 tenths.
So, 1.2 ÷ 0.3 = 4

Question 2.
0.45 ÷ 0.09 =
Go Math Grade 5 Answer Key Chapter 5 Divide Decimals img 9
_____

Answer:
0.45 ÷ 0.09 = 5

Explanation:
There are 5 groups. Each group has 9 hundredths.
0.09 x 5 = 0.45
So, 0.45 ÷ 0.09 = 5

Question 3.
0.96 ÷ 0.24 =
Go Math Grade 5 Answer Key Chapter 5 Divide Decimals img 10
_____

Answer:
0.96 ÷ 0.24 = 4

Explanation:
There are 4 groups. Each group has 2 tenths and 4 hundredths. 24 hundredths
0.24 x 4 = 0.96
0.96 ÷ 0.24 = 4

Question 4.
1 ÷ 0.5 =
Go Math Grade 5 Answer Key Chapter 5 Divide Decimals img 11
_____

Answer:
2

Explanation:
There are 2 groups. Each group has 5 tenths.
0.5 x 2 = 1.0 = 1
1 ÷ 0.5 = 2

Divide. Use decimal models.

Question 5.
1.8 ÷ 0.6 =
_____

Answer:
3

Explanation:
1.8
There are 1 one(s) and 8 tenth(s).
Cut 1.8 to show groups of 0.6.
There are 3 groups of 1.8 tenths.
1.8 ÷ 0.6 = 3

Question 6.
1.2 ÷ 0.3 =
_____

Answer:
4

Explanation:
1.2
There are 1 one(s) and 2 tenth(s).
Cut 1.2 to show groups of 0.3.
There are 4 groups of 1.2 tenths.
1.2 ÷ 0.3 = 4

Question 7.
0.24 ÷ 0.04 =
_____

Answer:
6

Explanation:
0.24
There are 0 one(s) and 2 tenth(s) and 4 hundredth(s).
Cut 0.24 to show groups of 0.04.
There are 6 groups of 0.04 hundredths.
0.24 ÷ 0.04 = 6

Question 8.
1.75 ÷ 0.35 =
_____

Answer:
5

Explanation:
1.75
There are 1 one(s) and 7 tenth(s) and 5 hundredth(s).
Cut 1.75 to show groups of 0.35.
There are 5 groups of 1.75 hundredths.
1.75 ÷ 0.35 = 5

Question 9.
2 ÷ 0.4 =
_____

Answer:
5

Explanation:
2
There are 2 one(s).
Cut 2 to show groups of 0.4.
There are 5 groups of 2 ones.
2 ÷ 0.4 = 5

Question 10.
2.7 ÷ 0.9 =
_____

Answer:
3

Explanation:
2.7
There are 2 one(s) and 7 tenth(s).
Cut 2.7 to show groups of 0.9.
There are 3 groups of 2.7 tenths.
2.7 ÷ 0.9 = 3

Question 11.
1.24 ÷ 0.62 =
_____

Answer:
2

Explanation:
1.24
There are 1 one(s), 2 tenth(s) and 4 hundredth(s).
Cut 1.24 to show groups of 0.62.
There are 2 groups of 1.24 hundredths.
1.24 ÷ 0.62 = 2

Question 12.
0.84 ÷ 0.14 =
_____

Answer:
6

Explanation:
0.84
There are 0 one(s), 8 tenth(s) and 4 hundredth(s).
Cut 0.84 to show groups of 0.14.
There are 6 groups of 0.84 hundredths.
0.84 ÷ 0.14 = 6

Question 13.
1.6 ÷ 0.4 =
_____

Answer:
4

Explanation:
1.6
There are 1 one(s) and 6 tenth(s).
Cut 1.6 to show groups of 0.4.
There are 4 groups of 1.6 tenths.
1.6 ÷ 0.4 = 4

Use the model to find the unknown value.

Question 14.
2.4 ÷ □ = 3
Go Math Grade 5 Answer Key Chapter 5 Divide Decimals img 12
_____

Answer:
0.8

Explanation:
2.4 ÷ □ = 3
There are 3 groups with 8 tenths in each group.
So, 2.4 ÷ 0.8 = 3
The unknown number is 0.8

Question 15.
□ ÷ 0.32 = 4
Go Math Grade 5 Answer Key Chapter 5 Divide Decimals img 13
_____

Answer:
1.28

Explanation:
□ ÷ 0.32 = 4
There are 4 groups with 32 hundredths in each group.
So, 0.32 x 4 = 1.28
The unknown number is 1.28

Question 16.
Make a model to find 0.6 ÷ 0.15. Describe your model.
_____

Answer:
0.6 ÷ 0.15
Divide 6 tenths into 15 hundredths each group.
15 hundredths + 15 hundredths + 15 hundredths + 15 hundredths
4 groups.
0.6 ÷ 0.15 = 4

Question 17.
Explain, using the model, what the equation represents in Exercise 15.
Type below:
_________

Answer:
Using the decimal model, we have to find the value of 0.6 ÷ 0.15
0.6 ÷ 0.15 = 4

Problem Solving – Page No. 222

Pose a Problem
Go Math Grade 5 Answer Key Chapter 5 Divide Decimals img 14

Question 18.
Emilio buys 1.2 kilograms of grapes. He separates the grapes into packages that contain 0.3 kilogram of grapes each. How many packages of grapes does Emilio make?
Go Math Grade 5 Answer Key Chapter 5 Divide Decimals img 15
1.2 ÷ 0.3 = 4
Emilio made 4 packages of grapes.
Write a new problem using a different amount for the weight in each package. The amount should be a decimal with tenths. Use a total amount of 1.5 kilograms of grapes. Then use decimal models to solve your problem.
Pose a problem.                          Solve your problem. Draw a picture of the
model you used to solve your problem.
• Explain why you chose the amount you did for your problem.
Type below:
_________

Answer:
Emilio buys 1.5 kilograms of grapes. He separates the grapes into packages that contain 0.5 kilogram of grapes each.
1.5/0.5 = 3
3 groups

Share and Show – Page No. 225

Copy and complete the pattern.

Question 1.
45 ÷ 9 = _____
4.5 ÷ _____ = 5
_____ ÷ 0.09 = 5

Answer:
45 ÷ 9 = 5
4.5 ÷ 0.9 = 5
0.45 ÷ 0.09 = 5

Explanation:
45 ÷ 9 = 5
4.5 ÷ _____ = 5
45 ÷ 10 = 4.5. So, 9 ÷ 10 = 0.9
4.5 ÷ 0.9 = 5
_____ ÷ 0.09 = 5
0.9 ÷ 10 = 0.09
4.5 ÷ 10 = 0.45
0.45 ÷ 0.09 = 5

Question 2.
175 ÷ 25 = _____
17.5 ÷ _____ = 7
_____ ÷ 0.25 = 7

Answer:
175 ÷ 25 = 7
17.5 ÷ 2.5 = 7
1.75 ÷ 0.25 = 7

Explanation:
175 ÷ 25 = 7
17.5 ÷ _____ = 7
175 ÷ 10 = 17.5. So, 25 ÷ 10 = 2.5
17.5 ÷ 2.5 = 7
_____ ÷ 0.25 = 7
2.5 ÷ 10 = 0.25
17.5 ÷ 10 = 1.75
1.75 ÷ 0.25 = 7

Question 3.
164 ÷ 2 = _____
16.4 ÷ _____ = 82
_____ ÷ 0.02 = 82

Answer:
164 ÷ 2 = 82
16.4 ÷ 0.2 = 82
1.64 ÷ 0.02 = 82

Explanation:
164 ÷ 2 = 82
16.4 ÷ _____ = 82
164 ÷ 10 = 16.4
2 ÷ 10 = 0.2
16.4 ÷ 0.2 = 82
_____ ÷ 0.02 = 82
0.2 ÷ 10 = 0.02
16.4 ÷ 10 = 1.64
1.64 ÷ 0.02 = 82

Divide.

Question 4.
1.6)\(\overline { 9.6 }\) = _____

Answer:
6

Explanation:
9.6 ÷ 1.6
9.6 x 10 = 96
1.6 x 10 = 16
96 ÷ 16 = 6
So, 9.6 ÷ 1.6 = 6

Question 5.
0.3)\(\overline { 0.24 }\) = _____

Answer:
0.8

Explanation:
0.24 ÷ 0.3
0.24 x 10 = 2.4
0.3 x 10 = 3
2.4 ÷ 3 = 0.8
0.24 ÷ 0.3 = 0.8

Question 6.
3.45 ÷ 1.5 = _____

Answer:
2.3

Explanation:
3.45 ÷ 1.5
3.45 x 10 = 34.5
1.5 x 10 = 15
34.5 ÷ 15 = 2.3
3.45 ÷ 1.5 = 2.3

On Your Own

Divide.

Question 7.
0.6)\(\overline { 13.2 }\) = _____

Answer:
22

Explanation:
13.2 ÷ 0.6
13.2 x 10 = 132
0.6 x 10 = 6
132 ÷ 6 = 22
13.2 ÷ 0.6 = 22

Question 8.
0.3)\(\overline { 0.9 }\) = _____

Answer:
3

Explanation:
0.9 ÷ 0.3
0.9 x 10 = 9
0.3 x 10 = 3
9 ÷ 3 = 3
0.9 ÷ 0.3 = 3

Question 9.
0.26)\(\overline { 1.56 }\) = _____

Answer:
6

Explanation:
1.56 ÷ 0.26
1.56 x 100 = 156
0.26 x 100 = 26
156 ÷ 26 = 6
1.56 ÷ 0.26 = 6

Question 10.
0.45)\(\overline { 5.85 }\) = _____

Answer:
13

Explanation:
5.85 ÷ 0.45
5.85 x 100 = 585
0.45 x 100 = 45
585 ÷ 45 = 13

Question 11.
0.3)\(\overline { 0.69 }\) = _____

Answer:
2.3

Explanation:
0.69 ÷ 0.3
0.69 x 10 = 6.9
0.3 x 10 = 3
6.9 ÷ 3 = 2.3

Question 12.
3.6 ÷ 0.4 = _____

Answer:
9

Explanation:
3.6 ÷ 0.4
3.6 x 10 = 36
0.4 x 10 = 4
36 ÷ 4 = 9
3.6 ÷ 0.4 = 9

Question 13.
1.26 ÷ 2.1 = _____

Answer:
0.6

Explanation:
1.26 ÷ 2.1
1.26 x 10 = 12.6
2.1 x 10 = 21
12.6 ÷ 21 = 0.6

Question 14.
7.84 ÷ 0.28 = _____

Answer:
28

Explanation:
7.84 ÷ 0.28
7.84 x 100 = 784
0.28 x 100 = 28
784 ÷ 28 = 28
7.84 ÷ 0.28 = 28

Question 15.
9.28 ÷ 2.9 = _____

Answer:
3.2

Explanation:
9.28 ÷ 2.9
9.28 x 10 = 92.8
2.9 x 10 = 29
92.8 ÷ 29 = 3.2
9.28 ÷ 2.9 = 3.2

Problem Solving – Page No. 226

Use the table to solve 16—19.
Go Math Grade 5 Answer Key Chapter 5 Divide Decimals img 16

Question 16.
Connie paid $1.08 for pencils. How many pencils did she buy?
_____ pencils

Answer:
9 pencils

Explanation:
1 pencil = $0.12
Connie paid $1.08 for pencils.
$1.08 ÷ $0.12 = 9
she buys 9 pencils

Question 17.
Albert has $2.16. How many more pencils can he buy than markers?
_____

Answer:
Albert can buy 12 more pencils than markers

Explanation:
Albert has $2.16.
1 pencil = $0.12
$2.16/$0.12 = 18 pencils
1 marker = $0.36
$2.16/$0.36 = 6 markers
18 – 6 = 12 more pencils

Question 18.
How many erasers can Ayita buy for the same amount that she would pay for one notepad?
_____ erasers

Answer:
Ayita can buy 13 erasers with one notepad money.

Explanation:
1 notepad = $0.65
$0.65/$0.05 = 13
Ayita can buy 13 erasers with one notepad money.

Question 19.
Ramon paid $3.25 for notepads and $1.44 for markers. What is the total number of items he bought?
_____ items

Answer:
9 items

Explanation:
Ramon paid $3.25 for notepads.
$3.25/$0.65 = 5
He paid $1.44 for markers
$1.44/$0.36 = 4
5 + 4 = 9
He bought 9 items

Question 20.
What’s the Error? Katie divided 4.25 by 0.25 and got a quotient of 0.17.
_____

Answer:
Katie divided 4.25 by 0.25
4.25 ÷ 0.25
4.25 x 100 = 425
0.25 x 100 = 25
425 ÷ 25 = 17
4.25 ÷ 0.25 = 17
But he said 0.17

Question 21.
Test Prep Marcus bought apples that cost $0.45 per pound. He paid $1.35 for the apples. How many pounds of apples did he buy?
Options:
a. 0.3 pound
b. 2.8 pounds
c. 3 pounds
d. 30 pounds

Answer:
c. 3 pounds

Explanation:
Marcus bought apples that cost $0.45 per pound.
He paid $1.35 for the apples.
$1.35 ÷ $0.45 = 3 pounds

Share and Show – Page No. 229

Write the quotient with the decimal point placed correctly.

Question 1.
5 ÷ 0.8 = 625
_____

Answer:
5 ÷ 0.8 = 6.25

Explanation:
5 ÷ 0.8 = 5 ÷ 0.800
50 ÷ 8 = 6.25
Divide 5 ÷ 0.8 until you get remainder 0
Divide: 50 ÷ 8 = 6
8 x 6 = 48; Subtract: 50 – 48 = 2
Divide: 20 ÷ 8 = 2;
Subtract: 20 – 16 = 4
Divide: 40 ÷ 8 = 5
8 x 5 = 40; Subtract: 40 – 40 = 0
So, 6.25

Question 2.
26.1 ÷ 6 = 435
_____

Answer:

Explanation:
26.1 ÷ 6 = 26.1 ÷ 6.0
261 ÷ 60.0
Divide 26.1 ÷ 6 until you get remainder 0
Divide: 261 ÷ 60 = 4;
60 x 4 = 240 Subtract: 261 – 240 = 21
Divide: 210 ÷ 60 = 3
60 x 3 = 180; Subtract: 210 – 180 = 30
Divide: 300 ÷ 60 = 5;
60 x 5 = 300; Subtract: 300 – 300 = 0
261 ÷ 60 = 4.35

Question 3.
0.42 ÷ 0.35 = 12
_____

Answer:
1.2

Explanation:
0.42 ÷ 0.35 = 42 ÷ 35
Divide 0.42 ÷ 0.35 until you get remainder 0
Divide: 42 ÷ 35 = 1;
35 x 1 = 35 Subtract: 42 – 35 = 7
Divide: 70 ÷ 35 = 2
35 x 2 = 70; Subtract: 70 – 70 = 0
0.42 ÷ 0.35 = 1.2

Question 4.
80 ÷ 50 = 16
_____

Answer:
1.6

Explanation:
80 ÷ 50
8 ÷ 5
Divide 8 ÷ 5
5 x 1 = 5 : Subtract: 8 – 5 = 3
Divide 30 ÷ 5 = 6
80 ÷ 50 = 1.6

Divide.

Question 5.
4)\(\overline { 32.6 }\) = _____

Answer:
8.15

Explanation:
32.6 ÷ 4
Divide: 32/4 = 8
8 x 4 = 32; Subtract: 32 – 32 = 0
Divide: 6/4 = 1
4 x 1 = 4; Subtract: 6 – 4 = 2
Add zero
20 ÷ 4 = 5
So, 8.15

Question 6.
1.2)\(\overline { 9 }\) = _____

Answer:
0.1333

Explanation:
1.2/9 = 12/90; 12 < 90
12 cannot be divided by 90. So add zero to 12.
120/90 = 1; 120 – 90 = 30; 30 < 90
300/90 = 3; 300 – 270 = 30
So, 0.13333

Question 7.
15)\(\overline { 42 }\) = _____

Answer:
2.8

Explanation:
42/15 = 2
15 x 2 = 30; 42 – 30 = 12
120/15 = 8;
So, 2.8

Question 8.
0.14)\(\overline { 0.91 }\) = _____

Answer:
6.5

Explanation:
0.91/0.14 = 91/14
91/14 = 6
14 x 6 = 84; 91 – 84 = 7; 7 < 14
70/14 = 5
So, 6.5

On Your Own

Divide.

Question 9.
8)\(\overline { 84 }\) = _____

Answer:
10.5

Explanation:
84/8 = 10
8 x 10 = 80; 84 – 80 = 4;4 < 8
40/8 = 5
So, 10.5

Question 10.
2.5)\(\overline { 4 }\) = _____

Answer:

Explanation:
4/2.5 = 40/25
40/25 = 1;
40 – 25 = 15; 15 < 25
150/25 = 6
So, 1.6

Question 11.
5)\(\overline { 16.2 }\) = _____

Answer:
3.24

Explanation:
16.2/5 = 162/50
162/50 = 3
50 x 3 = 150; 162 – 150 = 12; 12 < 50
120/50 = 2
50 x 2 = 100; 120 – 100 = 20; 20 < 50
200/50 = 4
So, 3.24

Question 12.
0.6)\(\overline { 2.7 }\) = _____

Answer:
4.5

Explanation:
2.7/0.6 = 27/6
27/6 = 4
4 x 6 = 24; 27 – 24 = 3; 3 < 6
30/6 = 5
So, 4.5

Question 13.
18 ÷ 7.5 = _____

Answer:
2.4

Explanation:
18/7.5 = 180/75
180/75 = 2
75 x 2 = 150; 180 – 150 = 30; 30 < 75
300/75 = 4
So, 2.4

Question 14.
34.8 ÷ 24 = _____

Answer:
1.45

Explanation:
34.8/24 = 348/240
348/240 = 1;
240 x 1 = 240; 348 – 240 = 108 < 240
1080/240 = 4
240 x 4 = 960; 1080 – 960 = 120; 120 < 240
1200/240 = 5
So, 1.45

Question 15.
5.16 ÷ 0.24 = _____

Answer:
21.5

Explanation:
5.16/0.24 = 516/24
516/24 = 21
24 x 21 = 504: 516 – 504 = 12; 12 < 24
120/24 = 5;
So, 21.5

Question 16.
81 ÷ 18 = _____

Answer:
4.5

Explanation:
81/18 = 4
18 x 4 = 72; 81 – 72 = 9; 9 < 18
90/18 = 5
4.5

Practice: Copy and Solve Divide.

Question 17.
1.6)\(\overline { 20 }\) = _____

Answer:
12.5

Explanation:
20/1.6 = 200/16
200/16 = 12
16 x 12 = 192; 200 – 192 = 8; 8 < 16
80/16 = 5;
12.5

Question 18.
15)\(\overline { 4.8 }\) = _____

Answer:
0.32

Explanation:
4.8/15 = 48/150; 48 < 150
480/150 = 3
150 x 3 = 450; 480 – 450 = 30 < 150
300/150 = 2
0.32

Question 19.
0.54)\(\overline { 2.43 }\) = _____

Answer:
4.5

Explanation:
2.43/0.54 = 243/54
243/54 = 4
54 x 4 = 216; 243 – 216 = 27 < 54
270/54 = 5
4.5

Question 20.
28)\(\overline { 98 }\) = _____

Answer:
3.5

Explanation:
98/28 = 3
28 x 3 = 84; 98 – 84 = 14 < 28
140/28 = 5
3.5

Question 21.
1.8 ÷ 12 = _____

Answer:
0.15

Explanation:
1.8/12 = 18/120; 18 < 120
180/120 = 1;
120 x 1 = 120; 180 – 120 = 60; 60 < 120
600/120 = 5
0.15

Question 22.
3.5 ÷ 2.5 = _____

Answer:
1.4

Explanation:
3.5/2.5 = 35/25
35/25 = 1;
25 x 1 = 25; 35 – 25 = 10; 10 < 25
100/25 = 4
1.4

Question 23.
40 ÷ 16 = _____

Answer:
2.5

Explanation:
40/16 = 2
16 x 2 = 32; 40 – 32 = 8 < 16
80/16 = 5
2.5

Question 24.
2.24 ÷ 0.35 = _____

Answer:
6.4

Explanation:
2.24/0.35 = 224/35
224/35 = 6
35 x 6 = 210; 224 – 210 = 14 < 35
140/35 = 4
6.4

Problem Solving – Page No. 230

Solve.

Question 25.
Jerry takes trail mix on hikes. A package of dried apricots weighs 25.5 ounces. Jerry divides the apricots equally among 6 bags of trail mix. How many ounces of apricots are in each bag?
_____ ounces

Answer:
4.25 ounces

Explanation:
Jerry takes trail mix on hikes. A package of dried apricots weighs 25.5 ounces. Jerry divides the apricots equally among 6 bags of trail mix.
25.5/6 = 4.25 ounces

Question 26.
Amy has 3 pounds of raisins. She divides the raisins equally into 12 bags. How many pounds of raisins are in each bag? Tell how many zeros you had to write at the end of the dividend.
_____ pound

Answer:
4 pound

Explanation:
Amy has 3 pounds of raisins. She divides the raisins equally into 12 bags.
12/3 = 4 pound. There are no zeros required at the end of the dividend.

Question 27.
Find 65 ÷ 4. Write your answer using a remainder, a fraction, and a decimal. Then tell which form of the answer you prefer. Explain your choice.
Type below:
_________

Answer:
65 ÷ 4 = 16 and 1 remainder.
Fraction = 65/4
65/4 = 16.25
The decimal answer is best to choose as there are no mathematical operations required.

Question 28.
Test Prep Todd has a piece of rope that is 1.6 meters long. He cuts the rope into 5 equal pieces. What is the length of each piece?
Options:
a. 0.8 meter
b. 0.32 meter
c. 3.2 meters
d. 8 meters

Answer:
b. 0.32 meter

Explanation:
Todd has a piece of rope that is 1.6 meters long. He cuts the rope into 5 equal pieces.
1.6/5 = 0.32 meter

Connect to Science

Rate of Speed Formula

The formula for velocity, or rate of speed, is r = d ÷ t, where r represents rate of speed, d represents distance, and t represents time. For example, if an object travels 12 feet in 10 seconds, you can find its rate of speed by using the formula.
r = d ÷ t
r = 12 ÷ 10
r = 1.2 feet per second
Use division and the formula for rate of speed to solve.
Go Math Grade 5 Answer Key Chapter 5 Divide Decimals img 17

Question 29.
A car travels 168 miles in 3.2 hours. Find the car’s rate of speed in miles per hour.
_____ miles per hour

Answer:
52.5 miles per hour

Explanation:
168/3.2 = 52.5 miles per hour

Question 30.
A submarine travels 90 kilometers in 4 hours. Find the submarine’s rate of speed in kilometers per hour.
_____ kilometers per hour

Answer:
22.5 kilometers per hour

Explanation:
A submarine travels 90 kilometers in 4 hours.
For an hour, 90/4 = 22.5 kilometers per hour

Share and Show – Page No. 233

Question 1.
Hector spent $36.75 for 2 DVDs with the same cost. The sales tax was $2.15. Hector also used a coupon for $1.00 off his purchase. How much did each DVD cost?
First, make a flowchart to show the information and show how you would work backward.
Type below:
_________

Answer:
grade 5 Chapter 5 Divide Decimals 232 image 1
Question 1.
Then, work backward to find the cost of 2 DVDs.
Type below:
_________

Answer:
Price of the DVD is s
2s + $2.15 – $1.00 = $36.75
2s = $35.60

Question 1.
Finally, find the cost of one DVD.
So, each DVD costs ________ .
$ _____

Answer:
2s = $35.60
s = $35.60/2
s = $17.8

Question 2.
What if Hector spent $40.15 for the DVDs, the sales tax was $2.55, and he didn’t have a coupon? How much would each DVD cost?
$ _____

Answer:

Explanation:
If Hector spent $40.15 for the DVDs. The sales tax was $2.55, and he didn’t have a coupon
Price of the DVD is s
2s + $2.55 = $40.15
2s = $37.60
s = $18.8
Each DVD price =$18.8

Question 3.
Sophia spent $7.30 for school supplies. She spent $3.00 for a notebook and $1.75 for a pen. She also bought 3 large erasers. If each eraser had the same cost, how much did she spend for each eraser?
$ _____

Answer:
$0.85

Explanation:
Sophia spent $7.30 for school supplies. She spent $3.00 for a notebook and $1.75 for a pen.
$3.00 + $1.75 = $4.75
$7.30 – $4.75 = $2.55
If she bought 3 erasers, $2.55/3 = $0.85
Each eraser cost = $0.85

On Your Own – Page No. 234

Question 4.
The change from a gift purchase was $3.90. Each of 6 students donated an equal amount for the gift. How much change should each student receive?
$ _____

Answer:
$0.65

Explanation:
The change from a gift purchase was $3.90. Each of 6 students donated an equal amount for the gift.
$3.90/6 = $0.65

Question 5.
If you divide this mystery number by 4, add 8, and multiply by 3, you get 42. What is the mystery number?
_____

Answer:
24

Explanation:
Let the unknown number = s.
(s/4 + 8) x 3 = 42
s/4 + 8 = 42/3
s/4 + 8 = 14
s/4 = 6
s = 6 x 4 = 24
s = 24

Question 6.
A mail truck picks up two boxes of mail from the post office. The total weight of the boxes is 32 pounds. One box is 8 pounds heavier than the other box. How much does each box weigh?
Type below:
_________

Answer:
20 pounds.
other box = 12 pounds

Explanation:
If one box weight is s, the other box weight is 8 pounds heavier than the other box. s – 8.
Total weight of two boxes =32
s + s – 8 = 32
2s = 40
s = 20

Question 7.
Stacy buys 3 CDs in a set for $29.98. She saved $6.44 by buying the set instead of buying the individual CDs. If each CD costs the same amount, how much does each of the 3 CDs cost when purchased individually?
$ _____

Answer:
$12.14

Explanation:
Stacy buys 3 CDs in a set for $29.98. She saved $6.44 by buying the set instead of buying the individual CDs.
Cost to buy 3 CDs = $29.98 + $6.44 = $36.42
$36.42/3 = $12.14

Question 8.
A school cafeteria sold 1,280 slices of pizza the first week, 640 the second week, and 320 the third week. If this pattern continues, in what week will the cafeteria sell 40 slices? Explain how you got your answer.
_____ th week

Answer:
sixth week

Explanation:
A school cafeteria sold 1,280 slices of pizza the first week, 640 the second week, and 320 the third week.
1280, 640, 320
1280/2 = 640
640/2 = 320
320/2 = 160
160/2 = 80
80/2 = 40
6th term shows 40. So, the answer is the sixth week.

Question 9.
Test Prep While working at the school store, John sold a jacket for $40.00 and notebooks for $1.50 each. If he collected $92.50, how many notebooks did he sell?
Options:
a. 3.5
b. 6.1
c. 35
d. 61

Answer:
c. 35

Explanation:
John sold a jacket for $40.00 and notebooks for $1.50 each.
$40 + $1.50S = $92.50
$1.50S = $52.5
S = $52.5/$1.50 = 35

Chapter Review/Test – Page No. 235

Concepts and Skills

Complete the pattern.

Question 1.
341 ÷ 1 = ________
341 ÷ 10 = ________
341 ÷ 100 = ________
341 ÷ 1,000 = ________

Answer:
341 ÷ 1 = 341
341 ÷ 10 = 34.1
341 ÷ 100 = 3.41
341 ÷ 1,000 = 0.341

Explanation:
341 ÷ 1 = 341
341 ÷ 10 = 34.1
341 ÷ 100 = 3.41
341 ÷ 1,000 = 0.341

Question 2.
15 ÷ 1 = ________
15 ÷ 10 = ________
15 ÷ 100 = ________
15 ÷ 1,000 = ________

Answer:
15 ÷ 1 = 15
15 ÷ 10 = 1.5
15 ÷ 100 = 0.15
15 ÷ 1,000 = 0.015

Explanation:
15 ÷ 1 = 15
15 ÷ 10 = 1.5
15 ÷ 100 = 0.15
15 ÷ 1,000 = 0.015

Question 3.
68.2 ÷ 100 = ________
68.2 ÷ 101 = ________
68.2 ÷ 102 = ________

Answer:
68.2 ÷ 100 = 68.2
68.2 ÷ 101 = 6.82
68.2 ÷ 102 = 0.682

Explanation:
68.2 ÷ 100 = 68.2 ÷ 1 = 68.2
68.2 ÷ 101 = 68.2 ÷ 10 = 6.82
68.2 ÷ 102 = 68.2 ÷ 100 = 0.682

Estimate the quotient.

Question 4.
49.3 ÷ 6 = _____

Answer:
About 8

Explanation:
49.3 is closer to 48
48 ÷ 6 = 8

Question 5.
3.5 ÷ 4 = _____

Answer:
about 1

Explanation:
3.5 is closer to 4
4 ÷ 4 = 1

Question 6.
396.5 ÷ 18 = _____

Answer:
About 22

Explanation:
396.5 is closer to 396
396 ÷ 18 = 22

Divide.

Question 7.
6)\(\overline { 3.24 }\) = _____

Answer:
0.54

Explanation:
3.24 ÷ 6
Divide. 3 ones ÷ 6
3 one(s) cannot be shared among 6 groups without regrouping.
30 + 2 = 32 tenths
Divide. 32 tenths ÷ 6
Multiply. 6 x 5 tenths = 30 tenths. Subtract. 32 tenths − 30 tenths = 2 tenths
Check. 30 tenths shared among 6 groups as 5 tenths.
2 tenths(s) cannot be shared among 6 groups without regrouping.
1 tenth = 10 hundredths.
20 + 4 = 24 hundredths
Divide. 24 hundredths ÷ 6
Multiply. 6 x 4 hundredths = 24 hundredths. Subtract. 24 hundredths − 24 hundredths = 0 hundredths
Check. 24 hundredths shared among 6 groups as 4 hundredths.
So, 0.54

Question 8.
5)\(\overline { 6.55 }\) = _____

Answer:
1.51

Explanation:
6.55 ÷ 5
Divide. 6 ones ÷ 5
Multiply. 5 × 1 ones = 5 ones. Subtract. 6 ones − 5 ones = 1 ones
Check. 5 ones shared among 5 groups as 1 whole.
1 one(s) cannot be shared among 5 groups without regrouping.
10 + 5 = 15 tenths
Divide. 15 tenths ÷ 5
Multiply. 5 x 3 tenths = 15 tenths. Subtract. 15 tenths − 15 tenths = 0 tenths
Check. 35 tenths shared among 5 groups as 3 tenths.
Divide. 5 hundredths ÷ 5
Multiply. 5 x 1 hundredths = 5 hundredths. Subtract. 5 hundredths − 5 hundredths = 0 hundredths
Check. 5 hundredths shared among 5 groups as 1 hundredth.
So, 1.51

Question 9.
26)\(\overline { 96.2 }\) = _____

Answer:
3.7

Explanation:
96.2 ÷ 26
Divide. 96 ones ÷ 26
Multiply. 26 × 3 ones = 78 ones. Subtract. 96 ones − 78 ones = 18 ones
Check. 78 ones shared among 26 groups as 3 wholes.
18 one(s) cannot be shared among 26 groups without regrouping.
180 + 2 = 182 tenths
Divide. 182 tenths ÷ 26
Multiply. 26 x 7 tenths = 182 tenths. Subtract. 182 tenths − 182 tenths = 0 tenths
So, 3.7

Question 10.
1.08 ÷ 0.4 = _____

Answer:
2.7

Explanation:
1.08 ÷ 0.4
1.08 x 10 = 10.8
0.4 x 10 = 4
10.8 ÷ 4 = 2.7

Question 11.
8.84 ÷ 0.68 = _____

Answer:
13

Explanation:
8.84 ÷ 0.68
8.84 x 100 = 884
0.68 x 100 = 68
884 ÷ 68 = 13

Question 12.
7.31 ÷ 1.7 = _____

Answer:
4.3

Explanation:
7.31 ÷ 1.7
7.31 x 10 = 73.1
1.7 x 10 = 17
73.1 ÷ 17 = 4.3

Question 13.
9.18 ÷ 0.9 = _____

Answer:
10.2

Explanation:
9.18 ÷ 0.9
9.18 x 10 = 91.8
0.9 x 10 = 9
91.8 ÷ 9 = 10.2

Question 14.
12.7 ÷ 5 = _____

Answer:
2.54

Explanation:
12.7 ÷ 5
12.7 x 10 = 127
5 x 10 = 50
127 ÷ 50 = 2.54

Question 15.
8.33 ÷ 0.34 = _____

Answer:
24.5

Explanation:
8.33 ÷ 0.34
8.33 x 100 = 833
0.34 x 100 = 34
833 ÷ 34 = 24.5

Chapter Review/Test – Page No. 236

Fill in the bubble completely to show your answer.

Question 16.
The Orchard Pie Company uses 95 pounds of apples to make 100 pies. Each pie contains the same amount of apples. How many pounds of apples are used in each pie?
Options:
A. 0.095 pound
B. 0.95 pound
C. 9.5 pounds
D. 95 pounds

Answer:
B. 0.95 pound

Explanation:
The Orchard Pie Company uses 95 pounds of apples to make 100 pies. Each pie contains the same amount of apples.
95/100 = 0.95 pounds

Question 17.
During a special sale, all CDs have the same price. Mr. Ortiz pays $228.85 for 23 CDs. Which is the best estimate of the price of each CD?
Options:
A. $9
B. $10
C. $12
D. $13

Answer:
B. $10

Explanation:
During a special sale, all CDs have the same price. Mr. Ortiz pays $228.85 for 23 CDs.
$228.85/23 = $9.95

Question 18.
Ryan earns $20.16 working for 3 hours. How much does he earn per hour?
Options:
A. $60.48
B. $6.82
C. $6.72
D. $6.71

Answer:
C. $6.72

Explanation:
Ryan earns $20.16 working for 3 hours.
$20.16/3 = $6.72

Question 19.
Anna hikes 6.4 miles during a 4-day vacation. If she hikes the same distance each day, how many miles does she hike each day?
Options:
A. 1.06 miles
B. 1.1 miles
C. 1.4 miles
D. 1.6 miles

Answer:
D. 1.6 miles

Explanation:
Anna hikes 6.4 miles during a 4-day vacation. If she hikes the same distance each day, 6.4/4 = 1.6 miles

Chapter Review/Test – Page No. 237

Fill in the bubble completely to show your answer.

Question 20.
Karina pays $1.92 for pencil erasers. The erasers cost $0.08 each. How many erasers does she buy?
Options:
A. 2.4
B. 2.5
C. 24
D. 25

Answer:
C. 24

Explanation:
Karina pays $1.92 for pencil erasers. The erasers cost $0.08 each.
$1.92/$0.08 = 24

Question 21.
Wyatt has 25.4 ounces of fruit juice. He divides the juice equally into 4 glasses. How much juice is in each glass?
Options:
A. 6 ounces
B. 6.35 ounces
C. 6.4 ounces
D. 6.45 ounces

Answer:
B. 6.35 ounces

Explanation:
Wyatt has 25.4 ounces of fruit juice. He divides the juice equally into 4 glasses.
25.4/4 = 6.35 ounces

Question 22.
Jacob walks 70.4 feet in 0.2 hour. If he walks at the same rate the whole time, what is his speed in feet per hour?
Options:
A. 352 feet per hour
B. 140.8 feet per hour
C. 35.2 feet per hour
D. 14.08 feet per hour

Answer:
A. 352 feet per hour

Explanation:
Jacob walks 70.4 feet in 0.2 hour. If he walks at the same rate the whole time,
7.4/0.2 = 352 feet per hour

Question 23.
Meghan earns $20.00 by walking dogs. She uses all of her earnings to buy a shirt for $12.85 and some stickers for $0.65 each. How many stickers does she buy?
Options:
A. 4.65
B. 11
C. 46
D. 110

Answer:
B. 11

Explanation:
Meghan earns $20.00 by walking dogs. She uses all of her earnings to buy a shirt for $12.85 and some stickers for $0.65 each.
$20 – $12.85 = $7.15
$7.15/$0.65 = 11

Chapter Review/Test – Page No. 238

Constructed Response

Question 24.
Percy buys tomatoes that cost $0.58 per pound. He pays $2.03 for the tomatoes. How many pounds of tomatoes does he buy? Show your work using words, pictures, or numbers. Explain how you know your answer is reasonable.
_____ pounds

Answer:
3.5 pounds

Explanation:
Percy buys tomatoes that cost $0.58 per pound. He pays $2.03 for the tomatoes.
$2.03/$0.58
200/50 = 4; Using Compitable numbers
Multiply the divisor by a power of 10 to make it a whole number. Then, multiply the dividend by the same power of 10.
0.58 x 100 = 58
2.03 x 100 = 203
203/58
Divide until you have an amour less than the divisor left over.
Insert a decimal point and a zero at the end of the dividend.
Place a decimal point in the quotient above the decimal point in the dividend.
203/58 = 3.5 pounds

Performance Task

Question 25.
Isabella is buying art supplies. The table at the right shows the prices of the items she wants to buy.
A Isabella spends $2.25 on poster boards. How many poster boards does she buy?
Go Math Grade 5 Answer Key Chapter 5 Divide Decimals Chapter Review/Test img 18
_____ posters

Answer:
3 posters

Explanation:
1 Poster Board = $0.75.
Isabella spends $2.25 on poster boards.
$2.25/$0.75 = 3

Question 25.
B Isabella spends $4.87 on paintbrushes and paint. How many of each item does she buy? Explain how you found your answer.
_______ paint brushes
_______ jars of paint

Answer:
2 paint brushes
3 jars of paint

Explanation:
paintbrushes = $0.95
Paint = $0.99
If she buys 2 paint brushes and 2 paints, she spent $1.9 for paintbrushes and $1.98 for 2 paints. The remaining amount is $0.99. So, she can buy one more paint with them. So, she can buy 2 paint brushes and 3 jars of paint.

Question 25.
C. Isabella spends less than $14.00 for glass beads, paintbrushes, poster board, and paint. She spends $1.68 on beads and $3.96 on paint. She buys more than 3 poster boards and more than 3 paintbrushes. Find how many ounces of glass beads and how many jars of paint she buys. Then, suggest the number of poster boards and paintbrushes she might buy for the total spent.
Type below:
_________

Answer:
Isabella spends less than $14.00 for glass beads, paintbrushes, poster board, and paint.
She spends $1.68 on beads and $3.96 on paint.
Each beads = $0.28
$1.68/$0.28 = 6 beads
Each paint = $0.99
$3.96/$0.99 = 4 paints
$14 – ($1.68 + $3.96) = $8.36.
So, $8.36 needs to spend on more than 3 poster boards and more than 3 paintbrushes.
So, Each poster board = $0.75
paintbrushes = $0.95
If she buys more than 3, 3 x $0.75 = $2.25
$0.95 x 3 = $2.85
$2.25 + $2.85 = $5.1
So, $8.36 – $5.1 = $3.26
If $0.75 x 3 = $2.25
$0.95 x 1 = $0.95
$2.25 + $0.95 = 3.2
So, she can buy 3 + 3 = 6 poster boards
and 3 + 1 = 4 paintbrushes.

Conclusion

We have given Go Math Grade 5 Answer Key Chapter 5 Divide Decimals here for free of cost. Learn the Simple Tricks and Tips to Solve Various Models on Divide Decimals. Step by Step Solutions is provided for all the Questions to make it easy for you to understand.

Go Math Grade 6 Answer Key Chapter 13 Variability and Data Distributions

go-math-grade-6-chapter-13-variability-and-data-distributions-answer-key

Go Math Grade 6 Answer Key Chapter 13 deals with Variability and Data Distributions. The HMH Go Math 6th Grade Answer Key is a very helpful resource for students to prepare for the exams. The solutions are mentioned topic-wise to all the questions for chapter 13 Variability and Data Distributions. You can understand the problem-solving methods in a better way by using Go Math Grade 6 Answer Key Chapter 13 Variability and Data Distributions for free.

Go Math Grade 6 Answer Key Chapter 13 Variability and Data Distributions

The main concepts of chapter 13 Variability and Data Distributions are discussed in the below sections. So, tap the links and practice the problems. This Go Math Grade 6 Answer Key Ch 13 Variability and Data Distributions helps you to secure the highest marks in the exams. For better performance, you try to compare the problems with real time.

Chapter 13 – Lesson: 1

Chapter 13 – Lesson: 2

Chapter 13 – Lesson: 3

Chapter 13 – Lesson: 4

Chapter – 13 – Mid-Chapter Checkpoint

Chapter 13 – Lesson: 5

Chapter 13 – Lesson: 5

Chapter 13 – Lesson: 6

Chapter 13 – Lesson: 7

Chapter 13 – Review/Test

Share and Show – Page No. 709

For 1–3, use the dot plot.

Question 1.
The dot plot shows the number of paintings students in the art club displayed at the art show. Does the dot plot contain any gaps?
If so, where?
Go Math Grade 6 Answer Key Chapter 13 Variability and Data Distributions img 1
Type below:
_________________

Answer: Between the intervals of 4 – 7 excluding 4 and 7

Explanation:
Go Math Grade 6 Answer Key Chapter 13 Variability and Data Distributions img 1
The dots are filled from 1 – 4 and 7 but the region between these two intervals is left unfilled so the region containing gaps is 5-6 including 5 and 6

Question 2.
Identify any clusters in the data.
Type below:
_________________

Answer: 1-4

Explanation:
A group of dots is called a cluster
The dots form a cluster at 1 – 4

Question 3.
Summarize the information in the dot plot.
Type below:
_________________

Answer: It says about the number of paintings done by each student in the art club.

Explanation:
The number of paintings is represented by the number line. The dots represent the students.
Therefore we can say that It says about the number of paintings done by each student in the art club.

On Your Own

Question 4.
What patterns do you see in the histogram data?
Go Math Grade 6 Answer Key Chapter 13 Variability and Data Distributions img 2
Type below:
_________________

Answer:

Explanation:
STEP 1 Identify any peaks in the data.
The histogram has 6 peaks.
The interval representing the greatest number of visitors is for ages between 60 and 69 age group.
STEP 2 The data changes across the intervals.
The number of visitors increases from 0 to 29 age group and from 40 to 69 age group.
So, the data values increase to one peak in the interval from 0 to 9 age group and then decrease.
The visitors of the age group 30 – 39 did not visit the zoo.

Question 5.
The dot plot shows the number of errors made by a baseball team in the first 16 games of the season. For numbers 5a-5e, choose Yes or No to indicate whether the statement is correct.
Go Math Grade 6 Answer Key Chapter 13 Variability and Data Distributions img 3
5a. There is a gap from 4 to 5.
5b. There is a peak at 0.
5c. The dot plot has symmetry.
5d. There are two modes.
5e. There is one cluster.
5a. __________
5b. __________
5c. __________
5d. __________
5e. __________

Answer:
5a. Yes
5b. Yes
5c. No
5d. No
5e. No

Explanation:
5a. There are dots between 4-5 so we can say that there is a gap from 4 to 5.
5b. The number of dots is more at the interval 0 So we can say that there is a peak at 0.
5c. The symmetrical view is nothing but having the same number of dots on both sides of the figure but we cannot observe it in the above figure. Therefore we can say that the dot plot has no symmetry.
5d. The most frequently occurring observation is known as a mode. One dot repeats in all the intervals so we can say that the mode is 1.
5e. A group of observations form a cluster, there are more than 1 group of dots in the figure given above.

Big Cats – Page No. 710

There are 41 species of cats living in the world today. Wild cats live in places as different as deserts and the cold forests of Siberia, and they come in many sizes. Siberian tigers may be as long as 9 feet and weigh over 2,000 pounds, while bobcats are often just 2 to 3 feet long and weigh between 15 and 30 pounds.

You can find bobcats in many zoos in the United States. The histogram below shows the weights of several bobcats. The weights are rounded to the nearest pound.
Go Math Grade 6 Answer Key Chapter 13 Variability and Data Distributions img 4
Use the histogram for 6 and 7.

Question 6.
Look for a Pattern Describe the overall shape of the histogram.
Type below:
_________________

Answer: The graph starts from a small interval and increases to the highest and then decreases to the smallest interval.
The histogram has rectangles which are closely packed.

Explanation:
STEP 1 Identify any peaks in the data.
The histogram has 1 peak(s).
The interval representing the greatest number of bobcats is for weights between 18 and 20 pounds.
STEP 2 Describe how the data changes across the intervals. The number of bobcats increases from 12 to 17 pounds and from 21 to 29 pounds.
STEP 3 Describe any symmetry the graph has. If I draw a vertical line through the interval for 18 to 20 pounds, the left and right parts of the histogram are very close to being mirror images. The histogram has line symmetry.

So, the data values increase to one peak in the interval for 18 to 20 pounds and then decrease. The data set has a vertical line
symmetry about the peak.

Question 7.
Sense or Nonsense? Sunny says that the graph might have a different shape if it was redrawn as a bar graph with one bar for each number of pounds. Is Sunny’s statement sense or nonsense? Explain.
Type below:
_________________

Answer: Sense

Explanation:
Bar graph also contains rectangles but they are not closely packed hence the statement is correct which is said by Sunny as a bar graph with one bar for each number of pounds.

Patterns in Data – Page No. 711

For 1–2, use the dot plot.
Go Math Grade 6 Answer Key Chapter 13 Variability and Data Distributions img 5

Question 1.
The dot plot shows the number of omelets ordered at Paul’s Restaurant each day. Does the dot plot contain any gaps?
Type below:
_________________

Answer: Yes, the dot plot contain gaps

Explanation:
The dots are filled from 10 – 11, from 14 – 16 and from 18 – 19 but the region between these two intervals is left unfilled so the region containing gaps is 12-13 including 12 and 13, 17 is also left unfilled.

Question 2.
Identify any clusters in the data.
Type below:
_________________

Answer: 14 – 16 and 18 – 19

Explanation:
A group of dots is called a cluster. The dots which form a cluster are 14 – 16 and from 18 – 19.

For 3–4, use the histogram.
Go Math Grade 6 Answer Key Chapter 13 Variability and Data Distributions img 6

Question 3.
The histogram shows the number of people that visited a local shop each day in January. How many peaks does the histogram have?
Type below:
_________________

Answer: The histogram has only one peak.

Explanation:
The rectangle with tall length represents the highest peak in the graph given above.
The number of people who visited a local shop each day in January were among 0 – 9  visitors and this was the highest frequency having 14 days.
The highest peak is in the interval of 0 – 9.

Question 4.
Describe how the data values change across the intervals.
Type below:
_________________

Answer: They decrease from highest to low values in the given picture above.

Explanation:
The graph represents the number of visitors in the month of January the visitors of number 0 – 9 have the highest frequency,
10 – 19 are the second-highest among the visitors who went to the local shop in the month of January, followed by 20 – 29,
30 – 39

Problem Solving

Question 5.
Look at the dot plot at the right. Does the graph have symmetry? Explain.
Go Math Grade 6 Answer Key Chapter 13 Variability and Data Distributions img 7
Type below:
_________________

Answer: Yes, the graph has a symmetry

Explanation:
If I draw a vertical line through the interval for _ to_ pounds, the left and right parts of the histogram are very close to being mirror images. The histogram __ line symmetry.
A geometric figure has line symmetry if you can draw a line through it so that the two parts are mirror images of each other.
So, the data values increase to one peak in the interval for _ to _ pounds and then decrease. The data set __ line symmetry about the peak.

Question 6.
A histogram that shows the ages of students at a library has intervals 1–5, 6–10, 11–15, 16–20, and 21–25. There is a peak at 11–15 years and the graph is symmetric. Sketch what the histogram could look like and describe the patterns you see in the data.
Type below:
_________________

Answer:

The histogram shows the ages of students at a library has intervals 1–5, 6–10, 11–15, 16–20, and 21–25. There is a peak at 11–15 years and the graph is symmetric.

Explanation:

The histogram is a graph with continuous rectangles which are closely packed.
The asymmetric graph is a graph which has a mirror-like view with equal rectangles on each side.
The graph with the highest peak represents the highest number of students who visit the library in that age group 11 – 15

Lesson Check – Page No. 712

Question 1.
What interval in the histogram has the greatest frequency?
Go Math Grade 6 Answer Key Chapter 13 Variability and Data Distributions img 8
Type below:
_________________

Answer: 10 – 14 interval has highest frequency of 6

Explanation:
The rectangle with a peak can be said as it has the greatest frequency. The interval with a peak is 11 – 15 and the frequency of the peak is 6

Question 2.
Meg makes a dot plot for the data 9, 9, 4, 5, 5, 3, 4, 5, 3, 8, 8, 5. Where does a gap occur?
Type below:
_________________

Answer: 6 – 7 including 6 and 7

Explanation:
Let us consider an axis with 3 to 9 numbers on it plot the dots as given in the question at the points 3,4,5,8 and 9 the gap occurs between 6 and 7 including 6 and 7.

Spiral Review

Question 3.
A rectangular fish tank is 20 inches long, 12 inches wide, and 20 inches tall. If the tank is filled halfway with water, how much water is in the tank?
________ in.

Answer: 37500 cubic centimeter

Explanation:
The length of the rectangle of the rectangular fish tank = 20 inches x 2.5 cm = 50 cm (since 1 inch = 2.5 cm)
The breadth of the rectangle of the rectangular fish tank = 12 inches x 2.5 cm = 30 cm (since 1 inch = 2.5 cm)
The height of the rectangle of the rectangular fish tank = 20 inches x 2.5 cm = 50 cm (since 1 inch = 2.5 cm)
Water filled in the tank = Volume of the tank = 50 x 50 x 30 = 75000 cubic centimeter
If the tank is filled halfway = volume of the tank / 2 = 37500 cubic centimeter

Question 4.
Look at the histogram below. How many students scored an 81 or higher on the math test?
________ students

Answer: 14

Explanation:
The interval 81 – 90 has 10 frequency and the interval 91 – 100 has 4 frequency. So the total number of students = 14

Question 5.
The Little League coach uses a radar gun to measure the speed of several of Kyle’s baseball pitches. The speeds, in miles per hour, are 52, 48, 63, 47, 47. What is the median of Kyle’s pitch speeds?
The median is ________ miles.

Answer: Median is 48

Explanation:
First, write the observations in ascending order or descending order.
The formula to calculate the median is (n+1/2)th observation, for odd number of observations
n = 5 (odd)
Median = (5 + 1 / 2) = (6/2) = 3rd observation = 48
Therefore the median is 48.

Share and Show – Page No. 715

Find the median, lower quartile, and upper quartile of the data.

Question 1.
the scores of 11 students on a geography quiz:
87, 72, 80, 95, 86, 80, 78, 92, 88, 76, 90
Type below:
_________________

Answer: Median: 86, lower quartile: 72, upper quartile: 95

Explanation:
First, write the observations in ascending order or descending order.
The formula to calculate the median is (n+1/2)th observation, for odd number of observations
n = 11 (odd)
Median = (11 + 1 / 2) = (12/2) = 6th observation = 86
Therefore the median is 86.

Lower quartile: 72  Upper quartile: 95

Question 2.
the lengths, in seconds, of 9 videos posted online:
50, 46, 51, 60, 62, 50, 65, 48, 53
Type below:
_________________

Answer: Median: 51 Lower quartile: 46 Upper quartile: 65

Explanation:
First, write the observations in ascending order or descending order.
The formula to calculate the median is (n+1/2)th observation, for odd number of observations
n = 9 (odd)
Median = (9 + 1 / 2) = (10/2) = 5th observation = 51
Therefore the median is 51.

Lower quartile: 46 Upper quartile: 65

Question 3.
Make a box plot to display the data set in Exercise 2.
Type below:
_________________

Answer: The box plot is drawn on the topic: Lengths of the videos (in seconds) posted in the online.

Explanation:
The box is drawn to understand the clear view of the raw data, in a precise manner.
This box gives us information about lengths of the videos posted in the online. We can directly say the median, lower quartile, upper quartile seeing the box plot.

On Your Own

Find the median, lower quartile, and upper quartile of the data.

Question 4.
13, 24, 37, 25, 56, 49, 43, 20, 24
Type below:
_________________

Answer: 25

Explanation:
First, write the observations in ascending order or descending order.
The formula to calculate the median is (n+1/2)th observation, for odd number of observations
n = 9 (odd)
Median = (9 + 1 / 2) = (10/2) = 5th observation =25
Therefore the median is 25.

Question 5.
61, 23, 49, 60, 83, 56, 51, 64, 84, 27
Type below:
_________________

Answer: 58

Explanation:
First, write the observations in ascending order or descending order.
The formula to calculate the median is mean of (n/2) and (n/2+1)th observations, for even number of observations
n = 10 (even)
Median = Mean of (5)th and (6)th observations = 56 + 60 divided by 2 = 116/2 = 58
Therefore the median is 58.

Question 6.
The chart shows the height of trees in a park. Display the data in a box plot.
Go Math Grade 6 Answer Key Chapter 13 Variability and Data Distributions img 9
Type below:
_________________

Answer:

Explanation:
Lower limit: 8
Upper limit: 30
Median:
First, write the observations in ascending order or descending order.
The formula to calculate the median is mean of (n/2) and (n/2+1)th observations, for even number of observations
n = 12 (even)
Median = Mean of (6)th and (7)th observations = 18 + 20 divided by 2 = 38/2 = 19
Therefore the median is 19.

Question 7.
Analyze Eric made this box plot for the data set below. Explain his error.
Go Math Grade 6 Answer Key Chapter 13 Variability and Data Distributions img 10
Go Math Grade 6 Answer Key Chapter 13 Variability and Data Distributions img 11
Type below:
_________________

Answer: The lower and upper limits are marked wrong.

Explanation:
The box drew above the number line is wrong.
It does not show the correct upper and lower limits.
The lower limit is 5 and the upper limit is 35.

Problem Solving + Applcations – Page No. 716

Pose a Problem

Question 8.
The box plots show the number of flights delayed per day for two different airlines. Which data set is more spread out?
Go Math Grade 6 Answer Key Chapter 13 Variability and Data Distributions img 12
Airline A: greatest value − least value = _____
Airline B: greatest value − least value = _____
So, the data for _____ is more spread out.
Write a new problem that can be solved using the data in the box plots.
Type below:
_________________

Answer:
Airline A: greatest value − least value = 8
Airline B: greatest value − least value = 10
The data for airline B is more spread out.

A problem which can be solved using the box plot can be:

Find the median, lower and upper limits.

Explanation:
The greatest value and lowest value can be identified by seeing the box drew above the number line. The ends represent the lower and upper limits in both the box plots.

The solution to the question framed:
The start and end of the rectangle represent the lower and upper limits. And the middle line represents the median.
The lower limit is 5
Upper limit is 35
Median:
First, write the observations in ascending order or descending order.
The formula to calculate the median is mean of (n/2) and (n/2+1)th observations, for even number of observations
n = 6 (even)
Median = Mean of (3)th and (4)th observations = 15 + 25 divided by 2 = 40/2 = 20
Therefore the median is 20.

Question 9.
The data set shows the cost of the dinner specials at a restaurant on Friday night.
Go Math Grade 6 Answer Key Chapter 13 Variability and Data Distributions img 13
The median is _____.
The lower quartile is _____.
The upper quartile is _____.

Answer:
Median: 24
The lower quartile is 16.
The upper quartile is 30.

Explanation:
Seeing the data in the box we can identify the lower and upper quartiles.
Median:
First, write the observations in ascending order or descending order.
The formula to calculate the median is (n+1/2)th observation, for odd number of observations
n = 11 (odd)
Median = (11 + 1 / 2) = (12/2) = 6th observation =24
Therefore the median is 24.

Box Plots – Page No. 717

Find the median, lower quartile, and upper quartile of the data.

Question 1.
the amounts of juice in 12 glasses, in fluid ounces:
11, 8, 4, 9, 12, 14, 9, 16, 15, 11, 10, 7
Type below:
_________________

Answer:
Median: 10.5
Lower quartile: 4
Upper quartile: 16

Explanation:
Median:
First, write the observations in ascending order or descending order.
The formula to calculate the median is mean of (n/2) and (n/2+1)th observations, for even number of observations
n = 12 (even)
Median = Mean of (6)th and (7)th observations = 10 + 11 divided by 2 = 21/2 = 10.5
Therefore the median is 10.5.
After writing in ascending or descending order the first and last terms justify the lower and upper limits respectively.
They are:
Lower quartile: 4
Upper quartile: 16

Question 2.
the lengths of 10 pencils, in centimeters:
18, 15, 4, 9, 14, 17, 16, 6, 8, 10
Type below:
_________________

Answer:
Median: 12
Lower quartile: 4
Upper quartile: 18

Explanation:
Median:
First, write the observations in ascending order or descending order.
The formula to calculate the median is mean of (n/2) and (n/2+1)th observations, for even number of observations
n = 10 (even)
Median = Mean of (5)th and (6)th observations = 10 + 14 divided by 2 = 24/2 = 12
Therefore the median is 12.
After writing in ascending or descending order the first and last terms justify the lower and upper limits respectively.
They are:
Lower quartile: 4
Upper quartile: 18

Question 3.
Make a box plot to display the data set in Exercise 2.
Type below:
_________________

Answer:

The above box plot represents the lower and upper quartiles, the median.

Explanation:
Box plot is drawn using the number line and the rectangle which is drawn above it.
The ends of the rectangles say about the lower and upper limits and the middle line indicates the median.

Question 4.
The numbers of students on several teams are 9, 4, 5, 10, 11, 9, 8, and 6. Make a box plot for the data.
Type below:
_________________

Answer:

Explanation:
Box plot is drawn using the number line and the rectangle which is drawn above it.
The ends of the rectangles say about the lower and upper limits and the middle line indicates the median.
Therefore the lower and upper quartiles are 4 and 11 respectively.
Median:
First, write the observations in ascending order or descending order.
The formula to calculate the median is mean of (n/2) and (n/2+1)th observations, for even number of observations
n = 8 (even)
Median = Mean of (4)th and (5)th observations = 8 + 9 divided by 2 = 17/2 = 8.5
Therefore the median is 8.5.

Problem Solving

Question 5.
The amounts spent at a gift shop today are $19, $30, $28, $22, $20, $26, and $26. What is the median? What is the lower quartile?
Type below:
_________________

Answer:
Median: $26
Lower quartile: $19
Upper quartile: $30

Explanation:
Median:
First, write the observations in ascending order or descending order.
The formula to calculate the median is (n+1/2)th observation, for odd number of observations
n = 7 (odd)
Median = (7 + 1 / 2) = (8/2) = 4th observation =26
Therefore the median is 26.
After writing in ascending or descending order the first and last terms justify the lower and upper limits respectively.
They are:
Lower quartile: $19
Upper quartile: $30

Question 6.
The weights of six puppies in ounces are 8, 5, 7, 5, 6, and 9. What is the upper quartile of the data?
Type below:
_________________

Answer: Upper quartile: 9

Explanation:
The highest value in the data is defined as the upper quartile.
The highest value in the raw data given is 9

Question 7.
Draw a box plot to display this data: 81, 22, 34, 55, 76, 20, 56.
Type below:
_________________

Answer:

Explanation:
A box plot gives information about the lower and upper quartiles and about the median.
The box plot is drawn using a rectangle and the number line.

Lesson Check – Page No. 718

Question 1.
The values in a data set are 15, 7, 11, 12, 6, 3, 10, and 6. Where would you draw the box in a box plot for the data?
Type below:
_________________

Answer: The box is drawn above the number line.

Explanation:
Example:

The rectangle which can be seen above the number line is the box plot which is drawn.
The box plot gives information about the lower and upper quartiles and about the median.

Question 2.
What is the lower quartile of the following data set?
22, 27, 14, 21, 22, 26, 18
Type below:
_________________

Answer: 14

Explanation:
The value which is lowest in the given data is called the lowest quartile.
Therefore the lowest quartile in the given data is 14.

Spiral Review

Question 3.
Jenn says that “What is the average number of school lunches bought per day?” is a statistical question. Lisa says that “How many lunches did Mark buy this week?” is a statistical question. Who is NOT correct?
Type below:
_________________

Answer: Lisa’s statement is wrong.

Explanation:

Question 4.
The prices of several chairs are $89, $76, $81, $91, $88, and $70. What is the mean of the chair prices?
The mean is $ _________

Answer: $82.5

Explanation:
Number of observations= 6
Mean of the observations= $89 + $76 + $81+ $91+$88+ $70/ 6= 495/6 = $82.5

Question 5.
By how much does the mean of the following data set change if the outlier is removed?
13, 19, 16, 40, 12
Type below:
_________________

Answer: The mean shows a difference if the lower limit is removed the mean increases and if the upper limit is removed the mean decreases.

Explanation:
Outliers are nothing but both upper and lower limits.
The actual mean is 20
But when the lower limit is removed the mean increases to 22 while when the upper limit is removed the mean decreases to 15.
Therefore, we can say that the mean shows a difference when the outliers are removed.

Question 6.
Where in the dot plot does a cluster occur?
Go Math Grade 6 Answer Key Chapter 13 Variability and Data Distributions img 14
Type below:
_________________

Answer: 52 – 54

Explanation:
A cluster is nothing but a group of dots.
In the intervals 52 – 54 a cluster has occurred.

Share and Show – Page No. 721

Use counters, a dot plot, or iTools to find the mean absolute deviation of the data.

Question 1.
Find the mean absolute deviation for both data sets. Explain which data set is more spread out.
the number of laps Shawna swam on 5 different days:
5, 6, 6, 8, 10
mean = 7

the number of laps Lara swam on 5 different days:
1, 3, 7, 11, 13
mean = 7
Type below:
_________________

Answer: Case 2 is more spread out.

Explanation:
CASE1
The number of laps Shawna swam on 5 different days:
5,6,6,8,10
Mean = 7
Deviations:
7 – 5 = 2
7 – 6 = 1
7 – 6 = 1
7 -8 = -1
7 -10=-3
Mean of deviations = 2+1+1+1+3/5 = 8/5 = 1.6

CASE2
The number of laps Lara swam on 5 different days:
1, 3, 7, 11, 13
Mean = 7
Deviations:
7 – 1 = 6
7 – 3 = 4
7 – 7 = 0
7 -11= -4
7 -13= -6
Mean of deviations = 6+ 4 + 0 + 4 + 6 / 5 = 20/5 = 4

Use the dot plot to find the mean absolute deviation of the data.

Question 2.
mean = 7 books
Go Math Grade 6 Answer Key Chapter 13 Variability and Data Distributions img 15
______ books

Answer: Mean absolute deviation is 2.4

Explanation:
STEP 1 Label each dot with its distance from the mean.
Starting from left to right:
4: 7-4=3
5: 7-5=2
6: 7-6=1
9: 7-9=-2
10: 7-10=-3
11: 7-11=-4

STEP 2 Find the mean of the distances.
(3) + (2) +(2) +(2) +(2) +(1) + (2) +(3) +(3) +(4) / 10 = 24/10 = 2.4

So, the mean absolute deviation of the data is 2.4

Question 3.
mean = 29 pounds
Go Math Grade 6 Answer Key Chapter 13 Variability and Data Distributions img 17
_______ pounds

Answer: Mean Absolute deviation is 3.2

Explanation:
STEP 1 Label each dot with its distance from the mean.
Starting from left to right:
26: 29-26=03
27: 29-27=02
32: 29-32=-3
33: 29-33=-4
35: 29-35=-6

STEP 2 Find the mean of the distances.
(3) + (2) +(3) +(4) +(6) +(3) + (3) +(2) / 8 = 26/8 = 3.2

So, the mean absolute deviation of the data is 3.2

Question 4.
The mean absolute deviation of the number of daily visits to Scott’s website for February is 167.7. In March, the absolute mean deviation is 235.9. In which month did the number of visits to Scott’s website vary more? Explain how you know.
Type below:
_________________

Answer: As the mean absolute deviation is more in the month of February we can say that there are more visitors in this month.

Explanation:
As the mean of the month of February is less it means that the number of observations are more.
Similarly, as the mean of the month of March is more it means that the number of observations are less.
Therefore we can say that the number of visitors were more in the month of February compared to March.

Question 5.
Write an Inequality Algebra In April, the data for Scott’s website visits are less spread out than they were in February. Use a to represent the mean absolute deviation for April. Write an inequality to describe the possible values of a.
Type below:
_________________

Answer: a < February

Explanation:
Since the data is more spread out in the month of April than they were in February. Therefore the inequality represents “less than” sign.

Problem Solving + Applcations – Page No. 722

Question 6.
Use the table.
Go Math Grade 6 Answer Key Chapter 13 Variability and Data Distributions img 18
The mean of the data is 11. What is the mean absolute deviation of the data?
_______ days

Answer: 3

Explanation:
STEP 1 Label each observation with its distance from the mean.
Starting from left to right:
10: 11-10= 1
12: 11-12=-1
13: 11-13=-2
18: 11-18=-7
10: 11-10= 1
08: 11-08= 3
07: 11-07= 4
06: 11-06= 5
16: 11-16=-5
14: 11-14=-3
08: 11-08= 3
10: 11-10= 1

STEP 2 Find the mean of the distances.
1+1 +2 +7 +1 +3 +4 +5+5+3+3+1/ 12
= 36/12 = 3

So, the mean absolute deviation of the data is 3.

Question 7.
Suppose all of the players on a basketball team had the same height. Explain how you could use reasoning to find the mean absolute deviation of the players’ heights.
Type below:
_________________

Answer: 0

Explanation:
If the players on a basketball team had the same height.
The mean deviation will be equal to the 0 because the difference between the mean and the observations is 0.
Let the observations be 2,2,2,2,2
Mean = 10/5 = 2
Mean deviation =  (2-2)+(2-2)+(2-2)+(2-2)+(2-2)/5 = 0/5 = 0

Question 8.
Explain Tell how an outlier that is much greater than the mean would affect the mean absolute deviation of the data set. Explain your reasoning.
Type below:
_________________

Answer: An outlier increases the mean absolute deviation of the data set.

Explanation:
The difference between the outlier and the mean is a greater number when added in the sum of observations the mean absolute deviation increases.

Question 9.
The data set shows the number of soccer goals scored by players in 3 games.
Go Math Grade 6 Answer Key Chapter 13 Variability and Data Distributions img 19
For numbers 9a–9c, choose Yes or No to indicate whether the statement is correct.
9a. The mean absolute deviation of Player A is 1.
9b. The mean absolute deviation of Player B is 0.
9c. The mean absolute deviation of Player C is greater than the mean absolute deviation of Player A.
9a. __________
9b. __________
9c. __________

Answer:
9a. No
9b. Yes
9c. No

Explanation:
Player A
Mean = 1+2+3/3 = 6/3 = 2
Mean absolute deviation = 1+0+1/3 = 2/3 = 0.6

Player B
Mean =2+2+2/3 = 6?3 = 2
Mean absolute deviation = 0/3 = 0

Player C
Mean = 1+2+1/3 = 4/3 = 1.3
Mean absolute deviation = 0.3+0+0.3/3 = 0.2

Mean Absolute Deviation – Page No. 723

Use counters and a dot plot to find the mean absolute deviation of the data.

Question 1.
the number of hours Maggie spent practicing soccer for 4 different weeks:
9, 6, 6, 7
mean = 7 hours
_______ hour

Answer: The mean absolute deviation of the data is 1.

Explanation:
STEP 1 Label each observation with its distance from the mean.
Starting from left to right:
9: 7-9=-2
6: 7-6=-1
6: 7-6=-1
7: 7-7= 0

STEP 2 Find the mean of the distances.
2 +1 +1+0/ 4
= 4 /4 = 1

So, the mean absolute deviation of the data is 1.

Question 2.
the heights of 7 people in inches:
60, 64, 58, 60, 70, 71, 65
mean = 64 inches
_______ inches

Answer: The mean absolute deviation of the data is 4.

Explanation:
STEP 1 Label each observation with its distance from the mean.
Starting from left to right:
60: 64-60= 4
64: 64-64= 0
58: 64-58= 6
60: 64-60= 4
70: 64-70=-6
71: 64-71=-7
65: 64-65=-1

STEP 2 Find the mean of the distances.
4+0+6+ 4+6+7+1/7
= 28/7 = 4

So, the mean absolute deviation of the data is 4.

Use the dot plot to find the mean absolute deviation of the data.

Question 3.
mean = 10
Go Math Grade 6 Answer Key Chapter 13 Variability and Data Distributions img 20
_______ year

Answer: The mean absolute deviation of the data is 1

Explanation:
STEP 1 Label each dot with its distance from the mean.
Starting from left to right:
08: 10-08=02
09: 10-09=01
10: 10-10= 0
11: 10-11=-1
12: 10-12=-2

STEP 2 Find the mean of the distances.
(2) + (1) +(0) +(1) +(2) +(2)+(1)+(0)+(0)+(0)+(1)+(2) /12 = 12/12= 1

So, the mean absolute deviation of the data is 1

Question 4.
mean = 8
Go Math Grade 6 Answer Key Chapter 13 Variability and Data Distributions img 21
_______ hours

Answer: The mean absolute deviation of the data is 2.4

Explanation:
STEP 1 Label each dot with its distance from the mean.
Starting from left to right:
03: 8-03=05
04: 8-04=04
05: 8-05=03
07: 8-07=01
08: 8-08= 0
09: 8-09=-1
10: 8-10=-2
11: 8-11=-3
12: 8-12=-4

STEP 2 Find the mean of the distances.
(5) + (4) +(3) +(1) +(0) +(1) + (2) +(3) +(4)+(5)+(0)+(1)+(1)+(2)+(4)/ 15 = 36/15 = 2.4

So, the mean absolute deviation of the data is 2.4

Problem Solving

Question 5.
In science class, Troy found the mass, in grams, of 6 samples to be 10, 12, 7, 8, 5, and 6. What is the mean absolute deviation?
_______ grams

Answer: The mean absolute deviation of the data is 2.

Explanation:
Mean = 10+12+7+8+5+6/6 = 48/6 = 8

Mean absolute deviation:

STEP 1 Label each observation with its distance from the mean.
Starting from left to right:
10: 8-10= -2
12: 8-12= -4
07: 8-07= 01
08: 8-08= 0
05: 8-05=03
06: 8-06=02

STEP 2 Find the mean of the distances.
2+4+1+0+3+2/6
= 12/6 = 2

So, the mean absolute deviation of the data is 2.

Question 6.
Five recorded temperatures are 71°F, 64°F, 72°F, 81°F, and 67°F. What is the mean absolute deviation?
_______ °F

Answer: The mean absolute deviation of the data is 4.4.

Explanation:
Mean = 71+64+72+81+67/5 = 355/5 = 71

Mean absolute deviation:

STEP 1 Label each observation with its distance from the mean.
Starting from left to right:
71: 71-71= 0
64: 71-64= 07
72: 71-72= -1
81: 71-81=-10
67: 71-67= 04

STEP 2 Find the mean of the distances.
0+7+1+10+4/5
= 22/5 = 4.4

So, the mean absolute deviation of the data is 4.4.

Question 7.
Make a dot plot of the following data: 10, 10, 11, 12, 12, 13, 13, 15. Use the dot plot to find the mean absolute deviation.
Type below:
_________________

Answer: The mean absolute deviation of the data is 1.25

Explanation:
Mean = 10+10+11+12+12+13+13+15/8 = 96/8 = 12

Mean absolute deviation:

Box plot:

STEP 1 Label each dot with its distance from the mean.
Starting from left to right:
10: 12-10=02
11: 12-11=01
12: 12-12=0
13: 12-13=-1
15: 12-15=-3

STEP 2 Find the mean of the distances.
(2) + (2) +(1) +(0) +(0) +(1) + (1) +(3) / 8 = 10/8 = 1.25

So, the mean absolute deviation of the data is 1.25

Lesson Check – Page No. 724

Question 1.
Six test grades are 86, 88, 92, 90, 82, and 84. The mean of the data is 87. What is the mean absolute deviation?
_______

Answer: The mean absolute deviation of the data is 3.5

Explanation:

Mean absolute deviation:

STEP 1 Label each observation with its distance from the mean.
Starting from left to right:
86: 87-86= 01
88: 87-88= -1
92: 87-92= -5
90: 87-81= 06
82: 87-82= 05
84: 87-84= 03

STEP 2 Find the mean of the distances.
1+5+1+6+5+3/6
= 21/6 = 3.5

So, the mean absolute deviation of the data is 3.5

Question 2.
Eight heights in inches are 42, 36, 44, 46, 48, 42, 48, and 46. The mean of the data is 44. What is the mean absolute deviation?
_______ inches

Answer:

Explanation: The mean absolute deviation of the data is 3

Mean absolute deviation:

STEP 1 Label each observation with its distance from the mean.
Starting from left to right:
42: 44-42= 02
36: 44-36= 08
44: 44-44= 0
46: 44-46= -2
48: 44-48= -4
42: 44-42= 02
48: 44-48= -4
46: 44-46= -2

STEP 2 Find the mean of the distances.
2+8+2+4+0+2+4+2/8
= 24/8 = 3

So, the mean absolute deviation of the data is 3

Spiral Review

Question 3.
What is the volume of a rectangular prism with dimensions 4 meters, 1 \(\frac{1}{2}\) meters, and 5 meters?
_______ m3

Answer: 30m3

Explanation:
Dimentions: 4 meters, 1 1/2 meters, 5 meters
Change the mixed fraction into improper fraction = 3/2
Volume of the rectangle = 4 x 3/2 x 5 = 30m3

Question 4.
Carrie is making a frequency table showing the number of miles she walked each day during the 30 days of September. What value should she write in the Frequency column for 9 to 11 miles?
Go Math Grade 6 Answer Key Chapter 13 Variability and Data Distributions img 22
_______

Answer: 1

Explanation:
Total number of days in the month of September = 30
Number of days given in the frequency table = 17+8+4 = 29
Frequency in the interval 9 – 11 = 30 – 29 = 1 day

Question 5.
The following data shows the number of laps each student completed. What number of laps is the mode?
9, 6, 7, 8, 5, 1, 8, 10
The mode is _______ laps.

Answer: The mode is 8 laps.

Explanation:
The most frequently occurring observation is known as mode.
8 is the mode in the above raw data given.

Question 6.
What is the upper quartile of the following data?
43, 48, 55, 50, 58, 49, 38, 42, 50
The upper quartile is _______

Answer: The upper quartile is 58

Explanation:
The highest observation in the data given is known as upper quartile. The upper quartile is 58

Share and Show – Page No. 727

Question 1.
Find the range and interquartile range of the data in the box plot.
Go Math Grade 6 Answer Key Chapter 13 Variability and Data Distributions img 23
The range is $ __________ .
The interquartile range is $ __________ .

Answer: $12, $3

Explanation:
The difference between the highest observation and the lowest observation is called a range.
Range = 19 – 7 = $12
The difference and the highest and the lowest dots of the dot plot is called as interquartile range.
Interquartile range = 15 – 12 = $3

Practice: Copy and Solve Find the mean absolute deviation for the data set.

Question 2.
heights in inches of several tomato plants:
16, 18, 18, 20, 17, 20, 18, 17
_______ inch

Answer: The mean absolute deviation of the data is 1

Explanation:
Mean:
Mean = 16+18+18 +20+17+20+18+17/8 = 144/8 = 18

Mean absolute deviation:

STEP 1 Label each observation with its distance from the mean.
Starting from left to right:
16: 18-16= 02
18: 18-18= 0
18: 18-18= 0
20: 18-20= -2
17: 18-17= 01
20: 18-20= -2
18: 18-18= 0
17: 18-17= 01

STEP 2 Find the mean of the distances.
2+0+0+2+1+2+0+1/8
= 8/8 = 1

So, the mean absolute deviation of the data is 1

Question 3.
times in seconds for students to run one lap:
68, 60, 52, 40, 64, 40
_______ seconds

Answer: The mean absolute deviation of the data is 10

Explanation:
Mean:
Mean = 68+60+52+40+64+40/6 = 54

Mean absolute deviation:

STEP 1 Label each observation with its distance from the mean.
Starting from left to right:
68: 54-68= -14
60: 54-60= -6
52: 54-52= 02
40: 54-40= 14
64: 54-64= -10
40: 54-40= 14

STEP 2 Find the mean of the distances.
14+6+2+14+10+14/6
= 60/6 = 10

So, the mean absolute deviation of the data is 10

On Your Own

Use the box plot for 4 and 5.
Go Math Grade 6 Answer Key Chapter 13 Variability and Data Distributions img 24

Question 4.
What is the range of the data?
$ _______

Answer: $24

Explanation:
The difference between the highest observation and the lowest observation is called a range.
Range = $56 – $32 = $24

Question 5.
What is the interquartile range of the data?
$ _______

Answer: $16

Explanation:
The difference and the highest and the lowest dots of the dot plot is called as interquartile range.
Interquartile range = $52 – $36 = $16

Practice: Copy and Solve Find the mean absolute deviation for the data set.

Question 6.
times in minutes spent on a history quiz:
35, 35, 32, 34, 34, 32, 34, 36
_______ minute

Answer: The mean absolute deviation of the data is 1

Explanation:
Mean:
Mean = 35+ 35+ 32+ 34+34+ 32+ 34+36/8 = 272/8 = 34

Mean absolute deviation:

STEP 1 Label each observation with its distance from the mean.
Starting from left to right:
35: 34-35= -1
35: 34-35= -1
32: 34-32= 02
34: 34-34= 0
34: 34-34= 0
32: 34-32=02
34: 34-34=0
36: 34-36=-2

STEP 2 Find the mean of the distances.
1+1+2+0+0+2+0+2/8
= 8/8 = 1

So, the mean absolute deviation of the data is 1

Question 7.
number of excused absences for one semester:
1, 2, 1, 10, 9, 9, 10, 6, 1, 1
_______

Answer: The mean absolute deviation of the data is 3.8

Explanation:

Mean:
Mean =1+2+1+10+9+9+10+6+1+1 /10 = 50/10 = 5

Mean absolute deviation:

STEP 1 Label each observation with its distance from the mean.
Starting from left to right:
1: 5-1= 4
2: 5-2= 3
1: 5-1= 4
10: 5-10= -5
9: 5-9= -4
9: 5-9=-4
10: 5-10=-5
6: 5-6=-1
1: 5-1=4
1: 5-1=4

STEP 2 Find the mean of the distances.
4+3+4+5+4+4+5+1+4+4/10
=38/10 = 3.8

So, the mean absolute deviation of the data is 3.8

Question 8.
The chart shows the price of different varieties of dog food at a pet store. Find the range, interquartile range, and the mean absolute deviation of the data set.
Go Math Grade 6 Answer Key Chapter 13 Variability and Data Distributions img 25
Type below:
_________________

Answer:

The mean absolute deviation of the data is 3.6
Range = 32-16 = 16
Interquartile range = 24 – 20 = 4

Explanation:

Mean:
Mean =18+24+20+26+24+20+32+20+16+20 /10 = 220/10 = 22

Mean absolute deviation:

STEP 1 Label each observation with its distance from the mean.
Starting from left to right:
18: 22-18= 4
24: 22-24= -2
20: 22-20= 2
26: 22-26= -4
24: 22-24= -2
20: 22-20= 2
32: 22-32=-10
20: 22-20= 2
16: 22-16= 6
20: 22-20= 2

STEP 2 Find the mean of the distances.
4+2+2+4+2+2+10+2+6+2/10
=36/10 = 3.6

So, the mean absolute deviation of the data is 3.6
The difference between the highest observation and the lowest observation is called a range.
Range = 32-16 = 16
The difference and the highest and the lowest dots of the dot plot is called as interquartile range.
Interquartile range = 24 – 20 = 4

Problem Solving + Applications – Page No. 728

Question 9.
Hyato’s family began a walking program. They walked 30, 45, 25, 35, 40, 30, and 40 minutes each day during one week. At the right, make a box plot of the data. Then find the interquartile range.
_______ minutes

Answer: 35 minutes

Explanation:
Ascending order: 25,30,30,35,40,40,45
n=7 (odd)
Median= Number of (n+1/2) = 8/2 = 4th observation = 35
Median:(four terms of the data)
Median = 30+30/2 = 60/2 = 30
Median:(last 3 terms of the data)
Median = (n+1/2) = 2nd observation = 40
Interquartile range = 30+40/2 = 70/2 = 35

Question 10.
Compare Jack recorded the number of minutes his family walked each day for a month. The range of the data is 15. How does this compare to the data for Hyato’s family?
Type below:
_________________

Answer: Jack’s family walked less number of minutes each day compared to Hyato’s family.

Explanation:
The range of Hyato’s family is 20 while the range of Jack’s family is 15. Therefore we can say that Hyato’s family walked more minutes compared to Jack’s family in a day.
Range can define the data with large observations and the data with least observations.

Question 11.
Sense or Nonsense? Nathan claims that the interquartile range of a data set can never be greater than its range. Is Nathan’s claim sense or nonsense? Explain.
Type below:
_________________

Answer: Nonsense, Interquartile range of a data set can be less than or greater than range.

Explanation:
The interquartile range is the difference between the medians of the observations.
Nathan’s claim is nonsense as he said that, ” The interquartile range can never be greater than its range.”
The range is the difference between the highest observation and the lowest observation.
The interquartile range can be less than greater than the range.

Example:
Ascending order: 25,30,30,35,40,40,45
n=7 (odd)
Median= Number of (n+1/2) = 8/2 = 4th observation = 35
Median:(four terms of the data)
Median = 30+30/2 = 60/2 = 30
Median:(last 3 terms of the data)
Median = (n+1/2) = 2nd observation = 40
Interquartile range = 30+40/2 = 70/2 = 35

Range= 45-25 = 20

In the above case the interquartile range is more than the range proving that the given statement is nonsense.

Question 12.
The box plot shows the heights of corn stalks from two different farms.
Go Math Grade 6 Answer Key Chapter 13 Variability and Data Distributions img 26
The range of Farm A’s heights is _____ the range of Farm B’s heights.

Answer: greater than

Explanation:
The range is the difference between the highest and the lowest observations.
Range of Farm A: 72-58 = 14
Range of Farm B: 70-55 = 15

Therefore, The range of Farm A’s heights is greater than the range of Farm B’s heights.

Measures of Variability – Page No. 729

Question 1.
Find the range and interquartile range of the data in the box plot.
Go Math Grade 6 Answer Key Chapter 13 Variability and Data Distributions img 27
The range is __________ miles.
The interquartile range is __________ miles.

Answer: 16, 8

Explanation:
The difference between the highest and the lowest observations is range.
Range = 17 – 1 = 16
The difference between the highest and lowest observations of the box is the interquartile range.
Interquartile range = 12 – 4 = 8

Use the box plot for 2 and 3.
Go Math Grade 6 Answer Key Chapter 13 Variability and Data Distributions img 28

Question 2.
What is the range of the data?
_____

Answer: 35

Explanation:
The difference between the highest and the lowest observations is range.
Range = 95 – 60 = 35

Question 3.
What is the interquartile range of the data?
_____

Answer: 20

Explanation:
The difference between the highest and lowest observations of the box is the interquartile range.
Interquartile range = 90 – 70 = 20

Find the mean absolute deviation for the set.

Question 4.
heights in centimetres of several flowers:
14, 7, 6, 5, 13
_____ cm

Answer: The mean absolute deviation of the data is 3.6

Explanation:

Mean:
Mean =14+7+ 6+5+13/5= 45/5 = 9

Mean absolute deviation:

STEP 1 Label each observation with its distance from the mean.
Starting from left to right:
14: 9-14= -5
07: 9-07= 02
06: 9-06= 03
05: 9-05= 04
13: 9-13= -4

STEP 2 Find the mean of the distances.
5+2+3+4+4/5
= 18/5 = 3.6

So, the mean absolute deviation of the data is 3.6

Question 5.
ages of several children:
5, 7, 4, 6, 3, 5, 3, 7
_____ years

Answer: The mean absolute deviation of the data is 1.25

Explanation:
Mean:
Mean = 5+7+4+6+ 3+5+3+7/8 = 40/8 = 5

Mean absolute deviation:

STEP 1 Label each observation with its distance from the mean.
Starting from left to right:
5: 5-5= 0
7: 5-7= -2
4: 5-4= 01
6: 5-6= -1
3: 5-3= 02
5: 5-5= 0
3: 5-3= 02
7: 5-7=-2

STEP 2 Find the mean of the distances.
0+2+1+1+2+0+2+2/8
= 10/8 = 1.25

So, the mean absolute deviation of the data is 1.25

Problem Solving

Question 6.
The following data set gives the amount of time, in minutes, it took five people to cook a recipe. What is the mean absolute deviation for the data?
33, 38, 31, 36, 37
_____ minutes

Answer: The mean absolute deviation of the data is 2.4

Explanation:
Mean:
Mean = 33+38+31+36+37/5 = 175/5 = 35

Mean absolute deviation:

STEP 1 Label each observation with its distance from the mean.
Starting from left to right:
33: 35-33= 02
38: 35-38= -3
31: 35-31= 04
36: 35-36= -1
37: 35-37= -2

STEP 2 Find the mean of the distances.
2+3+4+1+2/5
= 12/5 = 2.4

So, the mean absolute deviation of the data is 2.4

Question 7.
The prices of six food processors are $63, $59, $72, $68, $61, and $67. What are the range, interquartile range, and mean absolute deviation for the data?
Type below:
_________________

Answer: Range = $9 The mean absolute deviation of the data is 4

Explanation:
The difference between the highest and the lowest observations is range.
Range = $68 – $59 = $9
The difference between the highest and lowest observations of the box is the interquartile range.
Interquartile range = 12 – 4 = 8

Mean:
Mean = $63+$59+$72+$68+$61+$67/6 = 390/6 = 65

Mean absolute deviation:

STEP 1 Label each observation with its distance from the mean.
Starting from left to right:
63: 65-63= 02
59: 65-59= 06
72: 65-72= -7
68: 65-68= -3
61: 65-61= -4
67: 65-67= -2

STEP 2 Find the mean of the distances.
2+6+7+3+4+2/6
= 24/6 = 4

So, the mean absolute deviation of the data is 4

Question 8.
Find the range, interquartile range, and mean absolute deviation for this data set: 41, 45, 60, 61, 61, 72, 80.
Type below:
_________________

Answer: The mean absolute deviation of the data is 9.7

Explanation:

Mean:
Mean = 41+45+60+61+61+72+80 /7 = 420/7 = 60

Mean absolute deviation:

STEP 1 Label each observation with its distance from the mean.
Starting from left to right:
41: 60-41= 19
45: 60-45= 15
60: 60-60= 0
61: 60-61= -1
61: 60-61= -1
72: 60-72= -12
80: 60-80= -20

STEP 2 Find the mean of the distances.
19+15+0+1+1+12+20/7
= 68/7 = 9.7

So, the mean absolute deviation of the data is 9.7

Lesson Check – Page No. 730

Question 1.
Daily high temperatures recorded in a certain city are 65°F, 66°F, 70°F, 58°F, and 61°F. What is the mean absolute deviation for the data?
_____ °F

Answer: The mean absolute deviation of the data is 3.6

Explanation:

Mean:
Mean = 65+66+70+58+61 /5 = 320/5 = 64

Mean absolute deviation:

STEP 1 Label each observation with its distance from the mean.
Starting from left to right:
65: 64-65=-1
66: 64-66=-2
70: 64-70=-6
58: 64-58=06
61: 64-61=03

STEP 2 Find the mean of the distances.
1+2+6+6+3/5
= 18/5 = 3.6

So, the mean absolute deviation of the data is 3.6

Question 2.
Eight different cereals have 120, 160, 135, 144, 153, 122, 118, and 134 calories per serving. What is the interquartile range for the data?
_____ calories

Answer: 42cereals

Explanation:
Ascending order of the data: 118,120,122,134,135,144,153,160
Median:(for first 4 terms)
Median= 120+122/2 = 242/2 = 121
Median:(for first 4 terms)
Median= 144+153/2 = 297/2 = 148.5
The difference and the highest and the lowest dots of the dot plot is called as interquartile range.
Interquartile range = 148.5 – 121 = 27.5

Spiral Review

Question 3.
Look at the histogram. How many days did the restaurant sell more than 59 pizzas?
Go Math Grade 6 Answer Key Chapter 13 Variability and Data Distributions img 29
________

Answer: 20

Explanation:
After 59 there is 1 interval 60-79
Number of days the restaurant sell more than 59 pizzas = 20

Question 4.
Look at the histogram. Where does a peak in the data occur?
Go Math Grade 6 Answer Key Chapter 13 Variability and Data Distributions img 30
Type below:
_________________

Answer: 20 – 39

Explanation:
Number of days the restaurant sold the maximum pizzas = 30
Number pizzas sold in each day = 20 – 39

Question 5.
What is the mode of the data set?
14, 14, 18, 20
The mode is ________

Answer: 14

Explanation:
The most frequently occurring observation is known as a mode.
In the above data mode is 14.

Question 6.
The data set below lists the ages of people on a soccer team. The mean of the data is 23. What is the mean absolute deviation?
24, 22, 19, 19, 23, 23, 26, 27, 24
________

Answer: The mean absolute deviation of the data is 2

Explanation:

Mean absolute deviation:

STEP 1 Label each observation with its distance from the mean.
Starting from left to right:
24: 23-24=-1
22: 23-22= 1
19: 23-19= 4
19: 23-19= 4
23: 23-23=0
23: 23-23=0
26: 23-26=-3
27: 23-27=-4
24: 23-24=-1

STEP 2 Find the mean of the distances.
1+1+4+4+3+4+1+0+0/9
= 18/9 = 2

So, the mean absolute deviation of the data is 2

Mid-Chapter Checkpoint – Vocabulary – Page No. 731

Choose the best term from the box to complete the sentence.
Go Math Grade 6 Answer Key Chapter 13 Variability and Data Distributions img 31

Question 1.
The _____ is the difference between the upper quartile and the lower quartile of a data set.
Type below:
_________________

Answer: Range

Explanation:
The difference between the upper and lower quartiles of the data is known as range.

Question 2.
A graph that shows the median, quartiles and least and greatest values of a data set is called a(n) _____.
Type below:
_________________

Answer: Box plot

Explanation:
The figure which shows the median, quartiles and least and greatest values of a data set is called a box plot, a box plot is a figure which represents median with a horizontal line and the starting and ending line represents the upper and lower quartiles and the end dots represent the upper limit and the lower limit.

Question 3.
The difference between the greatest value and the least value in a data set is the _____.
Type below:
_________________

Answer: Range

Explanation:
Each data set consists of upper and lower limits the difference between these limits is called as range.

Question 4.
The _____ is the mean of the distances between the values of a data set and the mean of the data set.
Type below:
_________________

Answer: Mean absolute

Explanation:
Mean absolute deviation is calculated by subtracting each observation from the mean and then the mean is calculated for these observations.
Therefore we can say that the mean absolute is the mean of the distances between the values of a data set and the mean of the data set.

Concepts and Skills

Question 5.
Make a box plot for this data set: 73, 65, 68, 72, 70, 74.
Type below:
_________________

Answer: Median = 71

Explanation:
Median:
Ascending order: 65, 68, 70, 72, 73,74
n = even = 6
Median = Mean of 3rd and 4th terms
= 70+72/2 = 142/2 = 71

Find the mean absolute deviation of the data.

Question 6.
43, 46, 48, 40, 38
________

Answer: The mean absolute deviation of the data is 3.2

Explanation:
Mean:

Mean = 43+46+48+40+38/5 = 215/5 = 43

Mean absolute deviation:

STEP 1 Label each observation with its distance from the mean.
Starting from left to right:
43: 43-43=0
43: 43-46=-3
43: 43-48=-5
43: 43-40= 3
43: 43-38= 5

STEP 2 Find the mean of the distances.
0+3+5+3+5/5 = 16/5 = 3.2

So, the mean absolute deviation of the data is 3.2

Question 7.
26, 20, 25, 21, 24, 27, 26, 23
________

Answer: The mean absolute deviation of the data is 2.125

Explanation:
Mean:

Mean = 26+20+25+21+24+27+26+23/8 = 192/8 = 24

Mean absolute deviation:

STEP 1 Label each observation with its distance from the mean.
Starting from left to right:
26: 24-26=-3
20: 24-20= 4
25: 24-25= -1
21: 24-21= 3
24: 24-24=0
27: 24-27=-3
26: 24-26=-2
23: 24-23=1

STEP 2 Find the mean of the distances.
3+4+1+3+0+3+2+1/8
= 17/8 = 2.125

So, the mean absolute deviation of the data is 2.125

Question 8.
99, 70, 78, 85, 76, 81
________

Answer: The mean absolute deviation of the data is 2

Explanation:
Mean:

Mean = 99+70+78+85+76+81/6 = 489/6 = 81.5

Mean absolute deviation:

STEP 1 Label each observation with its distance from the mean.
Starting from left to right:
24: 23-24=-1
22: 23-22= 1
19: 23-19= 4
19: 23-19= 4
23: 23-23=0
23: 23-23=0
26: 23-26=-3
27: 23-27=-4
24: 23-24=-1

STEP 2 Find the mean of the distances.
1+1+4+4+3+4+1+0+0/9
= 18/9 = 2

So, the mean absolute deviation of the data is 2

Find the range and interquartile range of the data.

Question 9.
2, 4, 8, 3, 2
The range is _________ .
The interquartile range is _________ .

Answer: 6

Explanation:
The difference between the upper quartile and lower quartile.
Range = 8 – 2 =6

Question 10.
84, 82, 86, 87, 88, 83, 84
The range is _________ .
The interquartile range is _________ .

Answer: 6

Explanation:
The difference between the upper quartile and lower quartile.
Range = 88 – 82 = 6

Question 11.
39, 22, 33, 45, 42, 40, 28
The range is _________ .
The interquartile range is _________ .

Answer: 23

Explanation:
The difference between the upper quartile and lower quartile.
Range = 45 – 22 = 23

Page No. 732

Question 12.
Yasmine keeps track of the number of hockey goals scored by her school’s team at each game. The dot plot shows her data.
Go Math Grade 6 Answer Key Chapter 13 Variability and Data Distributions img 32
Where is there a gap in the data?
Type below:
_________________

Answer: There is a gap in the data in the intervals: between 1 and 2

Explanation:
As shown in the dot plot there is a gap between 1 and 2. This means that Yasmine’s team did not score only one goal when they played the game.

Question 13.
What is the interquartile range of the data shown in the dot plot with Question 12?
The interquartile range is _________ .

Answer: 2

Explanation:
The interquartile range is the difference between the lower and upper quartiles.
Interquartile range = 3-1 = 2

Question 14.
Randall’s teacher added up the class scores for the quarter and used a histogram to display the data. How many peaks does the histogram have? Explain how you know.
Go Math Grade 6 Answer Key Chapter 13 Variability and Data Distributions img 33
Type below:
_________________

Answer: The graph has only one peak

Explanation:
In the given graph there is are rectangles out of which one has a tall rectangle which can be addressed as peak.

Question 15.
In a box plot of the data below, where would the box be drawn?
55, 37, 41, 62, 50, 49, 64
Type below:
_________________

Answer: The box is drawn above the number line and this represents the median and the lower and upper limits.

Explanation:
A box is drawn to represent the median and  the upper and lower limits in a box plot.

Share and Show – Page No. 735

Question 1.
The distances in miles students travel to get to school are 7, 1, 5, 9, 9, and 8. Decide which measure(s) of center best describes the data set. Explain your reasoning.
Type below:
_________________

Answer: Mean: 6.5
Median: 7.5
Mode: 9

Explanation:
Mean:
7+1+5+9+9+8/6 = 39/6 = 6.5
Median:
Ascending order: 1,5,7,8,9,9
Median = Mean of 7 and 8 = 7+8/2 = 15/2 = 7.5
Mode:
The most frequently occurring observation is known as the mode.
The mode is 9.

Question 2.
Use Graphs The numbers of different brands of orange juice carried in several stores are 2, 1, 3, 1, 12, 1, 2, 2, and 5. Make a box plot of the data and find the range and interquartile range. Decide which measure better describes the data set and explain your reasoning.
Type below:
_________________

Answer: Range: 11      Interquartile: 3 Interquartile range is the best way to represent the data.

Explanation:
Range = 12 – 1 = 11
Interquartile range :
Median(of first 4 terms):
Median = 1+1/2 = 2/2 = 1
Median (of last 4 terms):
Median = 3+5/2 = 8/2 = 4
Interquartile range = 4 – 1 = 3

On Your Own

Question 3.
Use Reasoning The ages of students in a computer class are 14, 13, 14, 15, 14, 35, 14. Decide which measure of center(s) best describes the data set. Explain your reasoning.
Type below:
_________________

Answer: Mean:17  Median:14   Mode:14  Median and mode is the best ways to represent the data.

Explanation:
Mean:
14+13+14+15+14+35+14/7 = 119/7 = 17
Median:
Ascending order: 13,14,14,14,14,15,35
Median = 14
Mode:
The most frequently occurring observation is known as the mode.
The mode is 14.

Question 4.
Mateo scored 98, 85, 84, 80, 81, and 82 on six math tests. When a seventh math test score is added, the measure of center that best describes his scores is the median. What could the seventh test score be? Explain your reasoning.
Type below:
_________________

Answer: Median is the best way to represent the data.

Explanation:
Median:
Ascending order: 80,81,82,84,85,98
Median = Mean of 3 and 4 = 82+84/2 = 166/2 = 83
The seventh score can be 83

Unlock the Problem – Page No. 736

Question 5.
Jaime is on the community swim team. The table shows the team’s results in the last 8 swim meets. Jaime believes they can place in the top 3 at the next swim meet. Which measure of center should Jaime use to persuade her team that she is correct? Explain.
Go Math Grade 6 Answer Key Chapter 13 Variability and Data Distributions img 34
a. What do you need to find?
Type below:
_________________

Answer: Mean, median,mode

Explanation:
Mean= 1+2+2+3+3+1+18+2/8 = 32/8 = 4
Median:
Ascending order: 1,1,2,2,2,3,3,18
Median = 2+2/2 = 2
Mode:
The most frequently occurring observation is called a mode.
Mode=2

Question 5.
b. What information do you need to solve the problem?
Type below:
_________________

Answer: We need to have the data to find the centre of tendencies.

Explanation:
The given data can be used to find the mean, median and mode.

Question 5.
c. What are the measures of center?
Type below:
_________________

Answer: Mean = 4 Median = 2 Mode = 2

Explanation:
There are three measures to calculate their approximate values.

Question 5.
d. Which measure of center should Jaime use? Explain.
Type below:
_________________

Answer: Median or mode

Explanation:
Median or mode are nearer to the solution, therefore, they can be used.

Question 6.
The numbers of sit-ups students completed in one minute are 10, 42, 46, 50, 43, and 49. The mean of the data values is 40 and the median is 44.5. Which measure of center better describes the data, the mean or median? Use words and numbers to support your answer.
Type below:
_________________

Answer: Median is the better way to represent the data.

Explanation:
44.5 is closer and represents the more number of observations compared to the mean.

Choose Appropriate Measures of Center and Variability – Page No. 737

Question 1.
The distances, in miles, that 6 people travel to get to work are 14, 12, 2, 16, 16, and 18. Decide which measure(s) of center best describes the data set. Explain your reasoning.
Type below:
_________________

Answer: Mean= 13 miles Median= 15 miles Mode= 16 miles

Explanation:
Mean is less than the data points.
Median describes the data in the best way compared to mean and mode.

Question 2.
The numbers of pets that several children have are 2, 1, 2, 3, 4, 3, 10, 0, 1, and 0. Make a box plot of the data and find the range and interquartile range. Decide which measure better describes the data set and explain your reasoning.
Type below:
_________________

Answer: Range = 10-0 = 10
Interquartile range = 3.5 – 0.5 = 3
The interquartile range is the best way to represent the data.

Explanation:
Ascending order: 0,0,1,1,2,2,3,3,4,10
Median = 2+2/2 = 2
Lower quartile = 0.5
Upper quartile= 7/2 = 3.5
Highest observation= 10
Lowest observation = 0
Range = 10-0 = 10
Interquartile range = 3.5 – 0.5 = 3
The interquartile range is the best way to represent the data.

Problem Solving

Question 3.
Brett’s history quiz scores are 84, 78, 92, 90, 85, 91, and 0. Decide which measure(s) of center best describes the data set. Explain your reasoning.
Type below:
_________________

Answer: Mean is the best measure of centre to describe the data set.

Explanation:
Mean:
Mean= 84+78+92+90+85+91+0/7 = 74.2
Median:
Ascending order: 0,78,84,85,90,91,92
Median = 4th observation = 85

Question 4.
Eight students were absent the following number of days in a year: 4, 8, 0, 1, 7, 2, 6, and 3. Decide if the range or interquartile range better describes the data set, and explain your reasoning.
Type below:
_________________

Answer: 8 represents all the terms range is more preferable compared to the interquartile range.

Explanation:
Ascending order: 0,1,2,3,4,6,7,8
Range = 8-0 = 8
Median = Mean of 3 and 4 = 3+4/2 = 7/2 = 3.5
Median of first 3 terms = 1
Median of last 3 terms = 7
Interquartile range = 7-1 = 6
Since 8 represents all the terms range is more preferable compared to the interquartile range.

Question 5.
Create two sets of data that would be best described by two different measures of centre.
Type below:
_________________

Answer: The given below are the examples of two sets of data that would be best described by two different measures of centre.

Explanation:
Example 1:

The numbers of pets that several children have are 2, 1, 2, 3, 4, 3, 10, 0, 1, and 0. Make a box plot of the data and find the range and interquartile range. Decide which measure better describes the data set and explain your reasoning.
Type below:
_________________

Answer: Range = 10-0 = 10
Interquartile range = 3.5 – 0.5 = 3
The interquartile range is the best way to represent the data.

Explanation:
Ascending order: 0,0,1,1,2,2,3,3,4,10
Median = 2+2/2 = 2
Lower quartile = 0.5
Upper quartile= 7/2 = 3.5
Highest observation= 10
Lowest observation = 0
Range = 10-0 = 10
Interquartile range = 3.5 – 0.5 = 3
The interquartile range is the best way to represent the data.

Example 2:

Brett’s history quiz scores are 5,6,7,8,9,10. Decide which measure(s) of centre best describes the data set. Explain your reasoning.
Type below:
_________________

Answer: Mean and median are the best measure of centre to describe the data set.

Explanation:
Mean:
Mean= 5+6+7+8+9+10/6 = 7.5
Median:
Ascending order: 5,6,7,8,9,10
Median = Mean of 7 and 8 = 15/2 = 7.5

Lesson Check – Page No. 738

Question 1.
Chloe used two box plots to display some data. The box in the plot for the first data set is wider than the box for the second data set. What does this say about the data?
Type below:
_________________

Answer: The graphs say that the interquartile range is more for the second graph compared to the first.

Explanation:
The interquartile range is the difference between the lower and upper quartiles.
It is more for wider data compared to the compact data.

Question 2.
Hector recorded the temperature at noon for 7 days in a row. The temperatures are 20°F, 20°F, 20°F, 23°F, 23°F, 23°F, and 55°F. Which measure of center would best describe the data?
Type below:
_________________

Answer: Mode

Explanation:
The most frequently occurring observation is known as mode.
The mode of the above data describes the data well, the mode of the data is 20°F

Spiral Review

Question 3.
By how much does the median of the following data set change if the outlier is removed?
13, 20, 15, 19, 22, 26, 42
Type below:
_________________

Answer: 0.5

Explanation:
Median:
Ascending order: 13,15,19,20,22,26,42
Median = 20
If the outlier is removed then the median=
19+20/2 = 39/2 = 19.5
The difference in the medians = 0.5

Question 4.
What percent of the people surveyed spent at least an hour watching television?
Go Math Grade 6 Answer Key Chapter 13 Variability and Data Distributions img 35
_______ %

Answer: 8 people

Explanation:
Total number of people= 40
Percentage = 8/40 x 100 = 20%

Question 5.
What is the lower quartile of the following data?
12, 9, 10, 8, 7, 12
The lower quartile is _______ .

Answer:

Explanation:
Ascending order: 7,8,9,10,12,12
Median = 9+10/2 = 9.5
Lower quartile = 8

Question 6.
What is the interquartile range of the data shown in the box plot?
Go Math Grade 6 Answer Key Chapter 13 Variability and Data Distributions img 36
The interquartile range is _______ .

Answer: 5

Explanation:
The difference between the upper and lower quartiles is called as interquartile range.
Interquartile range = 14 – 9 = 5

Share and Show – Page No. 741

Question 1.
Zoe collected data on the number of points her favourite basketball players scored in several games. Use the information in the table to compare the data.
Go Math Grade 6 Answer Key Chapter 13 Variability and Data Distributions img 37
The mean of Player 1’s points is __ the mean of Player 2’s
points.
The interquartile range of Player 1’s points is __ the
interquartile range of Player 2’s points.
So, Player 2 typically scores __ points than Player 1, but
Player 2’s scores typically vary __ Player 1’s scores
Type below:
_________________

Answer: less than ; less than ; more ; more

Explanation:
The mean of Player 1’s points is less than the mean of Player 2’s points.
The interquartile range of Player 1’s points is less than the interquartile range of Player 2’s points.
So, Player 2 typically scores more points than Player 1, but Player 2’s scores typically vary in more Player 1’s scores

Question 2.
Mark collected data on the weights of puppies at two animal shelters. Find the median and range of each data set, and use these measures to compare the data.
Go Math Grade 6 Answer Key Chapter 13 Variability and Data Distributions img 38
Type below:
_________________

Answer: They differ slightly but on an average we can say that shalter B is more as compared to shelter A

Explanation:
Shelter A
Median:
Ascending order: 5,7,7,7,10,12,15
Median = 4th observation = 7
Range = 15-5 = 10
Shelter B
Median:
Ascending order: 4,5,5,11,11,13,15
Median = 4th observation = 11
Range = 15-4 = 11

On Your Own

Kwan analyzed data about the number of hours musicians in her band practice each week. The table shows her results. Use the table for Exercises 3–5.
Go Math Grade 6 Answer Key Chapter 13 Variability and Data Distributions img 39

Question 3.
Which two students typically practised the same amount each week, with about the same variation in practice times?
Type below:
_________________

Answer: Sally and Jennifer

Explanation:
They are slightly different but on a whole, the average shows no difference and we can say that Sally and Jennifer practised for the same amount each week

Question 4.
Which two students typically practised the same number of hours, but had very different variations in their practice times?
Type below:
_________________

Answer: Tim and Sally

Explanation:
They are different in range but on a whole, the average shows no difference and we can say that Sally and Tim practised for the same number of hours, but had very different variations in their practice times.

Question 5.
Which two students had the same variation in practice times, but typically practised a different number of hours per week?
Type below:
_________________

Answer: Matthew and Tim

Explanation:
Matthew and Tim practised for the same number of hours but they had a high variation in the range.

Problem Solving + Applications – Page No. 742

Question 6.
Compare The table shows the number of miles Johnny ran each day for two weeks. Find the median and the interquartile range of each data set, and use these measures to compare the data sets.
Go Math Grade 6 Answer Key Chapter 13 Variability and Data Distributions img 40
Type below:
_________________

Answer: Interquartile range is the best way to compare the data in the week 1
While the median is the best way to compare the data in the week 2

Explanation:
Week 1
Median:
Ascending order: 1,2,2,3,3,4,5
Median = 4th observation = 3
Lower quartile range= 2
Upper quartile range= 4
Interquartile range = 4-2 = 2

Week 2
Median:
Ascending order: 1,1,1,3,3,8,8
Median = 4th observation = 3
Lower quartile range= 1
Upper quartile range= 8
Interquartile range = 8-1 = 7

Question 7.
Sense or Nonsense? Yashi made the box plots at right to show the data he collected on plant growth. He thinks that the variation in bean plant growth was about the same as the variation in tomato plant growth. Does Yashi’s conclusion make sense? Why or why not?
Go Math Grade 6 Answer Key Chapter 13 Variability and Data Distributions img 41
Type below:
_________________

Answer: Sense

Explanation:
Yashi said that thinks that the variation in bean plant growth was about the same as the variation in tomato plant growth.
It is a true statement because the range of both bean and tomato plants growth is the same and they have the same medians.

Question 8.
Kylie’s teacher collected data on the heights of boys and girls in a sixth-grade class. Use the information in the table to compare the data.
Go Math Grade 6 Answer Key Chapter 13 Variability and Data Distributions img 42
The mean of the boys’ heights is _____ the mean of the girls’ heights.
The range of the boys’ heights is _____ the range of the girls’ heights.

Answer: more than ; more than

Explanation:
Mean of boys height:
Mean = 72+68+70+56+58+62+64/7 = 64.2
Range= 72-56 = 16

Mean of girls height:
Mean = 55+60+56+51+60+63+65/7 = 58.5
Range= 65-51 = 14
The mean of the boys’ heights is _more than____ the mean of the girls’ heights.
The range of the boys’ heights is _more than____ the range of the girls’ heights.

Apply Measures of Center and Variability – Page No. 743

Solve.

Question 1.
The table shows temperature data for two cities. Use the information in the table to compare the data.
Go Math Grade 6 Answer Key Chapter 13 Variability and Data Distributions img 43
The mean of City 1’s temperatures is the ———————– mean of City 2’s temperatures.
The ———————- of City 1’s temperatures is————— the —————–of City 2’s temperatures.
So, City 2 is typically —————-City 1, but City 2’s temperatures
vary ——————-City 1’s temperatures.
Type below:
_________________

Answer: less than; interquartile range; less than; interquartile range; warmer than; more than

Explanation:
The mean of City 1’s temperatures is the —-less than———- mean of City 2’s temperatures.
The –interquartile range——— of City 1’s temperatures is—less than—– the —-interquartile range—–of City 2’s temperatures.
So, City 2 is typically —warmer than—–City 1, but City 2’s temperatures
vary —–more than——-City 1’s temperatures.

Question 2.
The table shows weights of fish that were caught in two different lakes. Find the median and range of each data set, and use these measures to compare the data.
Go Math Grade 6 Answer Key Chapter 13 Variability and Data Distributions img 44
Type below:
_________________

Answer: Lake A’s average is greater but varies more.

Explanation:
Lake A
Median:
Ascending order: 4,6,7,9,10,12
Median = Mean 3rd and 4th observation = 7+9/2 = 8
Range = 12 – 4 = 8

Lake B
Median:
Ascending order: 4,4,5,6,6,7
Median = Mean 3rd and 4th observation = 5+6/2 = 5.5
Range = 7 – 3 = 4

Problem Solving

Question 3.
Mrs. Mack measured the heights of her students in two classes. Class 1 has a median height of 130 cm and an interquartile range of 5 cm. Class 2 has a median height of 134 cm and an interquartile range of 8 cm. Write a statement that compares the data.
Type below:
_________________

Answer: Class 2 is greater but varies more.

Explanation:
The interquartile range is the difference between the lower and upper quartiles. Since the interquartile range is more for class 2 we can say that the extremes are greater while the interquartile range is less for class 1 which means that the data is compact.

Question 4.
Richard’s science test scores are 76, 80, 78, 84, and 80. His math test scores are 100, 80, 73, 94, and 71. Compare the medians and interquartile ranges.
Type below:
_________________

Answer: Medians are equal but the interquartile range varies a large, math test scores are more spread out compared to science test scores.

Explanation:
Science test scores:
Median:
Ascending order: 76,78,80,80,84
Median = 3rd observation = 80
Interquartile range = 84 – 76 = 10

Math test scores:
Median:
Ascending order: 71,73,80,94,100
Median = 3rd observation = 80
Interquartile range = 100 – 71 = 29

Medians are equal but the interquartile range varies a large, math test scores are more spread out compared to science test scores.

Question 5.
Write a short paragraph to a new student that explains how you can compare data sets by examining the mean and the interquartile range.
Type below:
_________________

Answer: average and consistency

Explanation:
If the mean is more it means that the data has more observations or observations with more value.
Interquartile range and median range say about the consistency.

Lesson Check – Page No. 744

Question 1.
Team A has a mean of 35 points and a range of 8 points. Team B has a mean of 30 points and a range of 7 points. Write a statement that compares the data.
Type below:
_________________

Answer: Similar variation but team A average is more than team B

Explanation:
The range has only a difference of 1 point which can be said as a slight variation but while the average/ mean has a large variation.

Question 2.
Jean’s test scores have a mean of 83 and an interquartile range of 4. Ben’s test scores have a mean of 87 and an interquartile range of 9. Compare the students’ scores.
Type below:
_________________

Answer: Ben’s average is more than Jean’s but Ben is less consistent compared to Jean.

Explanation:
Ben’s average score is more than the average scores of Jean while the interquartile range

Spiral Review

Question 3.
Look at the box plots below. What is the difference between the medians for the two groups of data?
Go Math Grade 6 Answer Key Chapter 13 Variability and Data Distributions img 45
_______ students

Answer: 2 students

Explanation:
Median of students in a class of school A = 24
Median of students in a class of school B = 26
Difference between the medians of the schools = 26-24 = 2 students

Question 4.
The distances in miles that 6 people drive to get to work are 10, 11, 9, 12, 9, and 27. What measure of center best describes the data set?
Type below:
_________________

Answer: Median is the centre best describes the data set

Explanation:
Median:
AScending order: 9,9,10,11,12,27
Median= 3rd and 4th observations = 10+11/2 = 21/2 = 10.5

Question 5.
Which two teams typically practice the same number of hours, but have very different variations in their practice times?
Go Math Grade 6 Answer Key Chapter 13 Variability and Data Distributions img 46
Type below:
_________________

Answer: Team A and C

Explanation:
Team A and C have medians which nare only slightly different but the consistency varies a lot that is range.

Share and Show – Page No. 747

Connie asked people their ages as they entered the food court at the mall. Use the histogram of the data she collected for 1–5.
Go Math Grade 6 Answer Key Chapter 13 Variability and Data Distributions img 47

Question 1.
What statistical question could Connie ask about her data?
Type below:
_________________

Answer: Mean, Median, Mode

Explanation:
The graph shows the age and number of people. The questions which can be asked can be of mean, median, mode.

Question 2.
Describe any peak or gap in the data.
Type below:
_________________

Answer: Peak : 21-30   Gap : 61-70

Explanation:
There is a peak in the graph at the interval 21-30
There is a gap in between the bars of the histogram the bar which had a gap before it was 61-70

Question 3.
Does the graph have symmetry? Explain your reasoning.
Type below:
_________________

Answer: No the graph doesn’t have symmetry

Explanation:
The symmetry of the graph means there must be equal parts of the graph on both the sides of the line of the graph.
This is not possible in the above situation.

On Your Own

Question 4.
The lower quartile of the data set is 16.5 years, and the upper quartile is 51.5 years. Find the interquartile range. Is it a better description of the data than the range? Explain your reasoning.
Type below:
_________________

Answer: Interquartile range = 35; The interquartile range is better than the range.

Explanation:
The interquartile range is the difference between the upper quartile and the lower quartile.
Interquartile range = 51.5-16.5 = 35
The interquartile range is better than the range because if we take the example of the above graph we can see thatthe most of the data fall in the range of the interquartile range ie. 35.
Therefore we can say that the interquartile range is better than the range.

Question 5.
Make Arguments The mode of the data is 16 years old. Is the mode a good description of the center of the data? Explain
Type below:
_________________

Answer: No mode is not a good description of the data.

Explanation:
The mode is just a frequently occurring observation.
It cannot be the best way to describe the data.

Problem Solving + Applications – Page No. 748

Use the dot plot for 6–8.

Question 6.
Make Arguments Jason collected data about the number of songs his classmates bought online over the past 3 weeks. Does the data set have symmetry? Why or why not?
Go Math Grade 6 Answer Key Chapter 13 Variability and Data Distributions img 48
Type below:
_________________

Answer: No, the data has no symmetry.

Explanation:
No, the data has no symmetry. Because there are gaps between the dots drawn.

Question 7.
Jason claims that the median is a good description of his data set, but the mode is not. Does his statement make sense? Explain.
Type below:
_________________

Answer: Median can be a better centre of description. Therefore his statement makes a sense.

Explanation:
Median = 7+8/2 = 15/2 = 7.5
The number 7.5 represents more number of observations.

Question 8.
Trinni surveyed her classmates about how many siblings they have. A dot plot of her data increases from 0 siblings to a peak at 1 sibling and then decreases steadily as the graph goes to 6 siblings. How is Trinni’s dot plot similar to Jason’s? How is it different?
Type below:
_________________

Answer: Trinni graph represents a part of Jason’s graph

Explanation:
In Jason’s graph, there is a peak in the middle and then it decreases on both sides.
But according to Trinni graph, there is no peak in the middle.

Question 9.
Diego collected data on the number of movies seen last month by a random group of students.
Go Math Grade 6 Answer Key Chapter 13 Variability and Data Distributions img 49
Draw a box plot of the data and use it to find the interquartile range and range.
Type below:
_________________

Answer: Range = 12  Interquartile range = 2

Explanation:
The range is the difference between the highest and lowest observations.
Range = 12-0 = 12
The interquartile range is the difference between the upper and lower quartiles.
Ascending order: 0,0,1,1,2,2,2,2,3,3,3,5,12
Lower quartile = 1+1/2 = 2/2 = 1
Upper quartile = 3+3/2 = 6/2 = 3
Interquartile range = 3-1 = 2

Describe Distributions – Page No. 749

Chase asked people how many songs they have bought online in the past month. Use the histogram of the data he collected for 1–4.
Go Math Grade 6 Answer Key Chapter 13 Variability and Data Distributions img 50

Question 1.
What statistical question could Chase ask about the data?
Type below:
_________________

Answer: What is the median number of songs purchased?

Explanation:
Many questions can be formed from the data given to us
We can ask about the mean, median, mode.

Question 2.
Describe any peaks in the data.
Type below:
_________________

Answer: Peak : 0-4

Explanation:
The peak is a bar in the histogram which has the highest value. The peak of the given graph is 0-4

Question 3.
Describe any gaps in the data.
Type below:
_________________

Answer: There are no gaps in the graph.

Explanation:
The gap is something between a bar of the histogram and all the other adjacent bars.
There no such case in the graph. Therefore there are no gaps in the graph.

Question 4.
Does the graph have symmetry? Explain your reasoning.
Type below:
_________________

Answer: No the graph doesn’t have symmetry

Explanation:
The symmetry of the graph means there must be equal parts of the graph on both the sides of the line of the graph.
This is not possible in the above situation.

Problem Solving

Question 5.
Mr. Carpenter teaches five classes each day. For several days in a row, he kept track of the number of students who were late to class and displayed the results in a dot plot. Describe the data.
Go Math Grade 6 Answer Key Chapter 13 Variability and Data Distributions img 51
Type below:
_________________

Answer: Peaks: At 6 and 8
Gaps: Between 3 and 5 , 6 and 8
Clusters: Between 0-3 ; 9-11

Explanation:
The highest points in the graph are known as peaks.
They are at 6 and 8 in this graph.
The space between the dots in the dot plot graph is known as gaps.
Gaps are between 3 and 5; 6 and 8
The group of the dots in the dot plot are known as clusters.
The clusters are at the 0-3; 9-11

Question 6.
Describe how a graph of a data set can be used to understand the distribution of the data.
Type below:
_________________

Answer: Mean, median, mode

Explanation:
There are three measures of centre which can be used to describe the data given in the form of a graph.
The three measures of centre are mean, median, mode.

Lesson Check – Page No. 750

Question 1.
The ages of people in a restaurant are 28, 10, 44, 25, 18, 8, 47, and 30. What is the median age of the people in the restaurant?
_______ years old

Answer: Median age of the people in the restaurant is 26.5 approximately 27

Explanation:
Median:
Ascending order: 8,10,18,25,28,30,44,47
Median = Mean of 4th and 5th observations = 25+28/2 = 53/2 = 26.5

Question 2.
What is the median in the dot plot?
Go Math Grade 6 Answer Key Chapter 13 Variability and Data Distributions img 52
$ ________

Answer: 11

Explanation:
Median is the middlemost value and it is 11 in the above graph.
We need to consider the middle value by neglecting the same number on both the sides.

Spiral Review

Question 3.
Look at the dot plot. Where does a gap occur in the data?
Go Math Grade 6 Answer Key Chapter 13 Variability and Data Distributions img 53
Type below:
_________________

Answer: 30-33

Explanation:
The gap is a space between the intervals.
The intervals are 30-33.

Question 4.
Look at the dot plot. Where does a peak occur in the data?
Go Math Grade 6 Answer Key Chapter 13 Variability and Data Distributions img 54
Type below:
_________________

Answer: 37

Explanation:
The highest point in the graph is known as the peak.
The peak in the dot plot is 37.

Question 5.
Which two teams had similar variations in points earned, but typically earned a different number of points per game?
Go Math Grade 6 Answer Key Chapter 13 Variability and Data Distributions img 55
Type below:
_________________

Answer: Red and blue

Explanation:
The difference between the upper and lowest observations is called a range.
The range (consistency) in the data given is the same but they vary in the mean.
But we can say that Red and Blue teams typically earned a different number of points per game.

Question 6.
Manny’s monthly electric bills for the past 6 months are $140, $165, $145, $32, $125, and $135. What measure of center best represents the data?
Type below:
_________________

Answer: Median is the best way to represent the data. Median= 137.5

Explanation:
Median:
Ascending order: 32,125,135,140,145,165
Median = 135+140/2 = 275/2 = 137.5

Share and Show – Page No. 753

Question 1.
Josh is playing a game at the carnival. If his arrow lands on a section marked 25 or higher, he gets a prize. Josh will only play if most of the players win a prize. The carnival worker says that the average (mean) score is 28. The box plot shows other statistics about the game. Should Josh play the game? Explain your reasoning.
Go Math Grade 6 Answer Key Chapter 13 Variability and Data Distributions img 56
First, look at the median. The median is _ points.
Next, work backwards from the statistics.
The median is the __ value of the data.
So, at least __ of the values are scores
less than or equal to _.
Finally, use the statistics to draw a conclusion.
Type below:
_________________

Answer: middlemost value; 20; 15

Explanation:
First, look at the median. The median is 20 points.
Next, work backwards from the statistics.
The median is the middlemost value of the data.
So, at least 20 of the values are scores
less than or equal to 15.
Finally, use the statistics to draw a conclusion.

Question 2.
What if a score of 15 or greater resulted in a prize? How would that affect Josh’s decision? Explain.
Type below:
_________________

Answer: It doesn’t affect his decision.

Explanation:
Josh wanted to play only when most of the prizes were awarded.
Therefore if the minimum score was 15 or greater than it then they would get the prize. So there will be no difference in his decision.

Question 3.
A store collects data on the sales of DVD players each week for 3 months. The manager determines that the data has a range of 62 players and decides that the weekly sales were very inconsistent. Use the statistics in the table to decide if the manager is correct. Explain your answer.
Go Math Grade 6 Answer Key Chapter 13 Variability and Data Distributions img 57
Type below:
_________________

Answer: No the manager is not correct.

Explanation:
The range is given to be 62.
The range is correct when we check it with the help of the given data, therefore, the data is not inconsistent.

On Your Own – Page No. 754

Question 4.
Gerard is fencing in a yard that is 21 feet by 18 feet. How many yards of fencing material does Gerard need? Explain how you found your answer.
_______ yards

Answer: 78 yards of the fencing material is required

Explanation:
Length = 21 feet  Breadth = 18 feet
The perimeter of the rectangle = The number of yards of fencing material required = 2(l+b) = 2(21+18) = 2(39) = 78 yards

Question 5.
Susanna wants to buy a fish that grows to be about 4 in. long. Mark suggests she buys the same type of fish he has. He has five of these fish with lengths of 1 in., 1 in., 6 in., 6 in., and 6 in., with a mean length of 4 in. Should Susanna buy the type of fish that Mark suggests? Explain.
Type below:
_________________

Answer: Yes Susanna buy the type of fish that Mark suggests

Explanation:
The length of fish suggested by Mark is 4 in. long.
Mark has 5 fishes with a mean length of 4 in.
To buy a fish of that grows to be about 4 in. long. Susanna should buy the fishes suggested by Mark.

Question 6.
Look for a Pattern The graph shows the number of stamps that Luciano collected over several weeks. If the pattern continues, how many stamps will Luciano collect in Week 8? Explain.
Go Math Grade 6 Answer Key Chapter 13 Variability and Data Distributions img 58
_______ stamps

Answer: 7 stamps

Explanation:
In week 4 and 6, the number of stamps are 4, 5. Therefore in week 6 and 8, the number of stamps are 5, 7

Question 7.
The data set shows the number of hours Luke plays the piano each week. Luke says he usually plays the piano 3 hours per week. Why is Luke’s statement misleading?
Go Math Grade 6 Answer Key Chapter 13 Variability and Data Distributions img 59
Type below:
_________________

Answer: According to the question he should spend 3 hours per week. His statement is correct.

Explanation:
Sum of the data = 1+2+1+3+2+10+2 = 21
Number of days in a week = 7
Mean = 21/7 = 3 hours

Problem Solving Misleading Statistics – Page No. 755

Mr Jackson wants to make dinner reservations at a restaurant that has most meals costing less than $16. The Waterside Inn advertises that they have meals that average $15. The table shows the menu items.
Go Math Grade 6 Answer Key Chapter 13 Variability and Data Distributions img 60

Question 1.
What is the minimum price and the maximum price?
minimum: $ _________
maximum: $ _________

Answer: minimum: $6
maximum: $19

Explanation:
The minimum value is the most minimum price in the given data.
The maximum value is the most maximum price in the given data.

Question 2.
What is the mean of the prices?
$ ________

Answer: $15

Explanation:
Mean = sum of all the observations/ total number of observations = 6+16+18+16+18+19/6 = 93/6 = 15.2
Approximately therefore the mean of the observations is $15

Question 3.
Construct a box plot for the data.
Type below:
_________________

Answer: The box plot is a diagram which signifies the information about the data.

Explanation:

The box plot represents the range, lower and upper quartiles.

Question 4.
What is the range of the prices?
$ ________

Answer: the range is $13

Explanation:
The difference between the upper and lower observations is known as the range.
Range = 19-6 = $13

Question 5.
What is the interquartile range of the prices?
$ ________

Answer: $7.5

Explanation:
Ascending order: $6, $16, $16, $18, $18, $19
Median = 16+18/2 = 34/2 = 17
Lower quartile = 6+16/2 = 11
Upper quartile = 18+19/2 = 18.5
Interquartile range = 18.5-11 = 7.5

Question 6.
Does the menu match Mr. Jackson’s requirements? Explain your reasoning.
Type below:
_________________

Answer: Yes the menu matches Mr Jackson’s requirements.

Explanation:
Mr Jackson wants to make dinner arrangements with cost less than $16.
The mean of the items in the menu:
Mean = $6+$16+$16+$18+$18+$19/6 = 93/6 = $15.5
Therefore the requirements of Mr Jackson is satisfied.

Question 7.
Give an example of a misleading statistic. Explain why it is misleading.
Type below:
_________________

Answer: The Waterside Inn advertises the misleading statement.

Explanation:
According to the information given in the question, The Waterside Inn advertises that they have meals that average $15. But it is more than that, so this is the misleading statement.
The mean of the items in the menu:
Mean = $6+$16+$16+$18+$18+$19/6 = 93/6 = $15.5
Therefore the requirements of Mr Jackson is satisfied.

Lesson Check – Page No. 756

Question 1.
Mary’s science test scores are 66, 94, 73, 81, 70, 84, and 88. What is the range of Mary’s science test scores?
________

Answer: 28

Explanation:
The difference between the highest and the lowest observations is called a range.
Range = 94 – 66 = 28

Question 2.
The heights in inches of students on a team are 64, 66, 60, 68, 69, 59, 60, and 70. What is the interquartile range?
________

Answer: Interquartile range = 9

Explanation:
Ascending order: 59,60,60,64,66,68,69,70
Median = Mean of 64 and 66 = 64+66/2 = 130/2 = 65
Lower quartile = 60
Upper quartile = 69
Interquartile range = 69 – 60 = 9

Spiral Review

Question 3.
By how much does the median of the following data set change if the outlier is removed?
26, 21, 25, 18, 0, 28
Type below:
_________________

Answer: The median changes by 3.5

Explanation:
Ascending order: 0,18,21,25,26,28
Median = 21+28/2 = 49/2 = 24.5
If the outlier is removed then the
Median = 21
Difference between the 1st and 2nd median = 24.5 – 21 = 3.5

Question 4.
Look at the box plot. What is the interquartile range of the data?
Go Math Grade 6 Answer Key Chapter 13 Variability and Data Distributions img 61
________

Answer: Interquartile range = 6

Explanation:
The difference between the lower and upper quartiles is known as the interquartile range.
Interquartile range = 50 – 44 = 6

Question 5.
Erin is on the school trivia team. The table shows the team’s scores in the last 8 games. Erin wants to build confidence in her team so that they will do well in the last game. If a score of 20 is considered a good score, what measure of center would be best for Erin to use to motivate her teammates?
Go Math Grade 6 Answer Key Chapter 13 Variability and Data Distributions img 62
Type below:
_________________

Answer: Mean and median are the best centre of tendencies to compare the data.

Explanation:
Mean = 20+20+18+19+23+40+22+19/8 = 181/8 = 22.6
Median:
Ascending order: 18,19,19,20,20,22,23,40
Median = 40/2 = 20

Chapter 13 Review/Test – Page No. 757

Question 1.
The dot plot shows the number of chin-ups done by a gym class.
Go Math Grade 6 Answer Key Chapter 13 Variability and Data Distributions img 63
For numbers 1a–1e, choose Yes or No to indicate whether the statement is correct.
1a. There are two peaks.
1b. There are no clusters.
1c. There is a gap from 6 to 8.
1d. The most chin-ups anyone did was 15.
1e. The modes are 3, 4, and 9.
1a. ____________
1b. ____________
1c. ____________
1d. ____________
1e. ____________

Answer: 1a. Yes
1b. No
1c. Yes
1d. No
1e. Yes

Explanation:
1a. The highest point in the dot plot is called the peak. The peak in the given dot plot is at 5 and 11 the value of the peak is 3
1b. The group of dots form a cluster with 3 or more intervals.
1c. There is a gap between the intervals 6-8
1d. The maximum number of people did 11 chin-ups while only a single person did 15 chin-ups.
1e. The most frequently occurring observation is known as mode.
The mode of the given data is at the intervals 3,4 and 9.

Question 2.
The histogram shows the high temperatures in degrees Fahrenheit of various cities for one day in March.
Go Math Grade 6 Answer Key Chapter 13 Variability and Data Distributions img 64
Select the best word to complete each sentence.
The histogram has _____ peak(s).
The histogram _____ symmetry.

Answer: The histogram has 1 peak(s).
The histogram is symmetry.

Explanation:
The is one and only one peak at the interval 41 – 50
We can say that the graph is symmetrical because if we draw a line between the graph we can observe that the graph has two parts symmetric to each other.

Chapter 13 Review/Test – Page No. 758

Question 3.
The data set shows the scores of the players on the winning team of a basketball game.
Go Math Grade 6 Answer Key Chapter 13 Variability and Data Distributions img 65
The median is _____.
The lower quartile is _____.
The upper quartile is _____.

Answer:Median = 6
Lower quartile = 1
Upper quartile =19.5

Explanation:
Ascending order: 0,0,1,1,4,5,6,9,13,17,22,30,47
Median = 6
Lower quartile = Mean of 1 and 1 = 1+1/2 = 2/2 = 1
Upper quartile = Mean of 17 and 22 = 17+22/2 = 39/2 = 19.5

Question 4.
The data set shows the number of desks in 12 different classrooms.
Go Math Grade 6 Answer Key Chapter 13 Variability and Data Distributions img 66
Find the values of the points on the box plot.
Go Math Grade 6 Answer Key Chapter 13 Variability and Data Distributions img 67
Type below:
_________________

Answer: A= 16 B=17 C= 20 D= 21 E=24

Explanation:
Ascending order: 16,17,17,18,19,20,20,21,21,21,22,24
Median = 20+20/2 = 20
Lower quartile = 17
Upper quartile = 21

Question 5.
The box plot shows the number of boxes sold at an office supply store each day for a week.
Go Math Grade 6 Answer Key Chapter 13 Variability and Data Distributions img 68
For numbers 5a–5d, select True or False for each statement.
5a. The median is 18.
5b. The range is 15.
5c. The interquartile range is 9.
5d. The upper quartile is 18.
5a. ____________
5b. ____________
5c. ____________
5d. ____________

Answer: 5a.  false
5b. true
5c. true
5d. true

Explanation:
Median is the middlemost value of the given data.
Median of the data is 14
The range is the difference between the upper and lower observations.
Range = 21-6 = 15
The interquartile range is the difference between the upper and lower observations.
Upper quartile range: 18
Interquartile range = 18-9 = 9

Chapter 13 Review/Test – Page No. 759

Question 6.
The data set shows the number of glasses of water Dalia drinks each day for a week.
Go Math Grade 6 Answer Key Chapter 13 Variability and Data Distributions img 69
Part A
What is the mean number of glasses of water Dalia drinks each day?
_______ glasses

Answer: The mean number of glasses of water Dalia drinks each day is 8 glasses.

Explanation:
Mean = sum of all the observations/ total number of observations= 6+7+9+9+8+7+10/7 = 8

Question 6.
Part B
What is the mean absolute deviation of the number of glasses of water Dalia drinks each day? Round your answer to the nearest tenth. Use words and numbers to support your answer.
_______

Answer: Mean absolute deviation is 1.14

Explanation:

Mean:
Mean = sum of all the observations/ total number of observations= 6+7+9+9+8+7+10/7 = 8

Mean absolute deviation:

STEP 1 Label each observation with its distance from the mean.
Starting from left to right:
06: 8-6  = 02
07: 8-7  = 01
09: 8-9  = -1
09: 8-9  = -1
08: 8-8  =  0
07: 8-7  = 01
10: 8-10= -2

STEP 2 Find the mean of the distances.
2+1+1+1+0+1+2/7
= 8/7 = 1.14

So, the mean absolute deviation of the data is 1.14

Question 7.
The numbers of emails Megan received each hour are 9, 10, 9, 8, 7, and 2. The mean of the data values is 7.5 and the median is 8.5. Which measure of center better describes the data, the mean or median? Use words and numbers to support your answer.
Type below:
_________________

Answer: Mean is the best center of the tendency to represent the data given in the question

Explanation:
Ascending order of the data: 2,7,8,9,9,10
Mean = 7.5
Mean represents the observations 8,9,9,10 which come after 7.5
Therefore mean is the best way to represent the data.

Question 8.
The number of miles Madelyn drove between stops was 182, 180, 181, 184, 228, and 185. Which measure of center best describes the data?
Options:
a. mean
b. median
c. mode

Answer: b. Median

Explanation:
Ascending order: 180,181,182,184,185,228
Median = 182+184/2 = 183
183 represents all the observations after 182
So the median is the best way to represent the data.

Chapter 13 Review/Test – Page No. 760

Question 9.
The histogram shows the weekly earnings of part-time workers. What interval(s) represents the most common weekly earnings?
Go Math Grade 6 Answer Key Chapter 13 Variability and Data Distributions img 70
Type below:
_________________

Answer: 321-330 ; 341-350

Explanation:
The histogram has 2 intervals which show equal heights which means that the monthly earnings of these intervals is the same.

Question 10.
Jordan surveyed a group of randomly selected smartphone users and asked them how many applications they have downloaded onto their phones. The dot plot shows the results of Jordan’s survey. Select the statements that describe patterns in the data. Mark all that apply.
Go Math Grade 6 Answer Key Chapter 13 Variability and Data Distributions img 71
Options:
a. The modes are 37 and 42.
b. There is a gap from 38 to 40.
c. There is a cluster from 41 to 44.
d. There is a cluster from 35 to 36.

Answer: b. There is a gap from 38 to 40.

Explanation:
The dot plot represents a gap between 38-40. So we can say that there is a gap between the intervals 38 to 40.

Chapter 13 Review/Test – Page No. 761

Question 11.
Mrs. Gutierrez made a histogram of the birth month of the students in her class. Describe the patterns in the histogram by completing the chart.
Go Math Grade 6 Answer Key Chapter 13 Variability and Data Distributions img 72
Go Math Grade 6 Answer Key Chapter 13 Variability and Data Distributions img 73
Type below:
_________________

Answer: There are 2 peaks, Yes there is an increase across the intervals, Yes there is a decrease across the intervals

Explanation:
The highest point in the histogram is called is as a peak.
There is a peak near the month’s May and August.

There is an increase between the bars in the bar graph.
At the months February, March, November there is an increase in the graph.

There is a decrease between the bars in the bar graph.
At the months September, October, December.

Question 12.
Ian collected data on the number of children in 13 different families.
Go Math Grade 6 Answer Key Chapter 13 Variability and Data Distributions img 74
Draw a box plot of the data and use it to find the interquartile range and range.
Type below:
_________________

Answer: Range = 8-0 = 8 Interquartile range = 3-1 = 2

Explanation:

Ascending order:
0,0,1,1,1,1,2,2,2,3,3,4,8
Median = 2
Lower quartile = 1+1/2 = 1
Upper quartile = 3+3/2 = 3
Range = 8-0 = 8
Interquartile range = 3-1 = 2

Chapter 13 Review/Test – Page No. 762

Question 13.
Gavin wants to move to a county where it rains about 5 inches every month. The data set shows the monthly rainfall in inches for a county. The mean of the data is 5 and the median is 4.35. After analyzing the data, Gavin says that this county would be a good place to move. Do you agree or disagree with Gavin? Use words and numbers to support your answer.
Go Math Grade 6 Answer Key Chapter 13 Variability and Data Distributions img 75
Type below:
_________________

Answer: Yes I agree that it is a good place to move.

Explanation:
After analyzing the data we can say that this country would be a good place to move for Gavin.
Gavin wants to move to a place which has an average of 5 cm rainfall. So this country is the best choice because it has a mean equal to 5 and median equal to 4.35

Question 14.
The data set shows the number of books Peyton reads each month. Peyton says she usually reads 4 books per month. Why is Peyton’s statement misleading?
Go Math Grade 6 Answer Key Chapter 13 Variability and Data Distributions img 76
Type below:
_________________

Answer: No Peyton’s statement is not misleading because the mean of the data is 4.
Therefore Peyton says she usually reads 4 books per month.

Explanation:
Mean = 2+3+2+4+3+11+3/7 = 28/7 = 4

Question 15.
The data set shows the scores of three players for a board game.
Go Math Grade 6 Answer Key Chapter 13 Variability and Data Distributions img 77
For numbers 15a–15d, choose Yes or No to indicate whether the statement is correct.
15a. The mean absolute deviation of Player B’s scores is 0.
15b. The mean absolute deviation of Player A’s scores is 0.
15c. The mean absolute deviation of Player B’s scores is greater than the mean absolute deviation of Player C’s scores.
15a. ___________
15b. ___________
15c. ___________

Answer:15a. No
15b. Yes
15c.  Yes

Explanation:
Mean of player A = 90+90+90/3 = 90

Mean absolute deviation:

STEP 1 Label each observation with its distance from the mean.
Starting from left to right:
90: 90-90  = 0
90: 90-90  = 0
90: 90-90  = 0

STEP 2 Find the mean of the distances.
0+0+0/3 = 0
So, the mean absolute deviation of player A is 0

Mean of player B = 110+100+90/3 = 100

Mean absolute deviation:

STEP 1 Label each observation with its distance from the mean.
Starting from left to right:
100: 100-110  = -10
100: 100-100  = 0
100: 100-90    = 10

STEP 2 Find the mean of the distances.
10+0+10/3 = 20/3 = 6.67
So, the mean absolute deviation of player B is 6.67

Mean of player C = 95+100+95/3 = 96.67

Mean absolute deviation:

STEP 1 Label each observation with its distance from the mean.
Starting from left to right:
96.67: 96.67-95    = 1.67
96.67: 96.67-100  = -3.33
96.67: 96.67-95    = 1.67

STEP 2 Find the mean of the distances.
1.67+3.33+1.67/3 = 6.67/3 = 2.22
So, the mean absolute deviation of player C is 2.22

Conclusion:

The solutions with explanations to all the questions are prepared by the math experts as per the latest syllabus. So, go through the solutions and overcome your difficulties in this chapter. Keep in touch with this page to get the solutions to all the 6th-grade chapters.

Go Math Grade 8 Answer Key Chapter 9 Transformations and Congruence

go-math-grade-8-chapter-9-transformations-and-congruence-answer-key

There are multiple ways to learn maths. But choosing the best material is also important for the students to score good marks and also to improve their knowledge. This will be possible only with the help of our Go Math Grade 8 Answer Key Chapter 9 Transformations and Congruence. You can get the explanation for all the questions in detail in Go Math Grade 8 Chapter 9 Transformations and Congruence Answer Key here. Refer to Go Math Grade 8 Answer Key for learning the problems in an easy manner.

Go Math Grade 8 Answer Key Chapter 9 Transformations and Congruence

Practice makes you perfect become a master in maths. Without proper knowledge on the subject, you cannot solve the problems in real-time. This will affect your exam. So, keeping this in our mind we have prepared the solutions briefly which will help you to understand the concept in depth. Get free access to Download Go Math Grade 8 Chapter 9 Transformations and Congruence Solution Key pdf. In this article, you will get the solutions according to the topics. Therefore, students who want to score good marks in the exam must practice with Go Math Grade 8 Answer Key Chapter 9 Transformations and Congruence.

Lesson 1: Properties of Translations

Lesson 2: Properties of Reflections

Lesson 3: Algebraic Representations of Transformations

Lesson 4: Congruent Figures

Model Quiz

Mixed Review

Guided Practice – Properties of Translations – Page No. 282

Question 1.
Vocabulary A __________________is a change in the position, size, or shape of a figure.
____________

Answer:
transformation

Explanation:
A transformation is a change in the position, size, or shape of a figure.

Question 2.
Vocabulary When you perform a transformation of a figure on the coordinate plane, the input of the transformation is called the ________________, and the output of the transformation is called the_________________ .
Type below:
____________

Answer:
pre-image
image

Explanation:
When you perform a transformation of a figure on the coordinate plane, the input of the transformation is called the pre-image, and the output of the transformation is called the image.

Question 3.
Joni translates a right triangle 2 units down and 4 units to the right. How does the orientation of the image of the triangle compare with the orientation of the preimage?
Orientation is: _______

Answer:
Orientation is: Same

Explanation:
Since translation does not change the shape and size of a geometric figure, the two triangles are identical in shape and size, so they are congruent and the orientation is the same

Question 4.
Rashid drew rectangle PQRS on a coordinate plane. He then translated the rectangle 3 units up and 3 units to the left and labeled the image P ‘Q ‘R ‘S ‘. How do rectangle PQRS and rectangle P ‘Q ‘R ‘S ‘ compare?
They are: _______

Answer:
congruent

Explanation:
Since translation does not change the shape and size of a geometric figure, the two rectangles are identical in shape and size, so they are congruent.

Question 5.
The figure shows trapezoid WXYZ. Graph the image of the trapezoid after a translation of 4 units up and 2 units to the left.
Go Math Grade 8 Answer Key Chapter 9 Transformations and Congruence Lesson 1: Properties of Translations img 1
Type below:
____________

Answer:
After translation:
W'(-4, 3)
X'(2, 3)
Y'(1, 1)
Z'(-3, 1)

ESSENTIAL QUESTION CHECK-IN

Question 6.
What are the properties of translations?
Type below:
____________

Answer:
-> a translation is a geometric transformation that moves every point of a figure or space by the same amount in a given direction.
-> So the figures are identical and are congruent.

9.1 Independent Practice – Properties of Translations – Page No. 283

Question 7.
The figure shows triangle DEF.
Go Math Grade 8 Answer Key Chapter 9 Transformations and Congruence Lesson 1: Properties of Translations img 2
a. Graph the image of the triangle after the translation that maps point D to point D ‘.
Type below:
____________

Answer:
2 left, and 4 down

Question 7.
b. How would you describe the translation?
Type below:
____________

Answer:
It has the same size, shape. and orientation, but a different location

Question 7.
c. How does the image of triangle DEF compare with the preimage?
____________

Answer:
congruent

Question 8.
a. Graph quadrilateral KLMN with vertices K(-3, 2), L(2, 2), M(0, -3), and N(-4, 0) on the coordinate grid.
Go Math Grade 8 Answer Key Chapter 9 Transformations and Congruence Lesson 1: Properties of Translations img 3
Type below:
____________

Question 8.
b. On the same coordinate grid, graph the image of quadrilateral KLMN after a translation of 3 units to the right and 4 units up.
Type below:
____________

Answer:
grade 8 chapter 9 image 1

Question 8.
c. Which side of the image is congruent to side \(\overline { LM } \)?
___________
Name three other pairs of congruent sides.
___________
Type below:
____________

Answer:
Line LM is congruent to Line L!M!
Line KL is equal to K’L’
Line MN is equal to M’N’
Line KN is equal to K’N’

Draw the image of the figure after each translation.

Question 9.
4 units left and 2 units down
Go Math Grade 8 Answer Key Chapter 9 Transformations and Congruence Lesson 1: Properties of Translations img 4
Type below:
____________

Answer:
After translation
P'(-3, 1)
Q'(0, 2)
R'(0, -1)
S'(-3, -3)

Question 10.
5 units right and 3 units up
Go Math Grade 8 Answer Key Chapter 9 Transformations and Congruence Lesson 1: Properties of Translations img 5
Type below:
____________

Answer:
After translation
A'(0, 4)
B'(3, 5)
C'(3, 1)
D'(0, 0)

Properties of Translations – Page No. 284

Question 11.
The figure shows the ascent of a hot air balloon. How would you describe the translation?
Go Math Grade 8 Answer Key Chapter 9 Transformations and Congruence Lesson 1: Properties of Translations img 6
Type below:
____________

Answer:
4 units along positive X and 5 units along positive Y

Explanation:
Initial coordinate of balloon = ( -2 , -4)
Final coordinates of the balloon = (2,1)
Translation along x axis = 2 – (-2)
= 4 units along positive x direction
Translation along y axis = 1-(-4)
= 5 units along the positive y direction

Question 12.
Critical Thinking Is it possible that the orientation of a figure could change after it is translated? Explain.
_________

Answer:
No, it is not possible to change the orientation just by translation. As translation means, a transformation in which a figure is moved to another location without any change in size or orientation.

FOCUS ON HIGHER ORDER THINKING

Question 13.
a. Multistep Graph triangle XYZ with vertices X(-2, -5), Y(2, -2), and Z(4, -4) on the coordinate grid.
Go Math Grade 8 Answer Key Chapter 9 Transformations and Congruence Lesson 1: Properties of Translations img 7

Question 13.
b. On the same coordinate grid, graph and label triangle X’Y’Z’, the image of triangle XYZ after a translation of 3 units to the left and 6 units up.

Question 13.
c. Now graph and label triangle X”Y”Z”, the image of triangle X’Y’Z’ after a translation of 1 unit to the left and 2 units down.
Type below:
____________

Answer:
grade 8 chapter 9 image 2

Question 13.
d. Analyze Relationships How would you describe the translation that maps triangle XYZ onto triangle X”Y”Z”?
Type below:
____________

Answer:
Triangle XYZ has translated 4 units up and 4 units to the left

Question 14.
Critical Thinking The figure shows rectangle P’Q’R’S’, the image of rectangle PQRS after a translation of 5 units to the right and 7 units up. Graph and label the preimage PQRS.
Go Math Grade 8 Answer Key Chapter 9 Transformations and Congruence Lesson 1: Properties of Translations img 8
Type below:
____________

Answer:
grade 8 chapter 9 image 3

Question 15.
Communicate Mathematical Ideas Explain why the image of a figure after a translation is congruent to its preimage.
Type below:
____________

Answer:
A translation is a geometric transformation that moves every point of a figure or space by the same amount in a given direction. So the 2 figures are identical and the translated figure is congruent to its pre-image.

Guided Practice – Properties of Reflections – Page No. 288

Question 1.
Vocabulary A reflection is a transformation that flips a figure across a line called the __________ .
____________

Answer:
Reflection Axis

Explanation:
A reflection is a transformation that flips a figure across a line called the Reflection Axis.

Question 2.
The figure shows trapezoid ABCD.
Go Math Grade 8 Answer Key Chapter 9 Transformations and Congruence Lesson 2: Properties of Reflections img 9
a. Graph the image of the trapezoid after a reflection across the x-axis. Label the vertices of the image.
Type below:
____________

Answer:
A'(-3, -4)
B'(1, -4)
C'(3, -1)
D'(-3, -1)

Question 2.
b. How do trapezoid ABCD and trapezoid A’B’C’D’ compare?
____________

Answer:
congruent

Explanation:
trapezoid ABCD and trapezoid A’B’C’D’ are congruent

Question 2.
c. What If? Suppose you reflected trapezoid ABCD across the y-axis. How would the orientation of the image of the trapezoid compare with the orientation of the preimage?
Type below:
____________

Answer:
The orientation would be reversed horizontally.

ESSENTIAL QUESTION CHECK-IN

Question 3.
What are the properties of reflections?
Type below:
____________

Answer:
properties of reflections
-> the size stays the same
-> the shape stays the same
-> the orientation does NOT stay the same

9.2 Independent Practice – Properties of Reflections – Page No. 289

The graph shows four right triangles. Use the graph for Exercises 4-7.
Go Math Grade 8 Answer Key Chapter 9 Transformations and Congruence Lesson 2: Properties of Reflections img 10

Question 4.
Which two triangles are reflections of each other across the x-axis?
Type below:
____________

Answer:
Triangles A and C are the reflections of each other across the x-axis.

Question 5.
For which two triangles is the line of reflection the y-axis?
Type below:
____________

Answer:
For triangle C & D the line of reflection is y-axis.

Question 6.
Which triangle is a translation of triangle C? How would you describe the translation?
Type below:
____________

Answer:
Triangle B is the translation of triangle C.
Lets take any one point of the triangle = (-2, -6)
Lets take the corresponding side of triangle B = (4,2)
Translation across x axis = 4 -(-2) = 6 units
Translation across y axis = 2 -(-6) = 8 units

Question 7.
Which triangles are congruent? How do you know?
Type below:
____________

Answer:
All the 4 triangles A, B, C, D are congruent.
The length of base and height of all the four triangles are 3 units, 4 units respectively.

Explanation:
All the 4 triangles A, B, C, D are congruent.
If base and height are equal then the hypotenuse should also be equal. Thus all three sides of the triangles A,B,C,D are equal. Thus these triangles are congruent,
The length of base and height of all the four triangles are 3 units, 4 units respectively.

Question 8.
a. Graph quadrilateral WXYZ with vertices W(-2, -2), X(3, 1), Y(5, -1), and Z(4, -6) on the coordinate grid.
Type below:
____________

Question 8.
b. On the same coordinate grid, graph quadrilateral W’X’Y’Z’, the image of quadrilateral WXYZ after a reflection across the x-axis.
Type below:
____________

Answer:
grade 8 chapter 9 image 4

Question 8.
c. Which side of the image is congruent to side \(\overline { YZ } \)?
_______________
Name three other pairs of congruent sides.
_______________
Type below:
____________

Answer:
Line YZ = Line Y’Z’
Line WX = Line W’X’
Line XY = Line X’Y’
Line WZ = Line W’Z’

Question 8.
d. Which angle of the image is congruent to ∠X?
_______________
Name three other pairs of congruent angles.
_______________
Type below:
____________

Answer:
Angle X’
Angle W and Angle W’
Angle Y and Angle Y’
Angle Z and Angle Z’

Properties of Reflections – Page No. 290

Question 9.
Critical Thinking Is it possible that the image of a point after a reflection could be the same point as the preimage? Explain.
________

Answer:
Yes

Explanation:
It is possible that the image of a point after a reflection could be the same point as the preimage

FOCUS ON HIGHER ORDER THINKING

Question 10.
a. Graph the image of the figure shown after a reflection across the y-axis.
Go Math Grade 8 Answer Key Chapter 9 Transformations and Congruence Lesson 2: Properties of Reflections img 11
Type below:
____________

Answer:
grade 8 chapter 9 image 5

Question 10.
b. On the same coordinate grid, graph the image of the figure you drew in part a after a reflection across the x-axis.
Type below:
____________

Answer:
grade 8 chapter 9 image 6

Question 10.
c. Make a Conjecture What other sequence of transformations would produce the same final image from the original preimage? Check your answer by performing the transformations. Then make a conjecture that generalizes your findings.
Type below:
____________

Answer:
The same image can be obtained by reflecting first across the x-axis and then across the y-axis.
Reflecting a figure first across the y-axis and then across the x-axis has the same outcome,. reflecting first across the x-axis and then across the y-axis.

Question 11.
a. Graph triangle DEF with vertices D(2, 6), E(5, 6), and F(5, 1) on the coordinate grid.

Question 11.
b. Next graph triangle D ′E ′F ′, the image of triangle DEF after a reflection across the y-axis.
Type below:
____________

Question 11.
c. On the same coordinate grid, graph triangle D′′ E′′ F′′, the image of triangle D ′E ′F ′ after a translation of 7 units down and 2 units to the right.
Type below:
____________

Answer:
grade 8 chapter 9 image 7

Question 11.
d. Analyze Relationships Find a different sequence of transformations that will transform triangle DEF to triangle D ′′E ′′F ′′.
Type below:
____________

Answer:
Translate triangle DEF 7 units down and 2 units to the left. Then reflect the image across the y-axis.

Guided Practice – Properties of Reflections – Page No. 294

Question 1.
Vocabulary A rotation is a transformation that turns a figure around a given _____ called the center of rotation.
____________

Answer:
point

Explanation:
A rotation is a transformation that turns a figure around a given point called the center of rotation.

Siobhan rotates a right triangle 90° counterclockwise about the origin.

Question 2.
How does the orientation of the image of the triangle compare with the orientation of the preimage?
Type below:
____________

Answer:
Each leg in the preimage is perpendicular to its corresponding leg in the image.

Question 3.
Is the image of the triangle congruent to the preimage?
______

Answer:
Yes

Explanation:
The image of the triangle is congruent to the preimage

Draw the image of the figure after the given rotation about the origin.

Question 4.
90° counterclockwise
Go Math Grade 8 Answer Key Chapter 9 Transformations and Congruence Lesson 3: Properties of Rotation img 12
Type below:
____________

Answer:
grade 8 chapter 9 image 8

Question 5.
180°
Go Math Grade 8 Answer Key Chapter 9 Transformations and Congruence Lesson 3: Properties of Rotation img 13
Type below:
____________

Answer:
After 180° rotation
A'(-2, -3)
B'(-4, -1)
C'(-2, 0)
D'(0, -1)

ESSENTIAL QUESTION CHECK-IN

Question 6.
What are the properties of rotations?
Type below:
____________

Answer:
Rotations preserve size and shape but change orientation.

9.3 Independent Practice – Properties of Reflections – Page No. 295

Question 7.
The figure shows triangle ABC and a rotation of the triangle about the origin.
Go Math Grade 8 Answer Key Chapter 9 Transformations and Congruence Lesson 3: Properties of Rotation img 14
a. How would you describe the rotation?
____________

Answer:
ABC was rotated 90º counterclockwise about the origin

Question 7.
b. What are the coordinates of the image?
Type below:
____________

Answer:
A'(3, 1)
B'(2, 3)
C'(-1, 4)

Question 8.
The graph shows a figure and its image after a transformation.
Go Math Grade 8 Answer Key Chapter 9 Transformations and Congruence Lesson 3: Properties of Rotation img 15
a. How would you describe this as a rotation?
____________

Answer:
The figure was rotated 180º about the origin.

Question 8.
b. Can you describe this as a transformation other than a rotation? Explain.
____________

Answer:
Yes

Explanation:
This can also be described as a reflection across the y-axis.

Question 9.
What type of rotation will preserve the orientation of the H-shaped figure in the grid?
Go Math Grade 8 Answer Key Chapter 9 Transformations and Congruence Lesson 3: Properties of Rotation img 16
____________

Answer:
A 180º rotation about the origin will preserve the orientation of the H-shaped figure in the grid.

Question 10.
A point with coordinates (-2, -3) is rotated 90° clockwise about the origin. What are the coordinates of its image?
(_______ , _______)

Answer:
(-3, 2)

Explanation:
The new coordinates are (-3, 2)

Complete the table with rotations of 180° or 90°. Include the direction of rotation for rotations of 90°.

Question 11.
Go Math Grade 8 Answer Key Chapter 9 Transformations and Congruence Lesson 3: Properties of Rotation img 17
Type below:
____________

Answer:
grade 8 chapter 9 image 10

Properties of Reflections – Page No. 296

Draw the image of the figure after the given rotation about the origin.

Question 14.
180°
Go Math Grade 8 Answer Key Chapter 9 Transformations and Congruence Lesson 3: Properties of Rotation img 18
Type below:
____________

Answer:
After 180°
A'(4, 0)
B'(2, -1)
C'(0, 0)
D'(2, 1)

Question 15.
270° counterclockwise
Go Math Grade 8 Answer Key Chapter 9 Transformations and Congruence Lesson 3: Properties of Rotation img 19
Type below:
____________

Answer:
After 270º counterclockwise rotation
A'(1, 2)
B'(2, -1)
C'(4, 2)

Question 16.
Is there a rotation for which the orientation of the image is always the same as that of the preimage? If so, what?
______

Answer:
Yes

Explanation:
A 360º rotation will always be the same as the original image

FOCUS ON HIGHER ORDER THINKING

Question 17.
Problem Solving Lucas is playing a game where he has to rotate a figure for it to fit in an open space. Every time he clicks a button, the figure rotates 90 degrees clockwise. How many times does he need to click the button so that each figure returns to its original orientation?
Figure A ____________
Figure B ____________
Figure C ____________
Go Math Grade 8 Answer Key Chapter 9 Transformations and Congruence Lesson 3: Properties of Rotation img 20
Figure A: _________ time(s)
Figure B: _________ time(s)
Figure C: _________ time(s)

Answer:
Figure A: 2 time(s)
Figure B: 1 time(s)
Figure C: 4 time(s)

Explanation:
2 times to return to original orientation
1 time to return to original orientation
4 times to return to original orientation

Question 18.
Make a Conjecture Triangle ABC is reflected across the y-axis to form the image A′B′C′. Triangle A′B′C′ is then reflected across the x-axis to form the image A″B″C″. What type of rotation can be used to describe the relationship between triangle A″B″C″ and triangle ABC?
Type below:
____________

Answer:
Triangle A’B’C’ is a 90º rotation of triangle ABC
Triangle A”B”C” is a 90º rotation of triangle A’B’C’
Therefore, Triangle A”B”C” is a 180º rotation of triangle ABC

Question 19.
Communicate Mathematical Ideas Point A is on the y-axis. Describe all possible locations of image A′ for rotations of 90°, 180°, and 270°. Include the origin as a possible location for A.
Type below:
____________

Answer:
If Point A is on the y-axis, Point A’ will be on the x-axis for 190° and 270° rotations and on the y-axis for 180° rotation
If point A is at the origin,
A’ is at the origin for any rotation about the origin.

Guided Practice – Algebraic Representations of Transformations – Page No. 300

Question 1.
Triangle XYZ has vertices X(-3, -2), Y(-1, 0), and Z(1, -6). Find the vertices of triangle X′Y′Z′ after a translation of 6 units to the right. Then graph the triangle and its image.
Go Math Grade 8 Answer Key Chapter 9 Transformations and Congruence Lesson 4: Algebraic Representations of Transformations img 21
Type below:
____________

Answer:
After a translation of 6 units to the right:
X'(3, -2)
Y'(5, 0)
Z'(7, -6)

Question 2.
Describe what happens to the x- and y-coordinates after a point is reflected across the x-axis.
Type below:
____________

Answer:
The x-coordinate remains the same, while the sign of the y-coordinate changes

Question 3.
Use the rule (x, y) → (y, -x) to graph the image of the triangle at right. Then describe the transformation.
Go Math Grade 8 Answer Key Chapter 9 Transformations and Congruence Lesson 4: Algebraic Representations of Transformations img 22
Type below:
____________

Answer:
The triangle is rotated 90º clockwise about the origin

ESSENTIAL QUESTION CHECK-IN

Question 4.
How do the x- and y-coordinates change when a figure is translated right a units and down b units?
Type below:
____________

Answer:
The x-coordinates increase by a, and the y-coordinates decrease by b

9.4 Independent Practice – Algebraic Representations of Transformations – Page No. 301

Write an algebraic rule to describe each transformation.Then describe the transformation.

Question 5.
Go Math Grade 8 Answer Key Chapter 9 Transformations and Congruence Lesson 4: Algebraic Representations of Transformations img 23
Type below:
____________

Answer:
algebraic rule
(x, y) -> (x-2, y-5)
transformation
translation of 2 units to the left and 5 units down
new coordinates
M'(-4, -2)
N'(-2, -2)
O'(-1, -4)
P'(-4, -4)

Question 6.
Go Math Grade 8 Answer Key Chapter 9 Transformations and Congruence Lesson 4: Algebraic Representations of Transformations img 24
Type below:
____________

Answer:
algebraic rule
(x, y) -> (-x, -y)
transformation
rotation of 180º
new coordinates
A'(0, 0)
B'(0, -3)
C'(2, -3)
D'(2, 0)

Question 7.
Triangle XYZ has vertices X(6, -2.3), Y(7.5, 5), and Z(8, 4). When translated, X′ has coordinates (2.8, -1.3). Write a rule to describe this transformation. Then find the coordinates of Y′ and Z′.
Type below:
____________

Answer:
algebraic rule
(x, y) -> (x-3.2, y+1)
new coordinates
Y'(4.3, 6)
Z'(4.8, 5)

Question 8.
Point L has coordinates (3, -5). The coordinates of point L′ after a reflection are (-3, -5). Without graphing, tell which axis point L was reflected across. Explain your answer.
____________

Answer:
Point L was reflected on the y-axis.
When you reflect a point across the y-axis, the sign of the x-coordinate changes, and the sign of the y-coordinate remains the same

Question 9.
Use the rule (x, y) → (x – 2, y – 4) to graph the image of the rectangle. Then describe the transformation.
Go Math Grade 8 Answer Key Chapter 9 Transformations and Congruence Lesson 4: Algebraic Representations of Transformations img 25
Type below:
____________

Answer:
The rectangle is translated 2 units to the left and 4 units down

Question 10.
Parallelogram ABCD has vertices A(−2, −5\(\frac{1}{2}\)), B(−4, −5\(\frac{1}{2}\)),C(-3, -2), and D(-1, -2). Find the vertices of parallelogram A′B′C′D′ after a translation of 2 \(\frac{1}{2}\) units down.
Type below:
__________

Answer:
after a translation of 2 \(\frac{1}{2}\) units
A'(-2, -8)
B'(-4, -8)
C'(-3, -4 \(\frac{1}{2}\))
D'(-1, -4 \(\frac{1}{2}\))

Algebraic Representations of Transformations – Page No. 302

Question 11.
Alexandra drew the logo shown on half-inch graph paper. Write a rule that describes the translation Alexandra used to create the shadow on the letter A.
Go Math Grade 8 Answer Key Chapter 9 Transformations and Congruence Lesson 4: Algebraic Representations of Transformations img 26
Type below:
__________

Answer:
(x,y) –> (x+1,y-0.5)
(x+1,y-0.5) –> (x+0.5,y-0.25)

Explanation:
translation in units
(x,y) –> (x+1,y-0.5)
This step converts translation rule in units to translation rule in inches. (Divide by 2 since graph paper is half inch paper.
(x+1,y-0.5) –> (x+0.5,y-0.25)

Question 12.
Kite KLMN has vertices at K(1, 3), L(2, 4), M(3, 3), and N(2, 0). After the kite is rotated, K′ has coordinates (-3, 1). Describe the rotation, and include a rule in your description. Then find the coordinates of L′, M′, and N′.
Type below:
__________

Answer:
rotation
90 counterclockwise
rule
(x, y) -> (-y, x)
new coordinates
L'(-4, 2)
M'(-3, 3)
N'(0, 2)

FOCUS ON HIGHER ORDER THINKING

Question 13.
Make a Conjecture Graph the triangle with vertices (-3, 4), (3, 4), and (-5, -5). Use the transformation (y, x) to graph its image.
a. Which vertex of the image has the same coordinates as a vertex of the original figure? Explain why this is true.
Go Math Grade 8 Answer Key Chapter 9 Transformations and Congruence Lesson 4: Algebraic Representations of Transformations img 27
Type below:
__________

Answer:
(-5, 5) has the same coordinates

Question 13.
b. What is the equation of a line through the origin and this point?
Type below:
__________

Answer:
x and y are equal so switching x and y has no effect on the coordinates

Question 13.
c. Describe the transformation of the triangle.
Type below:
__________

Answer:
x and y are equal so switching x and y has no effect on the coordinates

Question 14.
Critical Thinking Mitchell says the point (0, 0) does not change when reflected across the x- or y-axis or when rotated about the origin. Do you agree with Mitchell? Explain why or why not.
_______

Answer:
Yes, I agree with Mitchell

Explanation:
Reflecting across the x-axis or y-axis changes the sign of the y or x coordinate 0 cannot change signs.
Rotating about the origin does not change the origin (0, 0)

Question 15.
Analyze Relationships Triangle ABC with vertices A(-2, -2), B(-3, 1), and C(1, 1) is translated by (x, y) → (x – 1, y + 3). Then the image, triangle A′B′C′, is translated by (x, y) → (x + 4, y – 1), resulting in A″B″C″.
a. Find the coordinates for the vertices of triangle A″B″C″.
Type below:
__________

Answer:
A”(-2-1+4, -2+3-1) = A”(1, 0)
B”(-3-1+4, 1+3-1) = B”(0, 3)
C”(1-1+4, 1+3-1) = C”(4, 3)

Question 15.
b. Write a rule for one translation that maps triangle ABC to triangle A″B″C″.
Type below:
__________

Answer:
(x, y) -> (x-1+4, y+3-1)
(x, y) -> (x+3, y+2)

Guided Practice – Congruent Figures – Page No. 306

Question 1.
Apply the indicated series of transformations to the rectangle. Each transformation is applied to the image of the previous transformation, not the original figure. Label each image with the letter of the transformation applied.
Go Math Grade 8 Answer Key Chapter 9 Transformations and Congruence Lesson 5: Congruent Figures img 28
A. Reflection across the y-axis
B. Rotation 90° clockwise around the origin
C. (x, y) → (x – 2, y)
D. Rotation 90° counterclockwise around the origin
E. (x, y) → (x – 7, y – 2)
Type below:
__________

Answer:
A. After transformation
(1, 3)
(1, 4)
(4, 4)
(4, 3)
B. After transformation
(3, -1)
(4, -1)
(4, -4)
(3, -4)
C. After transformation
(1, -1)
(2, -1)
(2, -4)
(1, -4)
D. After transformation
(1, 1)
(1, 2)
(4, 2)
(4, 1)
E. After transformation
(-6, -1)
(-6, 0)
(-3, 0)
(-3, -1)

Identify a sequence of transformations that will transform figure A into figure C.
Go Math Grade 8 Answer Key Chapter 9 Transformations and Congruence Lesson 5: Congruent Figures img 29

Question 2.
What transformation is used to transform figure A into figure B?
Type below:
__________

Answer:
Reflection across the y-axis

Explanation:
Reflection across the y-axis is used to transform figure A into figure B

Question 3.
What transformation is used to transform figure B into figure C?
Type below:
__________

Answer:
Translation 3 units right and 4 units down

Explanation:
Translation 3 units right and 4 units down is used to transform figure B into figure C

Question 4.
What sequence of transformations is used to transform figure A into figure C? Express the transformations algebraically.
Type below:
__________

Answer:
Reflection across the y-axis is used to transform figure A into figure B
Translation 3 units right and 4 units down is used to transform figure B into figure C
Algebraically:
(x, y) -> (-x, y)
(x, y) -> (x +3, y-4)

Question 5.
Vocabulary What does it mean for two figures to be congruent?
Type below:
__________

Answer:
Two figures are congruent when the figures have the same size and the same shape.

ESSENTIAL QUESTION CHECK-IN

Question 6.
After a sequence of translations, reflections, and rotations, what is true about the first figure and the final figure?
Type below:
__________

Answer:
After a sequence of translations, reflections, and rotations, the first and final figures have the same size and shape. (They are congruent)

9.5 Independent Practice – Congruent Figures – Page No. 307

For each given figure A, graph figures B and C using the given sequence of transformations. State whether figures A and C have the same or different orientation.

Question 7.
Go Math Grade 8 Answer Key Chapter 9 Transformations and Congruence Lesson 5: Congruent Figures img 30
Figure B: a translation of 1 unit to the right and 3 units up
Figure C: a 90° clockwise rotation around the origin
Type below:
__________

Answer:
Different orientation

Explanation:
grade 8 chapter 9 image 11
Different orientation

Question 8.
Go Math Grade 8 Answer Key Chapter 9 Transformations and Congruence Lesson 5: Congruent Figures img 31
Figure B: a reflection across the y-axis
Figure C: a 180° rotation around the origin
Type below:
__________

Answer:
Different orientation

Explanation:
grade 8 chapter 9 image 12
Different orientation

Question 9.
Go Math Grade 8 Answer Key Chapter 9 Transformations and Congruence Lesson 5: Congruent Figures img 32
Figure B: a reflection across the y-axis
Figure C: a translation 2 units down
Type below:
__________

Answer:
Different orientation

Explanation:
grade 8 chapter 9 image 13
Different orientation

Question 10.
Go Math Grade 8 Answer Key Chapter 9 Transformations and Congruence Lesson 5: Congruent Figures img 33
Figure B: a translation 2 units up
Figure C: a rotation of 180° around the origin
Type below:
__________

Answer:
Different orientation

Explanation:
grade 8 chapter 9 image 14
Different orientation

Congruent Figures – Page No. 308

Question 11.
Represent Real-World Problems A city planner wanted to place the new town library at site A. The mayor thought that it would be better at site B. What transformations were applied to the building at site A to relocate the building to site B? Did the mayor change the size or orientation of the library?
Go Math Grade 8 Answer Key Chapter 9 Transformations and Congruence Lesson 5: Congruent Figures img 34
Type below:
__________

Answer:
From Site A to Site B: Translation 2 units right and 4 units down
The size did NOT change
The orientation changed

Question 12.
Persevere in Problem Solving Find a sequence of three transformations that can be used to obtain figure D from figure A. Graph the figures B and C that are created by the transformations.
Go Math Grade 8 Answer Key Chapter 9 Transformations and Congruence Lesson 5: Congruent Figures img 35
Type below:
__________

Answer:
From figure A to D:
Reflection across the x-axis (-1, -5) (-1, -6) (2, -5) (4, -6)
90º clockwise rotation (4, -1) (5, -1) (5, -4) (4, -6)
translation 6 units left (4, -1) (5, -1) (5, -4) (4, -6)

FOCUS ON HIGHER ORDER THINKING

Question 13.
Counterexamples The Commutative Properties for Addition and Multiplication state that the order of two numbers being added or multiplied does not change the sum or product. Are translations and rotations commutative? If not, give a counterexample.
________

Answer:
No, Translation and rotations are not commutative

Explanation:
The point (2, 2) becomes (2, -4) when translated 2 units to the right then rotated 90 around the origin.
The point (2, 2) becomes (4, -2) when rotated 90 around the origin then translated 2 units to the right.
The above two points are not the same.

Question 14.
Multiple Representations For each representation, describe a possible sequence of transformations.
a. (x, y) → (-x – 2, y + 1)
Type below:
____________

Answer:
translation 2 units right and 1 unit up
reflection across y-axis

Question 14.
b. (x, y) → (y, -x – 3)
Type below:
____________

Answer:
rotation 90º clockwise around the origin
translation 3 units down

Ready to Go On? – Model Quiz – Page No. 309

9.1–9.3 Properties of Translations, Reflections, and Rotations

Question 1.
Graph the image of triangle ABC after a translation of 6 units to the right and 4 units down. Label the vertices of the image A’, B’, and C’.
Go Math Grade 8 Answer Key Chapter 9 Transformations and Congruence Model Quiz img 36
Type below:
____________

Answer:
After translation:
A'(2, 1)
B'(2, -1)
C'(5, -1)

Question 2.
On the same coordinate grid, graph the image of triangle ABC after a reflection across the x-axis. Label the vertices of the image A”, B”, and C”.
Type below:
____________

Answer:
After reflection:
A”(-4, -5)
B”(-4, -3)
C”(-1, -3)

Question 3.
Graph the image of HIJK after it is rotated 180° about the origin. Label the vertices of the image H’I’J’K’.
Go Math Grade 8 Answer Key Chapter 9 Transformations and Congruence Model Quiz img 37
Type below:
____________

Answer:
After rotation:
H'(0, -4)
I'(0, -1)
J'(2, -2)
K'(2, -3)

9.4 Algebraic Representations of Transformations

Question 4.
A triangle has vertices at (2, 3), (−2, 2), and (−3, 5). What are the coordinates of the vertices of the image after the translation (x, y) → (x + 4, y − 3)?
Type below:
____________

Answer:
After translation:
(6, 0)
(2, -1)
(1, 2)

9.5 Congruent Figures

Question 5.
Vocabulary Translations, reflections, and rotations produce a figure that is _____ to the original figure.
Type below:
____________

Answer:
congruent

Explanation:
Vocabulary Translations, reflections, and rotations produce a figure that is congruent to the original figure.

Question 6.
Use the coordinate grid for Exercise 3. Reflect H’I’J’K’ over the y-axis, then rotate it 180° about the origin. Label the new figure H″I″J″K″.
Type below:
____________

Answer:
after reflection
H'(0, -4)
I'(0, -1)
J'(-2, -2)
K'(-2, -3)
after rotation
H”(0, 4)
I”(0, 1)
J”(2, 2)
K”(2, 3)

ESSENTIAL QUESTION

Question 7.
How can you use transformations to solve real-world problems?
Type below:
____________

Answer:
Transformational properties allow the systematic movement of congruent figures while maintaining or adjusting their orientation.

Selected Response – Mixed Review – Page No. 310

Question 1.
What would be the orientation of the figure L after a translation of 8 units to the right and 3 units up?
Go Math Grade 8 Answer Key Chapter 9 Transformations and Congruence Mixed Review img 38
Options:
a. A
b. B
c. C
d. D

Answer:
c. C

Explanation:
After a translation of 8 units right and 3 units up, the orientation of figure L stays the same.

Question 2.
Figure A is reflected over the y-axis and then lowered 6 units. Which sequence describes these transformations?
Options:
a. (x, y) -> (x, -y) and (x, y) -> (x, y – 6)
b. (x, y) -> (-x, y) and (x, y) -> (x, y – 6)
c. (x, y) -> (x, -y) and (x, y) -> (x – 6, y)
d. (x, y) -> (-x, y) and (x, y) -> (x – 6, y)

Answer:
b. (x, y) -> (-x, y) and (x, y) -> (x, y – 6)

Explanation:
reflection over y-axis:
(x, y) -> (-x, y)
Translation 6 units down
(x, y) -> (x, y-6)

Question 3.
What quadrant would the triangle be in after a rotation of 90° counterclockwise about the origin?
Go Math Grade 8 Answer Key Chapter 9 Transformations and Congruence Mixed Review img 39
Options:
a. I
b. II
c. III
d. IV

Answer:
d. IV

Explanation:
After a rotation of 90° counterclockwise about the origin, the triangle will be in QIV

Question 4.
Which rational number is greater than −3 \(\frac{1}{3}\) but less than −\(\frac{4}{5}\)?
Options:
a. −0.4
b. −\(\frac{9}{7}\)
c. −0.19
d. −\(\frac{22}{5}\)

Answer:
b. −\(\frac{9}{7}\)

Question 5.
Which of the following is not true of a trapezoid that has been reflected across the x-axis?
Options:
a. The new trapezoid is the same size as the original trapezoid.
b. The new trapezoid is the same shape as the original trapezoid.
c. The new trapezoid is in the same orientation as the original trapezoid.
d. The x-coordinates of the new trapezoid are the same as the x-coordinates of the original trapezoid.

Answer:
d. The x-coordinates of the new trapezoid are the same as the x-coordinates of the original trapezoid.

Explanation:
The x-coordinates of the new trapezoid are the same as the x-coordinates of the original trapezoid.

Question 6.
A triangle with coordinates (6, 4), (2, −1), and (−3, 5) is translated 4 units left and rotated 180° about the origin. What are the coordinates of its image?
Options:
a. (2, 4), (-2, -1), (-7, 5)
b. (4, 6), (-1, 2), (5, -3)
c. (4, -2), (-1, 2), (5, 7)
d. (-2, -4), (2, 1), (7, -5)

Answer:
d. (-2, -4), (2, 1), (7, -5)

Question 7.
A rectangle with vertices (3, -2), (3, -4), (7, -2), (7, -4) is reflected across the x-axis and then rotated 90° counterclockwise.
a. In what quadrant does the image lie?
____________

Answer:
After reflection and rotation, the image lies in QII

Question 7.
b. What are the vertices of the image?
Type below:
____________

Answer:
image vertices
(-2, 3)
(-4, 3)
(-2, 7)
(-4, 7)

Question 7.
c. What other transformations produce the same image?
Type below:
____________

Answer:
A reflection across the y-axis and 90º clockwise rotation will produce the same result.

Conclusion:

The Go Math Grade 8 Answer Key Chapter 9 Transformations and Congruence pdf are available both online and offline. We have provided in pdf format so that students can practice the problems offline. We know that maths is the scoring and also typical among all the subjects. But you can make it easy if you understand the concept of the chapter. Students can refer to the Go Math Grade 8 Answer Key in their convenient way. Practice well and make maths your favorite subject. Best of Luck!!!

Go Math Grade 7 Answer Key Chapter 11 Analyzing and Comparing Data

go-math-grade-7-chapter-11-analyzing-and-comparing-data-answer-key

Download Go Math Grade 7 Answer Key Chapter 11 Analyzing and Comparing Data pdf for free of cost. Go through the solutions for all the topics in HMH Go Math Grade 7 Answer Key Chapter 11 Analyzing and Comparing Data. This Go Math Answer Key Chapter 11 Analyzing and Comparing Data helps the students to score top in the exams. Check out the solutions in  Go Math Grade 7 Answer Key Chapter 11 Analyzing and Comparing Data and start preparing now.

Go Math Grade 7 Answer Key Chapter 11 Analyzing and Comparing Data

Students can find the step by step explanation for each and every problem in Go Math 7th Grade Answer Key Chapter 11 Analyzing and Comparing Data. The 7th grade Go math answer key not only helps the students it also helps the teachers to find an easy method to teach the students. Know the different methods to solve the problems in Grade 7 Go Math Answer Key Chapter 11 Analyzing and Comparing Data.

Chapter 11 – Lesson: 1

Chapter 11 – Lesson: 2

Chapter 11 – Lesson: 3

Chapter 11 – Comparing Data Displayed in Dot Plots

Guided Practice – Page No. 338

The dot plots show the number of miles run per week for two different classes. For 1–5, use the dot plots shown.
Go Math Grade 7 Answer Key Chapter 11 Analyzing and Comparing Data img 1

Question 1.
Compare the shapes of the dot plots.

Answer: In Class A the dot plot is clustered around two areas and in Class B the dot plot is clustered in the middle.

Question 2.
Compare the centers of the dot plots.

Answer: In Class A the data is centered around 4 miles and 13 miles and in Class B the data is centered around 7 miles.

Question 3.
Compare the spreads of the dot plots.

Answer: In class A the spread of the dot plot is 4 miles to 14 miles and in Class B the spread is 3 miles to 9 miles.

Question 4.
Calculate the medians of the dot plots.

Answer: The median or the dot plots for Class A and Class B is 6.

Explanation: For Class A median is 4,4,4,4,4,5,5,5,6,6,12,13,13,13,13,14,14
= 6.
For Class B median is 3,4,4,4,5,5,5,5,6,6,7,7,7,7,7,8,8,9
= (6+6)/2
= 12/2
= 6.

Question 5.
Calculate the ranges of the dot plots.

Answer: The range of the dot plot For Class A is 10 and Class B is 6.

Explanation: For Class A the range is 14-4= 10.
For Class B the range is 9-3= 6.

Essential Question Check-In

Question 6.
What do the medians and ranges of two dot plots tell you about the data?

Answer: The median of dot plots tells that the values of each dot plot are centered and we can get to know which dot plot has greater values. The range of the dot plot tells about the spread of each value in each plot. The smaller the range, the closer will be the values.

Independent Practice – Page No. 339

The dot plot shows the number of letters in the spellings of the 12 months. Use the dot plot for 7–10.
Go Math Grade 7 Answer Key Chapter 11 Analyzing and Comparing Data img 2

Question 7.
Describe the shape of the dot plot.

Answer: There is a slight increase in the number 8.

Question 8.
Describe the center of the dot plot.

Answer: The center of the dot plot is 6.

Question 9.
Describe the spread of the dot plot.

Answer: The spread of the dot plot is from 3 to 9

Question 10.
Calculate the mean, median, and range of the data in the dot plot.

Answer:
The mean of the dot plot is 6.17.
The median of the dot plot is 6.5.
The range of the dot plot is 6.

Explanation: 3,4,4,5,5,6,7,7,8,8,8,9
The mean of the dot plot is \(\frac{3+4+4+5+5+6+7+7+8+8+8+9}{12}
= \frac{74}{12}\)
= 6.17.
The medain of the dot plot is \(\frac{6+7}{2}
= \frac{13}{2}\)
= 6.5.
The range of the dot plot is 9-3= 6.

The dot plots show the mean number of days with rain per month for two cities.
Go Math Grade 7 Answer Key Chapter 11 Analyzing and Comparing Data img 3

Question 11.
Compare the shapes of the dot plots.

Answer: The most number of days with rain for Montgomery is greater than 8 days and in Lynchburg, the number of days of rain is 12 days or less.

Question 12.
Compare the centers of the dot plots.

Answer: In Montgomery, the center of the dot plot is around 9 days. And in Lynchburg, the center of the dot plot is around 10 days.

Question 13.
Compare the spreads of the dot plots.

Answer: In Montgomery, the spread of the dot plot is from 1 to 12 days and the outlier is 1. And in Lynchburg, the spread of the data plot is from 8 to 12 days.

Question 14.
What do the dot plots tell you about the two cities with respect to their average monthly rainfall?

Answer: As the center of Lynchburg is greater than the center of Montgomery, so average monthly rainfall for Lynchburg is greater than the average monthly rainfall of Montgomery.

Page No. 340

The dot plots show the shoe sizes of two different groups of people.
Go Math Grade 7 Answer Key Chapter 11 Analyzing and Comparing Data img 4

Question 15.
Compare the shapes of the dot plots.

Answer: In Group A the shoe sizes are mostly less than 9. And in group B all the shoe sizes are 11.5 or less.

Question 16.
Compare the medians of the dot plots.

Answer:
The median of Group A is 8.
The median of Group A is 9.5.

Explanation: 6.5,7,7,7.5,7.5,7.5,8,8,8,8,8,8.5,8.5,9,13
The median of Group A is 8.
8.5,9,9,9,9,9.5,9.5,9.5,9.5,10,10,10.5,10.5,10.5,11.5
The median of Group B is 9.5.

Question 17.
Compare the ranges of the dot plots (with and without the outliers).

Answer:
The range with the outlier is 13-6.5= 6.5.
The range without the outlier is 9-6.5= 2.5.
The range is 11.5-8.5= 3.

Explanation: The outlier in Group A is 13
The range with the outlier is 13-6.5= 6.5.
The range without the outlier is 9-6.5= 2.5.
There is no outlier in Group B, so the range is 11.5-8.5= 3.

Question 18.
Make A Conjecture
Provide a possible explanation for the results of the dot plots.

Answer: Group A is Girls and Group B is boys. Because boys have large feet than girls.

H.O.T.

Focus on Higher Order Thinking

Question 19.
Analyze Relationships
Can two dot plots have the same median and range but have completely different shapes? Justify your answer using examples.

Answer: Yes, it is possible to have the same median and range with different shapes.

Explanation: Yes, it is possible to have the same median and range with different shapes. The median and the range of the below image is
image 1 data – 1,2,2,3,3,3,4,4,5.
The median of image 1 is 3.
image 2 data is – 2,2,2,2,3,3,4,4,5,5,6.
The median of image 2 is 3.
The range of image 1 is 5-1= 4.
The range of image 2 is 6-2= 4.

Go Math Grade 7 Answer Key Chapter 11 Analyzing and Comparing Data

Question 20.
Draw Conclusions
What value is most affected by an outlier, the median or the range? Explain. Can you see these effects in a dot plot?

Answer: The most affected by an outlier is range. The outlier increases the range as median values are in the middle, so the outlier will not mostly affect the median. Yes, in a dot plot we can see both range and median.

Guided Practice – Page No. 344

For 1–3, use the box plot Terrence created for his math test scores. Find each value.
Go Math Grade 7 Answer Key Chapter 11 Analyzing and Comparing Data img 5

Question 1.
Minimum = _____ ; Maximum = _____

Answer:
Minimum = 72.
Maximum = 88.

Explanation: The minimum value is the smallest value in the box plot, so the minimum value is 72, and the maximum value is the largest value in the box plot which is 88

Question 2.
Median = _____

Answer:
The Median is 79.

Explanation:
The data is 72,75,79,85,88
The Median is 79.

Question 3.
Range = _____ ; IQR = _____

Answer:
The range is 16.
The IQR is 10.

Explanation:
The range is 88-72= 16
IQR is the difference between upper quartiles and lower quartiles, so 85-75= 10.

For 4–7, use the box plots showing the distribution of the heights of hockey and volleyball players.
Go Math Grade 7 Answer Key Chapter 11 Analyzing and Comparing Data img 6

Question 4.
Which group has a greater median height?
_____

Answer:
The greater median height is Volleyball players with 74 in.

Explanation:
Hockey players data is 64,66,70,76,78.
The median height of hockey players is 70 in.
Volleyball players data is 67,68,74,78,85
The median height of the Volleyball player is 74 in.

Question 5.
Which group has the shortest player?
_____

Answer:
Hockey players have the shortest player with 64 in.

Explanation:
The minimum height of the hockey players is 64 in.
The minimum height of the Volleyball players is 67 in.

Question 6.
Which group has an interquartile range of about 10?
_____

Answer: The IQR for Hockey players and Volleyball players is 10.

Explanation:
The IQR for Hockey players is 76-66= 10.
The IQR for Volleyball players is 78-68= 10.

Essential Question Check-In

Question 7.
What information can you use to compare two box plots?

Answer: To compare two box plots we can use minimum, maximum values, ////////the median, the range, and the IQR.

Independent Practice – Page No. 345

For 8–11, use the box plots of the distances traveled by two toy cars that were jumped from a ramp.
Go Math Grade 7 Answer Key Chapter 11 Analyzing and Comparing Data img 7

Question 8.
Compare the minimum, maximum, and median of the box plots.

Answer:
The data of Car A is 165,170,180,195,210.
The data of Car B is 160,175,185,200,205.
The minimum value of Car A is 165.
The minimum value of Car B is 165.
The maximum value of Car A is 210.
The maximum value of Car B is 205.
The median of Car A is 180.
The median of Car B is 185.

Explanation:
The data of Car A is 165,170,180,195,210.
The data of Car B is 160,175,185,200,205.
The minimum value of Car A is 165.
The minimum value of Car B is 165.
The maximum value of Car A is 210.
The maximum value of Car B is 205.
The median of Car A is 180.
The median of Car B is 185.

Question 9.
Compare the ranges and interquartile ranges of the data in box plots.

Answer:
The range of Car A is 45.
The range of Car B is 45.
The IQR of Car A is  25.
The IQR of Car B is  25.

Explanation:
The range of Car A is 210-165= 45.
The range of Car B is 205-160= 45.
The IQR of Car A is 195-170= 25.
The IQR of Car B is 200-175= 25.

Question 10.
What do the box plots tell you about the jump distances of two cars?

Answer: The box plot tells about the minimum and the maximum jump distance, the median jump distance, and the spread of the jump distance.

Question 11.
Critical Thinking
What do the whiskers tell you about the two data sets?

Answer: The whiskers tells about the spread of maximum and minimum values of the bottom and top 25% of data.

For 12–14, use the box plots to compare the costs of leasing cars in two different cities.
Go Math Grade 7 Answer Key Chapter 11 Analyzing and Comparing Data img 8

Question 12.
In which city could you spend the least amount of money to lease a car? The greatest?
______

Answer: The least and the greatest amount is spent by City B.

Explanation:
The data set of City A is $425,$450,$475,$550,$600.
The data set of City B is $400,$425,$450,$475,$625.
The minimum cost of City A is $425 and the maximum is $600.
The minimum cost of City B is $400 and the maximum is $625.
The least and the greatest amount is spent by City B.

Question 13.
Which city has a higher median price? How much higher is it?
______

Answer: The higher median price is City A with $475 and $50 higher.

Explanation:
The median of City A is $475 and the median of City B is $450.
So the difference is $475-$425= $50.

Question 14.
Make a Conjecture
In which city is it more likely to choose a car at random that leases for less than $450? Why?
______

Answer: 450 corresponds to the first quartile of City A, which means 25% of the cars cost less than $450. 450 corresponds to the median for City B  which means 50% of the cars cost less than $450. So City B is more likely to have a car chosen randomly that costs less than $450.

Page No. 346

Question 15.
Summarize
Look back at the box plots for 12–14 on the previous page. What do the box plots tell you about the costs of leasing cars in those two cities?

Answer: City A has a smaller range than City B, but it has greater IQR. And City B has 4 key values of City A which means leasing a car is cheaper in City B.

H.O.T.

Focus on Higher Order Thinking

Question 16.
Draw Conclusions
Two box plots have the same median and equally long whiskers. If one box plot has a longer box than the other box plot, what does this tell you about the difference between the data sets?

Answer: If two box plots have the same median and equally long whiskers and one box is longer than the other, that means the box plot with the larger box has a greater range and IQR.

Question 17.
Communicate Mathematical Ideas
What you can learn about a data set from a box plot? How is this information different from a dot plot?

Answer: We can learn about the minimum and the maximum values, the median, the range, the IQR, and the range of 25% of the data.
and a data plot contains all data values. which a box plot doesn’t have.

Question 18.
Analyze Relationships
In mathematics, central tendency is the tendency of data values to cluster around some central value. What does a measure of variability tell you about the central tendency of a set of data? Explain.

Answer: If the range and IQR are small, the values are clustering around some central values.

Guided Practice – Page No. 350

The tables show the numbers of miles run by the students in two classes. Use the tables in 1–2.
Go Math Grade 7 Answer Key Chapter 11 Analyzing and Comparing Data img 9

Question 1.
For each class, what is the mean? What is the mean absolute deviation?
Class 1 mean: __________
Class 2 mean: __________
Class 1 MAD: __________
Class 2 MAD: __________

Answer:
Class 1 mean: 6
Class 2 mean: 11
Class 1 MAD: 3.067
Class 2 MAD: 3.067

Explanation:
The mean of Class 1 is \(\frac{12+6+1+10+1+2+3+10+3+8+3+9+8+6+8}{6}
= \frac{90}{15}\)
= 6
The mean of Class 2 is \(\frac{11+14+11+13+6+7+8+6+8+13+8+15+13+17+15}{15}
= \frac{165}{15}\)
= 11
The mean absolute deviation of Class 1 is
|12-6| = 6
|6-6| = 0
|1-6| = 5
|10-6| = 4
|1-6| = 5
|2-6| = 4
|3-6| = 3
|10-6| = 4
|3-6| = 3
|8-6| = 2
|3-6| = 3
|9-6| = 3
|8-6| = 2
|6-6| = 0
|8-6| = 2
The mean absolute deviation of Class 1 is \(\frac{6+0+5+4+5+4+3+4+3+2+3+3+2+0+2}{15}
= \frac{46}{15}\)
= 3.067

The mean absolute deviation of Class 2 is
|11-11| = 0
|14-11| = 3
|11-11| = 0
|13-11| = 2
|6-11| = 5
|7-11| = 4
|8-11| = 3
|6-11| = 5
|8-11| = 3
|13-11| = 2
|8-11| = 3
|15-11| = 4
|13-11| = 2
|17-11| = 6
|15-11| = 4
The mean absolute deviation of Class 2 is \(\frac{0+3+0+2+5+4+3+5+3+2+3+4+2+6+4}{15}
= \frac{46}{15}\)
= 3.067

Question 2.
The difference of the means is about _____ times the mean absolute deviations.
_____

Answer: 3, 1.67.

Explanation: The difference of the mean is 11-6=5, and the difference of the means is about 3 times the mean absolute deviations, so
5/3= 1.67.

Question 3.
Mark took 10 random samples of 10 students from two schools. He asked how many minutes they spend per day going to and from school. The tables show the medians and the means of the samples. Compare the travel times using distributions of the medians and means.
Go Math Grade 7 Answer Key Chapter 11 Analyzing and Comparing Data img 10

Answer:

Essential Question Check-In

Question 4.
Why is it a good idea to use multiple random samples when making comparative inferences about two populations?

Answer: It’s important to use multiple random samples, so you can draw more interferences about the populations. The more samples we use the more convincing arguments you can make about the distributions.

Independent Practice – Page No. 351

Josie recorded the average monthly temperatures for two cities in the state where she lives. Use the data for 5–7.
Go Math Grade 7 Answer Key Chapter 11 Analyzing and Comparing Data img 11

Question 5.
For City 1, what is the mean of the average monthly temperatures? What is the mean absolute deviation of the average monthly temperatures?
Mean: __________
MAD: __________

Answer:
Mean: 50 °F.
MAD: 13 °F.

Explanation:
The mean of city 1 is \(\frac{23+38+39+48+55+56+71+86+57+53+43+31}{12}
= \frac{600}{12}\)
= 50 °F.
|23-50|= 27
|38-50|= 12
|39-50|= 11
|48-50|= 2
|55-50|= 5
|56-50|= 6
|71-50|= 21
|86-50|= 36
|57-50|= 7
|53-50|= 3
|43-50|= 7
|31-50|= 19
The mean absolute deviation is \(\frac{27+12+11+2+5+6+21+36+7+3+7+19}{12}
= \frac{156}{12}\)
= 13 °F.

Question 6.
What is the difference between each average monthly temperature for City 1 and the corresponding temperature for City 2?
_______ °F

Answer: The difference between each average monthly temperature for City 1 and the corresponding temperature for City 2 is 15 °F

Explanation:
|23-8|= 15
|38-23|= 15
|39-24|= 15
|48-33|= 15
|55-40|= 15
|56-41|= 15
|71-56|= 15
|86-71|= 15
|57-42|= 15
|53-38|= 15
|43-28|= 15
|31-16|=  15
The difference between each average monthly temperature for City 1 and the corresponding temperature for City 2 is 15 °F

Question 7.
Draw Conclusions
Based on your answers to Exercises 5 and 6, what do you think the mean of the average monthly temperatures for City 2 is? What do you think the mean absolute deviation of the average monthly temperatures for City 2 is? Give your answers without actually calculating the mean and the mean absolute deviation. Explain your reasoning.
Mean = __________ °F
MAD __________ °F

Answer:
Mean =35 °F
MAD = 13°F

Explanation: As all the values of City 2 are 15 below the values of City 1, so the mean of the City 2 will be 50 less than the mean of City 1. Which means 50-15= 35. All of City 2’s values deviate from the mean the same way City 1’s values do which means that the mean absolute deviation is 13

Question 8.
What is the difference in the means as a multiple of the mean absolute deviations?
_______ (MAD)

Answer: 1.15.

Explanation:
(50-35)/13
= 15/13
= 1.15.
The difference in the means as a multiple of the mean absolute deviations 1.15.

Question 9.
Make a Conjecture
The box plots show the distributions of mean weights of 10 samples of 10 football players from each of two leagues, A and B. What can you say about any comparison of the weights of the two populations? Explain.
Go Math Grade 7 Answer Key Chapter 11 Analyzing and Comparing Data img 12

Answer: As both leagues have a lot of variability since the ranges and IQR’s are both very large. The middle halves overlap entirely. The variation and overlap in the distributions make it hard to make any convincing comparison.

Page No. 352

Question 10.
Justify Reasoning
Statistical measures are shown for the ages of middle school and high school teachers in two states.
State A: Mean age of middle school teachers = 38, mean age of high school teachers = 48, mean absolute deviation for both = 6
State B: Mean age of middle school teachers = 42, mean age of high school teachers = 50, mean absolute deviation for both = 4
In which state is the difference in ages between members of the two groups more significant? Support your answer.
_____________

Answer: State B has a difference in ages between members of the two groups more significant.

Explanation:
For State A the difference in the mean as a multiple of the mean absolute deviation is (48-38)/6
= 10/6
= 1.67.
So for State B, (50-42)/4
= 8/4
= 2.
As State B has a larger multiple, the differences in ages between members of the two groups are more significant.

Question 11.
Analyze Relationships
The tables show the heights in inches of all the adult grandchildren of two sets of grandparents, the Smiths and the Thompsons. What is the difference in the medians as a multiple of the ranges?
Go Math Grade 7 Answer Key Chapter 11 Analyzing and Comparing Data img 13
______ x range

Answer: The difference in the median is 1.75.

Explanation:
Smith: 64,65,65,66,66,67,68,68,69,70.
The Median is (66+67)/2
= 133/2
= 66.5.
The range is 70-64= 6.
Thompsons: 74,75,75,76,77,77,78,79,79,80.
The Median is (77+77)/2
= (154)/2
= 77.
The range is 80-74= 6.
The difference in the median is (77-66.5)/6
= 10.5/6
= 1.75.

H.O.T.

Focus on Higher Order Thinking

Question 12.
Critical Thinking
Jill took many samples of 10 tosses of a standard number cube. What might she reasonably expect the median of the medians of the samples to be? Why?
Median of the medians: ______

Answer:
Median of the medians: 3.5.

Explanation: The possible outcome of a number cube is 1,2,3,4,5,6. So median is
= (3+4)/2
= 7/2
= 3.5
The median of the medians should be close to the median of the populations, so it will also be about 3.5.

Question 13.
Analyze Relationships
Elly and Ramon are both conducting surveys to compare the average numbers of hours per month that men and women spend shopping. Elly plans to take many samples of size 10 from both populations and compare the distributions of both the medians and the means. Ramon will do the same, but will use a sample size of 100. Whose results will probably produce more reliable inferences? Explain.
_____________

Answer: The larger the sample size, the less variability there should be in the distributions of the medians and means. And Ramon will most likely produce more reliable inferences since he will be using a much larger sample size.

Question 14.
Counterexamples
Seth believes that it is always possible to compare two populations of numerical values by finding the difference in the means of the populations as a multiple of the mean absolute deviations. Describe a situation that explains why Seth is incorrect.

Answer: In order to compare two populations by finding the difference in the means of the populations as a multiple of the mean absolute deviations, so the mean absolute deviations of both populations need to be about the same. So if the mean absolute deviations are significantly different, like 5 and 10 and we cannot compare the populations this way.

11.1 Comparing Data Displayed in Dot Plots – Page No. 353

The two dot plots show the number of miles run by 14 students at the start and at the end of the school year. Compare each measure for the two dot plots. Use the data for 1–3.
Go Math Grade 7 Answer Key Chapter 11 Analyzing and Comparing Data img 14

Question 1.
Means
Start: _________
End: _________

Answer:
Mean
Start: 7.5 miles.
End: 8.2 miles.

Explanation:
The data for the start of the school year is 5,6,6,7,7,7,7,8,8,8,8,9,9,10.
The mean is \(\frac{5+6+6+7+7+7+7+8+8+8+8+9+9+10}{14}
= \frac{105}{14}\)
= 7.5 miles.
The data for the end of the school year is 6,6,7,7,8,8,8,8,9,9,9,10,10,10.
The mean is \(\frac{6+6+7+7+8+8+8+8+9+9+9+10+10+10}{14}
= \frac{115}{14}\)
= 8.2 miles.

Question 2.
Medians
Start: _________
End: _________

Answer:
Median
Start: 7.5 miles.
End: 8 miles.

Explanation:
The median for the start of the school year is
= (7+8)/2
= 15/2
= 7.5 miles.
The median for the end of the school year is
= (8+8)/2
= 16/2
= 8 miles.

Question 3.
Ranges
Start: _________
End: _________

Answer:
Ranges
Start: 5 miles.
End: 4 miles.

Explanation:
The range for the Start of the school year is 10-5= 5 miles.
The range for the end of the school year is 10-6= 4 miles.

11.2 Comparing Data Displayed in Box Plots

The box plots show lengths of flights in inches flown by two model airplanes. Use the data for 4–5.
Go Math Grade 7 Answer Key Chapter 11 Analyzing and Comparing Data img 15

Question 4.
Which has a greater median flight length?
_____________

Answer:
The greater median flight length is Airplane A which is 210 in.

Explanation:
The median of Airplane A is 210 in and the median of Airplane B is 204 in. So greater median flight length is Airplane A which is 210 in.

Question 5.
Which has a greater interquartile range?
_____________

Answer: The greater IQR is Airplane B with 35 in.

Explanation:
The IQR for Airplane A is 225-208= 17 in and The IQR for Airplane B is 230-195= 35 in. So the greater IQR is Airplane B.

11.3 Using Statistical Measures to Compare Populations

Question 6.
Roberta grows pea plants, some in shade and some in sun. She picks 8 plants of each type at random and records the heights.
Go Math Grade 7 Answer Key Chapter 11 Analyzing and Comparing Data img 16
Express the difference in the means as a multiple of their ranges.
______

Answer: The difference in the means as a multiple of their ranges is 2.4 in.

Explanation:
The mean of Shade plant heights is \(\frac{7+11+11+12+9+12+8+10}{8}
= \frac{80}{8}\)
= 10 in.
The range of Shade plant heights is 12-7= 5 in.
The mean of Sun plant heights is \(\frac{21+24+19+19+22+23+24+24}{8}
= \frac{176}{8}\)
= 22 in.
The range of Sun plant heights is 24-19= 5 in.
The difference in the means as a multiple of their ranges is  (22-10)/5
= 12/5
= 2.4 in.

Essential Question

Question 7.
How can you use and compare data to solve real-world problems?

Answer: We can use and compare data to solve real-world problems by determining if one set is larger than the other set in terms of values, means, and medians.

Selected Response – Page No. 354

Question 1.
Which statement about the data is true?
Go Math Grade 7 Answer Key Chapter 11 Analyzing and Comparing Data img 17
Options:
a. The difference between the medians is about 4 times the range.
b. The difference between the medians is about 4 times the IQR.
c. The difference between the medians is about 2 times the range.
d. The difference between the medians is about 2 times the IQR.

Answer: b is true.

Explanation:
Set 1 median is 60 and Set 2 median is 76
The range of Set 1 is 68-55= 13
The range of Set 2 is 80-65= 15
The IQR of Set 1 is 63-59= 4
The IQR of Set 2 is 77-73= 4
The difference in medians is 76-60= 16, So the difference between the medians is about 4 times the IQR.

Question 2.
Which is a true statement based on the box plots below?
Go Math Grade 7 Answer Key Chapter 11 Analyzing and Comparing Data img 18
Options:
a. The data for City A has a greater range.
b. The data for City B is more symmetric.
c. The data for City A has a greater interquartile range.
d. The data for City B has a greater median.

Answer: C is true.

Explanation: The length of the box for City A is much larger than for City B, so IQR for City A is greater.

Question 3.
What is −3 \(\frac{1}{2}\) written as a decimal?
Options:
a. -3.5
b. -3.05
c. -0.35
d. -0.035

Answer: a.

Explanation: −3 \(\frac{1}{2}\)
=  \(\frac{-7}{2}\)
= -3.5.

Question 4.
Which is a true statement based on the dot plots below?
Go Math Grade 7 Answer Key Chapter 11 Analyzing and Comparing Data img 19
Options:
a. Set A has the lesser range
b. Set B has a greater median.
c. Set A has the greater mean.
d. Set B is less symmetric than Set A.

Answer: c is a true statement.

Explanation:
The median of Set A is 30 and the median of Set B is 40, so Set A has the greater mean.

Mini-Task

Question 5.
The dot plots show the lengths of a random sample of words in a fourth-grade book and a seventh-grade book.
Go Math Grade 7 Answer Key Chapter 11 Analyzing and Comparing Data img 20
a. Compare the shapes of the plots.

Answer:
For Fourth grade, most of the words have a length of 6 or less and with two outliers 9 and 10.
For Seventh grade, most of the words have a length of 8 or less with 5 exceptions.

Question 5.
b. Compare the ranges of the plots. Explain what your answer means in terms of the situation.

Answer:
The Seventh grade has a larger range, so it has more variability.

Explanation:
The range for the fourth grade is 10-1=9.
The range for the seventh grade 14-2= 12.
As the Seventh grade has a larger range it has more variability.

EXERCISES – Page No. 356

Question 1.
Molly uses the school directory to select, at random, 25 students from her school for a survey on which sports people like to watch on television. She calls the students and asks them, “Do you think basketball is the best sport to watch on television?”
a. Did Molly survey a random sample or a biased sample of the students at her school?
_____________

Answer: Yes, Molly surveyed a random sample. As she selected 25 students from a school directory of the entire student’s population in her school.

Question 1.
b. Was the question she asked an unbiased question? Explain your answer.
_____________

Answer: No, the question is not unbiased. The question is biased because it assumes the person watches basketball on television.

Question 2.
There are 2,300 licensed dogs in Clarkson. A random sample of 50 of the dogs in Clarkson shows that 8 have ID microchips implanted. How many dogs in Clarkson are likely to have ID microchips implanted?
______ dogs

Answer: 368 dogs.

Explanation: Let the dogs in Clarkson to have ID microchips be X, so
X/2300 = 8/50
X= (8×2300)/50
X= 18,400/50
X= 368.

Question 3.
A store gets a shipment of 500 MP3 players. Twenty-five of the players are defective, and the rest are working. A graphing calculator is used to generate 20 random numbers to simulate a random sample of the players.
A list of 20 randomly generated numbers representing MP3 players is :
Go Math Grade 7 Answer Key Chapter 11 Analyzing and Comparing Data img 21
a. Let numbers 1 to 25 represent players that are _____
_____________

Answer: As there are twenty-five defective players, let the numbers 1 to 25 represent players that are defective.

Question 3.
b. Let numbers 26 to 500 represent players that are _____
_____________

Answer: Let the numbers 26 to 500 represent players that are working.

Question 3.
c. How many players in this sample are expected to be defective?
______ players

Answer: As there are 2 numbers in from 1 and 25 which are 5 and 9 are the players in the sample are expected to be defective.

Question 3.
d. If 300 players are chosen at random from the shipment, how many are expected to be defective based on the sample? Does the sample provide a reasonable inference? Explain.
______ players

Answer: 30 players.

Explanation:
X/300 = 2/20
X = (2×300)/20
X = 600/20
X = 30.
We may expect 25 out of 500 or 5% of the 300 players to be defective, which is only 15 players because the sample doesn’t provide a reasonable inference.

EXERCISES – Page No. 357

The dot plots show the number of hours a group of students spends online each week, and how many hours they spend reading. Compare the dot plots visually.
Go Math Grade 7 Answer Key Chapter 11 Analyzing and Comparing Data

Question 1.
Compare the shapes, centers, and spreads of the dot plots.

Answer:
Shape:
Time spent online- Most of the students spend 4 hours are more.
Time spent reading- The students spent a maximum of 6 hours.
Centers:,6
The no.of hours spent online is centered around 6 hours.
The no.of hours spent reading is centered around 5 hours.
Spread:
The range for time spent online is 7-0=7.
The range for time spent reading is 6-0=6.

Question 2.
Calculate the medians of the dot plots.
Time online: __________
Time reading: __________

Answer:
Time online: 6 hours.
Time reading: 5 hours.

Explanation:
The data of time online is 0,4,4,5,5,6,6,6,6,6,6,7,7,7,7
The Median is 6 hours.
The data of time reading is 0,0,0,0,1,1,2,5,5,5,6,6,6,6,6
The Median is 5 hours.

Question 3.
Calculate the ranges of the dot plots.
Time online: __________
Time reading: __________

Answer:
Time online: 7 hours.
Time reading: 6 hours.

Explanation:
The range of time online is 7-0= 7.
The range of time reading is 6-0= 6.

Page No. 358

Question 4.
The average times (in minutes) a group of students spend studying and watching TV per school day are given.
Studying: 25, 30, 35, 45, 60, 60, 70, 75
Watching TV: 0, 35, 35, 45, 50, 50, 70, 75
a. Find the mean times for studying and for watching TV.
Studying: __________
Watching TV: __________

Answer:
Studying: 50.
Watching TV: 40.

Explanation:
The mean for studying is \(\frac{25+30+35+45+60+60+0+75}{8}
= \frac{400}{8}\)
= 50.
The mean for watching TV is \(\frac{0+35+35+45+50+50+70+75}{8}
= \frac{360}{8}\)
= 45.

Question 4.
b. Find the mean absolute deviations (MADs) for each data set.
Studying: __________
Watching TV: __________

Answer:
Studying: 16.25
Watching TV: 16.25

Explanation:
|25-50|= 25
|30-50|= 20
|35-50|= 15
|45-50|= 5
|60-50|= 10
|60-50|= 10
|70-50|= 20
|75-50|= 25
The mean absolute deviation is \(\frac{25+20+15+5+10+10+20+25}{8}
= \frac{130}{8}\)
= 16.25.
|0-45|= 45
|35-45|= 10
|35-45|= 10
|45-45|= 0
|50-45|= 5
|50-45|= 5
|70-45|= 25
|75-45|= 30
The mean absolute deviation is \(\frac{45+10+10+0+5+5+25+30}{8}
= \frac{130}{8}\)
= 16.25.

Question 4.
c. Find the difference of the means as a multiple of the MAD, to two decimal places.
_____

Answer: 0.31.

Explanation: (50-45)/16.25 = 5/16.25
= 0.31.

Unit 5 Performance Tasks

Question 5.
Entomologist
An entomologist is studying how two different types of flowers appeal to butterflies. The box-and-whisker plots show the number of butterflies that visited one of two different types of flowers in a field. The data were collected over a two-week period, for one hour each day.
Go Math Grade 7 Answer Key Chapter 11 Analyzing and Comparing Data img 23
a. Find the median, range, and interquartile range for each data set.

Answer:
Type A:
The median is 11.5
The range is 4
The IQR is 3
Type B:
The median is 11
The range is 10
The IQR is 2

Explanation:
Type A:
The median is (11+12)/2
= 23/2
= 11.5
The range is 13-9= 4
The IQR is 12-9= 3
Type B:
The median is 11
The range is 17-7= 10
The IQR is 12-10= 2

Question 5.
b. Which measure makes it appear that flower type A had a more consistent number of butterfly visits? Which measure makes it appear that flower type B did? If you had to choose one flower as having the more consistent visits, which would you choose? Explain your reasoning.

Answer: As type A has a smaller range, the range makes it appear as if type A has a more consistent number of butterflies visits. And type B had a smaller IQR, the IQR makes it appear as if type A has a more consistent number of butterflies visits. We would choose type A has to have a more consistent number of butterflies visits and it has a much smaller range. The range of the fourth quartile for type Bis larger than the range for the entire data set of type A.

Selected Response – Page No. 359

Question 1.
Which is a true statement based on the dot plots below?
Go Math Grade 7 Answer Key Chapter 11 Analyzing and Comparing Data img 24
Options:
a. Set B has a greater range.
b. Set B has a greater median.
c. Set B has the greater mean.
d. Set A is less symmetric than Set B.

Answer: a

Explanation:
Set A has a range of 60-20= 40
Set B has a range of 60-10= 50.
So Set B has a greater range.

Question 2.
Which is a solution to the equation 7g − 2 = 47?
Options:
a. g = 5
b. g = 6
c. g = 7
d. g = 8

Answer: c

Explanation:
7g-2= 47
7g= 47+2
7g= 49
g= 49/7
g= 7.

Question 3.
Which is a true statement based on the box plots below?
Go Math Grade 7 Answer Key Chapter 11 Analyzing and Comparing Data img 25
Options:
a. The data for Team B has a greater range.
b. The data for Team A is more symmetric.
c. The data for Team B has a greater interquartile range.
d. The data for Team A has a greater median.

Answer: c

Explanation: The box of Team B is much larger than the box of Team A, so the data for Team B have the greater interquartile range.

Question 4.
Which is the best way to choose a random sample of people from a sold-out movie audience for a survey?
Options:
a. Survey all audience members who visit the restroom during the movie.
b. Assign each seat a number, write each number on a slip of paper, and then draw several slips from a hat. Survey the people in those seats.
c. Survey all of the audience members who sit in the first or last row of seats in the movie theater.
d. Before the movie begins, ask for volunteers to participate in a survey. Survey the first twenty people who volunteer.

Answer: b

Explanation:
A is not random because the people are being chosen are being surveyed in one place.
B is random as all members of the population can be chosen and each member has an equal chance of being selected.
C is may not assign every member of the population an equal chance of being chosen since the number of seats in the first or last rows may have more or fewer seats than the other rows.
D is not random because participants are self selecting to do the survey.

Question 5.
Find the percent change from 84 to 63.
Options:
a. 30% decrease
b. 30% increase
c. 25% decrease
d. 25% increase

Answer: c

Explanation:
(84-63)/84 = 21/84
= 0.25
= 25% decrease

Question 6.
A survey asked 100 students in a school to name the temperature at which they feel most comfortable. The box plot below shows the results for temperatures in degrees Fahrenheit. Which could you infer based on the box plot below?
Go Math Grade 7 Answer Key Chapter 11 Analyzing and Comparing Data img 26
Options:
a. Most students prefer a temperature less than 65 degrees.
b. Most students prefer a temperature of at least 70 degrees.
c. Almost no students prefer a temperature of fewer than 75 degrees.
d. Almost no students prefer a temperature of more than 65 degrees.

Answer: b.

Explanation: The last half of the data is about 73-85 which means 50% prefer a temperature above 73. This means that the most prefer a temperature of at least 70 degrees since more than 50% of the box plot is 70 degrees are more.

Page No. 360

Question 7.
The box plots below show data from a survey of students under 14 years old. They were asked on how many days in a month they read and draw. Based on the box plots, which is a true statement about students?
Go Math Grade 7 Answer Key Chapter 11 Analyzing and Comparing Data img 27
Options:
a. Most students draw at least 12 days a month.
b. Most students read less than 12 days a month.
c. Most students read more often than they draw.
d. Most students draw more often than they read.

Answer: c

Explanation: As 4 out of 5 key values for reading are greater than the corresponding values for drawing which means most of the students read more often than they draw.

Question 8.
Which describes the relationship between ∠NOM and ∠JOK in the diagram?
Go Math Grade 7 Answer Key Chapter 11 Analyzing and Comparing Data img 28
Options:
a. adjacent angles
b. complementary angles
c. supplementary angles
d. vertical angles

Answer: d

Explanation: ∠NOM and ∠JOK are vertical angles.

Question 9.
The tables show the typical number of minutes spent exercising each week for a group of fourth-grade students and a group of seventh-grade students.
Go Math Grade 7 Answer Key Chapter 11 Analyzing and Comparing Data img 29
a. What is the mean number of minutes spent exercising for fourth graders? For seventh graders?
4th grade: __________
7th grade: __________

Answer:
4th grade: 129
7th grade: 221

Explanation:
The mean for fourth grade is \(\frac{120+75+30+30+240+90+100+180+125+300}{10}
= \frac{1290}{10}\)
= 129
The mean for fourth grade is \(\frac{410+145+240+250+125+95+210+190+245+300}{10}
= \frac{2210}{10}\)
= 221

Question 9.
b. What is the mean absolute deviation of each data set?
4th grade: __________
7th grade: __________

Answer:
4th grade: 66.6
7th grade: 68

Explanation:
|120-129|= 9
|75-129|= 54
|30-129|= 99
|30-129|= 99
|240-129|=111
|90-129|= 39
|100-129|= 29
|180-129|= 51
|125-129|= 4
|300-129|= 171
The mean absolute deviation for fourth grade is \(\frac{9+54+99+99+111+39+29+51+4+171}{10}
= \frac{666}{10}\)
= 66.6
|410-221|= 189
|145-221|= 76
|240-221|= 19
|250-221|= 29
|125-221|= 96
|95-221|= 126
|210-221|= 11
|190-221|= 31
|245-221|= 24
|300-221|= 79
The mean absolute deviation for fourth grade is \(\frac{189+76+19+29+96+126+11+31+24+79}{10}
= \frac{680}{10}\)
= 68

Question 9.
c. Compare the two data sets with respect to their measures of center and their measures of variability.

Answer: The center of the fourth grade is much smaller than the center for 7th grade. The range is much smaller for a fourth grade than 7th grade which means that fourth graders spend less time exercising and have less variability in the number of minutes that they exercise.

Explanation:
The data of fourth grade is 30,30,75,90,100,120,125,180,240,300
Median is (100+120)/2
= 220/2
= 110
The range is 300-30= 270
The data of seventh grade is 95,125,145,190,210,240,245,250,300,410
Median is (210+240)/2
= 450/2
= 225
The range is 410-95= 315.
The center of the fourth grade is much smaller than the center for 7th grade. The range is much smaller for a fourth grade than 7th grade which means that fourth graders spend less time exercising and have less variability in the number of minutes that they exercise.

Question 9.
d. How many times the MADs is the difference between the means, to the nearest tenth?
_______

Answer: As the MADs are not the same we will find the average of them and then find the difference of the mean and divide by the average of the MADs.

Explanation:
(66.6+68)/2
= 134.6/2
= 67.3
(221-129)/67.3
= 92/67.3
= 1.37

Guided Practice – Page No. 371

Question 1.
In a hat, you have index cards with the numbers 1 through 10 written on them. Order the events from least likely to happen (1) to most likely to happen (8) when you pick one card at random. In the boxes, write a number from 1 to 8 to order the eight different events.
You pick a number greater than 0. __________
You pick an even number. __________
You pick a number that is at least 2. __________
You pick a number that is at most 0. __________
You pick a number divisible by 3. __________
You pick a number divisible by 5. __________
You pick a prime number. __________
You pick a number less than the greatest prime number. __________

Answer: 8,5,7,1,3,24,6.

Explanation:
As there are 10 numbers from 1 to 10 and thus there will be 10 possible outcomes. So,
The number greater than 0 is 1,2,3,4,5,6,7,8,9,10.
Even numbers are 2,4,6,8,10.
The number at least 2 is 2,3,4,5,6,7,8,9,10.
The number that is at most 0: as none of the integers are from 1 to 10 are at most 0.
The number divisible by 3 is 3,6,9.
The number divisible by 5 is 5,10.
The prime numbers are 2,3,5,7.
The number less than the greatest prime numbers are 1,2,3,4,5,6 as 7 is the greatest prime number from the numbers 1 to 10.
The more favorable outcomes correspond with an event, the more likely the events happen. Thus the number is at most 0 is the least likely and the greater than 0 is the most likely.
The number of events from the least likely to the most likely is
The number greater than 0 is 8
Even numbers are 5
The number at least 2 is 7
The number that is at most 0: 1
The number divisible by 3 is 3
The number divisible by 5 is 2
The prime numbers are 4
The number less than the greatest prime number is 6.

Final Words:

In addition to the exercise problems, students can also find solutions for homework exercises. Thus the Go Math Grade 7 Answer Key Chapter 11 helps to complete the homework in time. We wish the pdf helped you a lot in scoring marks in the exams. Keep in touch with us to get the latest information regarding all chapters in grade 7.

Go Math Grade 1 Answer Key Chapter 7 Compare Numbers

Do you want to get the best results in math tests? Refer to Go Math Grade 1 Chapter 7 Answer Key Compare Numbers. Get the free pdf of Go Math Grade 1 Answer Key Chapter 7 Compare Numbers now to start your practice. Just follow and grab more knowledge by referring to Go Math Grade 1 Chapter 7 Solution Key Compare Numbers.

Go Math Grade 1 Chapter 7 Answer Key Compare Numbers

Apply the math to real-time examples by learning the tricks using HMH Go Math Grade 1 Chapter 7 Answer Key Compare Numbers. The quick way of solving math problems will help the students to save time. So, students can practice more questions utilizing the time properly. If you want the best way of learning then you must use Go Math 1st Grade Chapter 7 Answer Key Compare Numbers.

Chapter 7: Compare Numbers

Lesson: 1 Algebra • Greater Than

Lesson: 2 Algebra • Less Than

Lesson: 3 Algebra • Use Symbols to Compare

Mid-Chapter Checkpoint

Lesson: 4 Problem Solving • Compare Numbers

Lesson: 5 10 Less, 10 More

Review

Compare Numbers Show What You Know

Model More
Draw lines to match.
Circle the set that has more.
Question 1.
1st Grade Go Math Answer Key Chapter 7 Compare Numbers 1.1
Answer:
1st-Grade-Go-Math-Answer-Key-Chapter-7-Compare-Numbers-1.1

Question 2.
1st Grade Go Math Answer Key Chapter 7 Compare Numbers 1.2
Answer:
1st-Grade-Go-Math-Answer-Key-Chapter-7-Compare-Numbers-1.2

More, Fewer
Question 3.
Circle the row that has more.
1st Grade Go Math Answer Key Chapter 7 Compare Numbers 1.3
Answer:
1st-Grade-Go-Math-Answer-Key-Chapter-7-Compare-Numbers-1.3

Question 4.
Circle the row that has fewer
1st Grade Go Math Answer Key Chapter 7 Compare Numbers 1.4
Answer:
1st-Grade-Go-Math-Answer-Key-Chapter-7-Compare-Numbers-1.4

Draw Equal Groups
Question 5.
Draw a ball for each glove.
1st Grade Go Math Answer Key Chapter 7 Compare Numbers 1.5
Answer:
1st-Grade-Go-Math-Answer-Key-Chapter-7-Compare-Numbers-1.5

Compare Numbers Vocabulary Builder

1st Grade Go Math Answer Key Chapter 7 Compare Numbers 1.6
Visualize It
Draw pictures in the box to show more, fewer, or the same number.
1st Grade Go Math Answer Key Chapter 7 Compare Numbers 1.7
Answer:
1st-Grade-Go-Math-Answer-Key-Chapter-7-Compare-Numbers-1.7

Understand Vocabulary
Complete the sentences with review words.
Question 1.
I see 2 white cats and 4 yellow cats. I see ______ yellow cats than white cats.
Answer: I see 2 white cats and 4 yellow cats. I see more yellow cats than white cats.

Question 2.
Dave has 9 grapes. Ann has 6 grapes. Ann has _______ grapes than Dave.
Answer: Dave has 9 grapes. Ann has 6 grapes. Ann has less grapes than Dave.

Question 3.
5 ducks and 5 swans are at the pond. There are the ______ number of ducks and swans.
Answer: 5 ducks and 5 swans are at the pond. There are the same number of ducks and swans.

Compare Numbers Game Rainy Day Bingo

Materials
1st Grade Go Math Answer Key Chapter 7 Compare Numbers 2.1
Play with a partner.

  1. Toss the 1st Grade Go Math Answer Key Chapter 7 Compare Numbers 2.3
  2. Use 1st Grade Go Math Answer Key Chapter 7 Compare Numbers 2.2 to cover one space that shows a number that is 1 more.
  3. If you do not have a space that shows the number, your turn is over.
  4. The other player takes a turn.
  5. The first player to cover all of his or her spaces wins.
    1st Grade Go Math Answer Key Chapter 7 Compare Numbers 2.4

Compare Numbers Vocabulary Game

Guess the Word
1st Grade Go Math Answer Key Chapter 7 Compare Numbers 2.5
Materials
timer
How to Play
Play with a partner.

  1. Choose a math word from the Word Box. Do not tell your partner.
  2. Set the timer.
  3. Give a one-word clue.
  4. Your partner tries to guess the secret word.
  5. Repeat with a new one-word clue until your partner guesses correctly or time runs out.
  6. Take turns.
  7. The first player to correctly guess 5 words wins.

The Write Way
Reflect
Choose one idea. Draw and write about it.

  • Write two questions you have about comparing numbers.
  • Explain how to compare 43 and 35.

Lesson 7.1 Algebra • Greater Than

Essential Question How can you compare two numbers to find which is greater?

Listen and Draw

Use 1st Grade Go Math Answer Key Chapter 7 Compare Numbers 7.1 1 or an iTool to solve. Draw quick pictures to show your work.
1st Grade Go Math Answer Key Chapter 7 Compare Numbers 7.1 2

MATHEMATICAL PRACTICES
How did you use tools to decide which number is greater?
Answer:

Share and Show

Use your MathBoard and 1st Grade Go Math Answer Key Chapter 7 Compare Numbers 7.1 1 to show each number.
1st Grade Go Math Answer Key Chapter 7 Compare Numbers 7.1 3
Answer:
1st-Grade-Go-Math-Answer-Key-Chapter-7-Compare-Numbers-7.1-3

On Your Own

MATHEMATICAL PRACTICE Use a Concrete Model Use 1st Grade Go Math Answer Key Chapter 7 Compare Numbers 7.1 1 if you need to.
1st Grade Go Math Answer Key Chapter 7 Compare Numbers 7.1 4
Answer:
1st-Grade-Go-Math-Answer-Key-Chapter-7-Compare-Numbers-7.1-4

Write or draw to solve.
Question 6.
THINK SMARTER
Pam and Jake play a game for points. Pam’s points are 1 ten 6 ones. Jake’s points are 1 one 6 ten’s. Who has the greatest number of points?
1st Grade Go Math Answer Key Chapter 7 Compare Numbers 7.1 5
________
Answer: pam=1 ten +6 ones=10+6=16
jake=1 one +6 ten’s=1+60 =61
61>16

jake’s points are greatest than pam.

Question 7.
GO DEEPER
John has 51 cards. Paul has 32 cards. George has a stack of cards greater than either Paul or John. How many cards might George have?
____________
____________
Answer: John has 51 cards.
Paul has 32 cards.
George has a number of cards greater than either paul or john.
This means Gorge does have the number of cards greater than both paul and john.
The number of cards with john is 51 and the number of cars with Pail is 32 cards.
Therefore the number of cards that george may have is in between 33 and 51.
If George has any number of cards greater than 32 and less than 52, then this number of cards is greater than cards with Paul bus less than the cards with John.
The number of cards George has is >32 but <52

Problem Solving • Applications

Question 8.
THINK SMARTER
Color the balloons that show numbers greater than 56.
1st Grade Go Math Answer Key Chapter 7 Compare Numbers 7.1 6
Answer:
1st-Grade-Go-Math-Answer-Key-Chapter-7-Compare-Numbers-7.1-6

Question 9.
THINK SMARTER
Compare. Is the math sentence true? Choose Yes or No.
1st Grade Go Math Answer Key Chapter 7 Compare Numbers 7.1 7
Answer:
1st-Grade-Go-Math-Answer-Key-Chapter-7-Compare-Numbers-7.1-7

TAKE HOME ACTIVITY • Write 38, 63, 68, and 83 on slips of paper. Show your child two numbers, and ask which number is greater. Repeat with different pairs of numbers.

Algebra • Greater Than Homework & Practice 7.1

Use 1st Grade Go Math Answer Key Chapter 7 Compare Numbers 7.1 8 if you need to.
1st Grade Go Math Answer Key Chapter 7 Compare Numbers 7.1 9
Answer:
1st-Grade-Go-Math-Answer-Key-Chapter-7-Compare-Numbers-7.1-9

Problem Solving
Question 4.
Color the blocks that show numbers greater than 47.
1st Grade Go Math Answer Key Chapter 7 Compare Numbers 7.1 10
Answer:
1st-Grade-Go-Math-Answer-Key-Chapter-7-Compare-Numbers-7.1-10

Question 5.
WRITE
Write a number that is greater than 29. Draw quick pictures to explain.
Answer: 30
1st-Grade-Go-Math-Answer-Key-Chapter-7-Compare-Numbers-7.1-10 (1)

Lesson Check
Question 1.
Circle the number that is greater than 65. Write the numbers.
37
49
56
66
_____ is greater than _____.
_____ > _____
Answer: 66
66 is greater than 65.
66 > 65

Question 2.
Circle the number that is greater than 29. Write the numbers.
19
20
28
92
_____ is greater than _____.
_____ > _____
Answer:
92 is greater than 29.
92 > 29

Spiral Review
Question 3.
What is 5 + 7? Write the sum.
5 + 7 = _____
Answer:5 + 7 = 12

Question 4.
Count forward. Write the number that is missing.
110, 111, ____, 113, 114
Answer:110, 111, 112, 113, 114

Lesson 7.2 Algebra • Less Than

Essential Question How can you compare two numbers to find which is less?

Listen and Draw

Use 1st Grade Go Math Answer Key Chapter 7 Compare Numbers 7.2 1 to solve. Draw quick pictures to show your work.
1st Grade Go Math Answer Key Chapter 7 Compare Numbers 7.2 2

MATHEMATICAL PRACTICES
Compare How does your drawing show which number is less? Explain.
Answer:

Share and Show

Use your MathBoard and 1st Grade Go Math Answer Key Chapter 7 Compare Numbers 7.2 1 to show each number.
1st Grade Go Math Answer Key Chapter 7 Compare Numbers 7.2 3
Answer:
1st-Grade-Go-Math-Answer-Key-Chapter-7-Compare-Numbers-7.2-3

On Your Own

MATHEMATICAL PRACTICE Use a Concrete Model
GO DEEPER Use 1st Grade Go Math Answer Key Chapter 7 Compare Numbers 7.2 1 if you need to.
1st Grade Go Math Answer Key Chapter 7 Compare Numbers 7.2 4
Answer:
1st-Grade-Go-Math-Answer-Key-Chapter-7-Compare-Numbers-7.2-4

Write a number to solve.
Question 8.
THINK SMARTER
Ella has 22 beads. Lauren has fewer beads than Ella. How many beads might Lauren have?
_______
Answer: 21
Ella has 22 deads
If Lauren has less than Ella
Then Lauren must have beads less than 22
21<22

Problem Solving • Applications

Write a number to solve.
Question 9.
THINK SMARTER
Nan makes the number 46. Marty makes a number that is less than 46. What could be a number Marty makes?
__________
Answer: 45
45 is less than 46.
4546

Question 10.
THINK SMARTER
Jack makes the number 92. Kit makes a number that has fewer ones than 92. What could be a number Kit makes?
1st Grade Go Math Answer Key Chapter 7 Compare Numbers 7.2 5
_________
Answer: 91
91 is less than 92.
91 < 92

Question 11.
THINK SMARTER
Write a number that is less than 67.

How do you know your number is less than 67?
Answer: 66
67=6 ten’s 7ones
66=6 ten’s 6ones
Therefore ones helps to identift the lesser number

TAKE HOME ACTIVITY • Write 47, 54, 57, and 74 on slips of paper. Show your child two numbers, and ask which number is less. Repeat with different pairs of numbers.

Algebra • Less Than Homework & Practice 7.2

Use 1st Grade Go Math Answer Key Chapter 7 Compare Numbers 7.2 6 if you need to.
1st Grade Go Math Answer Key Chapter 7 Compare Numbers 7.2 7
Answer:
1st-Grade-Go-Math-Answer-Key-Chapter-7-Compare-Numbers-7.2-7

Problem Solving
Write a number to solve.
Question 4.
Lori makes the number 74. Gabe makes a number that is less than 74. What could be a number Gabe makes?
________
Answer:73
73 is less than 74.
73 < 74

Question 5.
WRITE
Write a number that is less than 41. Draw quick pictures to explain.
Answer: 40
1st-Grade-Go-Math-Answer-Key-Chapter-7-Compare-Numbers-7.1-10 (2)

Lesson Check
Question 1.
Circle the number that is less than 52. Write the numbers.
25
52
64
88
_____ is less than _____
____ < ____
Answer:25
25 is less than 52
25 < 52

Question 2.
Circle the number that is less than 76. Write the numbers.
100
81
77
59
_____ is less than _____
____ < ____
Answer:59
59 is less than 76.
59 < 76

Spiral Review
Question 3.
Write the number. What number does the model show?
1st Grade Go Math Answer Key Chapter 7 Compare Numbers 7.2 8
_____ ten’s and ____ ones = ____
Answer: 30 ten’s and 7 ones = 37

Question 4.
Count by ten’s. Write the missing numbers.
8, 18, 28, ____, ____, 58
Answer: 8, 18, 28, 38, 48, 58

Lesson 7.3 Algebra • Use Symbols to Compare

Essential Question How can you use symbols to show how numbers compare?

Listen and Draw

Use Go Math 1st Grade Answer Key Chapter 7 Compare Numbers 7.3 1. Draw quick pictures to show your work. Write the numbers to compare.
Go Math 1st Grade Answer Key Chapter 7 Compare Numbers 7.3 2

Answer:
Go-Math-1st-Grade-Answer-Key-Chapter-7-Compare-Numbers-7.3-2

MATHEMATICAL PRACTICES
Represent Compare 47 and 32 in two ways. What two symbols do you use?
Answer: 47 is greater than 32
32 is less than 47
47>32
32<47

Share and Show

Use Go Math 1st Grade Answer Key Chapter 7 Compare Numbers 7.3 1. Draw to show each number. Write <, >, or =. Complete the sentence.
Question 1.
Go Math 1st Grade Answer Key Chapter 7 Compare Numbers 7.3 3
Answer:
Go-Math-1st-Grade-Answer-Key-Chapter-7-Compare-Numbers-7.3-3

Question 2.
Go Math 1st Grade Answer Key Chapter 7 Compare Numbers 7.3 4
Answer:
Go-Math-1st-Grade-Answer-Key-Chapter-7-Compare-Numbers-7.3-4

Question 3.
Go Math 1st Grade Answer Key Chapter 7 Compare Numbers 7.3 5
Answer:
Go-Math-1st-Grade-Answer-Key-Chapter-7-Compare-Numbers-7.3-5

Question 4.
Go Math 1st Grade Answer Key Chapter 7 Compare Numbers 7.3 6
Answer:
Go-Math-1st-Grade-Answer-Key-Chapter-7-Compare-Numbers-7.3-6

On Your Own

Go Math 1st Grade Answer Key Chapter 7 Compare Numbers 7.3 7
MATHEMATICAL PRACTICE Use Symbols
GO DEEPER Write <, >, or =. Draw a quick picture if you need to.
Question 5.
Go Math 1st Grade Answer Key Chapter 7 Compare Numbers 7.3 8
Answer:
Go-Math-1st-Grade-Answer-Key-Chapter-7-Compare-Numbers-7.3-8

Question 6.
Go Math 1st Grade Answer Key Chapter 7 Compare Numbers 7.3 9
Answer:
Go-Math-1st-Grade-Answer-Key-Chapter-7-Compare-Numbers-7.3-9

Question 7.
Go Math 1st Grade Answer Key Chapter 7 Compare Numbers 7.3 10
Answer:
Go-Math-1st-Grade-Answer-Key-Chapter-7-Compare-Numbers-7.3-10

Question 8.
Go Math 1st Grade Answer Key Chapter 7 Compare Numbers 7.3 11
Answer:
Go-Math-1st-Grade-Answer-Key-Chapter-7-Compare-Numbers-7.3-11

Question 9.
Go Math 1st Grade Answer Key Chapter 7 Compare Numbers 7.3 12
Answer:
Go-Math-1st-Grade-Answer-Key-Chapter-7-Compare-Numbers-7.3-12

Question 10.
THINK SMARTER
Gill and Rob win tokens in a game. Gill has 86 tokens. Rob has 61 tokens. 70 tokens are needed for a prize. Who has enough tokens for a prize? Write the number.
Go Math 1st Grade Answer Key Chapter 7 Compare Numbers 7.3 13
______
Answer: Gill has enough money.
Gill has 86 tokens.
Rob has 61 tokens.
The total tokens needed is 70
Gill has more than 70.
Rob has less than 70.
86>70
61<70

THINK SMARTER Write numbers to solve.
Question 11.
96 = _____
Answer: 96=96

Question 12.
53 > _____
Answer: 53 > 52

Question 13.
83 < _____
Answer: 83 < 84

Question 14.
40 < _____
Answer: 40 < 41

Question 15.
71 > ____
Answer: 71 > 70

Question 16.
29 = _____
Answer: 29 = 29

TAKE HOME ACTIVITY • Have your child show you how to write <, >, and = to compare two numbers. Ask him or her to use words to explain each comparison.

Algebra • Use Symbols to Compare Homework & Practice 7.3

Write <, >, or =. Draw a quick picture if you need to.
Question 1.
Go Math 1st Grade Answer Key Chapter 7 Compare Numbers 7.3 14
Answer:
Go-Math-1st-Grade-Answer-Key-Chapter-7-Compare-Numbers-7.3-14

Question 2.
Go Math 1st Grade Answer Key Chapter 7 Compare Numbers 7.3 15
Answer:
Go-Math-1st-Grade-Answer-Key-Chapter-7-Compare-Numbers-7.3-15

Question 3.
Go Math 1st Grade Answer Key Chapter 7 Compare Numbers 7.3 16
Answer:
Go-Math-1st-Grade-Answer-Key-Chapter-7-Compare-Numbers-7.3-16

Question 4.
Go Math 1st Grade Answer Key Chapter 7 Compare Numbers 7.3 17
Answer:
Go-Math-1st-Grade-Answer-Key-Chapter-7-Compare-Numbers-7.3-17

Question 5.
Go Math 1st Grade Answer Key Chapter 7 Compare Numbers 7.3 18
Answer:
Go-Math-1st-Grade-Answer-Key-Chapter-7-Compare-Numbers-7.3-18

Question 6.
Go Math 1st Grade Answer Key Chapter 7 Compare Numbers 7.3 19
Answer:
Go-Math-1st-Grade-Answer-Key-Chapter-7-Compare-Numbers-7.3-19

Question 7.
Go Math 1st Grade Answer Key Chapter 7 Compare Numbers 7.3 20
Answer:
Go-Math-1st-Grade-Answer-Key-Chapter-7-Compare-Numbers-7.3-20

Question 8.
Go Math 1st Grade Answer Key Chapter 7 Compare Numbers 7.3 21
Answer:
Go-Math-1st-Grade-Answer-Key-Chapter-7-Compare-Numbers-7.3-21

Problem Solving
Write <, >, or = to solve. Circle your answer.
Question 9.
Tracey has 26 shells. Heba has 29 shells. Who has a greater number of shells?
Tracey Heba
Go Math 1st Grade Answer Key Chapter 7 Compare Numbers 7.3 22
Answer:
Go-Math-1st-Grade-Answer-Key-Chapter-7-Compare-Numbers-7.3-22

Question 10.
WRITE
Choose some numbers to compare to 55. Use <, >, and =.
Answer: 55<56
55>54
55=55
55= 5 tens 5 ones
54= 5 tens 4 ones
56=5 tens 6 ones

Lesson Check
Question 1.
Compare each pair of numbers. Write <, >, or =.
Go Math 1st Grade Answer Key Chapter 7 Compare Numbers 7.3 23
Answer:
Go-Math-1st-Grade-Answer-Key-Chapter-7-Compare-Numbers-7.3-23

Question 2.
Compare each pair of numbers. Write <, >, or =
Go Math 1st Grade Answer Key Chapter 7 Compare Numbers 7.3 24
Answer:
Go-Math-1st-Grade-Answer-Key-Chapter-7-Compare-Numbers-7.3-24

Spiral Review
Question 3.
What number does the model show? Write the numbers.
Go Math 1st Grade Answer Key Chapter 7 Compare Numbers 7.3 25
____ ten’s and ____ ones = ____
Answer: 7 ten’s and 9 ones = 79

Question 4.
Use the model. Write the number three different ways.
Go Math 1st Grade Answer Key Chapter 7 Compare Numbers 7.3 26
____ ten ____ones
___ + ____
_____
Answer: 1 ten 8 ones
10 + 8
18

Compare Numbers Mid-Chapter Checkpoint

Concepts and Skills

Circle the greater number. Write the numbers.
Question 1.
38
83
____ is greater than _____.
____ > ____
Answer: 83
83 is greater than 38.
83 > 38

Circle the number that is less. Write the numbers.
Question 2.
61
29
____ is less than _____.
____ < ____
Answer: 29
29 is less than 61.
29 < 61

Question 3.
Matt scores 34 points and wins the game. Lee scores points and does not win. The number of Lee’s points is less than the number of Matt’s points. Is Lee’s score 49 or 29?
_______
Answer: 29
Matt score is 34
34= 3 tens 4 ones.
49= 4 tens 9 ones.
29= 2 tens 9 ones.
49 is greater than 34.
49>34
Therefore lee’s score must be less than 34.
29 is less than 34.
29<34.

Question 4.
THINK SMARTER
Circle the symbol that makes the math sentence true.
Go Math 1st Grade Answer Key Chapter 7 Compare Numbers 7.3 27
Answer:
Go-Math-1st-Grade-Answer-Key-Chapter-7-Compare-Numbers-7.3-27

Lesson 7.4 Problem Solving • Compare Numbers

Essential Question How can making a model help you compare numbers?

Cassidy has the number cards shown below. She gives away the cards with numbers less than 49 and greater than 53. Which number cards does Cassidy have now?

HOME CONNECTION • Your child made a model of the problem. The numbers crossed out are less than 49 and also greater than 53. The remaining numbers solve the problem.

Try Another Problem

Go Math Answer Key Grade 1 Chapter 7 Compare Numbers 7.4 1
Make a model to solve.
Question 1.
Tony has these number cards. He gives away the cards with numbers less than 16 and greater than 19. Which number cards does Tony have now?
Go Math Answer Key Grade 1 Chapter 7 Compare Numbers 7.4 2
Tony has number cards _____ .
Answer: Tony has number cards 17,18,22.

Question 2.
Carol has these number cards. She keeps the cards with numbers greater than 98 and less than 95. Circle the number cards Carol keeps.
Go Math Answer Key Grade 1 Chapter 7 Compare Numbers 7.4 3
Carol keeps number cards ______.
Answer: Carol keeps number cards 99,91,90.

MATHEMATICAL PRACTICES
Model Explain how you can find the number cards Tony has now.
Answer:

Share and Show

MATHEMATICAL PRACTICE Use Models Make a model to solve.
Question 3.
Felipe has these number cards. He gives away cards with numbers less than 60 and greater than 65. Which number cards does Felipe have now?
Go Math Answer Key Grade 1 Chapter 7 Compare Numbers 7.4 4
Felipe has number cards ______.
Answer: Felipe has number cards 58,68.

Question 4.
THINK SMARTER
Molly underlines the number cards greater than 76 and circles the number cards less than 84. Which number cards are both greater than 76 and less than 84?
Go Math Answer Key Grade 1 Chapter 7 Compare Numbers 7.4 5
Number cards ________ are both greater than 76 and less than 84.
Answer: Number cards 78 and 82 are both greater than 76 and less than 84.

On Your Own

WRITE Choose a way to solve. Draw or write to explain.
Question 5.
GO DEEPER
Some cows were in the field. 6 more cows walked there. Then there were 13 cows. How many cows were in the field before?
______ cows
Answer: 7 cows

Explanation:
Given that,
Some cows were in the field. 6 more cows walked there.
Then there were 13 cows.
6+x=13
x=13-6
x=7

Question 6.
THINK SMARTER
Ed has 6 marbles. How many marbles can he put in a red cup and how many can he put in a blue cup?
Go Math Answer Key Grade 1 Chapter 7 Compare Numbers 7.4 6
____ + ____ = 6
Answer: 3+3-6

Question 7.
THINK SMARTER+
Lani has these number cards. Write each number in the box to show less than 24 or greater than 24.
Go Math Answer Key Grade 1 Chapter 7 Compare Numbers 7.4 7
Answer:
Go-Math-Answer-Key-Grade-1-Chapter-7-Compare-Numbers-7.4-7

TAKE HOME ACTIVITY • Ask your child to tell you a number that is greater than 59 and a number less than 59.

Problem Solving • Compare Numbers Homework & Practice 7.4

Make a model to solve.
Question 1.
Ava has these number cards. She gives away cards with numbers less than 34 and greater than 38. Which number cards does Ava have now?
Go Math Answer Key Grade 1 Chapter 7 Compare Numbers 7.4 8
Ava has number cards ______.
Answer: Ava has number cards 35,37.

Question 2.
Ron has these number cards. He keeps the cards with numbers greater than 60 and less than 56. Circle the number cards Ron keeps.
Go Math Answer Key Grade 1 Chapter 7 Compare Numbers 7.4 9
Ron keeps number cards ______.
Answer: Ron keeps number cards 61,54.

Question 3.
WRITE
Write your own problem. Choose a secret number. Write clues about the number using the words is greater than and is less than.
________________
________________
Answer: x is greater than 98 and less than 100
x>98
x<100

Lesson Check
Question 1.
Juan crosses out the numbers that are less than 45 and greater than 50. Circle the numbers that are left.
Go Math Answer Key Grade 1 Chapter 7 Compare Numbers 7.4 10
Answer:
Go-Math-Answer-Key-Grade-1-Chapter-7-Compare-Numbers-7.4-10

Spiral Review
Question 2.
Count back 3. Write is the difference.
9 – 3 = _____
Answer: 9-3=6

Question 3.
Write the number to complete the related facts.
4 + 7 = 11
7 + 4 = 11
11 – 4 = 7
____ – _____ = ____
Answer: 711=4

Lesson 7.5 10 Less, 10 More

Essential Question How can you identify numbers that are 10 less or 10 more than a number?

Listen and Draw

Use Go Math Grade 1 Answer Key Chapter 7 Compare Numbers 7.5 1 to solve. Draw quick pictures to show your work.
Go Math Grade 1 Answer Key Chapter 7 Compare Numbers 7.5 2
Pat ________
Tony _____
Jan ______

MATHEMATICAL PRACTICES
What number has one less 10 than 12? Explain.
Answer: 2
We are looking for a new number which is 10 less than 12.
We will get the new number by subtracting 10 from 12.
We write it down as:
12-10=2

Model and Draw

Go Math Grade 1 Answer Key Chapter 7 Compare Numbers 7.5 3
Answer:
Go-Math-Grade-1-Answer-Key-Chapter-7-Compare-Numbers-7.5-3

Share and Show

Use mental math. Write the numbers that are 10 less and 10 more.
Question 1.
Go Math Grade 1 Answer Key Chapter 7 Compare Numbers 7.5 4
Answer:
Go-Math-Grade-1-Answer-Key-Chapter-7-Compare-Numbers-7.5-4

Question 2.
Go Math Grade 1 Answer Key Chapter 7 Compare Numbers 7.5 5
Answer:
Go-Math-Grade-1-Answer-Key-Chapter-7-Compare-Numbers-7.5-5

Question 3.
Go Math Grade 1 Answer Key Chapter 7 Compare Numbers 7.5 6
Answer:
Go-Math-Grade-1-Answer-Key-Chapter-7-Compare-Numbers-7.5-6

Question 4.
Go Math Grade 1 Answer Key Chapter 7 Compare Numbers 7.5 7
Answer:
Go-Math-Grade-1-Answer-Key-Chapter-7-Compare-Numbers-7.5-7

Question 5.
Go Math Grade 1 Answer Key Chapter 7 Compare Numbers 7.5 8
Answer:
Go-Math-Grade-1-Answer-Key-Chapter-7-Compare-Numbers-7.5-8

Question 6.
Go Math Grade 1 Answer Key Chapter 7 Compare Numbers 7.5 9
Answer:
Go-Math-Grade-1-Answer-Key-Chapter-7-Compare-Numbers-7.5-9

Question 7.
Go Math Grade 1 Answer Key Chapter 7 Compare Numbers 7.5 10
Answer:
Go-Math-Grade-1-Answer-Key-Chapter-7-Compare-Numbers-7.5-10

Question 8.
Go Math Grade 1 Answer Key Chapter 7 Compare Numbers 7.5 11
Answer:
Go-Math-Grade-1-Answer-Key-Chapter-7-Compare-Numbers-7.5-11

On Your Own

MATHEMATICAL PRACTICE Apply Use mental math. Complete the chart. Explain your method.
Go Math Grade 1 Answer Key Chapter 7 Compare Numbers 7.5 12
Answer:
Go-Math-Grade-1-Answer-Key-Chapter-7-Compare-Numbers-7.5-12 (1)

Question 16.
THINK SMARTER
Solve. I have 89 rocks. I want to collect 10 more. How many rocks will I have then?
Go Math Grade 1 Answer Key Chapter 7 Compare Numbers 7.5 13
_______ rocks
Answer: 99
89+10=99

Problem Solving • Applications

Choose a way to solve. Draw or write to show your work.
Question 17.
The plant has 4 fewer ladybugs on it than the tree. The tree has 7 ladybugs on it. How many ladybugs are on the plant?
Go Math Grade 1 Answer Key Chapter 7 Compare Numbers 7.5 14
______ lady bugs
Answer: 3 lady bugs.
Tree has 7 ladybugs on it.
Plant has 4 less than tree.
7-4=3

Question 18.
Amy has 7 ribbons. Charlotte has 9 ribbons. How many more ribbons does Charlotte have than Amy?
Go Math Grade 1 Answer Key Chapter 7 Compare Numbers 7.5 15
______ more ribbons
Answer: 2 more ribbons.
Amy has 7 ribbons.
Charlotte has 9 ribbons.
9-7=2

Question 19.
GO DEEPER
Margo has 28 stamps. Chet has 10 more stamps than Margo. Luis has 10 more stamps than Chet. How many stamps does Luis have?
Go Math Grade 1 Answer Key Chapter 7 Compare Numbers 7.5 16
_______ stamps
Answer: 48 stamps.
Margo has 28 stamps.
Chet has 10 more stamps than Margo = 28 +10 = 38.
Luis has 10 more stamps than Chet = 38 + 10 = 48.

Question 20.
THINK SMARTER+
Draw a quick picture to show a number that is 10 less than the model.
Go Math Grade 1 Answer Key Chapter 7 Compare Numbers 7.5 17
What is the new number?Go Math Grade 1 Answer Key Chapter 7 Compare Numbers 7.5 18
Answer: What is the new number?Go-Math-Grade-1-Answer-Key-Chapter-7-Compare-Numbers-7.5-18

TAKE HOME ACTIVITY • Write a two-digit number, such as 25, 40, or 81. Ask your child to identify the numbers that are ten less than and ten more than that number. Repeat with other numbers.

10 Less, 10 More Homework & Practice 7.5

Use mental math. Complete the chart.
Go Math Grade 1 Answer Key Chapter 7 Compare Numbers 7.5 19
Answer:
Go-Math-Grade-1-Answer-Key-Chapter-7-Compare-Numbers-7.5-19

Problem Solving
Choose a way to solve. Draw or write to show your work.
Question 5.
Jim has 16 books. Doug has 10 fewer books than Jim. How many books does Doug have?
______ books
Answer: Doug has 6 books.
16 – 10 = 6

Question 6.
WRITE
Choose a number from 10 to 90. Draw and write to show the numbers that are 10 less and 10 more than your number.
Answer:
Go-Math-Grade-1-Answer-Key-Chapter-7-Compare-Numbers-7.5-4

Lesson Check
Question 1.
What number is 10 less than 67. Write the number.
_______
Answer:57
67-10=57

Question 2.
What number is 10 more than 39. Write the number.
________
Answer: 49
39+10=49

Spiral Review
Question 3.
How many ten’s and ones make this number? Write how many.
Go Math Grade 1 Answer Key Chapter 7 Compare Numbers 7.5 20
____ ten ____ ones
Answer: 1 ten 8 ones.

Question 4.
Look at the model. Write the number. What number does the model show?
Go Math Grade 1 Answer Key Chapter 7 Compare Numbers 7.5 21
_____ ten’s = ____
Answer: 7 ten’s = 70.

Compare Numbers Review/Test

Question 1.
Compare. Is the math sentence true? Choose Yes or No.
Go Math Grade 1 Chapter 7 Answer Key Pdf Compare Numbers rt 1
Answer:
Go-Math-Grade-1-Chapter-7-Answer-Key-Pdf-Compare-Numbers-rt-1

Question 2.
Choose all the numbers that are less than 71.
Go Math Grade 1 Chapter 7 Answer Key Pdf Compare Numbers rt 2
Answer:
Go-Math-Grade-1-Chapter-7-Answer-Key-Pdf-Compare-Numbers-rt-2

Question 3.
Circle the symbol that makes the math sentence true.
Go Math Grade 1 Chapter 7 Answer Key Pdf Compare Numbers rt 3
Answer:
Go-Math-Grade-1-Chapter-7-Answer-Key-Pdf-Compare-Numbers-rt-3

Question 4.
Megan has these number cards. Write each number in the box to show less than 33 or greater than 33.
Go Math Grade 1 Chapter 7 Answer Key Pdf Compare Numbers rt 4
Answer:
Go-Math-Grade-1-Chapter-7-Answer-Key-Pdf-Compare-Numbers-rt-4

Question 5.
Use mental math. Complete the chart.
Go Math Grade 1 Chapter 7 Answer Key Pdf Compare Numbers rt 5
Answer:
Go-Math-Grade-1-Chapter-7-Answer-Key-Pdf-Compare-Numbers-rt-5

Question 6.
Write a number that is less than 30.
__________________
How do you know your number is less than 30?
_______________
Answer: 29
30 = 3 ten’s
29= 2 ten’s 9 ones
then’s help to know the number less than 30

Question 7.
Choose all the math sentences that are true.
Go Math Grade 1 Chapter 7 Answer Key Pdf Compare Numbers rt 7
Answer:
Go-Math-Grade-1-Chapter-7-Answer-Key-Pdf-Compare-Numbers-rt-7

Question 8.
James circles the numbers that are less than 87 or greater than 91. Which numbers does James circle?
Go Math Grade 1 Chapter 7 Answer Key Pdf Compare Numbers rt 8
James circles ______ and _____.
Answer: James circles 86 and 92.

Question 9.
THINK SMARTER+
Draw a quick picture to show a number that is 10 more than the model.
Go Math Grade 1 Chapter 7 Answer Key Pdf Compare Numbers rt 9
Answer:
Go-Math-Grade-1-Chapter-7-Answer-Key-Pdf-Compare-Numbers-rt-10

Question 10.
Compare. Is the math sentence true? Circle yes or no.
Go Math Grade 1 Chapter 7 Answer Key Pdf Compare Numbers rt 10
Answer:
Go-Math-Grade-1-Chapter-7-Answer-Key-Pdf-Compare-Numbers-rt-10

Question 11.
GO DEEPER
Write <, >, or = to compare the numbers.
Go Math Grade 1 Chapter 7 Answer Key Pdf Compare Numbers rt 11
How do the drawings help you compare the numbers?
Answer:
Straight line indicates ten’s.
Circles indicates ones.
48=4 ten’s 8 ones.
36=3 ten’s 6 ones.
Therefore ten’s helping to compare the numbers.

Question 12.
Circle the words that make the sentence true.
Go Math Grade 1 Chapter 7 Answer Key Pdf Compare Numbers rt 12
Answer:
We have to compare the two numbers 88 and 90.
88 is less than 90.
Go-Math-Grade-1-Chapter-7-Answer-Key-Pdf-Compare-Numbers-rt-12

Final Words:

We hope the details mentioned in the above pdf are helpful for all the grade 1 students. Go Math Grade 1 Answer Key Chapter 7 Compare Numbers PDF for download. Get the explanation for every problem along with practice questions. Students can easily solve any math questions in minutes with the help of Go Math Grade 1 Answer Key Ch 7 Compare Numbers.

Go Math Grade 6 Answer Key Chapter 2 Fractions and Decimals

go-math-grade-6-chapter-2-fractions-and-decimals-answer-key

Are you looking for the best material to score top in the exams? Then, you are in the right place. HMH Go Math Grade 6 Answer Key Chapter 2 Fractions and Decimals is the best material for the 6th standard students. Here you can find the explanations for each and every question in different methods. Refer to Go Math Grade 6 Chapter 2 Fractions and Decimals Solution Key to learn the concepts of the chapter. So, Download HMH Go Math Grade 6 Answer Key Chapter 2 Fractions and Decimals for free.

Download Go Math Grade 6 Chapter 2 Fractions and Decimals Answer Key PDF

The Go Math Grade 6 Chapter 2 Fractions and Decimals Solution Key consists of various topics like compare and order fractions and decimals, multiply fractions, Divide Fractions, Model Mixed Number Division, etc. We have provided detailed explanations in simple methods here. All the solutions are prepared according to the topics in the Fractions and Decimals Chapter. So, access the links and start your preparation for the exams.

Lesson 1: Fractions and Decimals

Lesson 2: Compare and Order Fractions and Decimals

Lesson 3: Multiply Fractions

Lesson 4: Simplify Factors

Mid-Chapter Checkpoint

Lesson 5: Investigate • Model Fraction Division

Lesson 6: Estimate Quotients

Lesson 7: Divide Fractions

Lesson 8: Investigate • Model Mixed Number Division

Lesson 9: Divide Mixed Numbers

Lesson 10: Problem Solving • Fraction Operations

Chapter 2 Review/Test

Share and Show – Page No. 71

Write as a fraction or as a mixed number in simplest form.

Question 1.
95.5
_____ \(\frac{□}{□}\)

Answer:
\(\frac{1}{2}\)

Explanation:
95.5
95.5 is 95 ones and 5 tenths.
5 tenths = \(\frac{5}{10}\)
Simplify using the GCF.
The GCF of 5 and 10 is 10.
Divide the numerator and the denominator by 10
\(\frac{5 ÷ 10}{10 ÷ 10}\) = \(\frac{1}{2}\)

Question 2.
0.6
\(\frac{□}{□}\)

Answer:
\(\frac{3}{5}\)

Explanation:
0.6
6 tenths = \(\frac{6}{10}\)
Simplify using the GCF.
The GCF of 6 and 10 is 2.
Divide the numerator and the denominator by 10
\(\frac{6 ÷ 2}{10 ÷ 2}\) = \(\frac{3}{5}\)

Question 3.
5.75
_____ \(\frac{□}{□}\)

Answer:
5\(\frac{3}{4}\)

Explanation:
5.75 is 5 ones and 75 hundredths.
75 hundredths = \(\frac{75}{100}\)
Simplify using the GCF.
The GCF of 75 and 100 is 25.
Divide the numerator and the denominator by 25
5\(\frac{75 ÷ 25}{100 ÷ 25}\) = 5\(\frac{3}{4}\)

Write as a decimal.

Question 4.
\(\frac{7}{8}\)
_____

Answer:
0.875

Explanation:
Use division to rename the fraction part as a decimal.
7/8 = 0.875
The quotient has 3 decimal places.
Add the whole number to the decimal.
0 + 0.875 = 0.875.
So, \(\frac{7}{8}\) = 0.875

Question 5.
\(\frac{13}{20}\)
_____

Answer:
0.65

Explanation:
Use division to rename the fraction part as a decimal.
\(\frac{13}{20}\) = 0.65
The quotient has 2 decimal places.
Add the whole number to the decimal.
0 + 0.65 = 0.65.
So, \(\frac{13}{20}\) = 0.65

Question 6.
\(\frac{3}{25}\)
_____

Answer:
0.12

Explanation:
Use division to rename the fraction part as a decimal.
\(\frac{3}{25}\) = 0.12
The quotient has 2 decimal places.
Add the whole number to the decimal.
0 + 0.12 = 0.12.
So, \(\frac{3}{25}\)= 0.12

On Your Own

Write as a fraction or as a mixed number in simplest form.

Question 7.
0.27
\(\frac{□}{□}\)

Answer:
\(\frac{27}{100}\)

Explanation:
0.27 is 0 ones and 27 hundredths.
27 hundredths = \(\frac{27}{100}\)
Simplify using the GCF.
The GCF of 27 and 100 is 1.
Divide the numerator and the denominator by 1
\(\frac{27 ÷ 1}{100 ÷ 1}\) = \(\frac{27}{100}\)

Question 8.
0.055
\(\frac{□}{□}\)

Answer:
\(\frac{11}{200}\)

Explanation:
0.055 is 0 ones and 55 thousandths.
55 thousandths = \(\frac{55}{1000}\)
Simplify using the GCF.
The GCF of 55 and 1000 is 5.
Divide the numerator and the denominator by 5
\(\frac{55 ÷ 5}{1000 ÷ 5}\) = \(\frac{11}{200}\)

Question 9.
2.45
_____ \(\frac{□}{□}\)

Answer:
\(\frac{9}{20}\)

Explanation:
2.45 is 2 ones and 45 hundredths.
45 hundredths = \(\frac{45}{100}\)
Simplify using the GCF.
The GCF of 45 and 100 is 5.
Divide the numerator and the denominator by 1
\(\frac{45 ÷ 5}{100 ÷ 5}\) = \(\frac{9}{20}\)

Write as a decimal.

Question 10.
\(\frac{3}{8}\)
_____

Answer:
0.375

Explanation:
Use division to rename the fraction part as a decimal.
\(\frac{3}{8}\) = 0.375
The quotient has 3 decimal places.
Add the whole number to the decimal.
0 + 0.375 = 0.375.
So, \(\frac{3}{8}\) = 0.375

Question 11.
3 \(\frac{1}{5}\)
_____

Answer:
3.2

Explanation:
Use division to rename the fraction part as a decimal.
\(\frac{1}{5}\) = 0.2
The quotient has 1 decimal place.
Add the whole number to the decimal.
3 + 0.2 = 3.2.
So, 3 \(\frac{1}{5}\) = 3.2

Question 12.
2 \(\frac{11}{20}\)
_____

Answer:
2.55

Explanation:
Use division to rename the fraction part as a decimal.
\(\frac{11}{20}\) = 0.55
The quotient has 2 decimal places.
Add the whole number to the decimal.
2 + 0.55 = 2.55.
So, 2 \(\frac{11}{20}\) = 2.55

Identify a decimal and a fraction in simplest form for the point.
Go Math Grade 6 Answer Key Chapter 2 Fractions and Decimals 1

Question 13.
Point A
Type below:
__________

Answer:
0.2

Question 14.
Point B
Type below:
__________

Answer:
0.9

Explanation:
Point B is between 0.8 and 1.0. Every point is separated by 0.1. So, Point B is at 0.9

Question 15.
Point C
Type below:
__________

Answer:
0.5

Explanation:
Point C is between 0.4 and 0.6. Every point is separated by 0.1. So, Point C is at 0.5

Question 16.
Point D
Type below:
__________

Answer:
0.1

Explanation:
Point D is between 0 and 0.2. Every point is separated by 0.1. So, Point D is at 0.1

Problem Solving + Applications – Page No. 72

Use the table for 17 and 18.
Go Math Grade 6 Answer Key Chapter 2 Fractions and Decimals 2

Question 17.
Members of the Ozark Trail Hiking Club hiked a steep section of the trail in June and July. The table shows the distances club members hiked in miles. Write Maria’s July distance as a decimal.
_____ miles

Answer:
2.625 miles

Explanation:
Maria’s July distance = 2 \(\frac{5}{8}\)
Use division to rename the fraction part as a decimal.
\(\frac{5}{8}\) = 0.625
The quotient has 3 decimal places.
Add the whole number to the decimal.
2 + 0.625 = 2.625.
2 \(\frac{5}{8}\) = 2.625

Question 18.
How much farther did Zoey hike in June and July than Maria hiked in June and July? Explain how you found your answer.
_____ miles

Answer:
0.7 miles

Explanation:
Maria: June = 2.95, July = 2 \(\frac{5}{8}\) = 2.58
Zoey: June = 2.85, July = 3 \(\frac{3}{8}\) = 3.38
[2.85 + 3.38] – [2.95 + 2.58] = 0.7 miles

Question 19.
What’s the Error? Tabitha’s hiking distance in July was 2 \(\frac{1}{5}\) miles. She wrote the distance as 2.02 miles. What error did she make?
Type below:
__________

Answer:
Tabitha’s hiking distance in July was 2 \(\frac{1}{5}\) miles.
2 \(\frac{1}{5}\)
Use division to rename the fraction part as a decimal.
\(\frac{1}{5}\)  = 0.2
The quotient has 1 decimal place.
Add the whole number to the decimal.
2 + 0.2 = 2.2.
2 \(\frac{1}{5}\) = 2.2
She wrote the distance as 2.02 miles in mistake.

Question 20.
Use Patterns Write \(\frac{3}{8}, \frac{4}{8}, \text { and } \frac{5}{8}\) as decimals. What pattern do you see? Use the pattern to predict the decimal form of \(\frac{6}{8}\) and \(\frac{7}{8}\).
Type below:
__________

Answer:
\(\frac{3}{8}, \frac{4}{8}, \text { and } \frac{5}{8}\) as decimals.
0.375, 0.5, 0.625
Each decimal is separated by 0.125.
So, 6/8 = 0.625 + 0.125 = 0.75
7/8 = 0.75 + 0.125 = 0.875

Question 21.
Identify a decimal and a fraction in simplest form for the point.
Go Math Grade 6 Answer Key Chapter 2 Fractions and Decimals 3
Type below:
__________

Answer:
Point A: 0.5
Point B: 0.7
Point C: 0.3
Point D: 0.8

Explanation:
Every point is differentiated by 0.1 distance.
The A is between 0.4 and 0.6 which is 0.5
The B is between 0.6 and 0.8 which is 0.7
The C is between 0.1 and 0.6 which is 0.53

Fractions and Decimals – Page No. 73

Write as a fraction or as a mixed number in simplest form.

Question 1.
0.52
\(\frac{□}{□}\)

Answer:
\(\frac{13}{25}\)

Explanation:
0.52
0.52 is 52 hundredths.
52 hundredths = \(\frac{52}{100}\)
Simplify using the GCF.
The GCF of 52 and 100 is 4.
Divide the numerator and the denominator by 4
\(\frac{52 ÷ 4}{100 ÷ 4}\) = \(\frac{13}{25}\)

Question 2.
0.02
\(\frac{□}{□}\)

Answer:
\(\frac{1}{50}\)

Explanation:
0.02
0.02 is 2 hundredths.
2 hundredths = \(\frac{2}{100}\)
Simplify using the GCF.
The GCF of 2 and 100 is 2.
Divide the numerator and the denominator by 2
\(\frac{2 ÷ 2}{100 ÷ 2}\) = \(\frac{1}{50}\)

Question 3.
4.8
______ \(\frac{□}{□}\)

Answer:
\(\frac{4}{5}\)

Explanation:
4.8
4.8 is 4 ones and 8 tenths.
8 tenths = \(\frac{8}{10}\)
Simplify using the GCF.
The GCF of 8 and 10 is 2.
Divide the numerator and the denominator by 2
\(\frac{8 ÷ 2}{10 ÷ 2}\) = \(\frac{4}{5}\)

Question 4.
6.025
______ \(\frac{□}{□}\)

Answer:
\(\frac{1}{40}\)

Explanation:
6.025 is 6 ones and 25 thousandths.
25 thousandths = \(\frac{25}{1000}\)
Simplify using the GCF.
The GCF of 25 and 1000 is 25.
Divide the numerator and the denominator by 25
\(\frac{25 ÷ 25}{1000 ÷ 25}\) = \(\frac{1}{40}\)

Write as a decimal.

Question 5.
\(\frac{17}{25}\)
______

Answer:
0.68

Explanation:
Use division to rename the fraction part as a decimal.
17/25 = 0.68
The quotient has 2 decimal places.
Add the whole number to the decimal.
0 + 0.68 = 0.68.
So, \(\frac{17}{25}\) = 0.68

Question 6.
\(\frac{11}{20}\)
______

Answer:
0.55

Explanation:
Use division to rename the fraction part as a decimal.
11/20 = 0.55
The quotient has 2 decimal places.
Add the whole number to the decimal.
0 + 0.55 = 0.55.
So, \(\frac{11}{20}\) = 0.55

Question 7.
4 \(\frac{13}{20}\)
______

Answer:
4.65

Explanation:
Use division to rename the fraction part as a decimal.
\(\frac{13}{20}\) = 0.65
The quotient has 2 decimal places.
Add the whole number to the decimal.
4 + 0.65 = 4.65.
So, 4 \(\frac{13}{20}\) = 4.65

Question 8.
7 \(\frac{3}{8}\)
______

Answer:
7.375

Explanation:
Use division to rename the fraction part as a decimal.
\(\frac{3}{8}\) = 0.375
The quotient has 3 decimal places.
Add the whole number to the decimal.
7 + 0.375 = 7.375.
So, 7 \(\frac{3}{8}\) = 7.375

Identify a decimal and a fraction or mixed number in simplest form for each point.
Go Math Grade 6 Answer Key Chapter 2 Fractions and Decimals 4

Question 9.
Point A
Type below:
__________

Answer:
0.4

Explanation:

Point A is between 0 and 0.5. Every point is separated by 0.1. So, Point A is at 0.4

Question 10.
Point D
Type below:
__________

Answer:
1.9

Explanation:
Point D is between 1.5 and 2. Every point is separated by 0.1. So, Point D is at 1.9

Question 11.
Point C
Type below:
__________

Answer:
1.2

Explanation:
Point C is between 1 and 1.5. Every point is separated by 0.1. So, Point C is at 1.2

Question 12.
Point B
Type below:
__________

Answer:
0.6

Explanation:
Point C is between 0.5 and 1. Every point is separated by 0.1. So, Point C is at 0.6

Problem Solving

Question 13.
Grace sold \(\frac{5}{8}\) of her stamp collection. What is this amount as a decimal?
______

Answer:
0.625

Explanation:
Grace sold \(\frac{5}{8}\) of her stamp collection.
Use division to rename the fraction part as a decimal.
\(\frac{5}{8}\)  = 0.625
The quotient has 3 decimal places.
Add the whole number to the decimal.
0 + 0.625 = 0.625.
So, \(\frac{5}{8}\) = 0.625

Question 14.
What if you scored an 0.80 on a test? What fraction of the test, in simplest form, did you answer correctly?
\(\frac{□}{□}\)

Answer:
\(\frac{4}{5}\)

Explanation:
0.80 is 0 ones and 8 tenths.
8 tenths = \(\frac{8}{10}\)
Simplify using the GCF.
The GCF of 8 and 10 is 2.
Divide the numerator and the denominator by 2
\(\frac{8 ÷ 2}{10 ÷ 2}\) = \(\frac{4}{5}\)

Question 15.
What fraction in simplest form is equivalent to 0.45? What decimal is equivalent to \(\frac{17}{20}\)? Explain how you found your answers.
Type below:
__________

Answer:
0.45 is 0 ones and 45 hundredths.
45 hundredths = \(\frac{45}{100}\)
Simplify using the GCF.
The GCF of 45 and 100 is 5.
Divide the numerator and the denominator by 5
\(\frac{45 ÷ 5}{100 ÷ 5}\) = \(\frac{9}{20}\)
\(\frac{17}{20}\)
Use division to rename the fraction part as a decimal.
\(\frac{17}{20}\) = 0.85
The quotient has 2 decimal places.
Add the whole number to the decimal.
0 + 0.85 = 0.85.
So, \(\frac{17}{20}\) = 0.85

Lesson Check – Page No. 74

Question 1.
After a storm, Michael measured 6 \(\frac{7}{8}\) inches of snow. What is this amount as a decimal?
______ inches

Answer:
6.875 inches

Explanation:
Michael measured 6 \(\frac{7}{8}\) inches of snow.
Use division to rename the fraction part as a decimal.
\(\frac{7}{8}\) = 0.875
The quotient has 3 decimal places.
Add the whole number to the decimal.
6 + 0.875 = 6.875.
So, 6 \(\frac{7}{8}\) = 6.875.

Question 2.
A recipe calls for 3.75 cups of flour. What is this amount as a mixed number in simplest form?
______ \(\frac{□}{□}\) cups

Answer:
3 \(\frac{3}{4}\) cups

Explanation:
A recipe calls for 3.75 cups of flour.
3 + 0.75
0.75 is 0 ones and 75 hundredths.
75 hundredths = \(\frac{75}{100}\)
Simplify using the GCF.
The GCF of 75 and 100 is 25.
Divide the numerator and the denominator by 25
\(\frac{75 ÷ 25}{100 ÷ 25}\) = \(\frac{3}{4}\)
3 \(\frac{3}{4}\)

Spiral Review

Question 3.
Gina bought 2.3 pounds of red apples and 2.42 pounds of green apples. They were on sale for $0.75 a pound. How much did the apples cost altogether?
$ ______

Answer:
$3.54

Explanation:
Gina bought 2.3 pounds of red apples and 2.42 pounds of green apples. They were on sale for $0.75 a pound.
$0.75 x 2.3 = 1.725
$0.75 x 2.42 = 1.815
1.725 + 1.815 = 3.54
So the apples cost $3.54

Question 4.
Ken has 4.66 pounds of walnuts, 2.1 pounds of cashews, and 8 pounds of peanuts. He mixes them together and divides them equally among 18 bags. How many pounds of nuts are in each bag?
______ pounds

Answer:
0.82 pounds

Explanation:
Ken has 4.66 pounds of walnuts, 2.1 pounds of cashews, and 8 pounds of peanuts.
4.66 + 2.1 + 8 = 14.76
He mixes them together and divides them equally among 18 bags.
14.76/18 = 0.82

Question 5.
Mia needs to separate 270 blue pens and 180 red pens into packs. Each pack will have the same number of blue pens and the same number of red pens. What is the greatest number of packs she can make? How many red pens and how many blue pens will be in each pack?
Type below:
__________

Answer:
There are 2 red pens and 3 blue pens in each pack.

Explanation:
Mia needs to separate 270 blue pens and 180 red pens into packs.
The GCF of 270 and 180 is 90
The greatest number of packs she can make is 90.
Divide the total number of red pens by the total number of packs.
180/90 = 2
Divide the total number of blue pens by the total number of packs.
270/90 = 3
There are 2 red pens and 3 blue pens in each pack.

Question 6.
Evan buys 19 tubes of watercolor paint for $50.35. What is the cost of each tube of paint?
$ ______

Answer:
$2.65

Explanation:
Evan buys 19 tubes of watercolor paint for $50.35.
$50.35/19 = $2.65

Share and Show – Page No. 77

Order from least to greatest.

Question 1.
\(3 \frac{3}{6}, 3 \frac{5}{8}, 2 \frac{9}{10}\)
Type below:
__________

Answer:
2 \(\frac{9}{10}\) < 3 \(\frac{3}{6}\) < 3 \(\frac{5}{8}\)

Explanation:
\(3 \frac{3}{6}, 3 \frac{5}{8}, 2 \frac{9}{10}\)
Compare the whole numbers first.
2 < 3
If the whole numbers are the same, compare the fractions.
3 \(\frac{3}{6}\), 3 \(\frac{5}{8}\)
6 and 8 are multiples of 48.
So, 48 is a common denominator.
3 \(\frac{3 x 8}{6 x 8}\) = 3 \(\frac{24}{48}\), 3 \(\frac{5 x 6}{8 x 6}\) = 3 \(\frac{30}{48}\)
3 \(\frac{24}{48}\) < 3 \(\frac{30}{48}\)
So, 3 \(\frac{3}{6}\) < 3 \(\frac{5}{8}\)
Order the fractions from least to greatest.
2 \(\frac{9}{10}\) < 3 \(\frac{3}{6}\) < 3 \(\frac{5}{8}\)

Write <, >, or =.

Question 2.
0.8 _____ \(\frac{4}{12}\)

Answer:
0.8 < latex]\frac{4}{12}[/latex]

Explanation:
Write the decimal form of \(\frac{4}{12}\) = 0.3333
0.8 > 0.333
So, 0.8 < latex]\frac{4}{12}[/latex]

Question 3.
0.22 _____ \(\frac{1}{4}\)

Answer:
0.22 < \(\frac{1}{4}\)

Explanation:
Write the decimal form of \(\frac{1}{4}\) = 0.25
0.22 < 0.25
So, 0.22 < \(\frac{1}{4}\)

Question 4.
\(\frac{1}{20}\) _____ 0.06

Answer:
\(\frac{1}{20}\) < 0.06

Explanation:
Write the decimal form of \(\frac{1}{20}\) = 0.05
0.05 < 0.06
So, \(\frac{1}{20}\) < 0.06

Use a number line to order from least to greatest.

Question 5.
\(1 \frac{4}{5}, 1.25, 1 \frac{1}{10}\)
Type below:
__________

Answer:
1\(\frac{1}{10}\), 1.25, 1\(\frac{4}{5}\)

Explanation:
Write the decimal form of 1\(\frac{4}{5}\) = 1.8
Write the decimal form of 1\(\frac{1}{10}\) = 1.1
1.8, 1.25, 1.1
Locate each decimal on a number line.
So, from least to greatest, the order is 1.1, 1.25, 1.8
1\(\frac{1}{10}\), 1.25, 1\(\frac{4}{5}\)

On Your Own

Order from least to greatest.

Question 6.
0.6, \(\frac{4}{5}\), 0.75
Type below:
__________

Answer:
0.6, 0.75, \(\frac{4}{5}\)

Explanation:
Write the decimal form of \(\frac{4}{5}\) = 0.8
0.6, 0.8, 0.75
Compare decimals.
All ones are equal.
Compare tenths: 6 < 7 < 8
So, from least to greatest, the order is 0.6, 0.75, 0.8
So, 0.6, 0.75, \(\frac{4}{5}\)

Question 7.
\(\frac{1}{2}\), \(\frac{2}{5}\), \(\frac{7}{15}\)
Type below:
__________

Answer:
\(\frac{2}{5}\), \(\frac{7}{15}\), \(\frac{1}{2}\)

Explanation:
Write the decimal form of \(\frac{1}{2}\) = 0.5
Write the decimal form of \(\frac{2}{5}\) = 0.4
Write the decimal form of \(\frac{7}{15}\) = 0.466
0.5, 0.4, 0.466
Compare decimals.
All ones are equal.
Compare tenths: 4 < 5
Compare hundredths of 0.4 and 0.466; 0 < 6
So, from least to greatest, the order is 0.4 < 0.466 < 0.5
So, \(\frac{2}{5}\), \(\frac{7}{15}\), \(\frac{1}{2}\)

Question 8.
5 \(\frac{1}{2}\), 5.05, 5 \(\frac{5}{9}\)
Type below:
__________

Answer:
5.05, 5 \(\frac{1}{2}\), 5 \(\frac{5}{9}\)

Explanation:
Write the decimal form of 5 \(\frac{1}{2}\) = 5.5
Write the decimal form of 5 \(\frac{5}{9}\) = 5.555
5.5, 5.05, 5.5555
Compare decimals.
All ones are equal.
Compare tenths: 0 < 5
Compare hundredths of 5.5 and 5.55; 0 < 5
So, from least to greatest, the order is 5.05 < 5.5 < 5.55
So, 5.05, 5 \(\frac{1}{2}\), 5 \(\frac{5}{9}\)

Question 9.
\(\frac{5}{7}\), \(\frac{5}{6}\), \(\frac{5}{12}\)
Type below:
__________

Answer:
\(\frac{5}{12}\), \(\frac{5}{7}\), \(\frac{5}{6}\)

Explanation:
\(\frac{5}{7}\), \(\frac{5}{6}\), \(\frac{5}{12}\)
To compare fractions with the same numerators, compare the denominators.
So, from least to greatest, the order is \(\frac{5}{12}\), \(\frac{5}{7}\), \(\frac{5}{6}\)

Question 10.
\(\frac{7}{15}\) _____ \(\frac{7}{10}\)

Answer:
\(\frac{7}{15}\) < \(\frac{7}{10}\)

Explanation:
\(\frac{7}{15}\) and \(\frac{7}{10}\)
To compare fractions with the same numerators, compare the denominators.
So, \(\frac{7}{15}\) < \(\frac{7}{10}\)

Question 11.
\(\frac{1}{8}\) _____ 0.125

Answer:
\(\frac{1}{8}\) = 0.125

Explanation:
Write the decimal form of \(\frac{1}{8}\) = 0.125
0.125 = 0.125

Question 12.
7 \(\frac{1}{3}\) _____ 6 \(\frac{2}{3}\)

Answer:
7 \(\frac{1}{3}\) > 6 \(\frac{2}{3}\)

Explanation:
Compare the whole numbers first.
7 > 6.
So, 7 \(\frac{1}{3}\) > 6 \(\frac{2}{3}\)

Question 13.
1 \(\frac{2}{5}\) _____ 1 \(\frac{7}{15}\)

Answer:
1 \(\frac{2}{5}\) < 1 \(\frac{7}{15}\)

Explanation:
1 \(\frac{2}{5}\) _____ 1 \(\frac{7}{15}\)
If the whole numbers are the same, compare the fractions.
Compare \(\frac{2}{5}\) and \(\frac{7}{15}\)
5 and 15 are multiples of 15.
So, \(\frac{2 x 3}{5 x 3}\) = \(\frac{6}{15}\)
\(\frac{6}{15}\) < \(\frac{7}{15}\)
Use common denominators to write equivalent fractions.
1 \(\frac{2}{5}\) < 1 \(\frac{7}{15}\)

Question 14.
Darrell spent 3 \(\frac{2}{5}\) hours on a project for school. Jan spent 3 \(\frac{1}{4}\) hours and Maeve spent 3.7 hours on the project. Who spent the least amount of time? Show how you found your answer. Then describe another possible method.
Type below:
__________

Answer:
Jan spent the least amount of time.

Explanation:
Darrell spent 3 \(\frac{2}{5}\) hours on a project for school. Jan spent 3 \(\frac{1}{4}\) hours and Maeve spent 3.7 hours on the project.
Write the decimal form of 3 \(\frac{2}{5}\) = 3.4
Write the decimal form of 3 \(\frac{1}{4}\) = 3.25
3.4, 3.25, 3.7
3.25 is the least one.
So, Jan spent the least amount of time.

Problem Solving + Applications – Page No. 78

Use the table for 15–18.
Go Math Grade 6 Answer Key Chapter 2 Fractions and Decimals 5

Question 15.
In one week, Altoona, PA, and Bethlehem, PA, received snowfall every day, Monday through Friday. On which days did Altoona receive over 0.1 inch more snow than Bethlehem?
Type below:
__________

Answer:
Altoona received over 1 inch more snow than Bethlehem on Friday

Explanation:
Altoona (converted to decimal form): 2.25, 3.25, 2.625, 4.6, 4.75
Bethlehem: 2.6, 3.2, 2.5, 4.8, 2.7
Altoona received over 1 inch more snow than Bethlehem on Friday

Question 16.
What if Altoona received an additional 0.3 inch of snow on Thursday? How would the total amount of snow in Altoona compare to the amount received in Bethlehem that day?
Type below:
__________

Answer:
Altoona received 0.1 inches more snow than Bethlehem on Thursday

Explanation:
Altoona received an additional 0.3 inch of snow on Thursday = 4.6 + 0.3 = 4.9
Bethlehem received on Thursday = 4.8
Altoona received 0.1 inches more snow than Bethlehem on Thursday

Question 17.
Explain two ways you could compare the snowfall amounts in Altoona and Bethlehem on Monday.
Type below:
__________

Answer:

Explanation:
Altoona received on Monday = 2.25
Bethlehem received on Monday = 2.6
Bethlehem received 0.35 inches more snow than Altoona on Monday.
As the whole numbers are equal compare 1/4 and 0.6.
0.25 < 0.6
So, Altoona received less snow compared to Bethlehem on Monday.

Question 18.
Explain how you could compare the snowfall amounts in Altoona on Thursday and Friday.
Type below:
__________

Answer:
Altoona received on Thursday = 4.6
Altoona received on Friday = 4.75
4.6 < 4.75
Altoona received less snow on Thursday compared to Friday.

Question 19.
Write the values in order from least to greatest.
Go Math Grade 6 Answer Key Chapter 2 Fractions and Decimals 6
Type below:
__________

Answer:
1/3, 0.39, 2/5, 0.45

Explanation:
1/3 = 0.333
0.45
0.39
2/5 = 0.4
Compare tenths: 3 < 4
Compare hundredths:
0.33 < 0.39
0.4 < 0.45
So, 1/3, 0.39, 2/5, 0.45

Compare and Order Fractions and Decimals – Page No. 79

Write <, >, =.

Question 1.
0.64 _____ \(\frac{7}{10}\)

Answer:
0.64 < \(\frac{7}{10}\)

Explanation:
Write the decimal form of \(\frac{7}{10}\) = 0.7
Compare tenths: 6 < 7
So, 0.64 < 0.7
0.64 < \(\frac{7}{10}\)

Question 2.
0.48 _____ \(\frac{6}{15}\)

Answer:
0.48 > \(\frac{6}{15}\)

Explanation:
Write the decimal form of \(\frac{6}{15}\) = 0.4
Compare hundredths:
0.48 > 0.4
0.48 > \(\frac{6}{15}\)

Question 3.
0.75 _____ \(\frac{7}{8}\)

Answer:
0.75 < \(\frac{7}{8}\)

Explanation:
Write the decimal form of \(\frac{7}{8}\) = 0.875
Compare tenths:
7 < 8
0.75 < \(\frac{7}{8}\)

Question 4.
7 \(\frac{1}{8}\) _____ 7.025

Answer:
7 \(\frac{1}{8}\) > 7.025

Explanation:
Write the decimal form of 7 \(\frac{1}{8}\) = 7.125
Compare tenths:
1 > 0
7 \(\frac{1}{8}\) > 7.025

Order from least to greatest.

Question 5.
\(\frac{7}{15}\), 0.75, \(\frac{5}{6}\)
Type below:
__________

Answer:
\(\frac{7}{15}\), 0.75, \(\frac{5}{6}\)

Explanation:
Write the decimal form of \(\frac{7}{15}\) = 0.466
0.75
Write the decimal form of \(\frac{5}{6}\) = 0.833
Order from least to greatest: \(\frac{7}{15}\), 0.75, \(\frac{5}{6}\)

Question 6.
0.5, 0.41, \(\frac{3}{5}\)
Type below:
__________

Answer:
0.41, 0.5, \(\frac{3}{5}\)

Explanation:
Write the decimal form of \(\frac{3}{5}\) = 0.6
Compare tenths:
0.41, 0.5, 0.6
Order from least to greatest: 0.41, 0.5, \(\frac{3}{5}\)

Question 7.
3.25, 3 \(\frac{2}{5}\), 3 \(\frac{3}{8}\)
Type below:
__________

Answer:
3.25, 3 \(\frac{2}{5}\), 3 \(\frac{3}{8}\)

Explanation:
Write the decimal form of 3 \(\frac{2}{5}\) = 3.4
Write the decimal form of 3 \(\frac{3}{8}\) = 3.375
Compare tenths:
Order from least to greatest: 3.25, 3 \(\frac{2}{5}\), 3 \(\frac{3}{8}\)

Question 8.
0.9, \(\frac{8}{9}\), 0.86
Type below:
__________

Answer:
0.86, \(\frac{8}{9}\), 0.9

Explanation:
Write the decimal form of \(\frac{8}{9}\) = 0.88
Compare tenths:
0.86, 0.88, 0.9
Order from least to greatest: 0.86, \(\frac{8}{9}\), 0.9

Order from greatest to least.

Question 9.
0.7, \(\frac{7}{9}\), \(\frac{7}{8}\)
Type below:
__________

Answer:
\(\frac{7}{8}\), \(\frac{7}{9}\), 0.7

Explanation:
0.7 = 7/10
To compare fractions with the same numerators, compare the denominators.
7/10, 7/9, 7/8
Order from greatest to least: 7/8, 7/9, 7/10

Question 10.
0.2, 0.19, \(\frac{3}{5}\)
Type below:
__________

Answer:
\(\frac{3}{5}\), 0.2, 0.19

Explanation:
Write the decimal form of \(\frac{3}{5}\) = 0.6
Compare tenths:
0.6, 0.2, 0.19
Order from greatest to least: \(\frac{3}{5}\), 0.2, 0.19

Question 11.
6\(\frac{1}{20}\), 6.1, 6.07
Type below:
__________

Answer:

Explanation:
Write the decimal form of 6\(\frac{1}{20}\) = 121/20 = 6.05
Compare tenths:
6.1, 6.07, 6.05
Order from greatest to least: 6.1, 6.07, 6\(\frac{1}{20}\)

Question 12.
2 \(\frac{1}{2}\), 2.4, 2.35, 2 \(\frac{1}{8}\)
Type below:
__________

Answer:
2 \(\frac{1}{2}\), 2.4, 2.35, 2 \(\frac{1}{8}\)

Explanation:
Write the decimal form of 2 \(\frac{1}{2}\) = 2.5
Write the decimal form of 2 \(\frac{1}{8}\) = 2.125
Compare tenths: 2.5, 2.4, 2.35, 2.125
Order from greatest to least: 2 \(\frac{1}{2}\), 2.4, 2.35, 2 \(\frac{1}{8}\)

Question 13.
One day it snowed 3 \(\frac{3}{8}\) inches in Altoona and 3.45 inches in Bethlehem. Which city received less snow that day?
__________

Answer:
Altoona

Explanation:
One day it snowed 3 \(\frac{3}{8}\) inches in Altoona and 3.45 inches in Bethlehem.
Write the decimal form of 3 \(\frac{3}{8}\) = 27/8 = 3.375
3.375 < 3.45.
Altoona received less snow that day

Question 14.
Malia and John each bought 2 pounds of sunflower seeds. Each ate some seeds. Malia has 1 \(\frac{1}{3}\) pounds left, and John has 1 \(\frac{2}{5}\) pounds left. Who ate more sunflower seeds?
__________

Answer:
Malia

Explanation:
Malia and John each bought 2 pounds of sunflower seeds. Each ate some seeds. Malia has 1 \(\frac{1}{3}\) pounds left, and John has 1 \(\frac{2}{5}\) pounds left.
2 – 1 \(\frac{1}{3}\) = 0.667
2 – 1 \(\frac{2}{5}\) = 0.6
0.667 > 0.6
So, Malia ate more sunflower seeds

Question 15.
Explain how you would compare the numbers 0.4 and \(\frac{3}{8}\).
Type below:
__________

Answer:
Write the decimal form of \(\frac{3}{8}\) = 0.375
Compare tenths:
0.4 > 0.375

Lesson Check – Page No. 80

Question 1.
Andrea has 3 \(\frac{7}{8}\) yards of purple ribbon, 3.7 yards of pink ribbon, and 3 \(\frac{4}{5}\) yards of blue ribbon. List the numbers in order from least to greatest.
Type below:
__________

Answer:
Andrea has 3 \(\frac{7}{8}\) yards of purple ribbon, 3.7 yards of pink ribbon, and 3 \(\frac{4}{5}\) yards of blue ribbon.
Write the decimal form of 3 \(\frac{7}{8}\) = 3.875
3.7
Write the decimal form of 3 \(\frac{4}{5}\) = 3.8
Least to greatest : 3.7, 3 \(\frac{4}{5}\), 3 \(\frac{7}{8}\)

Question 2.
Nassim completed \(\frac{18}{25}\) of the math homework. Kara completed 0.7 of it. Debbie completed \(\frac{5}{8}\) of it. List the numbers in order from greatest to least.
Type below:
__________

Answer:
$1.39, $0.70, $0.63

Explanation:
Nassim completed \(\frac{18}{25}\) of the math homework. Kara completed 0.7 of it. Debbie completed \(\frac{5}{8}\) of it.
Write the decimal form of 18/25 = 1.39
0.7
Write the decimal form of 5/8 = 0.63
They are now in order from greatest to least.
Think of the amounts as money:
$1.39, $0.70, $0.63

Spiral Review

Question 3.
Tyler bought 3 \(\frac{2}{5}\) pounds of oranges. Graph 3 \(\frac{2}{5}\) on a number line and write this amount using a decimal.
Type below:
__________

Answer:
grade 6 chapter 2 image 1
Tyler bought 3 \(\frac{2}{5}\) pounds of oranges.
Decimal Form: 17/5 = 3.4

Question 4.
At the factory, a baseball card is placed in every 9th package of cereal. A football card is placed in every 25th package of the cereal. What is the first package that gets both a baseball card and a football card?
Type below:
__________

Answer:
225th package

Explanation:
Look for the first number where both 25 and 9 are a factor of.
25 x 1 = 25 which isn’t a factor of 9, so it won’t be 25.
25 x 2 = 50, which isn’t a factor of 9.
75 is not a factor of 9. (you know because you don’t get a whole number when you divide 75 into 9.)
100 is not a factor of 9, nor is 125, 150, 175, or 200.
However, 225 is a factor of both 25 and 9. This makes sense because 25 x 9 is 225.
This means that the first package with both will be the 225th package.

Question 5.
$15.30 is divided among 15 students. How much does each student receive?
$ _____

Answer:
$1.02

Explanation:
$15.30 is divided among 15 students.
$15.30/15 = $1.02
each student receive $1.02

Question 6.
Carrie buys 4.16 pounds of apples for $5.20. How much does 1 pound cost?
$ _____

Answer:
$1.25

Explanation:
Carrie buys 4.16 pounds of apples for $5.20.
$5.20/4.16 = $1.25
1 pound cost = $1.25

Share and Show – Page No. 83

Find the product. Write it in simplest form.

Question 1.
6 × \(\frac{3}{8}\)
\(\frac{□}{□}\)

Answer:
\(\frac{9}{4}\)

Explanation:
\(\frac{6 × 3}{1 × 8}\)
\(\frac{18}{8}\)
Simplify using the GCF.
The GCF of 18 and 8 is 2.
Divide the numerator and the denominator by 2.
\(\frac{18 ÷ 2}{8 ÷ 2}\) = \(\frac{9}{4}\)

Question 2.
\(\frac{3}{8}\) × \(\frac{8}{9}\)
\(\frac{□}{□}\)

Answer:
\(\frac{1}{3}\)

Explanation:
Multiply the numerators and Multiply the denominators.
\(\frac{3 × 8}{8 × 9}\) = \(\frac{24}{72}\)
Simplify using the GCF.
The GCF of 24 and 72 is 24.
Divide the numerator and the denominator by 24.
\(\frac{24 ÷ 24}{72 ÷ 24}\) = \(\frac{1}{3}\)

Question 3.
Sam and his friends ate 3 \(\frac{3}{4}\) bags of fruit snacks. If each bag contained 2 \(\frac{1}{2}\) ounces, how many ounces of fruit snacks did Sam and his friends eat?
\(\frac{□}{□}\)

Answer:
\(\frac{75}{8}\) ounces

Explanation:
Sam and his friends ate 3 \(\frac{3}{4}\) bags of fruit snacks. If each bag contained 2 \(\frac{1}{2}\) ounces
3 \(\frac{3}{4}\) x 2 \(\frac{1}{2}\)
\(\frac{15}{4}\) x \(\frac{5}{2}\)
\(\frac{15 x 5}{4 x 2}\) = \(\frac{75}{8}\)

Attend to Precision Algebra Evaluate using the order of operations.

Write the answer in simplest form.

Question 4.
\(\left(\frac{3}{4}-\frac{1}{2}\right) \times \frac{3}{5}\)
\(\frac{□}{□}\)

Answer:
\(\frac{3}{20}\)

Explanation:
\(\left(\frac{3}{4}-\frac{1}{2}\right) \times \frac{3}{5}\)
Perform operations in parentheses.
\(\frac{3}{4}\) – \(\frac{1}{2}\) = \(\frac{1}{4}\)
\(\frac{1}{4}\) x \(\frac{3}{5}\) = \(\frac{1 x 3}{4 x 5}\) = \(\frac{3}{20}\)

Question 5.
\(\frac{1}{3}+\frac{4}{9} \times 12\)
\(\frac{□}{□}\)

Answer:
\(\frac{28}{3}\)

Explanation:
\(\frac{1}{3}\) + \(\frac{4}{9}\) = \(\frac{7}{9}\)
\(\frac{7 x 12}{9 x 1}\) = \(\frac{84}{9}\)
Simplify using the GCF.
The GCF of 84 and 9 is 3.
Divide the numerator and the denominator by 3.
\(\frac{84 ÷ 3}{9 ÷ 3}\) = \(\frac{28}{3}\)

Question 6.
\(\frac{5}{8} \times \frac{7}{10}-\frac{1}{4}\)
\(\frac{□}{□}\)

Answer:
\(\frac{11}{16}\)

Explanation:
\(\frac{5 x 7}{8 x 10}\) = \(\frac{35}{80}\)
\(\frac{35}{80}\) – \(\frac{1}{4}\) = \(\frac{11}{16}\)

Question 7.
3 × (\(\frac{5}{18}\) + \(\frac{1}{6}\)) + \(\frac{2}{5}\)
\(\frac{□}{□}\)

Answer:
\(\frac{38}{15}\)

Explanation:
3 x \(\frac{4}{9}\) + \(\frac{2}{5}\)
3 x \(\frac{38}{45}\) = \(\frac{38}{15}\)

On Your Own

Practice: Copy and Solve Find the product. Write it in simplest form.

Question 8.
\(1 \frac{2}{3} \times 2 \frac{5}{8}\)
\(\frac{□}{□}\)

Answer:
\(\frac{35}{8}\)

Explanation:
1 \(\frac{2}{3}\) = \(\frac{5}{3}\)
2 \(\frac{5}{8}\) = \(\frac{21}{8}\)
\(\frac{5 × 21}{3 × 8}\) = \(\frac{105}{24}\)
Simplify using the GCF
The GCF of 105 and 24 is 3.
Divide the numerator and the denominator by 3.
\(\frac{105 ÷ 3}{24 ÷ 3}\) = \(\frac{35}{8}\)

Question 9.
\(\frac{4}{9} \times \frac{4}{5}\)
\(\frac{□}{□}\)

Answer:
\(\frac{16}{45}\)

Explanation:
\(\frac{4 × 4}{9 × 5}\) = \(\frac{16}{45}\)

Question 10.
\(\frac{1}{6} \times \frac{2}{3}\)
\(\frac{□}{□}\)

Answer:
\(\frac{1}{9}\)

Explanation:
\(\frac{1 × 2}{6 × 3}\) = \(\frac{2}{18}\)
Simplify using the GCF
The GCF of 2 and 18 is 2.
Divide the numerator and the denominator by 2.
\(\frac{2 ÷ 2}{18 ÷ 2}\) = \(\frac{1}{9}\)

Question 11.
\(4 \frac{1}{7} \times 3 \frac{1}{9}\)
\(\frac{□}{□}\)

Answer:
\(\frac{116}{7}\)

Explanation:
4\(\frac{1}{7}\) = \(\frac{29}{7}\)
3\(\frac{1}{9}\) = \(\frac{28}{9}\)
\(\frac{29 × 28}{7 × 9}\) = \(\frac{812}{63}\)
Simplify using the GCF
The GCF of 812 and 63 is 7.
Divide the numerator and the denominator by 7.
\(\frac{812 ÷ 7}{63 ÷ 7}\) = \(\frac{116}{7}\)

Question 12.
\(\frac{5}{6}\) of the 90 pets in the pet show are cats. \(\frac{4}{5}\) of the cats are calico cats. What fraction of the pets are calico cats? How many of the pets are calico cats?
Type below:
__________

Answer:
60 calico cats

Explanation:
5/6 x 90 = 450/6 = 150/2
150/2 x 4/5 = 60

Question 13.
Five cats each ate \(\frac{1}{4}\) cup of cat food. Four other cats each ate \(\frac{1}{3}\) cup of cat food. How much food did the nine cats eat?
Type below:
__________

Answer:
\(\frac{31}{12}\)

Explanation:
5 x 1/4 = 5/4
4 x 1/3 = 4/3
5/4 + 4/3 = 31/12

Attend to Precision Algebra Evaluate using the order of operations.

Write the answer in simplest form.

Question 14.
\(\frac{1}{4} \times\left(\frac{3}{9}+5\right)\)
\(\frac{□}{□}\)

Answer:
\(\frac{4}{3}\)

Explanation:
3/9 + 5 = 16/3
1/4 x 16/3
1 x 16 = 16
4 x 3 = 12
16/12
Simplify using the GCF
The GCF of 16 and 12 is 4.
Divide the numerator and the denominator by 4.
\(\frac{16 ÷ 4}{12÷ 4}\) = \(\frac{4}{3}\)

Question 15.
\(\frac{9}{10}-\frac{3}{5} \times \frac{1}{2}\)
\(\frac{□}{□}\)

Answer:
\(\frac{3}{5}\)

Explanation:
3/5 x 1/2 = 3/10
9/10 – 3/10 = 6/10
Simplify using the GCF
The GCF of 6 and 10 is 2.
Divide the numerator and the denominator by 2.
\(\frac{6 ÷ 2}{10 ÷ 2}\) = \(\frac{3}{5}\)

Question 16.
\(\frac{4}{5}+\left(\frac{1}{2}-\frac{3}{7}\right) \times 2\)
\(\frac{□}{□}\)

Answer:
\(\frac{33}{35}\)

Explanation:
1/2 – 3/7 = 1/14
1/14 x 2 = 1/7
4/5 + 1/7 = 33/35

Question 17.
\(15 \times \frac{3}{10}+\frac{7}{8}\)
\(\frac{□}{□}\)

Answer:
\(\frac{141}{8}\)

Explanation:
3/10 + 7/8 = 47/40
15 x 47/40 = 141/8
\(\frac{141}{8}\)

Page No. 84

Question 18.
Write and solve a word problem for the expression \(\frac{1}{4} \times \frac{2}{3}\). Show your work.
Type below:
__________

Answer:
\(\frac{1}{6}\)

Explanation:
\(\frac{1}{4} \times \frac{2}{3}\) = \(\frac{1 X 2}{4 X 3}\) = \(\frac{2}{12}\)
Simplify using the GCF
The GCF of 2 and 12 is 2.
Divide the numerator and the denominator by 2.
\(\frac{2 ÷ 2}{12 ÷ 2}\) = \(\frac{1}{6}\)

Question 19.
Michelle has a recipe that asks for 2 \(\frac{1}{2}\) cups of vegetable oil. She wants to use \(\frac{2}{3}\) that amount of oil and use applesauce to replace the rest. How much applesauce will she use?
Type below:
__________

Answer:
\(\frac{10}{6}\)

Explanation:
2 1/2 * 2/3 = 5/2 * 2/3 = 10/6 She will use 10/6 or 1 2/3 cups of vegetable oil

Question 20.
Cara’s muffin recipe asks for 1 \(\frac{1}{2}\) cups of flour for the muffins and \(\frac{1}{4}\) cup of flour for the topping. If she makes \(\frac{1}{2}\) of the original recipe, how much flour will she use for the muffins and topping?
Type below:
__________

Answer:
Cara will use 1\(\frac{1}{8}\) cups of flour.

Explanation:
For first we will find how many cups of flours need to makes the original recipe. Cara uses 1 1/2 cups of flour for the muffins and 1/4 cup off flour for the topping.
So, 1 1/2 + 1/4 cups of flour to make the original recipe.
1 1/2 = 3/2
3/2 + 1/4 = 7/4
To make the original recipe Cara needs 7/4 cups of flour.
If she makes \(\frac{1}{2}\) of the original recipe, then
7/4 x 1/2 = 7/8 = 1 1/8
Cara will use 1 1/8 cups of flour.

Multiply Fractions – Page No. 85

Find the product. Write it in simplest form.

Question 1.
\(\frac{4}{5} \times \frac{7}{8}\)
\(\frac{□}{□}\)

Answer:
\(\frac{7}{10}\)

Explanation:
Multiply the numerators and Multiply the denominators.
\(\frac{4 × 7}{5 × 8}\) = \(\frac{28}{40}\)
Simplify using the GCF.
The GCF of 28 and 40 is 4.
Divide the numerator and the denominator by 4.
\(\frac{28 ÷ 4}{40 ÷ 4}\) = \(\frac{7}{10}\)

Question 2.
\(\frac{1}{8} \times 20\)
\(\frac{□}{□}\)

Answer:
\(\frac{5}{2}\)

Explanation:
\(\frac{1 × 20}{1 × 8}\)
\(\frac{20}{8}\)
Simplify using the GCF.
The GCF of 20 and 8 is 4.
Divide the numerator and the denominator by 4.
\(\frac{20 ÷ 4}{8 ÷ 4}\) = \(\frac{5}{2}\)

Question 3.
\(\frac{4}{5} \times \frac{3}{8}\)
\(\frac{□}{□}\)

Answer:
\(\frac{3}{10}\)

Explanation:
Multiply the numerators and Multiply the denominators.
\(\frac{4 × 3}{5 × 8}\) = \(\frac{12}{40}\)
Simplify using the GCF.
The GCF of 12 and 40 is 4.
Divide the numerator and the denominator by 4.
\(\frac{12 ÷ 4}{40 ÷ 4}\) = \(\frac{3}{10}\)

Question 4.
\(1 \frac{1}{8} \times \frac{1}{9}\)
\(\frac{□}{□}\)

Answer:
\(\frac{1}{8}\)

Explanation:
1\(\frac{1}{8}\) = \(\frac{9}{8}\)
Multiply the numerators and Multiply the denominators.
\(\frac{9 × 1}{8 × 9}\) = \(\frac{9}{72}\)
Simplify using the GCF.
The GCF of 9 and 72 is 9.
Divide the numerator and the denominator by 9.
\(\frac{9 ÷ 9}{72 ÷ 9}\) = \(\frac{1}{8}\)

Question 5.
\(\frac{3}{4} \times \frac{1}{3} \times \frac{2}{5}\)
\(\frac{□}{□}\)

Answer:
\(\frac{1}{10}\)

Explanation:
Multiply the numerators and Multiply the denominators.
\(\frac{3 × 1 × 2}{4 × 3 × 5}\) = \(\frac{6}{60}\)
Simplify using the GCF.
The GCF of 6 and 60 is 6.
Divide the numerator and the denominator by 6.
\(\frac{6 ÷ 6}{60 ÷ 6}\) = \(\frac{1}{10}\)

Question 6.
Karen raked \(\frac{3}{5}\) of the yard. Minni raked \(\frac{1}{3}\) of the amount Karen raked. How much of the yard did Minni rake?
\(\frac{□}{□}\)

Answer:
\(\frac{1}{3}\)

Explanation:
Minni raked 1/5 of the yard.
So, minni raked 3/5 of 1/3 means 3/5 x 1/3
Multiply the numerators and Multiply the denominators.
\(\frac{3 × 1}{5 × 3}\) = \(\frac{3}{15}\)
Simplify using the GCF.
The GCF of 3 and 15 is 3.
Divide the numerator and the denominator by 3.
\(\frac{3 ÷ 3}{15 ÷ 3}\) = \(\frac{1}{3}\)

Question 7.
\(\frac{3}{8}\) of the pets in the pet show are dogs. \(\frac{2}{3}\) of the dogs have long hair. What fraction of the pets are dogs with long hair?
\(\frac{□}{□}\)

Answer:
\(\frac{1}{4}\) are dogs with long hair

Explanation:
\(\frac{3}{8}\) of the pets in the pet show are dogs. \(\frac{2}{3}\) of the dogs have long hair.
\(\frac{3}{8}\) of \(\frac{2}{3}\) = \(\frac{3 × 2}{8 × 3}\) = \(\frac{6}{24}\)
The GCF of 6 and 24 is 6.
Divide the numerator and the denominator by 6.
\(\frac{6 ÷ 6}{24 ÷ 6}\) = \(\frac{1}{4}\)
\(\frac{1}{4}\) are dogs with long hair

Evaluate using the order of operations.

Question 8.
\(\left(\frac{1}{2}+\frac{3}{8}\right) \times 8\)
______

Answer:
7

Explanation:
1/2 + 3/8 = 7/8
7/8 × 8 = 7

Question 9.
\(\frac{3}{4} \times\left(1-\frac{1}{9}\right)\)
\(\frac{□}{□}\)

Answer:
\(\frac{2}{3}\)

Explanation:
1 – 1/9 = 8/9
3/4 × 8/9 = 24/36
The GCF of 24 and 36 is 12.
Divide the numerator and the denominator by 12.
\(\frac{24 ÷ 12}{36 ÷ 12}\) = \(\frac{2}{3}\)

Question 10.
\(4 \times \frac{1}{8} \times \frac{3}{10}\)
\(\frac{□}{□}\)

Answer:
\(\frac{3}{20}\)

Explanation:
Multiply the numerators and Multiply the denominators.
\(\frac{4 × 1 × 3}{1 × 8 × 10}\) = \(\frac{12}{80}\)
Simplify using the GCF.
The GCF of 12 and 80 is 4.
Divide the numerator and the denominator by 4.
\(\frac{12 ÷ 4}{80 ÷ 4}\) = \(\frac{3}{20}\)

Question 11.
\(6 \times\left(\frac{4}{5}+\frac{2}{10}\right) \times \frac{2}{3}\)
______

Answer:
4

Explanation:
4/5 + 2/10 = 1
6 × 1 × 2/3 = 12/3
The GCF of 12 and 3 is 4.
Divide the numerator and the denominator by 3.
\(\frac{12 ÷ 3}{3 ÷ 3}\) = \(\frac{4}{1}\) = 4

Problem Solving

Question 12.
Jason ran \(\frac{5}{7}\) of the distance around the school track. Sara ran \(\frac{4}{5}\) of Jason’s distance. What fraction of the total distance around the track did Sara run?
\(\frac{□}{□}\)

Answer:
\(\frac{4}{7}\)

Explanation:
Jason ran \(\frac{5}{7}\) of the distance around the school track. Sara ran \(\frac{4}{5}\) of Jason’s distance.
\(\frac{5}{7}\) × \(\frac{4}{5}\) = 20/35
The GCF of 20 and 35 is 5.
Divide the numerator and the denominator by 5.
\(\frac{20 ÷ 5}{35 ÷ 5}\) = \(\frac{4}{7}\)

Question 13.
A group of students attend a math club. Half of the students are boys and \(\frac{4}{9}\) of the boys have brown eyes. What fraction of the group are boys with brown eyes?
\(\frac{□}{□}\)

Answer:
\(\frac{2}{9}\) group are boys with brown eyes

Explanation:
A group of students attend a math club. Half of the students are boys and \(\frac{4}{9}\) of the boys have brown eyes.
\(\frac{4}{9}\) × \(\frac{1}{2}\) = 4/18 = 2/9
2/9 group are boys with brown eyes

Question 14.
Write and solve a word problem that involves multiplying by a fraction.
Type below:
__________

Answer:
A group of students attends a math club. Half of the students are boys and \(\frac{6}{9}\) of the boys have brown eyes. What fraction of the group are boys with brown eyes?
\(\frac{□}{□}\)
Answer:
A group of students attends a math club. Half of the students are boys and \(\frac{6}{9}\) of the boys have brown eyes.
\(\frac{6}{9}\) × \(\frac{1}{2}\) = 6/18 = 1/3
1/3 group are boys with brown eyes.

Lesson Check – Page No. 86

Question 1.
Veronica’s mom left \(\frac{3}{4}\) of a cake on the table. Her brothers ate \(\frac{1}{2}\) of it. What fraction of the cake did they eat?
\(\frac{□}{□}\)

Answer:
\(\frac{2}{4}\)

Explanation:
Veronica’s mom left \(\frac{3}{4}\) of a cake on the table. Her brothers ate \(\frac{1}{2}\) of it.
Since the fraction of the eaten cake is 1/2, you can multiply the numerator and denominator by and get an equivalent fraction, which is 2/4.

Question 2.
One lap around the school track is \(\frac{5}{8}\) mile. Carin ran 3 \(\frac{1}{2}\) laps. How far did she run?
_____ \(\frac{□}{□}\)

Answer:
2\(\frac{3}{16}\)

Explanation:
One lap around the school track is \(\frac{5}{8}\) mile. Carin ran 3 \(\frac{1}{2}\) laps.
3 \(\frac{1}{2}\) = \(\frac{7}{2}\)
Therefore, the total distance covered = 7/2 × 5/8 = 35/16 = 2 3/16

Spiral Review

Question 3.
Tom bought 2 \(\frac{5}{16}\) pounds of peanuts and 2.45 pounds of cashews. Which did he buy more of? Explain.
Type below:
__________

Answer:

Explanation:
Tom bought 2 \(\frac{5}{16}\) pounds of peanuts and 2.45 pounds of cashews.
2 \(\frac{5}{16}\) = 2.3125
2.3125 < 2.45
He buys more cashews.

Question 4.
Eve has 24 stamps each valued at $24.75. What is the total value of her stamps?
$ _____

Answer:
$594

Explanation:
Eve has 24 stamps each valued at $24.75.
24 x $24.75 = $594

Question 5.
Naomi went on a 6.5-mile hike. In the morning, she hiked 1.75 miles, rested, and then hiked 2.4 more miles. She completed the hike in the afternoon. How much farther did she hike in the morning than in the afternoon?
_____ miles

Answer:
Naomi went on a 6.5-mile hike. In the morning, she hiked 1.75 miles, rested, and then hiked 2.4 more miles. She completed the hike in the afternoon.
To find how many miles she walked in the afternoon you just subtract the morning miles 4.15 from the total miles 6.5.
6.5 – 4.15  = 2.35
To find how many more miles she walked in the morning you just subtract the morning from the afternoon 4.15 – 2.35=1.8 miles.
She hiked 1.8 more miles in the morning

Question 6.
A bookstore owner has 48 science fiction books and 30 mysteries he wants to sell quickly. He will make discount packages with one type of book in each. He wants the most books possible in each package, but all packages must contain the same number of books. How many packages can he make? How many packages of each type of book does he have?
Type below:
__________

Answer:
18 packages

Explanation:
The bookstore owner can make 18 possible packages
48 – 30 = 18 packages

Share and Show – Page No. 89

Find the product. Simplify before multiplying.

Question 1.
\(\frac{5}{6} \times \frac{3}{10}\)
\(\frac{□}{□}\)

Answer:
\(\frac{1}{4}\)

Explanation:
\(\frac{5}{6} \times \frac{3}{10}\)
Multiply the numerators and Multiply the denominators.
\(\frac{5 × 3}{6 × 10}\) = \(\frac{15}{60}\)
Simplify using the GCF.
The GCF of 15 and 60 is 15.
Divide the numerator and the denominator by 15.
\(\frac{15 ÷ 15}{60 ÷ 15}\) = \(\frac{1}{4}\)

Question 2.
\(\frac{3}{4} \times \frac{5}{9}\)
\(\frac{□}{□}\)

Answer:
\(\frac{5}{12}\)

Explanation:
\(\frac{3}{4} \times \frac{5}{9}\)
Multiply the numerators and Multiply the denominators.
\(\frac{3 × 5}{4 × 9}\) = \(\frac{15}{36}\)
Simplify using the GCF.
The GCF of 15 and 36 is 3.
Divide the numerator and the denominator by 3.
\(\frac{15 ÷ 3}{36 ÷ 3}\) = \(\frac{5}{12}\)

Question 3.
\(\frac{2}{3} \times \frac{9}{10}\)
\(\frac{□}{□}\)

Answer:
\(\frac{3}{5}\)

Explanation:
\(\frac{2}{3} \times \frac{9}{10}\)
Multiply the numerators and Multiply the denominators.
\(\frac{2 × 9}{3 × 10}\) = \(\frac{18}{30}\)
Simplify using the GCF.
The GCF of 18 and 30 is 6.
Divide the numerator and the denominator by 6.
\(\frac{18 ÷ 6}{30 ÷ 6}\) = \(\frac{3}{5}\)

Question 4.
After a picnic, \(\frac{5}{12}\) of the cornbread is left over. Val eats \(\frac{3}{5}\) of the leftover cornbread. What fraction of the cornbread does Val eat?
\(\frac{□}{□}\)

Answer:
\(\frac{1}{4}\)

Explanation:
After a picnic, \(\frac{5}{12}\) of the cornbread is left over. Val eats \(\frac{3}{5}\) of the leftover cornbread.
\(\frac{5}{12} \times \frac{3}{5}\)
Multiply the numerators and Multiply the denominators.
\(\frac{5 × 3}{12 × 5}\) = \(\frac{15}{60}\)
Simplify using the GCF.
The GCF of 15 and 60 is 15.
Divide the numerator and the denominator by 15.
\(\frac{15 ÷ 15}{60 ÷ 15}\) = \(\frac{1}{4}\)

Question 5.
The reptile house at the zoo has an iguana that is \(\frac{5}{6}\) yd long. It has a Gila monster that is \(\frac{4}{5}\) of the length of the iguana. How long is the Gila monster?
\(\frac{□}{□}\)

Answer:
\(\frac{2}{3}\)

Explanation:
The reptile house at the zoo has an iguana that is \(\frac{5}{6}\) yd long. It has a Gila monster that is \(\frac{4}{5}\) of the length of the iguana.
\(\frac{5}{6} \times \frac{4}{5}\)
Multiply the numerators and Multiply the denominators.
\(\frac{5 × 4}{6× 5}\) = \(\frac{20}{30}\)
Simplify using the GCF.
The GCF of 20 and 30 is 10.
Divide the numerator and the denominator by 10.
\(\frac{20 ÷ 10}{30 ÷ 10}\) = \(\frac{2}{3}\)

On Your Own

Find the product. Simplify before multiplying.

Question 6.
\(\frac{3}{4} \times \frac{1}{6}\)
\(\frac{□}{□}\)

Answer:

Explanation:
\(\frac{3}{4} \times \frac{1}{6}\)
Multiply the numerators and Multiply the denominators.
\(\frac{3 × 1}{4 × 6}\) = \(\frac{3}{24}\)
Simplify using the GCF.
The GCF of 3 and 24 is 3.
Divide the numerator and the denominator by 3.
\(\frac{3 ÷ 3}{24 ÷ 3}\) = \(\frac{1}{8}\)

Question 7.
\(\frac{7}{10} \times \frac{2}{3}\)
\(\frac{□}{□}\)

Answer:
\(\frac{7}{15}\)

Explanation:
\(\frac{7}{10} \times \frac{2}{3}\)
Multiply the numerators and Multiply the denominators.
\(\frac{7 × 2}{10 × 3}\) = \(\frac{14}{30}\)
Simplify using the GCF.
The GCF of 14 and 30 is 2.
Divide the numerator and the denominator by 2.
\(\frac{14 ÷ 2}{30 ÷ 2}\) = \(\frac{7}{15}\)

Question 8.
\(\frac{5}{8} \times \frac{2}{5}\)
\(\frac{□}{□}\)

Answer:
\(\frac{1}{4}\)

Explanation:
\(\frac{5}{8} \times \frac{2}{5}\)
Multiply the numerators and Multiply the denominators.
\(\frac{5 × 2}{8 × 5}\) = \(\frac{10}{40}\)
Simplify using the GCF.
The GCF of 10 and 40 is 10.
Divide the numerator and the denominator by 10.
\(\frac{10 ÷ 10}{40 ÷ 10}\) = \(\frac{1}{4}\)

Question 9.
\(\frac{9}{10} \times \frac{5}{6}\)
\(\frac{□}{□}\)

Answer:
\(\frac{3}{4}\)

Explanation:
\(\frac{9}{10} \times \frac{5}{6}\)
Multiply the numerators and Multiply the denominators.
\(\frac{9 × 5}{10 × 6}\) = \(\frac{45}{60}\)
Simplify using the GCF.
The GCF of 45 and 60 is 15.
Divide the numerator and the denominator by 15.
\(\frac{45 ÷ 15}{60 ÷ 15}\) = \(\frac{3}{4}\)

Question 10.
\(\frac{11}{12} \times \frac{3}{7}\)
\(\frac{□}{□}\)

Answer:
\(\frac{11}{28}\)

Explanation:
\(\frac{11}{12} \times \frac{3}{7}\)
Multiply the numerators and Multiply the denominators.
\(\frac{11 × 3}{12 × 7}\) = \(\frac{33}{84}\)
Simplify using the GCF.
The GCF of 33 and 84 is 3.
Divide the numerator and the denominator by 3.
\(\frac{33 ÷ 3}{84 ÷ 3}\) = \(\frac{11}{28}\)

Question 11.
Shelley’s basketball team won \(\frac{3}{4}\) of their games last season. In \(\frac{1}{6}\) of the games they won, they outscored their opponents by more than 10 points. What fraction of their games did Shelley’s team win by more than 10 points?
\(\frac{□}{□}\)

Answer:
\(\frac{1}{8}\)

Explanation:
Let the total number of games be x.
Number of games Shelley’s team won = 3/4x
Number of games they outscored their opponents by more than 10 points = 1/6 X 3/4x = 1/8x
Hence, 1/8 of the total games, Shelley’s team won by 10 points.

Question 12.
Mr. Ortiz has \(\frac{3}{4}\) pound of oatmeal. He uses \(\frac{2}{3}\) of the oatmeal to bake muffins. How much oatmeal does Mr. Ortiz have left?
\(\frac{□}{□}\)

Answer:
\(\frac{1}{2}\)

Explanation:
Mr. Ortiz has \(\frac{3}{4}\) pound of oatmeal. He uses \(\frac{2}{3}\) of the oatmeal to bake muffins.
\(\frac{3}{4} \times \frac{2}{3}\)
Multiply the numerators and Multiply the denominators.
\(\frac{3 × 2}{4 × 3}\) = \(\frac{6}{12}\)
Simplify using the GCF.
The GCF of 6 and 12 is 6.
Divide the numerator and the denominator by 6.
\(\frac{6 ÷ 6}{12 ÷ 6}\) = \(\frac{1}{2}\)

Question 13.
Compare Strategies To find \(\frac{16}{27}\) × \(\frac{3}{4}\), you can multiply the fractions and then simplify the product or you can simplify the fractions and then multiply. Which method do you prefer? Explain.
Type below:
__________

Answer:
\(\frac{16}{27}\) × \(\frac{3}{4}\)
\(\frac{16 × 3}{27 × 4}\) = \(\frac{16 × 3}{4 × 27}\)
\(\frac{48}{96}\)
Simplify using the GCF.
The GCF of 48 and 96 is 48.
Divide the numerator and the denominator by 48.
\(\frac{48 ÷ 48}{96 ÷ 48}\) = \(\frac{1}{2}\)

Problem Solving + Applications – Page No. 90

Go Math Grade 6 Answer Key Chapter 2 Fractions and Decimals 7

Question 14.
Three students each popped \(\frac{3}{4}\) cup of popcorn kernels. The table shows the fraction of each student’s kernels that did not pop. Which student had \(\frac{1}{16}\) cup unpopped kernels?
__________

Answer:
Mirza

Explanation:
Three students each popped \(\frac{3}{4}\) cup of popcorn kernels. The table shows the fraction of each student’s kernels that did not pop.
Katie = 3/4 x 1/10 = 3/40
Mirza = 3/4 x 1/12 = 1/16

Question 15.
The jogging track at Francine’s school is \(\frac{3}{4}\) mile long. Yesterday Francine completed two laps on the track. If she ran \(\frac{1}{3}\) of the distance and walked the remainder of the way, how far did she walk?
____ mile

Answer:
1 mile

Explanation:
Length of jogging track at Francine’s school = 3/4 mile
Let the distance covered by running be = x
Let the distance covered by walking be = y
Total number of laps completed by Francine = 2
Total distance covered by Francine = number of laps X distance covered in one lap
2 x 3/4 = 3/25 mile
Now,
distance covered by running = 1/3 of the total distance
x = 1/3 x 3/2
distance covered by walking y = total distance – distance covered by running
3/2 – x = 3/2 – 1/2 = 1 mile
Hence, Francine walked for 1 mile.

Question 16.
At a snack store, \(\frac{7}{12}\) of the customers bought pretzels and \(\frac{3}{10}\) of those customers bought low-salt pretzels. Bill states that \(\frac{7}{30}\) of the customers bought low-salt pretzels. Does Bill’s statement make sense? Explain.
Type below:
__________

Answer:
Bill’s statement does not make sense because it is incorrect:
7/12 customers bought pretzels.
3/10 Of those customers bought low salt pretzels (x)
3/10 of 7/12 = x
21/120 = x
Simplify: 7/40
To be correct, Bill would have to say that 7/40 of the customers bought low salt pretzels, but instead, he had said 7/30.

Question 17.
The table shows Tonya’s homework assignment. Tonya’s teacher instructed the class to simplify each expression by dividing the numerator and denominator by the GCF. Complete the table by simplifying each expression and then finding the value.
Go Math Grade 6 Answer Key Chapter 2 Fractions and Decimals 8
Type below:
__________

Answer:
Garde 6 chapter 2 image 1

Simplify Factors – Page No. 91

Find the product. Simplify before multiplying.

Question 1.
\(\frac{8}{9} \times \frac{5}{12}\)
\(\frac{□}{□}\)

Answer:
\(\frac{10}{27}\)

Explanation:
\(\frac{8}{9} \times \frac{5}{12}\)
Multiply the numerators and Multiply the denominators.
\(\frac{8 × 5}{9 × 12}\) = \(\frac{40}{108}\)
Simplify using the GCF.
The GCF of 40 and 108 is 4.
Divide the numerator and the denominator by 4.
\(\frac{40 ÷ 4}{108 ÷ 4}\) = \(\frac{10}{27}\)

Question 2.
\(\frac{3}{4} \times \frac{16}{21}\)
\(\frac{□}{□}\)

Answer:
\(\frac{4}{7}\)

Explanation:
\(\frac{3}{4} \times \frac{16}{21}\)
Multiply the numerators and Multiply the denominators.
\(\frac{3 × 16}{4 × 21}\) = \(\frac{48}{84}\)
Simplify using the GCF.
The GCF of 48 and 84 is 12.
Divide the numerator and the denominator by 12.
\(\frac{48 ÷ 12}{84 ÷ 12}\) = \(\frac{4}{7}\)

Question 3.
\(\frac{15}{20} \times \frac{2}{5}\)
\(\frac{□}{□}\)

Answer:
\(\frac{3}{10}\)

Explanation:
\(\frac{15}{20} \times \frac{2}{5}\)
Multiply the numerators and Multiply the denominators.
\(\frac{15 × 2}{20 × 5}\) = \(\frac{30}{100}\)
Simplify using the GCF.
The GCF of 30 and 100 is 10.
Divide the numerator and the denominator by 10.
\(\frac{30 ÷ 10}{100 ÷ 10}\) = \(\frac{3}{10}\)

Question 4.
\(\frac{9}{18} \times \frac{2}{3}\)
\(\frac{□}{□}\)

Answer:
\(\frac{1}{3}\)

Explanation:
\(\frac{9}{18} \times \frac{2}{3}\)
Multiply the numerators and Multiply the denominators.
\(\frac{9 × 2}{18 × 3}\) = \(\frac{18}{54}\)
Simplify using the GCF.
The GCF of 18 and 54 is 18.
Divide the numerator and the denominator by 18.
\(\frac{18 ÷ 18}{54 ÷ 18}\) = \(\frac{1}{3}\)

Question 5.
\(\frac{3}{4} \times \frac{7}{30}\)
\(\frac{□}{□}\)

Answer:
\(\frac{7}{40}\)

Explanation:
\(\frac{3}{4} \times \frac{7}{30}\)
Multiply the numerators and Multiply the denominators.
\(\frac{3 × 7}{4 × 30}\) = \(\frac{21}{120}\)
Simplify using the GCF.
The GCF of 21 and 120 is 3.
Divide the numerator and the denominator by 3.
\(\frac{21 ÷ 3}{120 ÷ 3}\) = \(\frac{7}{40}\)

Question 6.
\(\frac{8}{15} \times \frac{15}{32}\)
\(\frac{□}{□}\)

Answer:
\(\frac{1}{4}\)

Explanation:
\(\frac{8}{15} \times \frac{15}{32}\)
Multiply the numerators and Multiply the denominators.
\(\frac{8 × 15}{15 × 32}\) = \(\frac{120}{480}\)
Simplify using the GCF.
The GCF of 120 and 480 is 120.
Divide the numerator and the denominator by 120.
\(\frac{120 ÷ 120}{480 ÷ 120}\) = \(\frac{1}{4}\)

Question 7.
\(\frac{12}{21} \times \frac{7}{9}\)
\(\frac{□}{□}\)

Answer:
\(\frac{4}{9}\)

Explanation:
\(\frac{12}{21} \times \frac{7}{9}\)
Multiply the numerators and Multiply the denominators.
\(\frac{12 × 7}{21 × 9}\) = \(\frac{84}{189}\)
Simplify using the GCF.
The GCF of 84 and 189 is 21.
Divide the numerator and the denominator by 21.
\(\frac{84 ÷ 21}{189 ÷ 21}\) = \(\frac{4}{9}\)

Question 8.
\(\frac{18}{22} \times \frac{8}{9}\)
\(\frac{□}{□}\)

Answer:
\(\frac{8}{11}\)

Explanation:
\(\frac{18}{22} \times \frac{8}{9}\)
Multiply the numerators and Multiply the denominators.
\(\frac{18 × 8}{22 × 9}\) = \(\frac{144}{198}\)
Simplify using the GCF.
The GCF of 144 and 198 is 18.
Divide the numerator and the denominator by 18.
\(\frac{144 ÷ 18}{198 ÷ 18}\) = \(\frac{8}{11}\)

Problem Solving

Question 9.
Amber has a \(\frac{4}{5}\)-pound bag of colored sand. She uses \(\frac{1}{2}\) of the bag for an art project. How much sand does she use for the project?
\(\frac{□}{□}\) pounds

Answer:
\(\frac{2}{5}\) pounds

Explanation:
Amber has a \(\frac{4}{5}\)-pound bag of colored sand. She uses \(\frac{1}{2}\) of the bag for an art project.
4/5 X 1/2 = 2/5

Question 10.
Tyler has \(\frac{3}{4}\) month to write a book report. He finished the report in \(\frac{2}{3}\) that time. How much time did it take Tyler to write the report?
\(\frac{□}{□}\) month

Answer:
\(\frac{1}{2}\) month

Explanation:
Tyler has \(\frac{3}{4}\) month to write a book report. He finished the report in \(\frac{2}{3}\) that time.
3/4 X 2/3 = 1/2

Question 11.
Show two ways to multiply \(\frac{2}{15} \times \frac{3}{20}\). Then tell which way is easier and justify your choice.
Type below:
__________

Answer:
\(\frac{2}{15} \times \frac{3}{20}\)
2/15 X 3/20 = 2/20 X 3/15 = 1/10 X 1/5 = 1/50

Lesson Check – Page No. 92

Find each product. Simplify before multiplying.

Question 1.
At Susie’s school, \(\frac{5}{8}\) of all students play sports. Of the students who play sports, \(\frac{2}{5}\) play soccer. What fraction of the students in Susie’s school play soccer?
\(\frac{□}{□}\)

Answer:
\(\frac{1}{4}\)

Explanation:
At Susie’s school, \(\frac{5}{8}\) of all students play sports. Of the students who play sports, \(\frac{2}{5}\) play soccer.
Multiply 5/8 X 2/5, and the answer is 0.25, which converts to 25/100 or 1/4

Question 2.
A box of popcorn weighs \(\frac{15}{16}\) pounds. The box contains \(\frac{1}{3}\) buttered popcorn and \(\frac{2}{3}\) cheesy popcorn. How much does the cheesy popcorn weigh?
\(\frac{□}{□}\)

Answer:
\(\frac{5}{8}\)

Explanation:
Total weight of a box of popcorn =15/16 pounds.
We are given two types of popcorns are there, butter popcorns and cheesy popcorns.
Butter popcorn is the one-third of the total weight = 1/3 of the Total weight
Plugging the value of the total weight, we get
= 1/3 * 15/16 = 5/16 pounds.
Cheesy popcorn = 2/3 of Total weight
Plugging the value of total weight, we get
= 2/3 * 15/16 = 10/16 or 5/8 pounds.
Therefore, cheesy popcorn weighs is 5/8 pounds.

Spiral Review

Question 3.
Ramòn bought a dozen ears of corn for $1.80. What was the cost of each ear of corn?
$ ______

Answer:
$0.15

Explanation:
Ramòn bought a dozen ears of corn for $1.80.
So, for the cost of each ear of corn, $1.80/12 = $0.15

Question 4.
A 1.8-ounce jar of cinnamon costs $4.05. What is the cost per ounce?
$ ______

Answer:
$2.25 per ounce

Explanation:
If a 1.8-ounce jar costs $4.05, do $4.05 divided by 1.8.
$4.05 / 1.8 = $2.25 per ounce.

Question 5.
Rose bought \(\frac{7}{20}\) kilogram of ginger candy and 0.4 kilogram of cinnamon candy. Which did she buy more of? Explain how you know.
Type below:
__________

Answer:
Rose bought ginger candy = 7/20 kilogram = 0.35 Kilogram
She bought cinnamon candy = 0.4 kilogram
0.4 > 0.35
Therefore, She bought cinnamon candy more.

Question 6.
Don walked 3 \(\frac{3}{5}\) miles on Friday, 3.7 miles on Saturday, and 3 \(\frac{5}{8}\) miles on Sunday. List the distances from least to greatest.
Type below:
__________

Answer:
3 \(\frac{3}{5}\), 3 \(\frac{5}{8}\), 3.7

Explanation:
3 \(\frac{3}{5}\) = 18/5 = 3.6
3 \(\frac{5}{8}\) = 29/8 = 3.625
3.6 < 3.625 < 3.7
3 \(\frac{3}{5}\), 3 \(\frac{5}{8}\), 3.7

Mid-Chapter Checkpoint – Vocabulary – Page No. 93

Choose the best term from the box to complete the sentence.
Go Math Grade 6 Answer Key Chapter 2 Fractions and Decimals 9

Question 1.
The fractions \(\frac{1}{2}\) and \(\frac{5}{10}\) are _____.
Type below:
__________

Answer:
Equivalent fractions

Question 2.
A _____ is a denominator that is the same in two or more fractions.
Type below:
__________

Answer:
Common Denominator

Concepts and Skills

Write as a decimal. Tell whether you used division, a number line, or some other method.

Question 3.
\(\frac{7}{20}\)
_____

Answer:
0.35

Explanation:
By using Division,
\(\frac{7}{20}\) = 0.35

Question 4.
8 \(\frac{39}{40}\)
_____

Answer:
8.975

Explanation:
By using Division,
8 \(\frac{39}{40}\) = 359/40 = 8.975

Question 5.
1 \(\frac{5}{8}\)
_____

Answer:
1.625

Explanation:
By using Division,
1 \(\frac{5}{8}\) = 13/8 = 1.625

Question 6.
\(\frac{19}{25}\)
_____

Answer:
0.76

Explanation:
By using Division,
\(\frac{19}{25}\) = 0.76

Order from least to greatest.

Question 7.
\(\frac{4}{5}, \frac{3}{4}, 0.88\)
Type below:
__________

Answer:
\(\frac{3}{4}\), \(\frac{4}{5}\),0.88

Explanation:
Write the decimal form of 4/5 = 0.8
Write the decimal form of 3/4 = 0.75
0.88
0.75 < 0.8 < 0.88

Question 8.
0.65, 0.59, \(\frac{3}{5}\)
Type below:
__________

Answer:
0.59, \(\frac{3}{5}\), 0.65

Explanation:
Write the decimal form of 3/5 = 0.6
0.59 < 0.6 < 0.65

Question 9.
\(1 \frac{1}{4}, 1 \frac{2}{3}, \frac{11}{12}\)
Type below:
__________

Answer:
\(\frac{11}{12}\), 1\(\frac{1}{4}\), 1\(\frac{2}{3}\)

Explanation:
Write the decimal form of 1 1/4 = 5/4 = 1.25
Write the decimal form of 1 2/3 = 5/3 = 1.66
Write the decimal form of 11/12 = 0.916
0.916 < 1.25 < 1.66

Question 10.
0.9, \(\frac{7}{8}\), 0.86
Type below:
__________

Answer:
0.86, \(\frac{7}{8}\), 0.9

Explanation:
Write the decimal form of \(\frac{7}{8}\) = 0.875
0.86 < 0.875 < 0.9

Find the product. Write it in simplest form.

Question 11.
\(\frac{2}{3} \times \frac{1}{8}\)
\(\frac{□}{□}\)

Answer:
\(\frac{1}{12}\)

Explanation:
\(\frac{2}{3} \times \frac{1}{8}\)
Multiply the numerators and Multiply the denominators.
\(\frac{2 × 1}{3 × 8}\) = \(\frac{2}{24}\)
Simplify using the GCF.
The GCF of 2 and 24 is 2.
Divide the numerator and the denominator by 2.
\(\frac{2 ÷ 2}{24 ÷ 2}\) = \(\frac{1}{12}\)

Question 12.
\(\frac{4}{5} \times \frac{2}{5}\)
\(\frac{□}{□}\)

Answer:
\(\frac{8}{25}\)

Explanation:
\(\frac{4}{5} \times \frac{2}{5}\)
Multiply the numerators and Multiply the denominators.
\(\frac{4 × 2}{5 × 5}\) = \(\frac{8}{25}\)

Question 13.
12 × \(\frac{3}{4}\)
_____

Answer:
9

Explanation:
12 × \(\frac{3}{4}\)
Multiply the numerators and Multiply the denominators.
\(\frac{12 × 3}{1 × 4}\) = \(\frac{36}{4}\) = 9

Question 14.
Mia climbs \(\frac{5}{8}\) of the height of the rock wall. Lee climbs \(\frac{4}{5}\) of Mia’s distance. What fraction of the wall does Lee climb?
\(\frac{□}{□}\)

Answer:
\(\frac{7}{40}\)

Explanation:
find the LCM (least common denominator) for 5/8 and 4/5.
5/8= 25/40 and 4/5= 32/40.
Subtract and you get 7/40.

Page No. 94

Question 15.
In Zoe’s class, \(\frac{4}{5}\) of the students have pets. Of the students who have pets, \(\frac{1}{8}\) have rodents. What fraction of the students in Zoe’s class have pets that are rodents? What fraction of the students in Zoe’s class have pets that are not rodents?
Type below:
__________

Answer:
\(\frac{1}{10}\) of the students in Zoe’s class have pets that are rodents
\(\frac{7}{10}\) of the students in Zoe’s class have pets that are not rodents

Explanation:
In Zoe’s class, \(\frac{4}{5}\) of the students have pets. Of the students who have pets, \(\frac{1}{8}\) have rodents.
4/5 X 1/8 = 1/10
4/5 – 1/10 = 7/10

Question 16.
A recipe calls for 2 \(\frac{2}{3}\) cups of flour. Terell wants to make \(\frac{3}{4}\) of the recipe. How much flour should he use?
_____ cups

Answer:
2 cups

Explanation:
2 \(\frac{2}{3}\) = 8/3
8/3 * 3/4 = 2

Question 17.
Following the Baltimore Running Festival in 2009, volunteers collected and recycled 3.75 tons of trash. Graph 3.75 on a number line and write the weight as a mixed number.
Type below:
__________

Answer:
Volunteers collected and recycled 3.75 tons of trash.
We need to convert 3.75 as a mixed number.
The mixed number consists of a whole number and a proper fraction.
In the given number 3.75, 3 as the whole number and convert 0.75 to a fraction.
3.75 = 3 + 0.75 = 3 + 75/100
We can reduce the fraction 75/ 100 = 3+ 3/4 = 3 3/4

Question 18.
Four students took an exam. The fraction of the total possible points that each received is given. Which student had the highest score? If students receive a whole number of points on every exam item, can the exam be worth a total of 80 points? Explain.
Go Math Grade 6 Answer Key Chapter 2 Fractions and Decimals 10
Type below:
__________

Answer:
22/25 = 0.88
17/20 = 0.85
4/5 = 0.8
3/4 = 0.75
Monica had the highest score
Let x be the total number of points:
(22/25 + 17/20 + 4/5 + 3/4)x = 80
x = 24.39
That is not a whole number of points.

Share and Show – Page No. 97

Use the model to find the quotient.

Question 1.
\(\frac{1}{2}\) ÷ 3
Go Math Grade 6 Answer Key Chapter 2 Fractions and Decimals 11
\(\frac{□}{□}\)

Answer:
\(\frac{1}{6}\)

Explanation:
1/2 groups of 3
\(\frac{1}{2}\) ÷ 3
1/2 × 1/3 = 1/6

Question 2.
\(\frac{3}{4} \div \frac{3}{8}\)
Go Math Grade 6 Answer Key Chapter 2 Fractions and Decimals 12
______

Answer:
2

Explanation:
3/4 groups of 3/8
3/4 × 8/3 = 2

Use fraction strips to find the quotient. Then draw the model.

Question 3.
\(\frac{1}{3}\) ÷ 4
\(\frac{□}{□}\)

Answer:
\(\frac{1}{12}\)
Garde 6 chapter 2 image 2

Explanation:
\(\frac{1}{3}\) ÷ 4
\(\frac{1}{3}\) × \(\frac{1}{4}\)
\(\frac{1}{12}\)

Question 4.
\(\frac{3}{5} \div \frac{3}{10}\)
______

Answer:
2

Explanation:
\(\frac{3}{5} \div \frac{3}{10}\)
\(\frac{3}{5}\) × \(\frac{10}{3}\)
2

Draw a model to solve. Then write an equation for the model. Interpret the result.

Question 5.
How many \(\frac{1}{4}\) cup servings of raisins are in \(\frac{3}{8}\) cup of raisins?
Type below:
__________

Answer:
1.5

Explanation:
3/8 × 1/4 = 1.5

Question 6.
How many \(\frac{1}{3}\) lb bags of trail mix can Josh make from \(\frac{5}{6}\) lb of trail mix?
Type below:
__________

Answer:
2

Explanation:
Multiply 1/3 with 2
1/3 × 2 = 2/6. 2/6 can go into 5/6 twice so the answer is two bags.

Question 7.
Pose a Problem Write and solve a problem for \(\frac{3}{4}\) ÷ 3 that represents how much in each of 3 groups.
Type below:
__________

Answer:
\(\frac{1}{4}\)

Explanation:
\(\frac{3}{4}\) ÷ 3
\(\frac{3}{4}\) × \(\frac{1}{3}\) = 1/4

Problem Solving + Applications – Page No. 98

The table shows the amount of each material that students in a sewing class need for one purse.

Use the table for 8–10. Use models to solve.
Go Math Grade 6 Answer Key Chapter 2 Fractions and Decimals 13

Question 8.
Mrs. Brown has \(\frac{1}{3}\) yd of blue denim and \(\frac{1}{2}\) yd of black denim. How many purses can be made using denim as the main fabric?
_____ purses

Answer:
5 purses

Explanation:
Mrs. Brown has \(\frac{1}{3}\) yd of blue denim and \(\frac{1}{2}\) yd of black denim.
3 + 2 = 5

Question 9.
One student brings \(\frac{1}{2}\) yd of ribbon. If 3 students receive an equal length of the ribbon, how much ribbon will each student receive? Will each of them have enough ribbon for a purse? Explain.
Type below:
__________

Answer:
One student brings \(\frac{1}{2}\) yd of ribbon. If 3 students receive an equal length of the ribbon,
\(\frac{1}{2}\) ÷ 3
1/2 × 1/3 = 1/6
They don’t have enough ribbon for a purse

Question 10.
Make Arguments There was \(\frac{1}{2}\) yd of purple and pink striped fabric. Jessie said she could only make \(\frac{1}{24}\) of a purse using that fabric as the trim. Is she correct? Use what you know about the meanings of multiplication and division to defend your answer.
Type below:
__________

Answer:
There was \(\frac{1}{2}\) yd of purple and pink striped fabric. Jessie said she could only make \(\frac{1}{24}\) of a purse using that fabric as the trim.
1/2 × 12 = 1/24
So, 12 is the answer

Question 11.
Draw a model to find the quotient.
\(\frac{1}{2}\) ÷ 4 =
Type below:
__________

Answer:
Garde 6 chapter 2 image 3

Explanation:
1/2 × 1/4 = 1/8

Model Fraction Division – Page No. 99

Use the model to find the quotient

Question 1.
\(\frac{1}{4}\) ÷ 3 =
Go Math Grade 6 Answer Key Chapter 2 Fractions and Decimals 14
\(\frac{□}{□}\)

Answer:
\(\frac{1}{12}\)

Explanation:
\(\frac{1}{4}\) ÷ 3
\(\frac{1}{4}\) × \(\frac{1}{3}\) = \(\frac{1}{12}\)

Question 2.
\(\frac{1}{2} \div \frac{2}{12}=\)
Go Math Grade 6 Answer Key Chapter 2 Fractions and Decimals 15
______

Answer:
3

Explanation:
\(\frac{1}{2} \div \frac{2}{12}=\)
\(\frac{1}{2}\) × \(\frac{12}{2}\) = \(\frac{12}{4}\) = 3

Use fraction strips to find the quotient.

Question 3.
\(\frac{5}{6} \div \frac{1}{2}=\)
______ \(\frac{□}{□}\)

Answer:
\(\frac{5}{3}\)

Explanation:
\(\frac{5}{6} \div \frac{1}{2}=\)
\(\frac{5}{6}\) × \(\frac{2}{1}\) = \(\frac{5}{3}\)

Question 4.
\(\frac{2}{3}\) ÷ 4 =
\(\frac{□}{□}\)

Answer:
\(\frac{1}{6}\)

Explanation:
\(\frac{2}{3}\) ÷ 4
\(\frac{2}{3}\) × \(\frac{1}{4}\) = \(\frac{2}{12}\) = 1/6

Question 5.
\(\frac{1}{2}\) ÷ 6 =
\(\frac{□}{□}\)

Answer:
\(\frac{1}{12}\)

Explanation:
\(\frac{1}{2}\) ÷ 6
\(\frac{1}{2}\) × \(\frac{1}{6}\) = \(\frac{1}{12}\)

Question 6.
\(\frac{1}{3} \div \frac{1}{12}\)
______

Answer:
4

Explanation:
\(\frac{1}{3} \div \frac{1}{12}\)
\(\frac{1}{3}\) × \(\frac{12}{1}\) = \(\frac{12}{3}\) = 4

Draw a model to solve. Then write an equation for the model. Interpret the result.

Question 7.
If Jerry runs \(\frac{1}{10}\) mile each day, how many days will it take for him to run \(\frac{4}{5}\) mile?
______ days

Answer:
8 days

Explanation:
If Jerry runs \(\frac{1}{10}\) mile each day,
\(\frac{4}{5}\) ÷ \(\frac{1}{10}\)
\(\frac{4}{5}\) × \(\frac{10}{1}\) = \(\frac{40}{5}\) = 8

Problem Solving

Question 8.
Mrs. Jennings has \(\frac{3}{4}\) gallon of paint for an art project. She plans to divide the paint equally into jars. If she puts \(\frac{1}{8}\) gallon of paint into each jar, how many jars will she use?
______ jars

Answer:
6 jars

Explanation:
Mrs. Jennings has 3/4 Gallons of paint for an art project.
In 1 jar she puts 1/8 gallon of paint.
The number of jars in which she plans to to divide the paint equally is given by,
n= 3/4 ÷ 1/8
n = \(\frac{3}{4}\) × \(\frac{8}{1}\) = \(\frac{24}{4}\) = 6

Question 9.
If one jar of glue weighs \(\frac{1}{12}\) pound, how many jars can Rickie get from \(\frac{2}{3}\) pound of glue?
______ jars

Answer:
8 jars

Explanation:
The weight of glue in one jar = 1/12 pound
To get 2/3 pound of glue Rickie can get the number of jars
2/3 ÷ 1/12
2/3 × 12/1 = 24/3 = 8

Question 10.
Explain how to use a model to show \(\frac{2}{6} \div \frac{1}{12}\) and \(\frac{2}{6}\) ÷ 4.
Type below:
__________

Answer:
Garde 6 chapter 2 image 4
Garde 6 chapter 2 image 2

Explanation:
\(\frac{2}{6} \div \frac{1}{12}\)
2/6 = 1/3
1/3 x 12/1 = 4
\(\frac{2}{6}\) ÷ 4
1/3 x 1/4 = 1/12

Lesson Check – Page No. 100

Question 1.
Darcy needs \(\frac{1}{4}\) yard of fabric to make a banner. She has 2 yards of fabric. How many banners can she make?
______ banners

Answer:
8 banners

Explanation:
Darcy needs \(\frac{1}{4}\) yard of fabric to make a banner. She has 2 yards of fabric.
2 ÷ \(\frac{1}{4}\) = 2 x 4 = 8

Question 2.
Lorenzo bought \(\frac{15}{16}\) pounds of ground beef. He wants to make hamburgers that weigh \(\frac{3}{16}\) pound each. How many hamburgers can he make?
______ hamburgers

Answer:
5 hamburgers

Explanation:
Lorenzo bought \(\frac{15}{16}\) pounds of ground beef. He wants to make hamburgers that weigh \(\frac{3}{16}\) pound each.
\(\frac{15}{16}\) ÷ \(\frac{3}{16}\)
15/3 = 5

Spiral Review

Question 3.
Letisha wants to read 22 pages a night. At that rate, how long will it take her to read a book with 300 pages?
______ nights

Answer:
14 nights

Explanation:
Letisha wants to read 22 pages a night. It takes her to read a book with 300 pages
300/22 = 13.6
13.6 is near to 14
So, it is for 2 weeks.

Question 4.
A principal wants to order enough notebooks for 624 students. The notebooks come in boxes of 28. How many boxes should he order?
______ boxes

Answer:
22 boxes

Explanation:
A principal wants to order enough notebooks for 624 students. The notebooks come in boxes of 28.
624/28 = 22.2857
22.2857 is closer to 22
22 boxes.

Question 5.
Each block in Ton’s neighborhood is \(\frac{2}{3}\) mile long. If he walks 4 \(\frac{1}{2}\) blocks, how far does he walk?
______ miles

Answer:
3 miles

Explanation:
If each block is 2/3 miles long, and he walks 4 1/2 blocks, we can simply multiply to two. It looks like this:
(2/3)(4 1/2)
to multiply, make 4 1/2 into an improper fraction and multiply normally
(2/3)(9/4)
Ton walks 3 miles total.

Question 6.
In Cathy’s garden, \(\frac{5}{6}\) of the area is planted with flowers. Of the flowers, \(\frac{3}{10}\) of them are red. What fraction of Cathy’s garden is planted with red flowers?
\(\frac{□}{□}\)

Answer:
\(\frac{1}{4}\)

Explanation:
In Cathy’s garden, \(\frac{5}{6}\) of the area is planted with flowers. Of the flowers, \(\frac{3}{10}\) of them are red.
5/6 x 3/10 = 1/4

Share and Show – Page No. 103

Estimate using compatible numbers.

Question 1.
\(22 \frac{4}{5} \div 6 \frac{1}{4}\)
_______

Answer:
4

Explanation:
22 \(\frac{4}{5}\) = 114/5 = 22.8
6 \(\frac{1}{4}\) = 25/4 = 6.25
22.8 is closer to 24
6.25 is closer to 6
24/6 = 4

Question 2.
\(12 \div 3 \frac{3}{4}\)
_______

Answer:
3

Explanation:
3 \(\frac{3}{4}\) = 15/4 = 3.75
3.75 is closer to 4
12/4 = 3

Question 3.
\(33 \frac{7}{8} \div 5 \frac{1}{3}\)
_______

Answer:
7

Explanation:
33 \(\frac{7}{8}\) = 271/8 = 33.875
5 \(\frac{1}{3}\) = 16/3 = 5.333
33.875 is closer to 35
5.333 is closer to 5
35/5 = 7

Question 4.
\(3 \frac{7}{8} \div \frac{5}{9}\)
_______

Answer:
4

Explanation:
3 \(\frac{7}{8}\) = 31/8 = 3.875
\(\frac{5}{9}\) = 0.555
3.875 is closer to 4
0.555 is closer to 1
4/1 = 4

Question 5.
\(34 \frac{7}{12} \div 7 \frac{3}{8}\)
_______

Answer:
5

Explanation:
34 \(\frac{7}{12}\) = 415/12 = 34.583
7 \(\frac{3}{8}\) = 59/8 = 7.375
34.583 is closer to 35
7.375 is closer to 7
35/7 = 5

Question 6.
\(1 \frac{2}{9} \div \frac{1}{6}\)
_______

Answer:
5

Explanation:
1 \(\frac{2}{9}\) = 11/9 = 1.222
\(\frac{1}{6}\) = 0.1666
1.222 is closer to 1
0.1666 is closer to 0.2
1/0.2 = 5

On Your Own

Estimate using compatible numbers.

Question 7.
\(44 \frac{1}{4} \div 11 \frac{7}{9}\)
_______

Answer:
4

Explanation:
44 \(\frac{1}{4}\) = 177/4 = 44.25
11 \(\frac{7}{9}\) = 106/9 = 11.77
44.25 is closer to 44
11.77 is closer to 11
44/11 = 4

Question 8.
\(71 \frac{11}{12} \div 8 \frac{3}{4}\)
_______

Answer:
8

Explanation:
71 \(\frac{11}{12}\) = 863/12 = 71.916
8 \(\frac{3}{4}\) = 35/4 = 8.75
71.916 is closer to 72
8.75 is closer to 9
72/9 = 8

Question 9.
\(1 \frac{1}{6} \div \frac{1}{8}\)
_______

Answer:
12

Explanation:
1 \(\frac{1}{6}\) = 7/6 = 1.166
\(\frac{1}{8}\) = 0.125
1.166 is closer to 1.2
0.125 is closer to 0.1
1.2/0.1 = 12

Estimate to compare. Write <, >, or =.

Question 10.
\(21 \frac{3}{10} \div 2 \frac{5}{6}\) _______ \(35 \frac{7}{9} \div 3 \frac{2}{3}\)

Answer:
\(21 \frac{3}{10} \div 2 \frac{5}{6}\) < \(35 \frac{7}{9} \div 3 \frac{2}{3}\)

Explanation:
21 \(\frac{3}{10}\) = 213/10 = 21.3
2 \(\frac{5}{6}\) = 17/6 = 2.833
21.3 is closer to 21
2.833 is closer to 3
21/3 = 7
35 \(\frac{7}{9}\) = 322/9 = 35.777
3 \(\frac{2}{3}\) = 11/3 = 3.666
35.777 is closer to 36
3.666 is closer to 4
36/4 = 9
7 < 9
So, \(21 \frac{3}{10} \div 2 \frac{5}{6}\) < \(35 \frac{7}{9} \div 3 \frac{2}{3}\)

Question 11.
\(29 \frac{4}{5} \div 5 \frac{1}{6}\) _______ \(27 \frac{8}{9} \div 6 \frac{5}{8}\)

Answer:
\(29 \frac{4}{5} \div 5 \frac{1}{6}\) > \(27 \frac{8}{9} \div 6 \frac{5}{8}\)

Explanation:
29 \(\frac{4}{5}\) = 149/5 = 29.8
5 \(\frac{1}{6}\) = 31/6 = 5.1666
29.8 is closer to 30
5.1666 is closer to 5
30/5 = 6
27 \(\frac{8}{9}\) = 251/9 = 27.888
6 \(\frac{5}{8}\) = 53/8 = 6.625
27.888 is closer to 30
6.625 is closer 7
30/7 = 5
6 > 5
\(29 \frac{4}{5} \div 5 \frac{1}{6}\) > \(27 \frac{8}{9} \div 6 \frac{5}{8}\)

Question 12.
\(55 \frac{5}{6} \div 6 \frac{7}{10}\) _______ \(11 \frac{5}{7} \div \frac{5}{8}\)

Answer:
\(55 \frac{5}{6} \div 6 \frac{7}{10}\) < \(11 \frac{5}{7} \div \frac{5}{8}\)

Explanation:
55 \(\frac{5}{6}\) = 335/6 = 55.833
6 \(\frac{7}{10}\) = 67/10 = 6.7
55.833 is closer to 56
6.7 is closer to 7
56/7 = 8
11 \(\frac{5}{7}\) = 82/7 = 11.714
\(\frac{5}{8}\) = 0.625
11.714 is closer to 12
0.625 is closer to 1
12/1 = 12
8 < 12

Question 13.
Marion is making school flags. Each flag uses 2 \(\frac{3}{4}\) yards of felt. Marion has 24 \(\frac{1}{8}\) yards of felt. About how many flags can he make?
About _______ flags

Answer:
About 8 flags

Explanation:
Marion is making school flags. Each flag uses 2 \(\frac{3}{4}\) yards of felt. Marion has 24 \(\frac{1}{8}\) yards of felt.
2 \(\frac{3}{4}\) = 11/4
24 \(\frac{1}{8}\) = 193/8
193/8 ÷ 11/4
193/8 x 4/11 = 8.77
About 8 flags

Question 14.
A garden snail travels about 2 \(\frac{3}{5}\) feet in 1 minute. At that speed, about how many hours would it take the snail to travel 350 feet?
About _______ hours

Answer:
About 2 hours

Explanation:
2 \(\frac{3}{5}\) = 2.6
That’s how long he travels in one minute. There are 60 minutes in an hour so multiply it by 60 and see if that gets you close to 350.
60 x 2.6 = 156
Now let’s add one more hour.
156 + 156 = 312
14 x 2.6 = 36.4
312 + 36.4 = 348.4
348.4 + 2.6 = 351
So two hours and fourteen minutes

Problem Solving + Applications – Page No. 104

What’s the Error?

Question 15.
Megan is making pennants from a piece of butcher paper that is 10 \(\frac{3}{8}\) yards long. Each pennant requires \(\frac{3}{8}\) yard of paper. To estimate the number of pennants she could make, Megan estimated the quotient 10 \(\frac{3}{8}\) ÷ \(\frac{3}{8}\).
Look at how Megan solved the problem. Find her error
Estimate:
10 \(\frac{3}{8}\) ÷ \(\frac{3}{8}\)
10 ÷ \(\frac{1}{2}\) = 5
Correct the error. Estimate the quotient.
So, Megan can make about _____ pennants.
Describe the error that Megan made
Explain Tell which compatible numbers you used to estimate 10 \(\frac{3}{8}\) ÷ \(\frac{3}{8}\). Explain why you chose those numbers.
Type below:
__________

Answer:
10 \(\frac{3}{8}\) ÷ \(\frac{3}{8}\)
10 \(\frac{3}{8}\) = 83/8 = 10.375
\(\frac{3}{8}\) = 0.375
She had written 10 ÷ \(\frac{1}{2}\) = 5
10.375 is closer to 10
0.375 is closer to 0.5
10/0.5 = 20
But she has written 5 instead of 20.
Megan can make about 20 pennants.

For numbers 16a–16c, estimate to compare. Choose <, >, or =.

Question 16.
16a. 18 \(\frac{3}{10} \div 2 \frac{5}{6}\) ? \(30 \frac{7}{9} \div 3 \frac{1}{3}\)
_____

Answer:
16a. 18 \(\frac{3}{10} \div 2 \frac{5}{6}\) < \(30 \frac{7}{9} \div 3 \frac{1}{3}\)

Explanation:
18 \(\frac{3}{10}\) = 183/10 = 18.3
2 \(\frac{5}{6}\) = 17/6 = 2.833
18.3 is closer to 18
2.833 is closer to 3
18/3 = 6
30 \(\frac{7}{9}\) = 277/9 = 30.777
3 \(\frac{1}{3}\) = 10/3 = 3.333
30.777 is closer to 30
3.333 is closer to 3
30/3 = 10
6 < 10

Question 16.
16b. 17 \(\frac{4}{5} \div 6 \frac{1}{6}\) ? \(19 \frac{8}{9} \div 4 \frac{5}{8}\)
_____

Answer:
17 \(\frac{4}{5} \div 6 \frac{1}{6}\) < \(19 \frac{8}{9} \div 4 \frac{5}{8}\)

Explanation:
17 \(\frac{4}{5}\) = 89/5 = 17.8
6 \(\frac{1}{6}\) = 37/6 = 6.1666
17.8 is closer to 18
6.1666 is closer to 6
18/6 = 3
19 \(\frac{8}{9}\) = 179/9 = 19.888
4 \(\frac{5}{8}\) = 37/8 = 4.625
19.888 is closer to 20
4.625 is closer to 5
20/5 = 4
3 < 4
17 \(\frac{4}{5} \div 6 \frac{1}{6}\) < \(19 \frac{8}{9} \div 4 \frac{5}{8}\)

Question 16.
16c. 17 \(\frac{5}{6} \div 6 \frac{1}{4}\) ? \(11 \frac{5}{7} \div 2 \frac{3}{4}\)
_____

Answer:
17 \(\frac{5}{6} \div 6 \frac{1}{4}\) < \(11 \frac{5}{7} \div 2 \frac{3}{4}\)

Explanation:
17 \(\frac{5}{6}\) = 107/6 = 17.833
6 \(\frac{1}{4}\) = 25/4 = 6.25
17.833 is closer to 18
6.25 is closer to 6
18/6 = 3
11 \(\frac{5}{7}\) = 82/7 = 11.714
2 \(\frac{3}{4}\) = 11/4 = 2.75
11.714 is closer to 12
2.75 is closer to 3
12/3 = 4
3 < 4
17 \(\frac{5}{6} \div 6 \frac{1}{4}\) < \(11 \frac{5}{7} \div 2 \frac{3}{4}\)

Estimate Quotients – Page No. 105

Estimate using compatible numbers.

Question 1.
\(12 \frac{3}{16} \div 3 \frac{9}{10}\)
______

Answer:
3

Explanation:
12 \(\frac{3}{16}\) = 195/16 = 12.1875
3 \(\frac{9}{10}\) = 39/10 = 3.9
12.1875 is closer to 12
3.9 is closer to 4
12/4 = 3

Question 2.
\(15 \frac{3}{8} \div \frac{1}{2}\)
______

Answer:
30

Explanation:
15 \(\frac{3}{8}\) = 123/8 = 15.375
\(\frac{1}{2}\) = 0.5
15.375 is closer to 15
0.5 is closer to 0.5
15/0.5 = 30

Question 3.
\(22 \frac{1}{5} \div 1 \frac{5}{6}\)
______

Answer:
11

Explanation:
22 \(\frac{1}{5}\) = 111/5 = 22.2
1 \(\frac{5}{6}\) = 11/6 = 1.8333
22.2 is closer to 22
1.8333 is closer to 2
22/2 = 11

Question 4.
\(7 \frac{7}{9} \div \frac{4}{7}\)
______

Answer:
16

Explanation:
7 \(\frac{7}{9}\) = 70/9 = 7.777
\(\frac{4}{7}\) = 0.571
7.777 is closer to 8
0.571 is closer to 0.5
8/0.5 = 16

Question 5.
\(18 \frac{1}{4} \div 2 \frac{4}{5}\)
______

Answer:
6

Explanation:
18 \(\frac{1}{4}\) = 73/4 = 18.25
2 \(\frac{4}{5}\) = 14/5 = 2.8
18.25 is closer to 18
2.8 is closer to 3
18/3 = 6

Question 6.
\(\frac{15}{16} \div \frac{1}{7}\)
______

Answer:
10

Explanation:
\(\frac{15}{16}\) = 0.9375
\(\frac{1}{7}\) = 0.1428
0.9375 is closer to 1
0.1428 is closer to 0.1
1/0.1 = 10

Question 7.
\(14 \frac{7}{8} \div \frac{5}{11}\)
______

Answer:
30

Explanation:
14 \(\frac{7}{8}\) = 119/8 = 14.875
\(\frac{5}{11}\) = 0.4545
14.875 is closer to 15
0.4545 is closer to 0.5
15/0.5 = 30

Question 8.
\(53 \frac{7}{12} \div 8 \frac{11}{12}\)
______

Answer:
6

Explanation:
53 \(\frac{7}{12}\) = 643/12 = 53.58
8 \(\frac{11}{12}\) = 107/12 = 8.916
53.58 is closer to 54
8.916 is closer to 9
54/9 = 6

Question 9.
\(1 \frac{1}{6} \div \frac{1}{9}\)
______

Answer:
10

Explanation:
1 \(\frac{1}{6}\) = 7/6 = 1.166
\(\frac{1}{9}\) = 0.111
1.166 is closer to 1
0.111 is closer to 0.1
1/0.1 = 10

Problem Solving

Question 10.
Estimate the number of pieces Sharon will have if she divides 15 \(\frac{1}{3}\) yards of fabric into 4 \(\frac{4}{5}\) yard lengths.
About ______ pieces

Answer:
About 3 pieces

Explanation:
Sharon will have if she divides 15 \(\frac{1}{3}\) yards of fabric into 4 \(\frac{4}{5}\) yard lengths.
3 7/36 is the answer.
So, about 3 pieces

Question 11.
Estimate the number of \(\frac{1}{2}\) quart containers Ethan can fill from a container with 8 \(\frac{7}{8}\) quarts of water.
About ______ containers

Answer:
About 18 containers

Question 12.
How is estimating quotients different from estimating products?
Type below:
__________

Answer:
To estimate products and quotients, you need to first round the numbers. To round to the nearest whole number, look at the digit in the tenths place. If it is less than 5, round down. If it is 5 or greater, round up. Remember that an estimate is an answer that is not exact, but is approximate and reasonable.
Let’s look at an example of estimating a product.
Estimate the product: 11.256×6.81
First, round the first number. Since there is a 2 in the tenths place, 11.256 rounds down to 11.
Next, round the second number. Since there is an 8 in the tenths place, 6.81 rounds up to 7.
Then, multiply the rounded numbers. 11×7=77
The answer is 77.
Let’s look at an example of estimating a quotient.
Estimate the quotient: 91.93÷4.39
First, round the first number. Since there is a 9 in the tenths place, 91.93 rounds up to 92.
Next, round the second number. Since there is a 3 in the tenths place, 4.39 rounds down to 4.
Then, divide the rounded numbers.
92÷4=23
The answer is 23.

Lesson Check – Page No. 106

Question 1.
Each loaf of pumpkin bread calls for 1 \(\frac{3}{4}\) cups of raisins. About how many loaves can be made from 10 cups of raisins?
About ______ loaves

Answer:
About 5 loaves

Explanation:
Divide 10 by 1 3/4.
The answer is 5.714285
So you can make about 5 loaves of bread with 10 cups of raisins if each loaf needs 1 3/4 cups of raisins.

Question 2.
Perry’s goal is to run 2 \(\frac{1}{4}\) miles each day. One lap around the school track is \(\frac{1}{3}\) mile. About how many laps must he run to reach his goal?
About ______ laps

Answer:
About 9 laps

Explanation:
Perry’s goal is to run 2 \(\frac{1}{4}\) miles each day. One lap around the school track is \(\frac{1}{3}\) mile.
2 \(\frac{1}{4}\) = 9/4 = 2.25
\(\frac{1}{3}\) = 0.333
Perry will have to run 9 laps to reach his goal.

Spiral Review

Question 3.
A recipe calls for \(\frac{3}{4}\) teaspoon of red pepper. Uri wants to use \(\frac{1}{3}\) of that amount. How much red pepper should he use?
\(\frac{□}{□}\) teaspoon

Answer:
\(\frac{1}{4}\) teaspoon

Explanation:
A recipe calls for \(\frac{3}{4}\) teaspoon of red pepper. Uri wants to use \(\frac{1}{3}\) of that amount.
\(\frac{1}{3}\) of \(\frac{3}{4}\) = \(\frac{1}{4}\)

Question 4.
A recipe calls for 2 \(\frac{2}{3}\) cups of apple slices. Zoe wants to use 1 \(\frac{1}{2}\) times this amount. How many cups of apples should Zoe use?
______ cups

Answer:
4 cups

Explanation:
A recipe calls for 2 2/3 cups of apple slices.
Zoe wants to use 1 1/2 times this amount.
We will multiply the number of apple slices to 1 1/2
2 2/3 X 1 1/2
8/3 X3/2 = 24/6 = 4 cups
Zoe will use 4 cups of apple slices.

Question 5.
Edgar has 2.8 meters of rope. If he cuts it into 7 equal parts, how long will each piece be?
______ meters

Answer:
0.4 meters

Explanation:
2.8/7 = 0.4 meters

Question 6.
Kami has 7 liters of water to fill water bottles that each hold 2.8 liters. How many bottles can she fill?
______ bottles

Answer:
2 bottles

Explanation:
7/2.8 = 2.5
she can only fill 2 because anything over that would 8.4 liters of water

Share and Show – Page No. 109

Estimate. Then find the quotient.

Question 1.
\(\frac{5}{6}\) ÷ 3
\(\frac{□}{□}\)

Answer:
\(\frac{3}{10}\)

Explanation:
5/6 = 0.8333 is closer to 0.9
0.9/3 = 0.3 = 3/10

Use a number line to find the quotient.

Question 2.
\(\frac{3}{4} \div \frac{1}{8}\)
_______

Answer:
grade 6 chapter 2 image 7

Explanation:
3/4 x 8 = 3 x 2 = 6

Question 3.
\(\frac{3}{5} \div \frac{3}{10}\)
_______

Answer:

Explanation:
3/5 x 10/3 = 2

Estimate. Then write the quotient in simplest form.

Question 4.
\(\frac{3}{4} \div \frac{5}{6}\)
\(\frac{□}{□}\)

Answer:
\(\frac{1}{1}\)

Explanation:
3/4 = 0.75 is closer to 0.8
5/6 = 0.8333 is closer to 0.8
0.8/0.8 = 1

Question 5.
\(3 \div \frac{3}{4}\)
_______

Answer:
4

Explanation:
3/4 = 0.75
3/0.75 = 4

Question 6.
\(\frac{1}{2} \div \frac{3}{4}\)
\(\frac{□}{□}\)

Answer:
\(\frac{625}{1000}\)

Explanation:
1/2 = 0.5
3/4 = 0.75 is closer to 0.8
0.5/0.8 = 0.625 = 625/1000

Question 7.
\(\frac{5}{12} \div 3\)
\(\frac{□}{□}\)

Answer:
\(\frac{2}{10}\)

Explanation:
5/12 = 0.4166 is closer to 0.6
0.6/3 = 0.2 = 2/10

On Your Own

Practice: Copy and Solve Estimate. Then write the quotient in simplest form

Question 8.
\(2 \div \frac{1}{8}\)
_______

Answer:
20

Explanation:
1/8 = 0.125 is closer to 0.1
2/0.1 = 20

Question 9.
\(\frac{3}{4} \div \frac{3}{5}\)
\(\frac{□}{□}\)

Answer:
\(\frac{1}{1}\)

Explanation:
3/4 = 0.75 is closer to 0.8
3/5 = is 0.6 closer to 0.8
0.8/0.8 = 1

Question 10.
\(\frac{2}{5} \div 5\)
\(\frac{□}{□}\)

Answer:
\(\frac{1}{10}\)

Explanation:
2/5 = 0.4 is closer to 0.5
0.5/5 = 0.1 = 1/10

Question 11.
\(4 \div \frac{1}{7}\)
_______

Answer:
40

Explanation:
1/7 = 0.1428 is closer to 0.1
4/0.1 = 40

Practice: Copy and Solve Evaluate using the order of operations.

Write the answer in simplest form.

Question 12.
\(\left(\frac{3}{5}+\frac{1}{10}\right) \div 2\)
\(\frac{□}{□}\)

Answer:
\(\frac{7}{20}\)

Explanation:
3/5 + 1/10 = 7/10 = 0.7
0.7/2 = 7/20

Question 13.
\(\frac{3}{5}+\frac{1}{10} \div 2\)
\(\frac{□}{□}\)

Answer:
\(\frac{13}{20}\)

Explanation:
\(\frac{3}{5}+\frac{1}{10} \div 2\)
(1/10)/2 = 1/20
3/5 + 1/20 = 0.65 = 13/20

Question 14.
\(\frac{3}{5}+2 \div \frac{1}{10}\)
_______ \(\frac{□}{□}\)

Answer:

Explanation:
2/(1/10) = 1/5
3/5 + 1/5 = 4/5

Question 15.
Generalize Suppose the divisor and the dividend of a division problem are both fractions between 0 and 1, and the divisor is greater than the dividend. Is the quotient less than, equal to, or greater than 1?
Type below:
__________

Answer:
Divisor and Dividend are fractions lying between 0 and 1
Also, Divisor > Dividend
A smaller number is being divided by a larger number
Whenever a smaller number is divided by a larger number, the quotient is less than 1
Example:
0,5/0,6 Here, they are both numbers between 0 and 1, and the divisor is greater than the dividend.
The result is 0,8333, LESS THAN 1
Hence, the answer is that the quotient will be less than 1

Problem Solving + Applications – Page No. 110

Use the table for 16–19.
Go Math Grade 6 Answer Key Chapter 2 Fractions and Decimals 16

Question 16.
Kristen wants to cut ladder rungs from a 6 ft board. How many ladder rungs can she cut?
_______ ladder rungs

Answer:
8 ladder rungs

Explanation:
Kristen wants to cut ladder rungs from a 6 ft board.
ladder rungs = 3/4 ft
6/(3/4) = 8 rungs

Question 17.
Pose a Problem Look back at Problem 16. Write and solve a new problem by changing the length of the board Kristen is cutting for ladder rungs.
Type below:
__________

Answer:
Kristen wants to cut ladder rungs from a 9 ft board. How many ladder rungs can she cut?
Kristen wants to cut ladder rungs from a 9 ft board.
ladder rungs = 3/4 ft
9/(3/4) = 12 rungs

Question 18.
Dan paints a design that has 8 equal parts along the entire length of the windowsill. How long is each part of the design?
\(\frac{□}{□}\) yards

Answer:
\(\frac{1}{16}\) yards

Explanation:
Dan paints a design that has 8 equal parts along the entire length of the windowsill.
(1/2)/8 = 1/2 x 1/8 = 1/16 yards

Question 19.
Dan has a board that is \(\frac{15}{16}\) yd. How many “Keep Out” signs can he make if the length of the sign is changed to half of the original length?
_______ signs

Answer:
3 signs

Explanation:
Dan has a board that is \(\frac{15}{16}\) yd.
If the length of the sign is changed to half of the original length, (5/8)/2 = 5/16
(15/16) ÷ 5/16 = 15/16 x 16/5 = 3

Question 20.
Lauren has \(\frac{3}{4}\) cup of dried fruit. She puts the dried fruit into bags, each holding \(\frac{1}{8}\) cup. How many bags will Lauren use? Explain your answer using words and numbers.
Type below:
__________

Answer:
6

Explanation:
Lauren has \(\frac{3}{4}\) cup of dried fruit. She puts the dried fruit into bags, each holding \(\frac{1}{8}\) cup.
3/4 ÷ 1/8 = 3/4 x 8 = 6
Lauren has 3/4 and in 1/4 there are 2 1/8s. That 3 fourths times two = 6 so 6 one eights

Divide Fractions – Page No. 111

Estimate. Then write the quotient in simplest form.

Question 1.
\(5 \div \frac{1}{6}\)
_____

Answer:
25

Explanation:
1/6 = 0.166 is closer to 0.2
5/0.2 = 25

Question 2.
\(\frac{1}{2} \div \frac{1}{4}\)
_____

Answer:
5

Explanation:
1/2 = 0.5 is closer to 1
1/4 = 0.25 is closer to 0.2
1/0.2 = 5

Question 3.
\(\frac{4}{5} \div \frac{2}{3}\)
_____ \(\frac{□}{□}\)

Answer:
1 \(\frac{1}{5}\)

Explanation:
4/5 = 0.8 is closer to 0.8
2/3 = 0.66 is closer to 0.6
0.8/0.6 = 1 1/5

Question 4.
\(\frac{14}{15} \div 7\)
\(\frac{□}{□}\)

Answer:
\(\frac{2}{15}\)

Explanation:
14/15 = 0.9333
0.9/7 = 2/15

Question 5.
\(8 \div \frac{1}{3}\)
_____

Answer:
20

Explanation:
1/3 = 0.33 is closer to 0.4
8/0.4 = 20

Question 6.
\(\frac{12}{21} \div \frac{2}{3}\)
\(\frac{□}{□}\)

Answer:
\(\frac{1}{1}\)

Explanation:
12/21 = 0.571 is closer to 0.6
2/3 = 0.666 is closer to 0.6
0.6/0.6 = 1

Question 7.
\(\frac{5}{6} \div \frac{5}{12}\)
_____

Answer:
2

Explanation:
5/6 = 0.833 is closer to 0.8
5/12 = 0.416 is closer to 0.4
0.8/0.4 = 2

Question 8.
\(\frac{5}{8} \div \frac{1}{2}\)
_____ \(\frac{□}{□}\)

Answer:
1 \(\frac{2}{10}\)

Explanation:
5/8 = 0.625 is closer to 0.6
1/2 = 0.5 is closer to 0.5
0.6/0.5 = 1.2 = 1 2/10

Question 9.
Joy ate \(\frac{1}{4}\) of a pizza. If she divides the rest of the pizza into pieces equal to \(\frac{1}{8}\) pizza for her family, how many pieces will her family get?
_____ pieces

Answer:
6 pieces

Explanation:
The pizza is divided into 4 pieces, Joy ate 1/4.
So, the left pices are 1 – 1/4 = 3/4
now, 3/4 of a pizza and Joy will divide this rest of the pizza in pieces equal to 1/8, so we need to make a division
(3/4) ÷ (1/8) = 24/4 = 6 pieces.

Question 10.
Hideko has \(\frac{3}{5}\) yard of ribbon to tie on balloons for the festival. Each balloon will need \(\frac{3}{10}\) yard of ribbon. How many balloons can Hideko tie with ribbon?
_____ balloons

Answer:
2 balloons

Explanation:
3/10 yard of ribbon required to tie = 1 balloon
3/5 yard of ribber can tie = (3/5) ÷ (3/10) = 2 ballons
With 3/5 yard, Hideko can tie 2 balloons

Problem Solving

Question 11.
Rick knows that 1 cup of glue weighs \(\frac{1}{18}\) pound. He has \(\frac{2}{3}\) pound of glue. How many cups of glue does he have?
_____ cups

Answer:
12 cups

Explanation:
For 1/18lb, 1 cup
For 2/3lb, x cups.
1/8x = 1 x 2/3
1/8x = 2/3
x = 2/3 x 18
x = 2 x 6 = 12 cups

Question 12.
Mrs. Jennings had \(\frac{5}{7}\) gallon of paint. She gave \(\frac{1}{7}\) gallon each to some students. How many students received paint if Mrs. Jennings gave away all the paint?
_____ students

Answer:
4 students

Explanation:
Mrs. Jennings had \(\frac{5}{7}\) gallon of paint. She gave \(\frac{1}{7}\) gallon each to some students.
\(\frac{5}{7}\) ÷ \(\frac{1}{7}\) = 25/7 = 3.571 is closer to 4

Question 13.
Write a word problem that involves two fractions. Include the solution.
Type below:
__________

Answer:
Mrs. Jennings had \(\frac{5}{7}\) gallon of paint. She gave \(\frac{1}{7}\) gallon each to some students. How many students received paint if Mrs. Jennings gave away all the paint?
Answer:
Mrs. Jennings had \(\frac{5}{7}\) gallon of paint. She gave \(\frac{1}{7}\) gallon each to some students.
\(\frac{5}{7}\) ÷ \(\frac{1}{7}\) = 25/7 = 3.571 is closer to 4

Lesson Check – Page No. 112

Question 1.
There was \(\frac{2}{3}\) of a pizza for 6 friends to share equally. What fraction of the pizza did each person get?
\(\frac{□}{□}\)

Answer:
\(\frac{1}{9}\)

Explanation:
There was \(\frac{2}{3}\) of a pizza for 6 friends to share equally.
\(\frac{2}{3}\) ÷ 6 = 2/3 x 1/6 = 2/18 = 1/9

Question 2.
Rashad needs \(\frac{2}{3}\) pound of wax to make a candle. How many candles can he make with 6 pounds of wax?
_____ candles

Answer:
9 candles

Explanation:
Rashad needs 2/3 pound a wax to make candles.
1 Candle = 2/3 pounds.
So, for 2 pounds,
3 x 2/3 = 3 candles
2 pounds = 3 candles
1 pound = 3/2 candles
So, for 6 pounds,
6 x 3/2 = 9 candles

Spiral Review

Question 3.
Jeremy had \(\frac{3}{4}\) of a submarine sandwich and gave his friend \(\frac{1}{3}\) of it. What fraction of the sandwich did the friend receive?
\(\frac{□}{□}\)

Answer:
\(\frac{1}{4}\)

Explanation:
Jeremy had \(\frac{3}{4}\) of a submarine sandwich and gave his friend \(\frac{1}{3}\) of it.
1/3 x 3/4 = 1/4

Question 4.
Ebony walked at a rate of 3 \(\frac{1}{2}\) miles per hour for 1 \(\frac{1}{3}\) hours. How far did she walk?
_____ \(\frac{□}{□}\)

Answer:
4 \(\frac{2}{3}\)

Explanation:
Ebony walked at a rate of 3 \(\frac{1}{2}\) miles per hour for 1 \(\frac{1}{3}\) hours.
3 1/2 miles = 7/2 miles … 1 hour
x miles = ? … 1 1/3 hours = 4/3 hours
7/2 x 4/3 = 1 x x
x = 7/2 x 4/3
x = 14/3 = 4 2/3 miles
The correct result would be 4 2/3 miles.

Question 5.
Penny uses \(\frac{3}{4}\) yard of fabric for each pillow she makes. How many pillows can she make using 6 yards of fabric?
_____ pillows

Answer:
8 pillows

Explanation:
Penny uses \(\frac{3}{4}\) yard of fabric for each pillow she makes.
Using 6 yards of fabric 6/(3/4) = 24/3 = 8

Question 6.
During track practice, Chris ran 2.5 laps in 81 seconds. What was his average time per lap?
_____ seconds

Answer:
32.4 seconds

Explanation:
During track practice, Chris ran 2.5 laps in 81 seconds.
81/2.5 = 32.4 seconds

Share and Show – Page No. 115

Use the model to find the quotient.

Question 1.
\(3 \frac{1}{3} \div \frac{1}{3}\)
Go Math Grade 6 Answer Key Chapter 2 Fractions and Decimals 17
_____

Answer:
21

Explanation:
Model 3 with 3 hexagonal blocks.
Model 1/2 with 1 trapezoid block.
For 1/6,
6 triangle blocks are equal to 1 hexagon.
So, a triangle block shows 1/6.
Count the triangles.
There are 21 triangle blocks.
So, 3 1/2 ÷ 1/6 = 21.

Question 2.
\(2 \frac{1}{2} \div \frac{1}{6}\)
Go Math Grade 6 Answer Key Chapter 2 Fractions and Decimals 18
_____

Answer:
15

Explanation:
Model 2 with 2 hexagonal blocks.
Model 1/2 with 1 trapezoid block.
For 1/6,
6 triangle blocks are equal to 1 hexagon.
So, a triangle block shows 1/6.
Count the triangles.
There are 15 triangle blocks.
So, \(2 \frac{1}{2} \div \frac{1}{6}\) = 15.

Use pattern blocks to find the quotient. Then draw the model.

Question 3.
\(2 \frac{2}{3} \div \frac{1}{6}\)
_____

Answer:
grade 6 chapter 2 image 1

Explanation:
2 2/3 = 8/3
8/3 ÷ 1/6 = 16

Question 4.
\(3 \frac{1}{2} \div \frac{1}{2}\)
_____

Answer:
grade 6 chapter 2 image 2

Explanation:
3 1/2 = 7/2
7/2 ÷ 1/2 = 7

Draw a model to find the quotient.

Question 5.
\(3 \frac{1}{2} \div 3\)
_____ \(\frac{□}{□}\)

Answer:
grade 6 chapter 2 image 3

Explanation:
3 1/2 = 7/2
7/2 ÷ 3 = 21/2

Question 6.
\(1 \frac{1}{4} \div 2\)
\(\frac{□}{□}\)

Answer:
grade 6 chapter 2 image 4

Explanation:
1/4 ÷ 2 = 1/2

Question 7.
Use Appropriate Tools Explain how models can be used to divide mixed numbers by fractions or whole numbers
Type below:
__________

Answer:
Multiply the whole number part by the fraction’s denominator. Add that to the numerator. Then write the result on top of the denominator.

Problem Solving + Applications – Page No. 116

Use a model to solve. Then write an equation for the model.

Question 8.
Use Models Eliza opens a box of bead kits. The box weighs 2 \(\frac{2}{3}\) lb. Each bead kit weighs \(\frac{1}{6}\) lb. How many kits are in the box? What does the answer mean?
Type below:
__________

Answer:
grade 6 chapter 2 image 6
16 kits are in the box

Explanation:
Eliza opens a box of bead kits. The box weighs 2 \(\frac{2}{3}\) lb. Each bead kit weighs \(\frac{1}{6}\) lb, 2 \(\frac{2}{3}\) ÷ \(\frac{1}{6}\) = 8/3 ÷ 1/6 = 16.
16 kits are in the box

Question 9.
Hassan has two boxes of trail mix. Each box holds 1 \(\frac{2}{3}\) lb of trail mix. He eats \(\frac{1}{3}\) lb of trail mix each day. How many days can Hassan eat trail mix before he runs out?
_____ days

Answer:
10 days

Explanation:
Hassan has two boxes of trail mix. Each box holds 1 \(\frac{2}{3}\) lb of trail mix.
1 \(\frac{2}{3}\) = 5/3
2 x (5/3) = 10/3
He eats \(\frac{1}{3}\) lb of trail mix each day.
10/3 ÷ 1/3 = 10
Hassan eats trail mix for 10 days before he runs out.

Question 10.
Sense or Nonsense? Steve made this model to show \(2 \frac{1}{3} \div \frac{1}{6}\). He says that the quotient is 7. Is his answer sense or nonsense? Explain your reasoning
Go Math Grade 6 Answer Key Chapter 2 Fractions and Decimals 19
Type below:
__________

Answer:
\(2 \frac{1}{3} \div \frac{1}{6}\) = 7/3 ÷ 1/6 = 14.
He said the quotient is 7.
His answer is Nonsense.

Question 11.
Eva is making muffins to sell at a fundraiser. She has 2 \(\frac{1}{4}\) cups of flour, and the recipe calls for \(\frac{3}{4}\) cup of flour for each batch of muffins. Explain how to use a model to find the number of batches of muffins Eva can make.
Type below:
__________

Answer:
3

Explanation:
Eva is making muffins to sell at a fundraiser. She has 2 \(\frac{1}{4}\) cups of flour, and the recipe calls for \(\frac{3}{4}\) cup of flour for each batch of muffins.
2 \(\frac{1}{4}\) ÷ \(\frac{3}{4}\) = 9/4 ÷ 3/4 = 3

Model Mixed Number Division – Page No. 117

Use the model to find the quotient.

Question 1.
\(4 \frac{1}{2} \div \frac{1}{2}\)
Go Math Grade 6 Answer Key Chapter 2 Fractions and Decimals 20
_____

Answer:
9

Explanation:
Count the number of trapezoids to find the answer.

Question 2.
\(3 \frac{1}{3} \div \frac{1}{6}\)
Go Math Grade 6 Answer Key Chapter 2 Fractions and Decimals 21
_____

Answer:
20

Use pattern blocks or another model to find the quotient. Then draw the model.

Question 3.
\(2 \frac{1}{2} \div \frac{1}{6}\)
_____

Answer:
grade 6 chapter 2 image 5

Explanation:
Model 2 with 2 hexagonal blocks.
Model 1/2 with 1 trapezoid block.
For 1/6,
6 triangle blocks are equal to 1 hexagon.
So, a triangle block shows 1/6.
Count the triangles.
There are 15 triangle blocks.
So, 212÷16 = 15.

Question 4.
\(2 \frac{3}{4} \div 2\)
_____

Answer:
grade 6 chapter 2 image 6

Explanation:
2 3/4 ÷ 2 = 11/2

Problem Solving

Question 5.
Marty has 2 \(\frac{4}{5}\) quarts of juice. He pours the same amount of juice into 2 bottles. How much does he pour into each bottle?
_____ \(\frac{□}{□}\) quarts

Answer:
1\(\frac{2}{5}\) quarts

Explanation:
Marty has 2 \(\frac{4}{5}\) quarts of juice. He pours the same amount of juice into 2 bottles.
2 \(\frac{4}{5}\) = 14/5 = 2.8
2.8/2 = 1.4 = 1 2/5

Question 6.
How many \(\frac{1}{3}\) pound servings are in 4 \(\frac{2}{3}\) pounds of cheese?
_____ pounds

Answer:
14 pounds

Explanation:
4 2/3 = 14/3
(14/3)/(1/3) = 14

Question 7.
Write a word problem that involves dividing a mixed number by a whole number. Solve the problem and describe how you found the answer.
Type below:
__________

Answer:
How many \(\frac{1}{3}\) pound servings are in 4 \(\frac{2}{3}\) pounds of cheese?
Explanation:
4 2/3 = 14/3
(14/3)/(1/3) = 14

Lesson Check – Page No. 118

Sketch a model to find the quotient.

Question 1.
Emma has 4 \(\frac{1}{2}\) pounds of birdseed. She wants to divide it evenly among 3 bird feeders. How much birdseed should she put in each?
_____ \(\frac{□}{□}\) pounds

Answer:
1\(\frac{1}{2}\) pounds

Explanation:
Emma has 4 1/2 pounds of birdseed.
Convert this to an improper fraction.
4 1/2 = 9/2
Emma wants to divide it evenly among 3 bird feeders.
So, she should put (9/2)/3 = 3/2 = 1 1/2

Question 2.
A box of crackers weighs 11 \(\frac{1}{4}\) ounces. Kaden estimates that one serving is \(\frac{3}{4}\) ounce. How many servings are in the box?
_____ servings

Answer:
15 servings

Explanation:
11 1/4 by 3/4
11 1/4 = 45/4
45/4 / 3/4 = 45/4 × 4/3 = 180/12 = 15
there are 15 servings

Spiral Review

Question 3.
The Ecology Club has volunteered to clean up 4.8 kilometers of highway. The members are organized into 16 teams. Each team will clean the same amount of highway. How much highway will each team clean?
_____ kilometers

Answer:
0.3 kilometers

Explanation:
The Ecology Club has volunteered to clean up 4.8 kilometers of highway. The members are organized into 16 teams.
The total length of the highway is given to clean = 4.8 kilometers
If the members are organized into 16 teams.
4.8/16 = 0.3
Hence, each team will clean 0.3 kilometers of the highway.

Question 4.
Tyrone has $8.06. How many bagels can he buy if each bagel costs $0.65?
_____ bagels

Answer:
12 bagels

Explanation:
$8.06/$0.65 = 12.4
12 bagels

Question 5.
A nail is 0.1875 inch thick. What is its thickness as a fraction? Is 0.1875 inch closer to \(\frac{1}{8}\) inch or \(\frac{1}{4}\) inch on a number line?
Type below:
__________

Answer:
0.1875 = 3/16 which is at the same distance to 1/4 and 1/8
It is the same distance apart.

Question 6.
Maria wants to find the product of 5 \(\frac{3}{20}\) × 3 \(\frac{4}{25}\) using decimals instead of fractions. How can she rewrite the problem using decimals?
Type below:
__________

Answer:
16.274

Explanation:
The decimal for 5 3/20 is 5.15
The decimal for 3 4/25 is 3.16
5.15 × 3.16 = 16.274

Share and Show – Page No. 121

Estimate. Then write the quotient in simplest form.

Question 1.
\(4 \frac{1}{3} \div \frac{3}{4}\)
______ \(\frac{□}{□}\)

Answer:
5\(\frac{375}{1000}\)

Explanation:
4 1/3 = 13/3 = 4.333 is closer to 4.3
3/4 = 0.75 is closer to 0.8
4.3/0.8 = 5.375 = 5 375/1000

Question 2.
Six hikers shared 4 \(\frac{1}{2}\) lb of trail mix. How much trail mix did each hiker receive?
\(\frac{□}{□}\)

Answer:
\(\frac{75}{100}\)

Explanation:
6 hikers = 4.5 lbs of trail mix
4.5/6= .75 lbs each hiker.

Question 3.
\(5 \frac{2}{3} \div 3\)
______ \(\frac{□}{□}\)

Answer:
2\(\frac{947}{1000}\)

Explanation:
5 2/3 = 17/3 = 5.666 is closer to 5.6
5.6/3 = 1.866 is closer to 1.9
5.6/1.9 = 2.947 = 2 947/1000

Question 4.
\(7 \frac{1}{2} \div 2 \frac{1}{2}\)
______

Answer:
3

Explanation:
7 1/2 = 15/2 = 7.5
2 1/2 = 5/2 = 2.5
7.5/2.5 = 3

On Your Own

Estimate. Then write the quotient in simplest form.

Question 5.
\(5 \frac{3}{4} \div 4 \frac{1}{2}\)
______ \(\frac{□}{□}\)

Answer:
1\(\frac{27}{100}\)

Explanation:
5 3/4 = 23/4 = 5.75
4 1/2 = 9/2 = 4.5
5.75/4.5 = 1.27 = 1 27/100

Question 6.
\(5 \div 1 \frac{1}{3}\)
______ \(\frac{□}{□}\)

Answer:
3\(\frac{84}{100}\)

Explanation:
1 1/3 = 4/3 = 1.33 is closer to 1.3
5/1.3 = 3.84 = 3 84/100

Question 7.
\(6 \frac{3}{4} \div 2\)
______ \(\frac{□}{□}\)

Answer:
3\(\frac{2}{5}\)

Explanation:
6 3/4 = 27/4 = 6.75 is closer to 6.8
6.8/2 = 3.4 = 3 2/5

Question 8.
\(2 \frac{2}{9} \div 1 \frac{3}{7}\)
______ \(\frac{□}{□}\)

Answer:
1\(\frac{571}{1000}\)

Explanation:
2 2/9 = 20/9 = 2.22 is closer to 2.2
1 3/7 = 10/7 = 1.428 is closer to 1.4
2.2/1.4 = 1.571 = 1 571/1000

Question 9.
How many 3 \(\frac{1}{3}\) yd pieces can Amanda get from a 3 \(\frac{1}{3}\) yd ribbon?
______

Answer:
1

Explanation:
(3 1/3) ÷ (3 1/3) = 1

Question 10.
Samantha cut 6 \(\frac{3}{4}\) yd of yarn into 3 equal pieces. Explain how she could use mental math to find the length of each piece
Type below:
__________

Answer:
27/12

Explanation:
Samantha cut 6 \(\frac{3}{4}\) yd of yarn into 3 equal pieces.
6 3/4 = 27/4
(27/4)/3
(27/4)(1/3) = 27/12

Evaluate Algebra Evaluate using the order of operations. Write the answer in simplest form.

Question 11.
\(1 \frac{1}{2} \times 2 \div 1 \frac{1}{3}\)
_____ \(\frac{□}{□}\)

Answer:
2\(\frac{1}{4}\)

Explanation:
(1 1/2) × 2 = 3/2 × 2 = 3
1 1/3 = 4/3
3/(4/3) = 9/4 = 2.25 = 2 1/4

Question 12.
\(1 \frac{2}{5} \div 1 \frac{13}{15}+\frac{5}{8}\)
_____ \(\frac{□}{□}\)

Answer:
1\(\frac{3}{8}\)

Explanation:
(1 2/5)/(1 13/15) = (7/5)/(28/15) = 3/4 = 0.75
0.75 + 0.625 = 1.375 = 1 3/8

Question 13.
\(3 \frac{1}{2}-1 \frac{5}{6} \div 1 \frac{2}{9}\)
_____

Answer:
2

Explanation:
(1 5/6)/(1 2/9) = (11/6)/11/9 = 3/2 = 1 1/2 = 1.5
3 1/2 = 7/2 = 3.5
3.5 – 1.5 = 2

Question 14.
Look for a Pattern Find these quotients: \(20 \div 4 \frac{4}{5}\), \(10 \div 4 \frac{4}{5}\), \(5 \div 4 \frac{4}{5}\). Describe a pattern you see.
Type below:
__________

Answer:
20 ÷ 4 4/5 = 20 ÷ 24/5 = 20/4.8 = 4.1666
10 ÷ 4 4/5 = 10 ÷ 24/5 = 10/4.8 = 2.08333
5 ÷ 4 4/5 = 5 ÷ 24/5 = 5/4.8 = 1.04166
The pattern is multiplied by 2 every time.

Page No. 122

Question 15.
Dina hikes \(\frac{1}{2}\) of the easy trail and stops for a break every 3 \(\frac{1}{4}\) miles. How many breaks will she take?
Go Math Grade 6 Answer Key Chapter 2 Fractions and Decimals 22
a. What problem are you asked to solve?
Type below:
__________

Answer:
How many breaks Dina will take when hikes \(\frac{1}{2}\) of the easy trail and stops for a break every 3 \(\frac{1}{4}\) mile.

Question 15.
b. How will you use the information in the table to solve the problem?
Type below:
__________

Answer:
Dina easy trail length, break time

Question 15.
c. How can you find the distance Dina hikes? How far does she hike?
______ \(\frac{□}{□}\) miles

Answer:
9\(\frac{3}{4}\) miles

Explanation:
19 1/2 × 1/2 = 39/2 × 1/2 = 39/4 = 9 3/4

Question 15.
d. What operation will you use to find how many breaks Dina takes?
Type below:
__________

Answer:
Division

Question 15.
e. How many breaks will Dina take?
______ breaks

Answer:
3 breaks

Explanation:
39/4 ÷ 13/4 = 3

Question 16.
Carlo packs 15 \(\frac{3}{4}\) lb of books in 2 boxes. Each book weighs 1 \(\frac{1}{8}\) lb. There are 4 more books in Box A than in Box B. How many books are in Box A? Explain your work.
______ books

Answer:
Carlo packs 15 \(\frac{3}{4}\) lb of books in 2 boxes. Each book weighs 1 \(\frac{1}{8}\) lb.
15 \(\frac{3}{4}\) ÷ 1 \(\frac{1}{8}\) = 63/4 ÷ 9/8 = 14
14 books available in 2 boxes.
There are 4 more books in Box A than in Box B.
Box A contains 5 + 4 = 9 books
Box B contains 5 books

Question 17.
Rex’s goal is to run 13 \(\frac{3}{4}\) miles over 5 days. He wants to run the same distance each day. Jordan said that Rex would have to run 3 \(\frac{3}{4}\) miles each day to reach his goal. Do you agree with Jordan? Explain your answer using words and numbers.
Type below:
__________

Answer:
Rex’s goal is to run 13 \(\frac{3}{4}\) miles over 5 days. He wants to run the same distance each day.
13 \(\frac{3}{4}\) ÷ 5 = 55/4 ÷ 5 = 11/4 or 2 3/4.
Jordan answer is wrong

Divide Mixed Numbers – Page No. 123

Estimate. Then write the quotient in simplest form.

Question 1.
\(2 \frac{1}{2} \div 2 \frac{1}{3}\)
______ \(\frac{□}{□}\)

Answer:
1\(\frac{1}{2}\)

Explanation:
2 1/2 = 5/2 = 2.5 is closer to 3
2 1/3 = 7/3 = 2.333 is closer to 2
3/2 = 1.5 = 1 1/2

Question 2.
\(2 \frac{2}{3} \div 1 \frac{1}{3}\)
______

Answer:
2

Explanation:
2 2/3 = 8/3 = 2.666 is closer to 2.6
1 1/3 = 4/3 = 1.333 is closer to 1.3
2.6/1.3 = 2

Question 3.
\(2 \div 3 \frac{5}{8}\)
\(\frac{□}{□}\)

Answer:
\(\frac{1}{2}\)

Explanation:
3 5/8 = 29/8 = 3.625 is closer to 3.6
2/3.6 = 0.5 = 1/2

Question 4.
\(1 \frac{13}{15} \div 1 \frac{2}{5}\)
\(\frac{□}{□}\)

Answer:
\(\frac{126}{100}\)

Explanation:
1 13/15 = 28/15 = 1.8666 is closer to 1.9
1 2/5 = 7/5 = 1.4 is closer to 1.5
1.9/1.5 = 1.266
126/100

Question 5.
\(10 \div 6 \frac{2}{3}\)
______ \(\frac{□}{□}\)

Answer:
1\(\frac{1}{2}\)

Explanation:
6 2/3 = 20/3 = 6.666 is closer to 6.7
10/6.7 = 3/2 = 1 1/2

Question 6.
\(2 \frac{3}{5} \div 1 \frac{1}{25}\)
______ \(\frac{□}{□}\)

Answer:
2\(\frac{3}{5}\)

Explanation:
2 3/5 = 13/5 = 2.6
1 1/25 = 26/25 = 1.04 is closer to 1
2.6/1 = 13/5 or 2 3/5

Question 7.
\(2 \frac{1}{5} \div 2\)
______ \(\frac{□}{□}\)

Answer:
1\(\frac{1}{10}\)

Explanation:
2 1/5 = 11/5 = 2.2 is closer to 2.2
2.2/2 = 1.1 = 11/10 = 1 1/10

Question 8.
Sid and Jill hiked 4 \(\frac{1}{8}\) miles in the morning and 1 \(\frac{7}{8}\) miles in the afternoon. How many times as far did they hike in the morning as in the afternoon?
______ \(\frac{□}{□}\) times

Answer:
2\(\frac{1}{5}\) times

Explanation:
Sid and Jill hiked 4 \(\frac{1}{8}\) miles in the morning and 1 \(\frac{7}{8}\) miles in the afternoon.
4 \(\frac{1}{8}\) = 33/8
1 \(\frac{7}{8}\) = 15/8
(33/8) ÷ (15/8) = 33/15 = 11/5 or 2 1/5

Problem Solving

Question 9.
It takes Nim 2 \(\frac{2}{3}\) hours to weave a basket. He worked Monday through Friday, 8 hours a day. How many baskets did he make?
______ baskets

Answer:
15 baskets

Explanation:
he worked (Mon – Fri) 5 days at 8 hrs per day = 5 × 8= 40 hrs
40/ (2 2/3) = 40 / (8/3) = 40 × 3/8 = 120/8 = 15 baskets

Question 10.
A tree grows 1 \(\frac{3}{4}\) feet per year. How long will it take the tree to grow from a height of 21 \(\frac{1}{4}\) feet to a height of 37 feet?
______ years

Answer:
9 years

Explanation:
A tree grows 1 3/4 = 7/4 feet per year.
If you would like to know how long will it take the tree to grow from a height of 21 1/4 = 85/4 feet to a height of 37 feet,
37 – 21 1/4 = 37 – 85/4 = 148/4 – 85/4 = 63/4 = 15 3/4
15 3/4 / 1 3/4 = 63/4 / 7/4 = 63/4 × 4/7 = 9 years

Question 11.
Explain how you would find how many 1 \(\frac{1}{2}\) cup servings there are in a pot that contains 22 \(\frac{1}{2}\) cups of soup.
Type below:
__________

Answer:
Given that, Total number of cups = 22 1/2
The number of cups required for each serving = 1 1/2
The number of servings = 22 1/2 ÷ 1 1/2
= 45/2 ÷ 3/2 = 45/3 = 15

Lesson Check – Page No. 124

Question 1.
Tom has a can of paint that covers 37 \(\frac{1}{2}\) square meters. Each board on the fence has an area of \(\frac{3}{16}\) square meters. How many boards can he paint?
______ boards

Answer:
200 boards

Explanation:
Tom has a can of paint that covers 37 \(\frac{1}{2}\) square meters. Each board on the fence has an area of \(\frac{3}{16}\) square meters.
37 \(\frac{1}{2}\) ÷ \(\frac{3}{16}\) = 200 square meters

Question 2.
A baker wants to put 3 \(\frac{3}{4}\) pounds of apples in each pie she makes. She purchased 52 \(\frac{1}{2}\) pounds of apples. How many pies can she make?
______ pies

Answer:
14 pies

Explanation:
A baker wants to put 3 \(\frac{3}{4}\) pounds of apples in each pie she makes. She purchased 52 \(\frac{1}{2}\) pounds of apples.
52 \(\frac{1}{2}\) ÷ 3 \(\frac{3}{4}\) = 14 pies

Spiral Review

Question 3.
The three sides of a triangle measure 9.97 meters, 10.1 meters, and 0.53 meter. What is the distance around the triangle?
______ meters

Answer:
20.6 meters

Explanation:
The distance around the triangle is call perimeter, to get it we must add the 3 sides.
So, 9.97 + 10.1 + 0.53 = 20.6 meters

Question 4.
Selena bought 3.75 pounds of meat for $4.64 per pound. What was the total cost of the meat?
$ ______

Answer:
$17.40

Explanation:
Selena bought 3.75 pounds of meat.
The cost of meat of one pound = $4.64
The total cost of the meat = 4.64 × 3.75 = $17.40
The total cost of 3.75 lb of meat was $17.40.

Question 5.
Melanie prepared 7 \(\frac{1}{2}\) tablespoons of a spice mixture. She uses \(\frac{1}{4}\) tablespoon to make a batch of barbecue sauce. Estimate the number of batches of barbecue sauce she can make using the spice mixture.
Type below:
__________

Answer:
30 batches of sauce

Explanation:
Melanie prepared 7 \(\frac{1}{2}\) tablespoons of a spice mixture. She uses \(\frac{1}{4}\) tablespoon to make a batch of barbecue sauce.
4 X 1/4 tbsp = 1 tbsp.
4 X 7 1/2 = 30.
she can make 30 batches of sauce

Question 6.
Arturo mixed together 1.24 pounds of pretzels, 0.78 pounds of nuts, 0.3 pounds of candy, and 2 pounds of popcorn. He then packaged it in bags that each contained 0.27 pounds. How many bags could he fill?
______ bags

Answer:
16 bags

Explanation:
Arturo mixed together 1.24 pounds of pretzels, 0.78 pounds of nuts, 0.3 pounds of candy, and 2 pounds of popcorn.
1.24 + 0.78 + 0.3 + 2 = 4.32
4.32/0.27 = 16

Page No. 127

Question 1.
There is \(\frac{4}{5}\) lb of sand in the class science supplies. If one scoop of sand weighs \(\frac{1}{20}\) lb, how many scoops of sand can Maria get from the class supplies and still leave \(\frac{1}{2}\) lb in the supplies?
Type below:
__________

Answer:
16 scoops

Explanation:
There is \(\frac{4}{5}\) lb of sand in the class science supplies. If one scoop of sand weighs \(\frac{1}{20}\) lb,
\(\frac{4}{5}\) ÷ \(\frac{1}{20}\) = 4/5 × 1/20 = 16 scoops

Question 2.
What if Maria leaves \(\frac{2}{5}\) lb of sand in the supplies? How many scoops of sand can she get?
______ scoops

Answer:
8 scoops

Explanation:
There is \(\frac{2}{5}\) lb of sand in the class science supplies. If one scoop of sand weighs \(\frac{1}{20}\) lb,
\(\frac{2}{5}\) ÷ \(\frac{1}{20}\) = 2/5 × 20 = 8

Question 3.
There are 6 gallons of distilled water in the science supplies. If 10 students each use an equal amount of the distilled water and there is 1 gal left in the supplies, how much will each student get?
\(\frac{□}{□}\) gallon

Answer:
\(\frac{1}{2}\) gallon

Explanation:
There are 6 gallons of distilled water in the science supplies.
There is 1 gal left in the supplies, 6 – 1 = 5
10 students each use an equal amount of the distilled water = 5/10 = 1/2
.5 gal for each student

On Your Own – Page No. 128

Question 4.
The total weight of the fish in a tank of tropical fish at Fish ‘n’ Fur was \(\frac{7}{8}\) lb. Each fish weighed \(\frac{1}{64}\) lb. After Eric bought some fish, the total weight of the fish remaining in the tank was \(\frac{1}{2}\) lb. How many fish did Eric buy?
______ fish

Answer:
386 fish

Explanation:
The total weight of the fish in a tank of tropical fish at Fish ‘n’ Fur was \(\frac{7}{8}\) lb. Each fish weighed \(\frac{1}{64}\) lb. After Eric bought some fish, the total weight of the fish remaining in the tank was \(\frac{1}{2}\) lb.
386 is the answer

Question 5.
Fish ‘n’ Fur had a bin containing 2 \(\frac{1}{2}\) lb of gerbil food. After selling bags of gerbil food that each held \(\frac{3}{4}\) lb, \(\frac{1}{4}\) lb of food was left in the bin. If each bag of gerbil food sold for $3.25, how much did the store earn?
$ ______

Answer:
$9.75

Explanation:
The store would earn 9.75$ because 3 bags of gerbil food is sold. Then you would multiply 3 by 3.25.

Question 6.
Describe Niko bought 2 lb of dog treats. He gave his dog \(\frac{3}{5}\) lb of treats one week and \(\frac{7}{10}\) lb of treats the next week. Describe how Niko can find how much is left.
Type below:
__________

Answer:
Niko bought 2 lb of dog treats. He gave his dog \(\frac{3}{5}\) lb of treats one week and \(\frac{7}{10}\) lb of treats the next week.
Let us find the amount of dog-food eaten by dogs in two months.
3/5 + 7/10 = 13/10
Now we will subtract the amount of food eaten by the dog from the amount of food initially to find the remaining amount of dog food.
2 – 13/10 = 7/10
Therefore, 7/10 pounds of food was remaining in the bag at the end of the two months.

Question 7.
There were 14 \(\frac{1}{4}\) cups of apple juice in a container. Each day, Elise drank 1 \(\frac{1}{2}\) cups of apple juice. Today, there is \(\frac{3}{4}\) cup of apple juice left. Derek said that Elise drank apple juice on nine days. Do you agree with Derek? Use words and numbers to explain your answer.
Type below:
__________

Answer:
Derek is correct.

Explanation:
An apple juice the container had 14 1/2 =14.25
She drank per day 1 1/2= 1.5
The left part in the container 3/4= .75
14.25 cups – .75 cup = 13.5 cups
13.5 cups ÷ 1.5 cups per day= 9 days

Problem Solving Fraction Operations – Page No. 129

Read each problem and solve.

Question 1.
\(\frac{2}{3}\) of a pizza was left over. A group of friends divided the leftover pizza into pieces each equal to \(\frac{1}{18}\) of the original pizza. After each friend took one piece, \(\frac{1}{6}\) of the original pizza remained. How many friends were in the group?
Go Math Grade 6 Answer Key Chapter 2 Fractions and Decimals 23
______ friends

Answer:
9 friends

Explanation:
Let us say that there are x friends.
Each one gets 1/18 of the original pizza: but this in turn leaves 1/6 of the 2/3 leftover.
1x/18 = 2/3 – 1/6
x = 12 – 3 = 9

Question 2.
Sarah’s craft project uses pieces of yarn that are \(\frac{1}{8}\) yard long. She has a piece of yarn that is 3 yards long. How many \(\frac{1}{8}\) -yard pieces can she cut and still have 1 \(\frac{1}{4}\) yards left?
______ pieces

Answer:
14 pieces

Explanation:
Sarah’s craft project uses pieces of yarn that are \(\frac{1}{8}\) yard long. She has a piece of yarn that is 3 yards long.
If she left 1 \(\frac{1}{4}\) yards left, 3 – 1 \(\frac{1}{4}\) = 7/4
7/4 ÷ \(\frac{1}{8}\) = 14

Question 3.
Alex opens a 1-pint container of orange butter. He spreads \(\frac{1}{16}\) of the butter on his bread. Then he divides the rest of the butter into \(\frac{3}{4}\) -pint containers. How many \(\frac{3}{4}\) -pint containers is he able to fill?
______ \(\frac{□}{□}\) containers

Answer:
1\(\frac{1}{4}\) containers

Explanation:
Alex opens a 1-pint container of orange butter. He spreads \(\frac{1}{16}\) of the butter on his bread.
1 – 1/16 = 15/16
Then he divides the rest of the butter into \(\frac{3}{4}\) -pint containers.
(15/16) ÷ (3/4) = 5/4 = 1 1/4

Question 4.
Kaitlin buys \(\frac{9}{10}\) a pound of orange slices. She eats \(\frac{1}{3}\) of them and divides the rest equally into 3 bags. How much is in each bag?
______ lb

Answer:
17/90 lb

Explanation:
Kaitlin buys \(\frac{9}{10}\) a pound of orange slices. She eats \(\frac{1}{3}\) of them and divides the rest equally into 3 bags.
If she starts with 9/10 pounds and has eaten 1/3 of them, 9/10 – 1/3 = 17/30
This is the amount she has left. Let’s divide this value by 3 to see how many pounds are in one bag.
(17/30)/3 = 17/90
There are 17/90 pounds in one bag.

Question 5.
Explain how to draw a model that represents \(\left(1 \frac{1}{4}-\frac{1}{2}\right) \div \frac{1}{8}\).
Type below:
__________

Answer:
Divide 2 bars into 8 quarters.
Below that draw 1 1/4 or 5 quarters.
Remove 1/2 or 2 quarters
Divide each of the 3 quarters left into 2 eighths

Explanation:
\(\left(1 \frac{1}{4}-\frac{1}{2}\right) \div \frac{1}{8}\)
1 1/4 -1/2 = 5/4 – 1/2 = 3/4
3/4 ÷ 1/8 = 6

Lesson Check – Page No. 130

Question 1.
Eva wanted to fill bags with \(\frac{3}{4}\) pounds of trail mix. She started with 11 \(\frac{3}{8}\) pounds but ate \(\frac{1}{8}\) pound before she started filling the bags. How many bags could she fill?
______ bags

Answer:
15 bags

Explanation:
11 and 3/8-1/8=11 and 2/8=11 and 1/4
3/4 times x bags=11 and 1/4
convert 11 and 1/4 to improper fraction
11 and 1/4 = 11 + 1/4 = 44/4 + 1/4 = 45/4
3/4 times x bags=45/4
x bags = 45/4 × 4/3 = 15 bags
she could fill 15 bags

Question 2.
John has a roll containing 24 \(\frac{2}{3}\) feet of wrapping paper. He wants to divide it into 11 pieces. First, though, he must cut off \(\frac{5}{6}\) foot because it was torn. How long will each piece be?
______ \(\frac{□}{□}\) feet

Answer:
2\(\frac{4}{25}\) feet

Explanation:
John had a roll containing wrapping paper = 24 2/3 = 74/3
First, he must cut off 5/6 foot because it was torn.
He wants to divide it into 11 pieces.
74/3 – 5/6
Taking the L.C.M of 3 and 6 is 6
(148-5)/6 = 143/6 = 23.83 feet
He wants to divide it into 11 pieces. length of the each piece = 23.83/11 = 2.16 feet

Spiral Review

Question 3.
Alexis has 32 \(\frac{2}{5}\) ounces of beads. How many necklaces can she make if each uses 2 \(\frac{7}{10}\) ounces of beads?
______ necklaces

Answer:
12 necklaces

Explanation:
Alexis has 32 \(\frac{2}{5}\) ounces of beads.
If each uses 2 \(\frac{7}{10}\) ounces of beads, 32 \(\frac{2}{5}\) × 2 \(\frac{7}{10}\)
32 \(\frac{2}{5}\) = 162/5
2 \(\frac{7}{10}\) = 27/10
162/5 × 27/10 = 12 necklaces

Question 4.
Joseph has $32.40. He wants to buy several comic books that each cost $2.70. How many comic books can he buy?
______ comic books

Answer:
12 comic books

Explanation:
Joseph has $32.40. He wants to buy several comic books that each cost $2.70.
$32.40/$2.70 = 12 comic books

Question 5.
A rectangle is 2 \(\frac{4}{5}\) meters wide and 3 \(\frac{1}{2}\) meters long. What is its area?
______ \(\frac{□}{□}\) m2

Answer:
9\(\frac{4}{5}\) m2

Explanation:
2 \(\frac{4}{5}\) = 14/5
3 \(\frac{1}{2}\) = 7/2
14/5 × 7/2 = 9 4/5

Question 6.
A rectangle is 2.8 meters wide and 3.5 meters long. What is its area?
______ m2

Answer:
9.8 m2

Explanation:
A rectangle is 2.8 meters wide and 3.5 meters long.
2.8 × 3.5 = 9.8

Chapter 2 Review/Test – Page No. 131

Question 1.
Write the values in order from least to greatest.
Go Math Grade 6 Answer Key Chapter 2 Fractions and Decimals 24
Type below:
__________

Answer:
0.45, 0.5, 5/8, 3/4

Explanation:
3/4 = 0.75
5/8 = 0.625
0.45, 0.5
0.45 < 0.5 < 0.625 < 0.75

Question 2.
For numbers 2a–2d, compare. Choose <, >, or =.
2a. 0.75 _____ \(\frac{3}{4}\)
2b. \(\frac{4}{5}\) _____ 0.325
2c. 1 \(\frac{3}{5}\) _____ 1.9
2d. 7.4 _____ 7 \(\frac{2}{5}\)

Answer:
2a. 0.75 = \(\frac{3}{4}\)
2b. \(\frac{4}{5}\) > 0.325
2c. 1 \(\frac{3}{5}\) < 1.9
2d. 7.4 = 7 \(\frac{2}{5}\)

Explanation:
2a. 3/4 = 0.75
0.75 = 0.75
2b. \(\frac{4}{5}\) = 0.8
0.8 > 0.325
2c. 1 \(\frac{3}{5}\) = 8/5 = 1.6
1.6 < 1.9
2d. 7 \(\frac{2}{5}\) = 37/5 = 7.4
7.4 = 7.4

Question 3.
The table lists the heights of 4 trees.
Go Math Grade 6 Answer Key Chapter 2 Fractions and Decimals 25
For numbers 3a–3d, select True or False for each statement.
3a. The oak tree is the shortest. True False
3b. The birch tree is the tallest. True False
3c. Two of the trees are the same height. True False
3d. The sycamore tree is taller than the maple tree. True False
Type below:
__________

Answer:
3a. The oak tree is the shortest. True
3b. The birch tree is the tallest. False
3c. Two of the trees are the same height. False
3d. The sycamore tree is taller than the maple tree. False

Explanation:
Sycamore = 15 2/3 = 47/3 = 15.666
Oak = 14 3/4 = 59/4 = 14.75
Maple = 15 3/4 = 63/4 = 15.75
Birch = 15.72

Page No. 132

Question 4.
For numbers 4a–4d, choose Yes or No to indicate whether the statement is correct.
Go Math Grade 6 Answer Key Chapter 2 Fractions and Decimals 26
4a. Point A represents 1.0. Yes No
4b. Point B represents \(\frac{3}{10}\). Yes No
4c. Point C represents 6.5. Yes No
4d. Point D represents \(\frac{4}{5}\). Yes No
Type below:
__________

Answer:
4a. Point A represents 1.0. Yes
4b. Point B represents \(\frac{3}{10}\). Yes
4c. Point C represents 6.5. No
4d. Point D represents \(\frac{4}{5}\). Yes

Question 5.
Select the values that are equivalent to one twenty-fifth. Mark all that apply.
Options:
a. 125
b. 25
c. 0.04
d. 0.025

Answer:
c. 0.04

Explanation:
one twenty-fifth = 1/25 = 0.04

Question 6.
The table shows Lily’s homework assignment. Lily’s teacher instructed the class to simplify each expression by dividing the numerator and denominator by the GCF. Complete the table by simplifying each expression and then finding the product.
Go Math Grade 6 Answer Key Chapter 2 Fractions and Decimals 27
Type below:
___________

Answer:
a. Simplified Expression: 1/10
Product: 0.1
b. Simplified Expression: 1/2
Product: 0.5
c. Simplified Expression: 15/56
Product: 0.267
d. Simplified Expression: 1/12
Product: 0.083

Explanation:
a. 2/5 × 1/4 = 2/20
Simplify using the GCF.
The GCF of 2 and 20 is 2.
Divide the numerator and the denominator by 2.
So, 1/10 is the answer.
Product: 0.1
b. 4/5 × 5/8 = 1/2
Product: 0.5
c. 3/7 × 5/8 = 15/ 56
Product: 0.267
d. 4/9 × 3/16 = 1/12
Product: 0.083

Page No. 133

Question 7.
Two-fifths of the fish in Gary’s fish tank are guppies. One fourth of the guppies are red. What fraction of the fish in Gary’s tank are red guppies? What fraction of the fish in Gary’s tank are not red guppies? Show your work.
Type below:
___________

Answer:
1/10 of the fish are red guppies.
and 9/10 of the fish are not red guppies.

Explanation:
two-fifths of the fish in Gary’s fish tank are guppies.
One-fourth of the guppies are red.
Let the total number of fish in Gary’s fish tank be x.
It is given that two-fifths of the fish in Gary’s fish tank are guppies.
So, the number of guppies in Gary’s fish tank is 2/5 × x
Given that One-fourth of the guppies are red.
number of red guppies = 1/4 × 2x/5 = x/10
So, 1/10 of the fish are red guppies.
1 – 1/10 = 9/10 of the fish are not red guppies.

Question 8.
One-third of the students at Finley High School play sports. Two-fifths of the students who play sports are girls. What fraction of all students are girls who play sports? Use numbers and words to explain your answer.
Type below:
___________

Answer:
One-third of the students at Finley High School play sports. Two-fifths of the students who play sports are girls.
1/3 × 2/5 = 2/15 of the girls in the school play sports.

Question 9.
Draw a model to find the quotient.
\(\frac{3}{4}\) ÷ 2 =
\(\frac{3}{4}\) ÷ \(\frac{3}{8}\) =
How are your models alike? How are they different?
Type below:
___________

Answer:
grade 6 chapter 2 image 5

Explanation:
\(\frac{3}{4}\) ÷ 2 = 3/4 × 1/2 = 3/8
\(\frac{3}{4}\) ÷ \(\frac{3}{8}\) = 3/4 × 8/3 = 2
Both models are multiplying with the 3/4.
The number line model shows how many groups of 3/8 are in 3/4.

Question 10.
Explain how to use a model to find the quotient.
2 \(\frac{1}{2}\) ÷ 2 =
Type below:
___________

Answer:
5/4

Explanation:
2 1/2 = 5/2
5/2 groups of 2
5/2 ÷ 2 = 5/2 × 1/2 = 5/4

Page No. 134

Divide. Show your work.

Question 11.
\(\frac{7}{8}\) ÷ \(\frac{3}{5}\) =
_______ \(\frac{□}{□}\)

Answer:
1 \(\frac{11}{24}\)

Explanation:
\(\frac{7}{8}\) ÷ \(\frac{3}{5}\)
\(\frac{7}{8}\) × \(\frac{5}{3}\) = 35/24 = 1 \(\frac{11}{24}\)

Question 12.
\(2 \frac{1}{10} \div 1 \frac{1}{5}=\) =
_______ \(\frac{□}{□}\)

Answer:
1 \(\frac{3}{4}\)

Explanation:
2 \(\frac{1}{10}\) = 21/10
1 \(\frac{1}{5}\) = 6/5
(21/10) ÷ (6/5) = 7/4 or 1 3/4

Question 13.
Sophie has \(\frac{3}{4}\) quart of lemonade. If she divides the lemonade into glasses that hold \(\frac{1}{16}\) quart, how many glasses can Sophie fill? Show your work
_______ glasses

Answer:
12 glasses

Explanation:
Let x be the number of glasses
1/16x = 3/4
x = 3/4 × 16 = 3 × 4 = 12 glasses

Question 14.
Ink cartridges weigh \(\frac{1}{8}\) pound. The total weight of the cartridges in a box is 4 \(\frac{1}{2}\) pounds. How many cartridges does the box contain? Show your work and explain why you chose the operation you did.
_______ cartridges

Answer:
36 cartridges

Explanation:
Weight of ink cartridges = 1/8 pounds
Total weight of the cartridges in a box = 4 1/2 = 9/2 pounds
so, the Number of cartridges that box contain is given by
9/2 ÷ 1/8 = 36
Hence, there are 36 cartridges that box contain.

Question 15.
Beth had 1 yard of ribbon. She used \(\frac{1}{3}\) yard for a project. She wants to divide the rest of the ribbon into pieces \(\frac{1}{6}\) yard long. How many \(\frac{1}{6}\) yard pieces of ribbon can she make? Explain your solution.
_______ pieces

Answer:
4 pieces

Explanation:
Beth had 1 yard of ribbon. She used \(\frac{1}{3}\) yard for a project.
1 – \(\frac{1}{3}\) = 2/3 yard left
She wants to divide the rest of the ribbon into pieces \(\frac{1}{6}\) yard long.
2/3 ÷ 1/6 = 4

Page No. 135

Question 16.
Complete the table by finding the products. Then answer the questions in Part A and Part B.
Go Math Grade 6 Answer Key Chapter 2 Fractions and Decimals 28
Part A
Explain how each pair of division and multiplication problems are the same, and how they are different.
Type below:
___________

Answer:
1/5 ÷ 3/4 = 4/15; 1/5 × 4/3 = 4/15
2/13 ÷ 1/5 = 10/13; 2/13 × 5/1 = 10/13
4/5 ÷ 3/5 = 4/3; 4/5 × 5/3 = 4/3
the product of the each pair of division and multiplication problems are the same.
They are different from the operation performed.

Question 16.
Part B
Explain how to use the pattern in the table to rewrite a division problem involving fractions as a multiplication problem.
Type below:
___________

Answer:
First, since it’s the division you have to change the second fraction which is called the reciprocal. That means the second fraction has to be flipped before you can multiple the fractions.

Page No. 136

Question 17.
Margie hiked a 17 \(\frac{7}{8}\) mile trail. She stopped every 3 \(\frac{2}{5}\) miles to take a picture. Martin and Tina estimated how many times Margie stopped.
Go Math Grade 6 Answer Key Chapter 2 Fractions and Decimals 29
Who made the better estimate? Use numbers and words to explain your answer.
Type below:
___________

Answer:
Margie hiked a 17 7/8 mile trail.
Distance hiked by Margie = 17 7/8 = 143/8 mile.
She stopped every 3 2/5 miles to take a picture = 17/5 mile
Number of pictures = (143/8) ÷ (17/5) = 715/136 = 5.28
So she can take a maximum of 6 pictures and a minimum of 5 pictures.
B is the correct answer.

Question 18.
Brad and Wes are building a tree house. They cut a 12 \(\frac{1}{2}\) foot piece of wood into 5 of the same length pieces. How long is each piece of wood? Show your work.
_______ \(\frac{□}{□}\) foot

Answer:
2 \(\frac{1}{2}\) foot

Explanation:
Brad and Wes cut a 12 1/2foot piece of wood into 5 of the same length.
Let the length of 1 piece be x
So, Length of 5 pieces = 5x
The total length of wood = 25/2
5x = 25/2
x = 5/2 = 2 1/2

Free Grade 6 HMH Go Math Answer Key PDF Download

In addition, to the exercise problems students of grade 6 can get the solutions for mid-chapter checkpoint and review test also. So, start solving the problems which are at the end of the chapter and check the solutions from here. Hope the information regarding Go Math 6th Grade Answer Key Chapter 2 Fractions and Decimals is helpful for you to overcome the issues in maths. Check out the links and start solving all the questions.